Onko enää idealisteja?

Järkisyitä

En juurikaan tunne idealisteja, jotka siis ajattelevat, että todellsuus on perimmäiseltä luonteeltaan henkinen tai mielen sisäinen.

Dualisteja toki tiedän, jotka uskovat materialismin lisäksi henkimaailmaan.

Mutta onko täällä ketään idealistia, joka haluaisi selventää ajatuksiaan?

178

3034

    Vastaukset

    Anonyymi (Kirjaudu / Rekisteröidy)
    5000
    • kiikkustuolista

      Idealismi ja materialismi ovat periaatteessa samankaltaisia oppirakennelmia. Toisessa vain mieli on yhdistetty materiaan ja toisessa vastaavasti materia mieleen. Molemmat suuntaukset ovat rationaalisesti hyvin perusteltuja. Siinä mielessä on aivan samantekevää kumpaa kannattaa tai suosii. Elämismaailma on molemmissa tapauksissa kuitenkin sama. Eroja alkaakin löytymään vasta asennetasolla.

      • AntinMateria

        En pidä idelismia hyvin perusteltuna. Mahtaako pitää moni muukaan luonnontieteilijä.


      • kiikkustuolista
        AntinMateria kirjoitti:

        En pidä idelismia hyvin perusteltuna. Mahtaako pitää moni muukaan luonnontieteilijä.

        Ilmeisesti liität idealismin johonkin uskonnollisuuteen. Tai sitten näkemyksesi idealismista on kovin rajoittunut. Jo pelkästään inhimillinen mieli koostuu ideoista. Näin ollen myös edustamasi luonnontiede on luonnontieteellisten ideoiden kyllästämä. Pelkään pahoin, että me joudumme näin hyväksymään idealismin ainakin osittain varteen otettavana ajatusrakennelmana. Tietenkin vulgäärimaterialistit kieltävät kaiken idealismiin viittaavankin.


      • Funtsittuina

        Järkisyitä kirjoitti:

        "Onko enää idealisteja?

        En juurikaan tunne idealisteja, jotka siis ajattelevat, että todellsuus on perimmäiseltä luonteeltaan henkinen tai mielen sisäinen.

        Dualisteja toki tiedän, jotka uskovat materialismin lisäksi henkimaailmaan.

        Mutta onko täällä ketään idealistia, joka haluaisi selventää ajatuksiaan?"

        ..

        Olemme Boltzmannin aivoissa.


      • Musta-Aurinko-Nousee

        Nyt ei useimmille kreationisteillekaan riitä tieto, että Jumala yksinkertaisesti loi maailman! Kuinka Hän sen teki? Mitä Hän käytti apunaan? Onko kaikki Materia osa Jumalan Henkeä jne...
        Tuohon viimeksimainittuun sanoisin, että kaikki todella on Jumalan henkeä, massa tiivistettynä hahtuvasta hengen olomuodostaan orjallisesti maailmankaikkuden lakeja noudattavaan materiaan.

        Fyysinen maailma syntyi jakautuneen tai itsensä pirstaloineen Yhden ja ainoan tosiolevan Hengen suhteista. Vaikka tämä Yksi (Jumala, Luoja, Maailmansielu, Logos jne.) monisti jakamalla itseään, se on edelleen yksi kuten rikkoutunut peili, joka monine sirpaleineen heijastaa samaa kuvaa, niin tämä Yksi Henki heijastuu maailman fyysisistä osista. Yhdestä tuli näin myös monta, ja näin tämä tosiolevainen Yksi kokee moninaisuutta, suhteita eri näkökulmista.
        Fyysinen ulotteisuus ja sen kvantittuneet ajan ja avaruuden yksiköt, ovat myös hengen erkaleita. Itse asiassa Yhdestä jakautuneet hengen erkaleet, ja niiden väliset vuorovaikutussuhteet muodostavat fyysisen ulotteisen maailman. Näin Henki on perustavanlaatuisempaa kuin Liha eli fyysinen olevainen. Erkaantuneet Hengen suhteet vain näkyvät fyysinä ulotteisina suhteina. Näin Yksi kokee moninaisuutta. Maailma on näin ollen vain automaatti, joka noudattaa järkähtämättömiä jakautuneen hengen erkaleiden vuorovaikutussuhteita. Tämä ylösnousemusautomaatti kulkee yhdestä moneen ja lopulta taas monesta yhteen Suuren Suunnitelman mukaan. Kaikessa pienimmästä suurimpaan on näin henki ja kokija, tuon Yhden ja ainoan näkökulma eli sielu. Myös me ihmiset olemme tuon automaatin kokijoita, Yhden eri näkökulmia. Mutta samalla lailla myös jokainen yksinkertaisempi objekti; kissa, sammakko, hyttynen, ameeba, vetyatomi, elektroni on henkinen kokija tässä maailmanautomaatissa. Yksinkertaiset henkiset rakenteet muodostavat yhdessä monimutkaisempia henkisiä rakenteita ja samalla myös monimutkaisempia fyysisiä systeemejä. Vaikka siis aivoissa oleva yksittäinen vetyatomi on henkinen kokija, se osana lukematonta atomijoukkoa muodostaa monimutkaisemman henkisen kokevan systeemin.


      • PyhäHenki

        Henki on Pyhä, ja se on ainut olevainen. Me olemme tuon Yhden Hengen sielullisia ilmentymiä, eli Yhden eri näkökulmia.

        Jos Henki loi aineen, se on ihme. Jos aine loi Hengen, se on ihmeiden ihme! Liha on hengestä, eikä Henki lihasta.


      • HenkiKokeeMeissä

        Ajattelen, siis olen olemassa. Eikös tämä jo kerro että henki on se ainut varma asia?


      • Hengessä
        Musta-Aurinko-Nousee kirjoitti:

        Nyt ei useimmille kreationisteillekaan riitä tieto, että Jumala yksinkertaisesti loi maailman! Kuinka Hän sen teki? Mitä Hän käytti apunaan? Onko kaikki Materia osa Jumalan Henkeä jne...
        Tuohon viimeksimainittuun sanoisin, että kaikki todella on Jumalan henkeä, massa tiivistettynä hahtuvasta hengen olomuodostaan orjallisesti maailmankaikkuden lakeja noudattavaan materiaan.

        Fyysinen maailma syntyi jakautuneen tai itsensä pirstaloineen Yhden ja ainoan tosiolevan Hengen suhteista. Vaikka tämä Yksi (Jumala, Luoja, Maailmansielu, Logos jne.) monisti jakamalla itseään, se on edelleen yksi kuten rikkoutunut peili, joka monine sirpaleineen heijastaa samaa kuvaa, niin tämä Yksi Henki heijastuu maailman fyysisistä osista. Yhdestä tuli näin myös monta, ja näin tämä tosiolevainen Yksi kokee moninaisuutta, suhteita eri näkökulmista.
        Fyysinen ulotteisuus ja sen kvantittuneet ajan ja avaruuden yksiköt, ovat myös hengen erkaleita. Itse asiassa Yhdestä jakautuneet hengen erkaleet, ja niiden väliset vuorovaikutussuhteet muodostavat fyysisen ulotteisen maailman. Näin Henki on perustavanlaatuisempaa kuin Liha eli fyysinen olevainen. Erkaantuneet Hengen suhteet vain näkyvät fyysinä ulotteisina suhteina. Näin Yksi kokee moninaisuutta. Maailma on näin ollen vain automaatti, joka noudattaa järkähtämättömiä jakautuneen hengen erkaleiden vuorovaikutussuhteita. Tämä ylösnousemusautomaatti kulkee yhdestä moneen ja lopulta taas monesta yhteen Suuren Suunnitelman mukaan. Kaikessa pienimmästä suurimpaan on näin henki ja kokija, tuon Yhden ja ainoan näkökulma eli sielu. Myös me ihmiset olemme tuon automaatin kokijoita, Yhden eri näkökulmia. Mutta samalla lailla myös jokainen yksinkertaisempi objekti; kissa, sammakko, hyttynen, ameeba, vetyatomi, elektroni on henkinen kokija tässä maailmanautomaatissa. Yksinkertaiset henkiset rakenteet muodostavat yhdessä monimutkaisempia henkisiä rakenteita ja samalla myös monimutkaisempia fyysisiä systeemejä. Vaikka siis aivoissa oleva yksittäinen vetyatomi on henkinen kokija, se osana lukematonta atomijoukkoa muodostaa monimutkaisemman henkisen kokevan systeemin.

        "Kokija, mentaalisen tapahtuman tai tilan tajunnallinen ja siis elollinen osallistuja."

        Jos ja kun maailma rakentuu pienimmistä paloista fyysistä ulotteisuutta aikaa ja avaruutta, ja nämä ajan ja avaruuden yksiköt -erkaleet,kvantit todelliset atomit eli jakamattomat ovat samalla pienin pala mentaalista -henkistä tilaa, niin eikö koko maailma ole tiedostava, kokeva eli elävä?

        Tämä pienin ajan ja avaruuden kvantti on pienin mentaalinen yksikkö, tajunnallinen yksikkö tai yksinkertaisin tajunnallinen kokija. Näistä fyysisen ja mentaalisen maailman yksiköistä _yhdessä_ muodostuu fyysinen systeemi, vaikkapa atomi, joka samalla on myös henkinen, tajunnallinen systeemi, mentaalisten yksiköiden järjestelmä eli kokija. Näin aivot eivät ole tajunnallisuuden tai kokijan luoja, vaan jokaiseen maailman osaseen ja systeemiin liittyy mentaalinen puoli. Näin ollen tämä 'prima materia' perusosanen on maailman pienin mentaalinen yksikkö -kokija ja koko maailma tai multiversumi kokonaisuutena suurin mentaalinen systeemi tai kokija.
        Tajunnallisella kokijalla on tietty rajattu näkökulma maailmaan, eli sielullinen tila, paitsi koko maailmalla tai multiversumilla kokonaisuutena joka kokee kaiken. Näin olemme määrittäneet sielun, se on tajunnallisen kokijan näkökulma maailmaan.

        Viesti perustuu oletukseen että maailma on tajunnallinen tai mentaalinen jo perustaltaan, ei ainoastaan ihmisaivojen tasolla. Siis aivot eivät luo henkistä tai tajunnallista kokijaa, vaan tietoisuus ilmenee ihmisaivoissa ja sen rakenteessa tietynlaisena, mutta on mukana jo maailman perusrakenteessa ja kaikissa sen osissa. Eli aine ei luo tietoisuutta, vaan tietoisuus on toinen pooli, joka on aina mukana. Samoin kuin aineen rakenneosat luovat aivot, yksinkertaisemmat tietoisuuden perusosat luovat ihmisaivojen kokeman itsetietoisuuden.


      • Musta_aurinko_nousee

        Jos henki on vahva niin pienikin työ riittää maailman luomiseen.


    • Belisario

      Onhan niitä idealisteja aina ollut ja tulee varmaan aina olemaan vaikka nykyisen kulttuurin ja valtavirtatieteen puitteissa se ei olekaan nykyään kovin suosittua ja muodikasta.

      Yksi tunnetuimmista nykyisistä idealisteista on Bernardo Kastrup joka kirjoitti äskettäin melkoista kohua herättäneen artikkelin Scientific American sivustolle.

      https://blogs.scientificamerican.com/observations/could-multiple-personality-disorder-explain-life-the-universe-and-everything/

      Kastrupin mukaan me kaikki kokijat ja havaitsijat olemme yhden kosmoksen tietoisuuden dissosiaatioita eli kaikki kärsimme jakaantuneesta persoonallisuudesta.

      Kvanttifysiikan pioneereista ainakin Max Planck, Werner Heisenberg ja Erwin Schrödiger olivat filosofiselta kannaltaan selkeästi jonkinlaisia idealisteja. Myös idän uskonnoissa ja maailmankatsomuksissa idealistinen näkemys oli ja on enemmän sääntö kuin poikkeus.

      Tietyssä mielessä idealismi on paljon koherentimpi maailmankuva kuin materialismi. Materialismissa ei kyetä mitenkään selittämään eikä kuvaamaan miten elämyksellinen tietoisuus eli 1. persoonan kokemus syntyy aineellisten aivojen tuotoksena. Idealismin kannalta taas on hyvin helppoa selittää ns. materian syntyminen tietoisuudesta koska se materia on vain 3. persoonan ulkopuolinen näkökulma asioihin.

      Solipsismia pidetään idealismin suurimpana ongelma. Todellisuudessa se ei ole mikään ongelma vaan yksi idealismin aspekteista eli se että olemme yhden yhteisen tietoisuuden jakautuneita havaitsijoita.

      Sekään ei ole mikään ongelma että idealismissa ns. aine on tavallaan illuusio koska aineen näennäinen konkreettisuus katoaa aina hyvin pienessä mittakaavassa.

      Tietysti voi toisaalta ajatella että länsimainen tiede ja tekniikka ei olisi koskaan kehittynyt nykyiselle tasolle jos valtafilosofiana olisi ollut idealismi. Nyt kuitenkin kun tämä materialismi on mennyt niin pitkälle että hyödyt alkavat olla pienempiä kuin materialismin tuottavat ongelmat olisi ehkä syytä siirtyä ainakin vähän enemmän idealismiin päin ja kehittää välillä sitä ihmisen henkistä puolta.

      • Belisario

        Lisäys:

        Matematiikka ei ole idealistista eikä materialistista.

        Tunti päivässä idealismia on yleensä riittävä annostus - lopun ajan voi sitten olla materialisti kunhan ei pistä aivan överiksi. (:-))


      • SeliSeli

        "Nyt kuitenkin kun tämä materialismi on mennyt niin pitkälle että hyödyt alkavat olla pienempiä kuin materialismin tuottavat ongelmat olisi ehkä syytä siirtyä ainakin vähän enemmän idealismiin päin ja kehittää välillä sitä ihmisen henkistä puolta. "

        Eli takaisin hyödyttömään taikauskoon.


      • tjtjtgjhty

        " Idealismin kannalta taas on hyvin helppoa selittää ns. materian syntyminen tietoisuudesta koska se materia on vain 3. persoonan ulkopuolinen näkökulma asioihin."

        Tarkoittaen niin kuin mitä?


      • kiikkustuolista
        tjtjtgjhty kirjoitti:

        " Idealismin kannalta taas on hyvin helppoa selittää ns. materian syntyminen tietoisuudesta koska se materia on vain 3. persoonan ulkopuolinen näkökulma asioihin."

        Tarkoittaen niin kuin mitä?

        Mieli on ollut materialisteille aina ongelmallinen siitäkin huolimatta, että heillä itselläänkin on sellainen. Suurimmat ongelmat nousevat kahdesta lähteestä, nimittäin erilaisista ominaisuuksista ja kausaalisuudesta. Aineen ominaisuus on ulottuvuus, vastaavasti mielellä se on ajattelu. Miten siis aineeton ajattelu ja ulottuvainen materia voivat olla kausaalisuhteessa keskenään?

        Ongelmaa on pyritty ratkaisemaan ns. ominaisuusdualismin avulla. Esimerkiksi Spinoza päätyi yhden substanssin malliin. Eli on vain yksi substanssi, joka on sekä ajatteleva, että ulottuvainen. Myöhemmin substanssin käsitteestä on siirrytty enemmän sen ominaisuuksien puolelle ja vastaavasti olevasta sen tapaan olla. Aineeton mieli sekä aineellinen substanssi eli keho, ovat edelleenkin erillisiä, mutta aineellinen puoli toimii mielen "realisaattorina". Tämän toivotaan olevan lähempänä intuitiotamme todellisuudesta.


      • koittakaakehitellä
        kiikkustuolista kirjoitti:

        Mieli on ollut materialisteille aina ongelmallinen siitäkin huolimatta, että heillä itselläänkin on sellainen. Suurimmat ongelmat nousevat kahdesta lähteestä, nimittäin erilaisista ominaisuuksista ja kausaalisuudesta. Aineen ominaisuus on ulottuvuus, vastaavasti mielellä se on ajattelu. Miten siis aineeton ajattelu ja ulottuvainen materia voivat olla kausaalisuhteessa keskenään?

        Ongelmaa on pyritty ratkaisemaan ns. ominaisuusdualismin avulla. Esimerkiksi Spinoza päätyi yhden substanssin malliin. Eli on vain yksi substanssi, joka on sekä ajatteleva, että ulottuvainen. Myöhemmin substanssin käsitteestä on siirrytty enemmän sen ominaisuuksien puolelle ja vastaavasti olevasta sen tapaan olla. Aineeton mieli sekä aineellinen substanssi eli keho, ovat edelleenkin erillisiä, mutta aineellinen puoli toimii mielen "realisaattorina". Tämän toivotaan olevan lähempänä intuitiotamme todellisuudesta.

        Itse olen sitä mieltä, että ns. mind-body ongelma johtuu väärästä kysymyksen asettelusta. Ajatellaan juuri tuohon tapaan, että mielen ominaisuus on aineeton ajattelu.

        En ukso aineettomaan ajatteluun enkä mieleen. Ne ovat varsin suorassa suhteessa fysikaalisiin aivotoimintoihin. Esim. aivovamma voi muuttaa persoonallisuutta. Päihteillä saadaan suoria vaikutuksia mielentilaan. En havaitse aineettomuutta, se on harhainen käsitys.

        Aineellinen mieli ei kumoa vapaata tahtoa, vaan sen selityksenä on informaation käsittelykyky, joka aivoilla on elämänkokemuksen myötä.


      • Belisario
        koittakaakehitellä kirjoitti:

        Itse olen sitä mieltä, että ns. mind-body ongelma johtuu väärästä kysymyksen asettelusta. Ajatellaan juuri tuohon tapaan, että mielen ominaisuus on aineeton ajattelu.

        En ukso aineettomaan ajatteluun enkä mieleen. Ne ovat varsin suorassa suhteessa fysikaalisiin aivotoimintoihin. Esim. aivovamma voi muuttaa persoonallisuutta. Päihteillä saadaan suoria vaikutuksia mielentilaan. En havaitse aineettomuutta, se on harhainen käsitys.

        Aineellinen mieli ei kumoa vapaata tahtoa, vaan sen selityksenä on informaation käsittelykyky, joka aivoilla on elämänkokemuksen myötä.

        "Itse olen sitä mieltä, että ns. mind-body ongelma johtuu väärästä kysymyksen asettelusta. "

        Niin varmaan johtuukin. Varsin keinotekoisesti on erotettu yksi osa kokemuksesta (aistihavainnot) "aineelliseksi" ja sitten se muu kokemus taas "henkiseksi" tai aineettomaksi.

        Aistikokemuksen säännönmukaista osaa on sitten alettu kutsua "luonnoksi" tai luonnonlakien kokoelmaksi. Yksilön aistihavainto on kuitenkin ihan yhtä subjektiivinen asia kuin esim. tunteet tai ajattelu.


        "En ukso aineettomaan ajatteluun enkä mieleen. Ne ovat varsin suorassa suhteessa fysikaalisiin aivotoimintoihin. Esim. aivovamma voi muuttaa persoonallisuutta. Päihteillä saadaan suoria vaikutuksia mielentilaan. En havaitse aineettomuutta, se on harhainen käsitys."

        Kyseessä on pelkkä korrelaatio. Yhtä hyvin voisi sanoa että aivot ovat se miltä tietoisuus näyttää kolmannen persoonan tai pelkästään aistihavaintojen kannalta. Ns. "aine" on ihan yhtä vaikea täsmällisesti määritellä kuin ns. "tietoisuus" tai "henki".

        Luonnollinen kieli on pääasiassa rakentunut ns. fysikaalisen maailman eli 3. persoonan kokemusta kuvaamaan. Toisen ihmisen 1. persoonan kokemuksista puhutaan usein vertauskuvin ja yhdistetään ne sitten omaan henk. kohtaiseen kokemuksiin niin hyvin kuin löytyy vastaavuuksia.

        Jyrkkä erottelu aineen ja hengen välillä juontaa juurensa Descartesin filosofiaan jossa hän ikäänkuin jakoi reviirit siten että henki kuuluu kirkolle ja aine tieteelle. Kyseessä lienee kuitenkin yksi "substanssi" jossa on sekä aineellisia että psyykkisiä/henkisiä ominaisuuksia. Kokemus sinänsä on aina yhtenäinen ja jakamaton. Käytetty kieli vain sotkee tässä tapauksessa ajattelua.

        On myös tapauksia joissa henkilöllä on hyvin vähän aivokudosta mutta silti älykkyys voi olla keskimääräistä korkeampi eikä ole havaittavaa häiriötä muissakaan toiminnoissa (esim. Lorber).

        Aineen ominaisuuksista (kuten esim. massa. varaus ja paikka ) on äärimmäisen vaikea tai mahdotonta loogisesti johtaa psyyken ominaisuuksia jotka eivät ole selkeästi paikallistettavissa. Kyse ei ole myöskään mistään laskentatehosta vaikka hyvin helposti rinnastetaan kunkin ajan huipputekniikka tietoisuuden toimintaan (esim. aivot tietokoneena).

        Mielestäni aivot ovat lähinnä jonkinlainen käyttöliittymä tai vastaanotin ns. fysikaaliseen tasoon ja itse psyyke taas enemmän jonkinlainen epälokaali korkeamman organisoitumistason ainetta tai energiaa. Aivojen tutkiminen tietoisuuden ominaisuuksien selvittämiseksi vastaa lähinnä tv-vastaanottimen tutkimista kun pitäisi tutkia esim. sitä ihmistä tai ilmiötä joka näkyy siinä tv:ssä.

        Ns. sivupersoonailmiö myös tukee tuota vastaanotin mallia koska siinä kehon ja aivojen käyttöjärjestelmän ottaa valtaansa toinen persoona (vrt. meediot, ns. kanavointi yms. ilmiöt)

        Henkiset toiminnot kuten ajattelu vaatii energiaa ja tarkkaavaisuutta ja ne kummatkin ovat rajoitettuja ihan samalla tavalla kuin mikä tahansa muukin ihmisen toiminta kuten juokseminen tai joku fyysisen työn tekeminen. Se fyysisen työn vaatima energia on kuitenkin erilaatuista kuin se ajatteluun tai opiskeluun tarvittava energia.


      • koittakaakehitellä
        Belisario kirjoitti:

        "Itse olen sitä mieltä, että ns. mind-body ongelma johtuu väärästä kysymyksen asettelusta. "

        Niin varmaan johtuukin. Varsin keinotekoisesti on erotettu yksi osa kokemuksesta (aistihavainnot) "aineelliseksi" ja sitten se muu kokemus taas "henkiseksi" tai aineettomaksi.

        Aistikokemuksen säännönmukaista osaa on sitten alettu kutsua "luonnoksi" tai luonnonlakien kokoelmaksi. Yksilön aistihavainto on kuitenkin ihan yhtä subjektiivinen asia kuin esim. tunteet tai ajattelu.


        "En ukso aineettomaan ajatteluun enkä mieleen. Ne ovat varsin suorassa suhteessa fysikaalisiin aivotoimintoihin. Esim. aivovamma voi muuttaa persoonallisuutta. Päihteillä saadaan suoria vaikutuksia mielentilaan. En havaitse aineettomuutta, se on harhainen käsitys."

        Kyseessä on pelkkä korrelaatio. Yhtä hyvin voisi sanoa että aivot ovat se miltä tietoisuus näyttää kolmannen persoonan tai pelkästään aistihavaintojen kannalta. Ns. "aine" on ihan yhtä vaikea täsmällisesti määritellä kuin ns. "tietoisuus" tai "henki".

        Luonnollinen kieli on pääasiassa rakentunut ns. fysikaalisen maailman eli 3. persoonan kokemusta kuvaamaan. Toisen ihmisen 1. persoonan kokemuksista puhutaan usein vertauskuvin ja yhdistetään ne sitten omaan henk. kohtaiseen kokemuksiin niin hyvin kuin löytyy vastaavuuksia.

        Jyrkkä erottelu aineen ja hengen välillä juontaa juurensa Descartesin filosofiaan jossa hän ikäänkuin jakoi reviirit siten että henki kuuluu kirkolle ja aine tieteelle. Kyseessä lienee kuitenkin yksi "substanssi" jossa on sekä aineellisia että psyykkisiä/henkisiä ominaisuuksia. Kokemus sinänsä on aina yhtenäinen ja jakamaton. Käytetty kieli vain sotkee tässä tapauksessa ajattelua.

        On myös tapauksia joissa henkilöllä on hyvin vähän aivokudosta mutta silti älykkyys voi olla keskimääräistä korkeampi eikä ole havaittavaa häiriötä muissakaan toiminnoissa (esim. Lorber).

        Aineen ominaisuuksista (kuten esim. massa. varaus ja paikka ) on äärimmäisen vaikea tai mahdotonta loogisesti johtaa psyyken ominaisuuksia jotka eivät ole selkeästi paikallistettavissa. Kyse ei ole myöskään mistään laskentatehosta vaikka hyvin helposti rinnastetaan kunkin ajan huipputekniikka tietoisuuden toimintaan (esim. aivot tietokoneena).

        Mielestäni aivot ovat lähinnä jonkinlainen käyttöliittymä tai vastaanotin ns. fysikaaliseen tasoon ja itse psyyke taas enemmän jonkinlainen epälokaali korkeamman organisoitumistason ainetta tai energiaa. Aivojen tutkiminen tietoisuuden ominaisuuksien selvittämiseksi vastaa lähinnä tv-vastaanottimen tutkimista kun pitäisi tutkia esim. sitä ihmistä tai ilmiötä joka näkyy siinä tv:ssä.

        Ns. sivupersoonailmiö myös tukee tuota vastaanotin mallia koska siinä kehon ja aivojen käyttöjärjestelmän ottaa valtaansa toinen persoona (vrt. meediot, ns. kanavointi yms. ilmiöt)

        Henkiset toiminnot kuten ajattelu vaatii energiaa ja tarkkaavaisuutta ja ne kummatkin ovat rajoitettuja ihan samalla tavalla kuin mikä tahansa muukin ihmisen toiminta kuten juokseminen tai joku fyysisen työn tekeminen. Se fyysisen työn vaatima energia on kuitenkin erilaatuista kuin se ajatteluun tai opiskeluun tarvittava energia.

        "On myös tapauksia joissa henkilöllä on hyvin vähän aivokudosta mutta silti älykkyys voi olla keskimääräistä korkeampi eikä ole havaittavaa häiriötä muissakaan toiminnoissa"

        Tämä johtuu aivojen mukautuvuudesta, joka on aivan eri asia.

        "Aineen ominaisuuksista (kuten esim. massa. varaus ja paikka ) on äärimmäisen vaikea tai mahdotonta loogisesti johtaa psyyken ominaisuuksia jotka eivät ole selkeästi paikallistettavissa."

        Se on periaatteessakin mahdotonta eikä reduktio päde monessa muussakaan asiassa.

        "Kyse ei ole myöskään mistään laskentatehosta vaikka hyvin helposti rinnastetaan kunkin ajan huipputekniikka tietoisuuden toimintaan (esim. aivot tietokoneena)."

        Periaatteellisesta mahdottomuudestaan huolimatta aivotoimintojen selittämiseksi fysikaalisen maailman prosessien kautta, vertaus aivojen ja tietokoneen (-ohjelman) kesken ei ole virheellinen. Aivotoimintoja voidaan lähestyä informaation käsittelyä koskevien lainalaisuuksien avulla.

        "Mielestäni aivot ovat lähinnä jonkinlainen käyttöliittymä (...) ns. fysikaaliseen tasoon ja itse psyyke taas enemmän jonkinlainen epälokaali korkeamman organisoitumistason ainetta tai energiaa."

        Jotakin tällaista ajattelen itsekin asiasta. Vertaus aivojen toiminnan ja tietokoneohjelman itseohjautuvien prosessien kanssa (tekoäly) on oikeansuuntainen. Ei vain pidä redusoida suoraan mitään prosesseja fysikaaliselle tasolle. Ne ovat informaation käsittelyä.


      • kiikkustuolista
        koittakaakehitellä kirjoitti:

        Itse olen sitä mieltä, että ns. mind-body ongelma johtuu väärästä kysymyksen asettelusta. Ajatellaan juuri tuohon tapaan, että mielen ominaisuus on aineeton ajattelu.

        En ukso aineettomaan ajatteluun enkä mieleen. Ne ovat varsin suorassa suhteessa fysikaalisiin aivotoimintoihin. Esim. aivovamma voi muuttaa persoonallisuutta. Päihteillä saadaan suoria vaikutuksia mielentilaan. En havaitse aineettomuutta, se on harhainen käsitys.

        Aineellinen mieli ei kumoa vapaata tahtoa, vaan sen selityksenä on informaation käsittelykyky, joka aivoilla on elämänkokemuksen myötä.

        Niin mieli, samoin kuin aivotkin koostuvat pelkästään ominaisuuksista. Tarkasteltiin niitä kuinka paljon tahansa, niin mitään muuta niistä ei löydy. Näin ollen kaikki tutkimus on ominaisuuksien tutkimista. Paitsi tietenkin naiivin arkirealismin tasolla.

        Mielen identiteettiteoria epäonnistuu kaikkein olennaisimmassa kysymyksessä. Jos aivot on yhtä kuin mieli, eli A = B ja varsinkin, jos mieli on aineellinen, silloin ajatusten pitäisi löytyä aivoista. En ole kuullut yhdenkään aivokirurgin löytäneen niitä.

        Mielenteoriana funktionalismi syntyi paikkaamaan identiteettiteorian puutteita. Funktionalismi pukee mielen tai tietoisuuden toiminnallisiin kausaalirooleihin. Vaikka funktionalismissa pidetäänkin mieltä aivotoimintojen tuotoksena, niin silti niiden katsotaan poikkeavan toisistaan. Aivot ovat kuitenkin niin monimutkainen elin kaikkine toimintoineen ja aputoimintoineen, että funktionalismikaan ei pysty aukottomasti selittämään mielen ja aivojen suhdetta.


      • kjljklklj
        koittakaakehitellä kirjoitti:

        Itse olen sitä mieltä, että ns. mind-body ongelma johtuu väärästä kysymyksen asettelusta. Ajatellaan juuri tuohon tapaan, että mielen ominaisuus on aineeton ajattelu.

        En ukso aineettomaan ajatteluun enkä mieleen. Ne ovat varsin suorassa suhteessa fysikaalisiin aivotoimintoihin. Esim. aivovamma voi muuttaa persoonallisuutta. Päihteillä saadaan suoria vaikutuksia mielentilaan. En havaitse aineettomuutta, se on harhainen käsitys.

        Aineellinen mieli ei kumoa vapaata tahtoa, vaan sen selityksenä on informaation käsittelykyky, joka aivoilla on elämänkokemuksen myötä.

        Olen samaa mieltä. Mieli, tietoisuus ja kogitio ovat keskushermoston tuottamia tiloja, joita ei ilman kehittynyttä biologista rakennetta ole.


      • SairasKeskushermosto
        Belisario kirjoitti:

        "Itse olen sitä mieltä, että ns. mind-body ongelma johtuu väärästä kysymyksen asettelusta. "

        Niin varmaan johtuukin. Varsin keinotekoisesti on erotettu yksi osa kokemuksesta (aistihavainnot) "aineelliseksi" ja sitten se muu kokemus taas "henkiseksi" tai aineettomaksi.

        Aistikokemuksen säännönmukaista osaa on sitten alettu kutsua "luonnoksi" tai luonnonlakien kokoelmaksi. Yksilön aistihavainto on kuitenkin ihan yhtä subjektiivinen asia kuin esim. tunteet tai ajattelu.


        "En ukso aineettomaan ajatteluun enkä mieleen. Ne ovat varsin suorassa suhteessa fysikaalisiin aivotoimintoihin. Esim. aivovamma voi muuttaa persoonallisuutta. Päihteillä saadaan suoria vaikutuksia mielentilaan. En havaitse aineettomuutta, se on harhainen käsitys."

        Kyseessä on pelkkä korrelaatio. Yhtä hyvin voisi sanoa että aivot ovat se miltä tietoisuus näyttää kolmannen persoonan tai pelkästään aistihavaintojen kannalta. Ns. "aine" on ihan yhtä vaikea täsmällisesti määritellä kuin ns. "tietoisuus" tai "henki".

        Luonnollinen kieli on pääasiassa rakentunut ns. fysikaalisen maailman eli 3. persoonan kokemusta kuvaamaan. Toisen ihmisen 1. persoonan kokemuksista puhutaan usein vertauskuvin ja yhdistetään ne sitten omaan henk. kohtaiseen kokemuksiin niin hyvin kuin löytyy vastaavuuksia.

        Jyrkkä erottelu aineen ja hengen välillä juontaa juurensa Descartesin filosofiaan jossa hän ikäänkuin jakoi reviirit siten että henki kuuluu kirkolle ja aine tieteelle. Kyseessä lienee kuitenkin yksi "substanssi" jossa on sekä aineellisia että psyykkisiä/henkisiä ominaisuuksia. Kokemus sinänsä on aina yhtenäinen ja jakamaton. Käytetty kieli vain sotkee tässä tapauksessa ajattelua.

        On myös tapauksia joissa henkilöllä on hyvin vähän aivokudosta mutta silti älykkyys voi olla keskimääräistä korkeampi eikä ole havaittavaa häiriötä muissakaan toiminnoissa (esim. Lorber).

        Aineen ominaisuuksista (kuten esim. massa. varaus ja paikka ) on äärimmäisen vaikea tai mahdotonta loogisesti johtaa psyyken ominaisuuksia jotka eivät ole selkeästi paikallistettavissa. Kyse ei ole myöskään mistään laskentatehosta vaikka hyvin helposti rinnastetaan kunkin ajan huipputekniikka tietoisuuden toimintaan (esim. aivot tietokoneena).

        Mielestäni aivot ovat lähinnä jonkinlainen käyttöliittymä tai vastaanotin ns. fysikaaliseen tasoon ja itse psyyke taas enemmän jonkinlainen epälokaali korkeamman organisoitumistason ainetta tai energiaa. Aivojen tutkiminen tietoisuuden ominaisuuksien selvittämiseksi vastaa lähinnä tv-vastaanottimen tutkimista kun pitäisi tutkia esim. sitä ihmistä tai ilmiötä joka näkyy siinä tv:ssä.

        Ns. sivupersoonailmiö myös tukee tuota vastaanotin mallia koska siinä kehon ja aivojen käyttöjärjestelmän ottaa valtaansa toinen persoona (vrt. meediot, ns. kanavointi yms. ilmiöt)

        Henkiset toiminnot kuten ajattelu vaatii energiaa ja tarkkaavaisuutta ja ne kummatkin ovat rajoitettuja ihan samalla tavalla kuin mikä tahansa muukin ihmisen toiminta kuten juokseminen tai joku fyysisen työn tekeminen. Se fyysisen työn vaatima energia on kuitenkin erilaatuista kuin se ajatteluun tai opiskeluun tarvittava energia.

        "Ns. sivupersoonailmiö myös tukee tuota vastaanotin mallia koska siinä kehon ja aivojen käyttöjärjestelmän ottaa valtaansa toinen persoona (vrt. meediot, ns. kanavointi yms. ilmiöt) "

        Ei tue. Kyseessä on mielenterveyden häiriö, kuten skitsofreniassakin. Sairaiden aivojen harhaa.


      • Järkisyitä

        "Sekään ei ole mikään ongelma että idealismissa ns. aine on tavallaan illuusio koska aineen näennäinen konkreettisuus katoaa aina hyvin pienessä mittakaavassa."

        Miten idealisti selittää itselleen sen, että hänen on pakko syödä, juoda ja hengittää tai muuten tietoisuus sammuu? Eli miten illuusio kykenee sammuttamaan mielen?


      • Järkisyitä
        kiikkustuolista kirjoitti:

        Niin mieli, samoin kuin aivotkin koostuvat pelkästään ominaisuuksista. Tarkasteltiin niitä kuinka paljon tahansa, niin mitään muuta niistä ei löydy. Näin ollen kaikki tutkimus on ominaisuuksien tutkimista. Paitsi tietenkin naiivin arkirealismin tasolla.

        Mielen identiteettiteoria epäonnistuu kaikkein olennaisimmassa kysymyksessä. Jos aivot on yhtä kuin mieli, eli A = B ja varsinkin, jos mieli on aineellinen, silloin ajatusten pitäisi löytyä aivoista. En ole kuullut yhdenkään aivokirurgin löytäneen niitä.

        Mielenteoriana funktionalismi syntyi paikkaamaan identiteettiteorian puutteita. Funktionalismi pukee mielen tai tietoisuuden toiminnallisiin kausaalirooleihin. Vaikka funktionalismissa pidetäänkin mieltä aivotoimintojen tuotoksena, niin silti niiden katsotaan poikkeavan toisistaan. Aivot ovat kuitenkin niin monimutkainen elin kaikkine toimintoineen ja aputoimintoineen, että funktionalismikaan ei pysty aukottomasti selittämään mielen ja aivojen suhdetta.

        "Mielen identiteettiteoria epäonnistuu kaikkein olennaisimmassa kysymyksessä. Jos aivot on yhtä kuin mieli, eli A = B ja varsinkin, jos mieli on aineellinen, silloin ajatusten pitäisi löytyä aivoista. En ole kuullut yhdenkään aivokirurgin löytäneen niitä."

        Aivojen lukemiseen pitää tarkastella niiden sähköistä toimintaa, jota aivokirurgit eivät tee.

        Aivojen lukeminen onnistuu tänä päivänä fMRI mittauksella. Aivoista on luettu kuvia siitä, mitä ihmiset näkivät tai pelkästään ajattelivat (!). Laite kalibroitiin jokaiselle ihmiselle erikseen, mutta luettu kuva ei ole mistään kuvakirjastosta vaan luettu suoraan aivoista. Kuvan tulkitsemiseen täytyi käyttää syvää koneoppimista, joka etsi korrelaatiot mittauksen ja kuvan välille.

        Tässä mallia, miten hyvin aivoista luettu kuva vastasi ihmisen näkemää.
        https://www.newscientist.com/article/2162862-ai-reconstructs-whatever-you-see-just-by-reading-a-brain-scan/


      • kiikkustuolista
        Järkisyitä kirjoitti:

        "Sekään ei ole mikään ongelma että idealismissa ns. aine on tavallaan illuusio koska aineen näennäinen konkreettisuus katoaa aina hyvin pienessä mittakaavassa."

        Miten idealisti selittää itselleen sen, että hänen on pakko syödä, juoda ja hengittää tai muuten tietoisuus sammuu? Eli miten illuusio kykenee sammuttamaan mielen?

        Idealistit eivät kiellä syömistä, juomista tai vaikkapa naimista, eivätkä ylipäänsä materiaalista maailmaa. He vain pitävät niitä pelkästään mielellisinä toimintoina, mielen luomina illuusioina. Valveilla olon "selvyyden" he selittävät pitkittyneiksi unelmiksi.

        Idealistit ovatkin pelkästään kiinnostuneita mielen sisällöstä, representaatioista ja intentioista. Vaikka en itse ole sen enempää idealisti kuin materialistikaan, niin idealismin etuna on tuoda esille mielen keskeinen asema maailman representoinnissa.


      • kiikkustuolista
        Järkisyitä kirjoitti:

        "Mielen identiteettiteoria epäonnistuu kaikkein olennaisimmassa kysymyksessä. Jos aivot on yhtä kuin mieli, eli A = B ja varsinkin, jos mieli on aineellinen, silloin ajatusten pitäisi löytyä aivoista. En ole kuullut yhdenkään aivokirurgin löytäneen niitä."

        Aivojen lukemiseen pitää tarkastella niiden sähköistä toimintaa, jota aivokirurgit eivät tee.

        Aivojen lukeminen onnistuu tänä päivänä fMRI mittauksella. Aivoista on luettu kuvia siitä, mitä ihmiset näkivät tai pelkästään ajattelivat (!). Laite kalibroitiin jokaiselle ihmiselle erikseen, mutta luettu kuva ei ole mistään kuvakirjastosta vaan luettu suoraan aivoista. Kuvan tulkitsemiseen täytyi käyttää syvää koneoppimista, joka etsi korrelaatiot mittauksen ja kuvan välille.

        Tässä mallia, miten hyvin aivoista luettu kuva vastasi ihmisen näkemää.
        https://www.newscientist.com/article/2162862-ai-reconstructs-whatever-you-see-just-by-reading-a-brain-scan/

        Niin, mitä tuossa on ei-mielellistä toimintaa? Tuo kyseinen tutkimus osoittaa vaan sen, että jossain esiintyy ajattelua ja lukemista. Ei mitään sen enempää. Et kai väitä, että tuohon episodiin ei sisälly mitään mielellistä?


      • Belisario
        kiikkustuolista kirjoitti:

        Idealistit eivät kiellä syömistä, juomista tai vaikkapa naimista, eivätkä ylipäänsä materiaalista maailmaa. He vain pitävät niitä pelkästään mielellisinä toimintoina, mielen luomina illuusioina. Valveilla olon "selvyyden" he selittävät pitkittyneiksi unelmiksi.

        Idealistit ovatkin pelkästään kiinnostuneita mielen sisällöstä, representaatioista ja intentioista. Vaikka en itse ole sen enempää idealisti kuin materialistikaan, niin idealismin etuna on tuoda esille mielen keskeinen asema maailman representoinnissa.

        "Idealistit eivät kiellä syömistä, juomista tai vaikkapa naimista, eivätkä ylipäänsä materiaalista maailmaa."

        Syöminen, juominen ja naiminen ovat kaikki kokemusta ja siinä mielessä juuri sellaisia kuin ne koetaan. Sen sijaan luonnontieteiden käsitys näistä samoista asioista on usein hyvin kaavamainen ja suppea ja siksi lähinnä melko epäolennaista ja siis siinä mielessä illuusiota.

        " He vain pitävät niitä pelkästään mielellisinä toimintoina, mielen luomina illuusioina. Valveilla olon "selvyyden" he selittävät pitkittyneiksi unelmiksi. "

        Ns. "tavallinen valvetila" on sekin unenkaltaista jatkuvan sisäisen monologin tai dialogin takia. Ihminen on hyvin suggestioaltis olio. Sen takia mainonta, propaganda ja "koulutus" (ehdollistaminen) toimii niin tehokkaasti. Yksilö saadaan uskomaan melkein mitä tahansa jos sitä asiaa toistetaan riittävästi. Kyse on oikeastaan sekä hypnoosista että itsehypnoosista jossa dissosioitu "minä" heijastelee lähinnä itseään.

        Todellista valveillaoloa koetaan vain hyvin satunnaisesti ja välähdyksittäin ja usein vain silloin kun tapahtuu jotain mikä järkyttää sekä minäkuvaa että maailmankuvaa.

        ....

        Aivot ovat aika pitkälle evoluution kehittämä suodatusjärjestelmä joka voidaan ohittaa esim. DMT-kemikaalilla ja ns. kuolemanrajakokemuksissa (NDE). Koko kehon käsittävä sähköinen hermojärjestelmä muodostaa eräänlaisen "antennin" tai ambience-aistin jolla koetaan ympäristön ja muiden ihmisten/eläinten tunnelma tai tunnetila.

        Kaukonäkemisessä (jota jotkut tutkijat pitävät kaikkein selkeimpänä psi-kykynä) taas tapahtuu etäaistimista kaikilla kuudella aistilla joka toteutetaan aina vähintään kaksoissokkokokeena.

        Mielenkiintoista kaukonäkemisessä on se että siinä nimenomaan pyritään sulkemaan tietoinen mieli ja päättely kokonaan pois ja keskitytään pelkästään kehon kokemuksiin. Kohde voi olla ihan missä ajassa ja paikassa tahansa. Kaukonäkemiseen liittyy myös kaukovaikuttaminen joka on taas melko kyseenalaista eettiseltä kannalta.

        Tässä pari mielenkiintoista videota jossa U.S.A:n kylmän sodan aikaiseen kaukonäkemisprojektiin osallistunut Lyn Buchanan kertoo kaukonäkemisestä

        https://youtu.be/qRpCysgpEFk

        https://youtu.be/wDO8CdJV_kc

        Kaukonäkeminen sopii aika huonosti vallitsevaan materialismiin mutta sitten kyllä vastaavasti vahvistaa idealistista näkökulmaa. Kyseessä on kuitenkin kyky jonka periaatteessa kuka tahansa voi harjoittelemalla hankkia ja jossa noudatetaan hyvin tarkkaa protokollaa. Useampia kaukonäkijöitä voidaan myös toisistaan riippumatta laittaa tutkimaan samaa kohdetta.

        ....

        Tällaisista asioista kirjoittaminen on tietenkin vähintään majesteettirikos tiedepalstalla joidenkin mielestä mutta puhiskoon he minun puolestani ihan rauhassa kiukkuaan...:)


      • informaatiosubstanssi

        Idealistinen substanssi voi olla myös informaatiota, joka ei ole ainetta tai henkeä.

        Jos oletetaan että aika, paikka, ja kaikki fysikaaliset suureet ovat kvantittuneita ja äärellisen suuruisia, niin riittää melko rajoitettu ideoiden entiteetti. Rajoitettuun ideoiden maailmaan kuuluu esim. numero 3, sekä muitakin lukuja, reaalilukuja ja irrationaalilukuja, mutta ei kuitenkaan matemaattisia operaattoreita, funktioita tai muita liian monimutkaisia ideoita, eikä edes kaikkia lukujakaan tarvita.

        Yksikin aito irrationaaliluku sisältää äärettömän pitkässä numerosarjassa mm. Shakespearen koko tuotannon, Windows 10:n lähdekoodin ja myös kokonaisia diskreettejä universumeja, loputtoman paljon ja mielivaltaisen suuria ja pitkäaikaisia äärellisiä diskreettejä universumeja.


      • Belisario
        informaatiosubstanssi kirjoitti:

        Idealistinen substanssi voi olla myös informaatiota, joka ei ole ainetta tai henkeä.

        Jos oletetaan että aika, paikka, ja kaikki fysikaaliset suureet ovat kvantittuneita ja äärellisen suuruisia, niin riittää melko rajoitettu ideoiden entiteetti. Rajoitettuun ideoiden maailmaan kuuluu esim. numero 3, sekä muitakin lukuja, reaalilukuja ja irrationaalilukuja, mutta ei kuitenkaan matemaattisia operaattoreita, funktioita tai muita liian monimutkaisia ideoita, eikä edes kaikkia lukujakaan tarvita.

        Yksikin aito irrationaaliluku sisältää äärettömän pitkässä numerosarjassa mm. Shakespearen koko tuotannon, Windows 10:n lähdekoodin ja myös kokonaisia diskreettejä universumeja, loputtoman paljon ja mielivaltaisen suuria ja pitkäaikaisia äärellisiä diskreettejä universumeja.

        "Yksikin aito irrationaaliluku sisältää äärettömän pitkässä numerosarjassa mm. Shakespearen koko tuotannon, Windows 10:n lähdekoodin ja myös kokonaisia diskreettejä universumeja, loputtoman paljon ja mielivaltaisen suuria ja pitkäaikaisia äärellisiä diskreettejä universumeja."

        Niinhän sitä väitetään mutta olen aina epäillyt tuota väitettä. Universumien tapauksessa se voi sisältää ainoastaan jonkinlaisen "pysäytyskuvan" mutta se kuva ei sisällä kaikkia niitä prosesseja mitä siinä universumissa tapahtuu ja voi tapahtua.

        Noissa muissakin tapauksissa pitäisi konkreettisesti tarkistaa kuinka pitkä irrationaaliluku tarvitaan että sieltä löytyisi edes tämän oman viestini mittainen numerosarja kirjaimiksi koodattuna yksi yhteen ja peräkkäin. Joitain lyhyitä lauseita voi löytyä mutta yksikin poikkeama siinä merkkijonossa pakottaa aloittamaan taas alusta.

        Numeerinen informaatio ei sellaisenaan muutenkaan tee yhtään mitään eli on täysin passiivinen merkkijono ei vaikuta siten myöskään mihinkään muuhun eikä edes itseensä.

        Simulaatiomallissa maailman "substanssi" voidaan rakentaa informaation pohjalta mutta siinäkin pitää erottaa toisistaan passiivinen data ja sitten toisaalta sitä maailmaa muokkaavat algoritmit (=luonnonlait tai sääntökokoelma). Tom Campbellin simulaatiomalli on juuri tällainen idealismiin ja puhtaasti tietotekniikkaan liittyvän simulaation välimuoto jossa sekä simulaation serveri että pelaajat ovat tietoisia entiteettejä ja kaikki muu taas informaatiota (dataa & algoritmeja).

        Sinänsä pelkästä informaatiosta ei voi mitenkään syntyä tietoisuutta vaan korkeintaan käyttäytymistä joka yrittää feikata tietoisuutta mutta ilman elämyksiä ja aitoja valintoja.

        Tekoäly kehittyneimmilläänkin jää aina jonkinlaisen automaatin tasolle jossa hyvin nopeasti ja mekaanisesti lasketaan loogisten vaihtoehtojen todennäköisyyksiä. Jonkinlainen ihmismielen ja tekoälyn yhdistelmä lienee kuitenkin mahdollinen mutta se ei välttämättä ole kovin miellyttävä tulevaisuuden kuva.

        Mikään algoritmi ei sinänsä voi tuottaa elämyksellistä tietoisuutta eikä vapaata tahtoa ja ne algoritmitkin toimivat parhaiten vain melko säännönmukaisessa ympäristössä jollaiseksi tosin urbaania kaupunkiympäristöä on kehitetty jo muutamia satoja vuosia ja lopputuloksena on melko tylsä(mielinen) ja ruma urbaanibarbaarinen ambience joka jo sellaisenaan aiheuttaa monille mielenhäiriöitä.

        Sekä tiede että älykkyys edellyttävät jonkinlaista vapaata tahtoa ja vapaa tahto taas edellyttää aina kokemuksellista tietoisuutta.


      • informaatiosubstanssi

        "Noissa muissakin tapauksissa pitäisi konkreettisesti tarkistaa kuinka pitkä irrationaaliluku tarvitaan että sieltä löytyisi edes tämän oman viestini mittainen numerosarja kirjaimiksi koodattuna yksi yhteen ja peräkkäin."

        Tuon viestin merkitsemiseen riittää pelkkä kokonaisluku, jonka pituus bitteinä on sama kuin merkkien määrä * 8. Esim. 2048 bittiä riittää 256 merkkiä sisältävän viestin ilmaisemiseen.

        Irrationaaliluku puolestaan ei ole kokonaisluku eikä kokonaislukujen osamäärä, vaan sisältää loputtomasti jaksottomia numeroita desimaalipilkun jälkeen. Sinunkin viestisi löytyy mistä tahansa irrationaaliluvusta tuhottoman monta kertaa. Mistä tahansa irrationaaliluvusta löytyy myös 1:1 kopio Jurassic Word Fallen Kingdom bluray-levystä, sekä loputtomasti diskreettejä universumeita, koska kyseessä on päättymätön numerosarja joka ei ole minkään kahden kokonaisluvun osamäärä.


      • puhtaasti_biologinen

        " pelkästä informaatiosta ei voi mitenkään syntyä tietoisuutta vaan korkeintaan käyttäytymistä joka yrittää feikata tietoisuutta mutta ilman elämyksiä ja aitoja valintoja. "

        Tietoisuus voi syntyä aivoissa puhtaasti biologisista syistä ilman että kvanttimekaniikalla on mitään osuutta asiaan. Eli vaikka perimmäinen "fyysinen" todellisuus on diskreetti, silti aivoissa syntyy biologisena ilmiönä tietoisuus. Tällöin vapaata tahtoa ei ole.


      • Belisario
        informaatiosubstanssi kirjoitti:

        "Noissa muissakin tapauksissa pitäisi konkreettisesti tarkistaa kuinka pitkä irrationaaliluku tarvitaan että sieltä löytyisi edes tämän oman viestini mittainen numerosarja kirjaimiksi koodattuna yksi yhteen ja peräkkäin."

        Tuon viestin merkitsemiseen riittää pelkkä kokonaisluku, jonka pituus bitteinä on sama kuin merkkien määrä * 8. Esim. 2048 bittiä riittää 256 merkkiä sisältävän viestin ilmaisemiseen.

        Irrationaaliluku puolestaan ei ole kokonaisluku eikä kokonaislukujen osamäärä, vaan sisältää loputtomasti jaksottomia numeroita desimaalipilkun jälkeen. Sinunkin viestisi löytyy mistä tahansa irrationaaliluvusta tuhottoman monta kertaa. Mistä tahansa irrationaaliluvusta löytyy myös 1:1 kopio Jurassic Word Fallen Kingdom bluray-levystä, sekä loputtomasti diskreettejä universumeita, koska kyseessä on päättymätön numerosarja joka ei ole minkään kahden kokonaisluvun osamäärä.

        " Sinunkin viestisi löytyy mistä tahansa irrationaaliluvusta tuhottoman monta kertaa. "

        Mietipä vähän tarkemmin asiaa. Missään luonnollisessa kielessä eivät kaikki kirjaimet eikä varsinkaan sanat esiinny yhtä suurella todennäköisyydellä. Irrationaaliluvusta voi poimia vaikka 2 numeroa kerrallaan vastaamaan 8-bittistä ASCII koodia ja sillä perusteella kääntää se irrationaaliluku alfanumeeriseen luonnollisen kielen muotoon.

        "Mistä tahansa irrationaaliluvusta löytyy myös 1:1 kopio Jurassic Word Fallen Kingdom bluray-levystä, sekä loputtomasti diskreettejä universumeita, koska kyseessä on päättymätön numerosarja joka ei ole minkään kahden kokonaisluvun osamäärä."

        Voisit vähän yrittää perustella eikä vaan ilmaista uskomustasi. Mikään matemaattinen todistus ei tässä riitä eikä sekään että irrationaaliluvut ovat päättymättömiä eli äärettömän pitkiä.

        Jos väitteesi pitäisi paikkansa niin koko tieteen resurssit voisi sitten siirtää tutkimaan mitä tahansa irrationaalilukua koska sieltähän sinun mukaasi löytyy kaikki tarvittava informaatio.

        Jossain SETI-projektissa hypättäisiin kattoon riemusta jos jostain kaukaisesta galaksista tulisi palautteena esim. 1960-luvun uutislähetys kommenteilla varustettuna tai joku Shakespearen kirjallinen tuotos.

        Tuo väite on yksi kaikkein ääliömäisimmistä väitteistä mihin olen koskaan törmännyt tieteen ja matematiikan puitteissa.

        ......

        "Tietoisuus voi syntyä aivoissa puhtaasti biologisista syistä ilman että kvanttimekaniikalla on mitään osuutta asiaan. "

        Mikä on "puhtaasti biologinen syy"? Kvanttimekaniikasta en kirjoittanut mitään edellä vaikka tuskin pelkkä klassinen mekaniikka riittää selittämään edes aivojen toimintaa. Taas pelkkä uskomuksen ilmaisu ilman perusteluita.

        "Eli vaikka perimmäinen "fyysinen" todellisuus on diskreetti, silti aivoissa syntyy biologisena ilmiönä tietoisuus. Tällöin vapaata tahtoa ei ole."

        Pelkkä uskon ilmaus tuokin on ja taaskin täysin ilman perusteluita tuokin. Luettelet pelkästään nykytieteen dogmeja jotka ovat juurin niin avuttomia kuin ovat koska materialismin ideologian/uskonnon pohjalta ei oikein muuhun pystytä.

        Tietoisuus ei ylipäätänsä voi olla koskaan tieteen eikä minkään muunkaan tutkimuksen objekti koska tietoisuus on aina sen tutkimuksen subjekti. Jos pystyt kuvailemaan jonkun asian niin ei se ei ole tietoisuutta. On olemassa vain yksi tietoisuus. Tietoisuudella ei ole monikkoa.


      • Järkisyitä
        kiikkustuolista kirjoitti:

        Niin, mitä tuossa on ei-mielellistä toimintaa? Tuo kyseinen tutkimus osoittaa vaan sen, että jossain esiintyy ajattelua ja lukemista. Ei mitään sen enempää. Et kai väitä, että tuohon episodiin ei sisälly mitään mielellistä?

        Tokihan me kaikki koemme maailmaa aina oman tietoisuutemme kautta. Mutta jos realimaailma osoittautuu sellaiseksi, että jopa ajatuksia voidaan lukea, niin se antaa vahvaa viitettä, ettei kyseessä ole pelkkä kuvitteellinen virtuaalimaailma vaan maailma, jossa ajatuksemmekin näkyvät mitattavana asiana.

        Toki jääräpäinen idealisti voi tulkita kaiken illuusioksi ja jonkin syvemmän alitajunnan tuotoksiksi. Jos hän kuitenkin hyväksyy, että on olemassa muitakin tietoisuuksia kuin hän itse, niin muiden tietoisuuksiin vaikuttaminen realimaailman kautta esimerkiksi lääkityksellä tuntuu vaikealta asialta selittää idealistille.


      • kiikkustuolista

        Pitää erottaa toisistaan naiivi arkirealismi ja tieteellinen realismi. Varmuus toisten persoonien ajatuksista on mittausmenetelmistä ja tulkinnoista riippumatta epäsuoraa. Näin ollen, koska emme pysty toteamaan toisten ajatuksia suoraan heidän ajatustensa kautta, emme voi olla niistä myöskään täysin varmoja. Mikäli jääräpäinen materialisti kieltää tuon, niin hän on itse tietämättään astunut idealismin puolelle.


    • Olisiko se idealismia, jos todellisuus = todennäköisyysjakauma?

    • Idealismiakin on tietysti monen sorttista.

      Minä olen filosofiassani yhdistänyt materialismin ja idealismin sillä tavalla, että koko universumi on ymmärrettävissä täysin materialistisesti, se ei ole syntynyt eikä luotu, vaan se on aina ollut ja on sellainen kuin on, oli sitten millainen vaan. Sensijaan maapallon nykyinen järjestys on kokonaan Jumalan luoma tilanne ja näkymä ihmisen hengellisen kehityksen tarpeisiin. Emme näe henkimaailmaa emmekä Jumalan maailmoja, emmekä tiedä mitä kaikkea noissa galakseissa ja muiden tähtien planeetoilla on.

      Ja kaikilla todellisuuden tasoilla on henki ja aine yhtä aikaa, riippuu näkökulmasta kumpi on ensisijainen. Aine ja energia, solut ja henki, ihmisen ruumis ja henki, täydellisen olennon ruumis ja henki. Täydellinen olento on jo ikuisuus sitten siirtynyt luomaan ainetta, elämää ja ihmisiä, tekee sitä nyt ja tekee ikuisesti. (Profiilista löytyy kotisivuni, joilla asia on tarkemmin aspekteineen filosofian kirjoissani).

      • ReeÄnnAaa

        Lyhyestä virsi kaunis: Me emme vain tiedä tarpeeksi.


      • kjlkjljlj

        "Profiilista löytyy kotisivuni, joilla asia on tarkemmin aspekteineen filosofian kirjoissani"

        Kutsutko uskonnollista löpinää filosofiaksi?


      • kjlkjljlj kirjoitti:

        "Profiilista löytyy kotisivuni, joilla asia on tarkemmin aspekteineen filosofian kirjoissani"

        Kutsutko uskonnollista löpinää filosofiaksi?

        Uskonto kuuluu filosofiaan, toiset filosofit uskoo, toiset ei. Löpinä on ilmeisesti sinun asiantunteva arviosi mielestäsi, mutta kuullostaa lähinnä perustelemattomalta propgandalta.


      • Järkisyitä

        Olet siis dualisti.

        Mutta pidätkö täysin materialistista maailmaa mahdollisena? Eli minkä ilmiön selittämiseksi tarvitset "henkeä"? Jos henkeä ei olisikaan, niin mikä muuttuisi?


      • Filosofiaako
        Olli.S kirjoitti:

        Uskonto kuuluu filosofiaan, toiset filosofit uskoo, toiset ei. Löpinä on ilmeisesti sinun asiantunteva arviosi mielestäsi, mutta kuullostaa lähinnä perustelemattomalta propgandalta.

        En tiedä ainuttakaan vähääkään merkittävää filosofia, joka olisi tunnustautunut uskovaksi. Filosofit taitavat olla jopa poikkeuksellisen ateistisia.

        Missään tapauksessa kirjoittelusi henkimaailmasta ja koetuspaikoista ei ole filosofiaa. Se on pelkkää uskonnollista jaarittelua. Filosofeilla ei ole pyhiä kirjoituksia.


      • Järkisyitä kirjoitti:

        Olet siis dualisti.

        Mutta pidätkö täysin materialistista maailmaa mahdollisena? Eli minkä ilmiön selittämiseksi tarvitset "henkeä"? Jos henkeä ei olisikaan, niin mikä muuttuisi?

        Se olisi fysikalismia ja materialismia, Jumalan henki poistuisi, henkimaailma poistuisi, tietoisuus poistuisi, sielu, psyyke, minä, persoonallisuus poistuisi, elämä poistuisi, energia poistuisi. Henki ja aine ovat aina yhtä aikaa, se on ykseyttä, dualismia ja monismia yhtä aikaa. Ja systeemin rakentamista, kokonaisuuden ymmärtämistä, yksityiskohtien tarkastelua, synteesiä ja analyysiä vuorotellen, ei yksinomaisesti. Samoin rationaalista ja empiiristä tietoa vuorotellen.

        Eikä korkeamman tason ilmiöitä voida kokonaan palauttaa alemman tason ilmiöihin, elämää aineeseen, ajattelua aivoihin jne.

        Realisti kai lähinnä olen. Tai monistinen dialektinen materialisti. Tai dualistinen dialektinen materialisti.


      • Järkisyitä
        Olli.S kirjoitti:

        Se olisi fysikalismia ja materialismia, Jumalan henki poistuisi, henkimaailma poistuisi, tietoisuus poistuisi, sielu, psyyke, minä, persoonallisuus poistuisi, elämä poistuisi, energia poistuisi. Henki ja aine ovat aina yhtä aikaa, se on ykseyttä, dualismia ja monismia yhtä aikaa. Ja systeemin rakentamista, kokonaisuuden ymmärtämistä, yksityiskohtien tarkastelua, synteesiä ja analyysiä vuorotellen, ei yksinomaisesti. Samoin rationaalista ja empiiristä tietoa vuorotellen.

        Eikä korkeamman tason ilmiöitä voida kokonaan palauttaa alemman tason ilmiöihin, elämää aineeseen, ajattelua aivoihin jne.

        Realisti kai lähinnä olen. Tai monistinen dialektinen materialisti. Tai dualistinen dialektinen materialisti.

        ”Eikä korkeamman tason ilmiöitä voida kokonaan palauttaa alemman tason ilmiöihin, elämää aineeseen, ajattelua aivoihin jne.”

        Tuosta olen materialistina luonnollisesti eri mieltä. Näen, että oppivat neuroverkot etenevät koko ajan kohti monimutkaisempaa todellisuuden mallintamista ja jossain vaiheessa syntyy kyky abstrsktiin ajatteluun ja ymmärrys itsensä olemassa oloon. Kun tälläinen neuroverkko alkaa mallintaa ja miettiä omaa käytöstään maailmassa, niin jossain vaiheessa tästä kohoaa se, mitä kutsumme tietoisuudeksi.

        Toki myönnän, että tämä on vielä uskomus, koska tietoisuuden mekanismia ei tunneta.


      • Järkisyitä kirjoitti:

        ”Eikä korkeamman tason ilmiöitä voida kokonaan palauttaa alemman tason ilmiöihin, elämää aineeseen, ajattelua aivoihin jne.”

        Tuosta olen materialistina luonnollisesti eri mieltä. Näen, että oppivat neuroverkot etenevät koko ajan kohti monimutkaisempaa todellisuuden mallintamista ja jossain vaiheessa syntyy kyky abstrsktiin ajatteluun ja ymmärrys itsensä olemassa oloon. Kun tälläinen neuroverkko alkaa mallintaa ja miettiä omaa käytöstään maailmassa, niin jossain vaiheessa tästä kohoaa se, mitä kutsumme tietoisuudeksi.

        Toki myönnän, että tämä on vielä uskomus, koska tietoisuuden mekanismia ei tunneta.

        Tuo on fysikalismia, ei edes materialismia, se on taakse jäänyt kanta filosofiassa.


      • kjlkjlkjlkjkl
        Olli.S kirjoitti:

        Tuo on fysikalismia, ei edes materialismia, se on taakse jäänyt kanta filosofiassa.

        "se on taakse jäänyt kanta filosofiassa"

        ...kuten jumalat ja henkiolennotkin.


      • kjlkjlkjlkjkl kirjoitti:

        "se on taakse jäänyt kanta filosofiassa"

        ...kuten jumalat ja henkiolennotkin.

        Ateistisessa filosofiassa, joka on kaukana totuudesta.


      • Järkisyitä
        Olli.S kirjoitti:

        Tuo on fysikalismia, ei edes materialismia, se on taakse jäänyt kanta filosofiassa.

        ”Tuo on fysikalismia, ei edes materialismia, se on taakse jäänyt kanta filosofiassa.”

        Eikö fysikalismi ja materialismi ole aika pitkälti sama asia? Fysikalismi on ymmärtääkseni materialismin vastaus henki-ruumisongelmaan.

        Materialismi on nykyisin hallitseva luonnontieteen metafysiikka. Sinänsä harmi, jos filosofia ei tosiaankaan ole kiinnostunut neurofysiikasta ja keinoälystä. Tietoisuuden materialistinen pohtiminen voi muutaman vuosikymmenien jälkeen olla ajankohtaisempaa kuin koskaan! Vasta nyt olemme ensimmäistä kertaa rakentaneet koneita, jotka oppivat itsenäisesti suorittamaan tehtäviä, joita koneen suunnittelijakaan ei osaisi ratkaista.


    • No tietysti todellisuus on tietoisuudessa, emmehän me muuten voisi olla tietoisia siitä. Että olisi muka joku erillinen maailma tietoisuuden "ulkopuolella" on täysin uskonvarainen käsite jota on mahdoton todistaa, koska tietoisuus on ainoa tapa millä voimme kokea mitään.

      Tunnelma ulkopuolisesta maailmasta syntyy siitä että meillä on keho, ja me voimme nähdä ja tuntea että kehon ulkopuolella on maailma joka on erillinen meidän kehosta. Mutta maailma ei ole erillinen tietoisuudesta, koska muuten me emme voisi kokea sitä.

    • kiikkustuolista

      Mielen tutkiminen on sikäli vaikeaa, että itsetajunnassamme emme näytä pääsevän yhteyteen itsemme kanssa, vaan pelkästään määrättyihin elämyksiin ja vaikutelmiin. Voidaksemme ymmärtää mieltä "mielenä", sen täytyisi olla omien rakenteidensa ehdollistamaa. Tämä voisi toteutua sitä kautta, että ymmärryksen kohteena oleva mieli toimisi ymmärryksen alustana. Tuona alustana voisi toimia esirefleksinen tietoisuus, mikä tarkoittaisi tietoisuutta siitä, että ollaan tietoisia, joka puolestaan merkitsisi tietoisuuden tietoisuutta omasta itsestään.


      Esirefleksiivisellä tietoisuudella näyttäisi olevan myös se etu, että se voisi estää äärettömän regression syntymisen, mikä näissäkin kirjoituksissa näyttää vallitsevan. Nimittäin, kun yritämme väkipakolla tavoittaa mielessämme olevaa "mieltä" itsereflektiolla suuntaamalla tietoisuutemme siihen itseensä, niin mieli objektivoituu, muuttuen näin "toiseksi" ja sitä kautta ikään kuin vieraantuu meistä.


      Tuo edellä esitetty pohdinta perustuu tietoisuuden ontologiseen määrittelyyn, jonka mukaan tietoisuus on tietoisuutta jostakin. Tietoisuuteen kuuluu siten transendenssi, eli tietoisuus syntyy sellaisen olion tukemana, mikä ei ole se itse. Mielen toiminnot ovat tapoja suhtautua todellisuuteen suhdeverkostoa hyväksi käyttäen. Siten viittaus mielen ulkopuoliseen todellisuuteen on kaikissa tapauksissa välttämätön riippumatta siitä, kuinka konkreettinen tuo todellisuus on.

    • Belisario

      "Tuona alustana voisi toimia esirefleksinen tietoisuus, mikä tarkoittaisi tietoisuutta siitä, että ollaan tietoisia, joka puolestaan merkitsisi tietoisuuden tietoisuutta omasta itsestään."

      Gurdjieff kutsui tuota itsemuistannaksi (self-remembering) jossa yhtä aikaa ollaan tietoisia sekä tarkkavaisuuden kohteestaettä siitä miten se kohde vaikuttaa itseen. Tavalllisesti ikäänkuin uppoudutaan siihen kohteeseen ja reagoidaan tiedottomasti. Itsemuistanta edellyttää korkeamman tasoista "energiaa" kuin pelkkä tarkkaavainen havaitseminen.


      "Siten viittaus mielen ulkopuoliseen todellisuuteen on kaikissa tapauksissa välttämätön riippumatta siitä, kuinka konkreettinen tuo todellisuus on."

      Kannattaa erottaa toisistaan tietoisuus ja sitten ne tietoisuuden sisällöt jotka nekin ovat tietoisuudessa mutta eivät ole se itse tietoisuus. Tietoisuus on ikään kuin sipuli jossa on useampia kerroksia päällekäin ja sitä sisintä kerrosta ei ole siinä mielessä että sitä voisi jollain ominaisuudella kuvata.

      .....

      Vuosittaisesta 'The Science of Consciousness' konferenssista on taas julkaistu luentoja Youtubessa. Tämä tapahtuma järjestettiin muutama vuosi sitten myös Helsingissä.

      Plenary 11 Idealism and Panpsychism:

      https://www.youtube.com/watch?v=2P6xgydJrO0

      David Chalmers tuon videon alussa esittää että tietoisuuden ymmärtämisen alkeellisin taso on materialismi josta sitten siirrytään panpsykismiin tai protopanpsykismiin ja lopulta idealismiin.

      Esim. toistaiseksi ainoa matematisoitu tietoisuuden teoria IIT (integroidun informaation teoria) on hyvin lähellä jonkinlaista panpsykismiä.

      • planeetta_ei.kirj

        Hmm. Kovasti kiinnostaa kyllä mitä Chalmers ja seuraajat saavat lisättyä olemassaolevaan ymmärrykseen. Ylipäätään ongelmallista lienee, jos koettu poikkeaa kovastikin fyysisestä todellisuudesta, joka taas joillekin voi olla hyvinkin sama, tai jos joidenkin reaalimaailma käsittää asioita laajemmin, kuin valtaosin muiden. Omaa kokemusta tai joukkokokemusta kun harvemmin tupataan sisällyttämään varsinaiseen tieteeseen, jos se vaatii tietynlaisen kokijan tai kokijajoukon.


      • Belisario
        planeetta_ei.kirj kirjoitti:

        Hmm. Kovasti kiinnostaa kyllä mitä Chalmers ja seuraajat saavat lisättyä olemassaolevaan ymmärrykseen. Ylipäätään ongelmallista lienee, jos koettu poikkeaa kovastikin fyysisestä todellisuudesta, joka taas joillekin voi olla hyvinkin sama, tai jos joidenkin reaalimaailma käsittää asioita laajemmin, kuin valtaosin muiden. Omaa kokemusta tai joukkokokemusta kun harvemmin tupataan sisällyttämään varsinaiseen tieteeseen, jos se vaatii tietynlaisen kokijan tai kokijajoukon.

        " Ylipäätään ongelmallista lienee, jos koettu poikkeaa kovastikin fyysisestä todellisuudesta, joka taas joillekin voi olla hyvinkin sama..."

        Oletko varma? Ylipäätänsä toisten kokemuksen samankaltaisuuden tai erilaisuuden voi päätellä vain niiden muiden kokemusten ja oman kokemuksen keskinäisestä rakenneyhtäläisyydestä ja tietenkin siitä että sitä todellisuutta kuvaava kieli on sama.

        Et voi kuitenkaan tietää onko esim. minun kokemani punainen sama kuin sinun kokemasi punainen. Tiede perustuu hyvin pitkälle pelkästään ihmisen lajityypilliselle näköaistille eli esim. jos lepakoila olisi tiedettä niin niiden tiede olisi varmaan aika erilaista kuin meidän.

        Sekin on melko ongelmallista miten määritellään "fysikaalinen todellisuus" . Mukana on aina suuri joukko oletuksia ja tulkintoja joita ei voi edes kokeellisesti eli tieteellisellä metodilla ainakaan helposti vahvistaa.

        Suurimman osan maailmankuva on jonkinlaista 1800-luvun junttimaterialismia jossa kaikki halutaan ymmärtää toisistaan erillisten kiinteiden kappaleiden kautta (kokonaisluvut, hiukkaset) ja vain asioiden määrällisten suhteiden kautta.

        "Omaa kokemusta tai joukkokokemusta kun harvemmin tupataan sisällyttämään varsinaiseen tieteeseen, jos se vaatii tietynlaisen kokijan tai kokijajoukon."

        Tiedehän nimenomaan vaatii tietynlaisen kokijan ja kokijajoukon - nimittäin länsimaisen maailmankuvan omaavan henkilön jollaisia kasvatetaan ja koulutetaan/ehdollistetaan koko ajan lisää.

        Olen muutenkin taipumassa sellaiselle kannalle että tiede ei oikeasti löydä mitään vaan pääsääntöisesti tuottaa koko ajan uutta todellisuutta oletustensa ja uskomustenssa pohjalta (esim. hiukkaskiihdyttimet ja kaikki tiede jossa mennään teoria edellä)

        Periaatteessa melkein mikä tahansa toimii kun se on vain riittävän koherenttia sisäisesti ja suhteessa muuhun ns. todellisuuteen. Tiede ja tekniikka muuttaa hyvinkin voimakkaasti ympäristöä ja kokemusmaailmaa eikä läheskään aina positiivisessa mielessä vaan tuottamalla umpikujia ja uusia ongelmia ihmislajille.

        Tavallaan voisi sanoa että ihmiskunta tieteensä ja tekniikkansa kanssa on kuin kissa joka jahtaa omaa häntäänsä.... ;-)


      • kiikkustuolista

        Panpsykismi on tavallaan paluuta aristoteliseen metafysiikkaan ja sen sisältämään terminaalikausaliteettiin. Kyse on eräästä monismin periaatteesta. Koska aineen ja tietoisuuden välinen yhteys on ongelmallinen, niin siinä aineen oletetaan olevan jo lähtökohdiltaan tietoista. Ajatellaan, että ilman tuota jo alkujaankin aineeseen sisältyvää tietoisuutta, tietoisen elämän syntyminenkin olisi mahdotonta. Puolestaan aine, nykyajan panpsykismissä, on määritelty uudella tavalla kvanttifysiikkaa hyväksi käyttäen.

        Kyseisessä mallissa elämismaailman oletetaan koostuvan pelkästään muodoista, piirteistä ja erilaisista olemisen tavoista. Nämä olisivat siten pelkkiä ilmennyksiä substanssin ollessa määrätyssä aikaan ja avaruuteen rajoittuvassa mielen tilassa. Tällöin meidänkin olemisemme olisi eräänlainen tihentymä tajunnan virran aika-avaruus-jatkumossa. Siten esimerkiksi liikahduksissamme ja ylipäänsä muutoksissa tajunnan tilan peräkkäiset alueet tihentyisivät ajan yli.

        Puolestaan muutoksia voisi verrata vaikkapa meren aaltoihin tai johonkin kohteeseen tv-ruudulla. Mikäli objektien liike halki tila-avaruuden olisi todella analoginen tv-ruudun poikki kulkevalle liikekuvalle, silloin liikettä voitaisiin pitää pelkkänä illuusiona. Tällöin tavallisetkin arkielämän objektit eivät olisi sen todellisempia kuin aallot ovat objekteja valtamerellä tai, että ne olisivat sen enempää objekteja kuin substanssejakaan, vaan erilaisia tapoja olla, joilla oleminen pyrkisi ilmenemään ja organisoitumaan tajunnan virrassa. Kvanttifysiikkaan liitettynä kyse voisi olla eräänlaisesta "häiriöstä" kvanttikentässä.


      • kiikkustuolista kirjoitti:

        Panpsykismi on tavallaan paluuta aristoteliseen metafysiikkaan ja sen sisältämään terminaalikausaliteettiin. Kyse on eräästä monismin periaatteesta. Koska aineen ja tietoisuuden välinen yhteys on ongelmallinen, niin siinä aineen oletetaan olevan jo lähtökohdiltaan tietoista. Ajatellaan, että ilman tuota jo alkujaankin aineeseen sisältyvää tietoisuutta, tietoisen elämän syntyminenkin olisi mahdotonta. Puolestaan aine, nykyajan panpsykismissä, on määritelty uudella tavalla kvanttifysiikkaa hyväksi käyttäen.

        Kyseisessä mallissa elämismaailman oletetaan koostuvan pelkästään muodoista, piirteistä ja erilaisista olemisen tavoista. Nämä olisivat siten pelkkiä ilmennyksiä substanssin ollessa määrätyssä aikaan ja avaruuteen rajoittuvassa mielen tilassa. Tällöin meidänkin olemisemme olisi eräänlainen tihentymä tajunnan virran aika-avaruus-jatkumossa. Siten esimerkiksi liikahduksissamme ja ylipäänsä muutoksissa tajunnan tilan peräkkäiset alueet tihentyisivät ajan yli.

        Puolestaan muutoksia voisi verrata vaikkapa meren aaltoihin tai johonkin kohteeseen tv-ruudulla. Mikäli objektien liike halki tila-avaruuden olisi todella analoginen tv-ruudun poikki kulkevalle liikekuvalle, silloin liikettä voitaisiin pitää pelkkänä illuusiona. Tällöin tavallisetkin arkielämän objektit eivät olisi sen todellisempia kuin aallot ovat objekteja valtamerellä tai, että ne olisivat sen enempää objekteja kuin substanssejakaan, vaan erilaisia tapoja olla, joilla oleminen pyrkisi ilmenemään ja organisoitumaan tajunnan virrassa. Kvanttifysiikkaan liitettynä kyse voisi olla eräänlaisesta "häiriöstä" kvanttikentässä.

        Olisi siis tietoista happea ja tietoista vetyä ja kun ne yhdistyvät syntyy tietoista vettä?


      • Järkisyitä
        kiikkustuolista kirjoitti:

        Panpsykismi on tavallaan paluuta aristoteliseen metafysiikkaan ja sen sisältämään terminaalikausaliteettiin. Kyse on eräästä monismin periaatteesta. Koska aineen ja tietoisuuden välinen yhteys on ongelmallinen, niin siinä aineen oletetaan olevan jo lähtökohdiltaan tietoista. Ajatellaan, että ilman tuota jo alkujaankin aineeseen sisältyvää tietoisuutta, tietoisen elämän syntyminenkin olisi mahdotonta. Puolestaan aine, nykyajan panpsykismissä, on määritelty uudella tavalla kvanttifysiikkaa hyväksi käyttäen.

        Kyseisessä mallissa elämismaailman oletetaan koostuvan pelkästään muodoista, piirteistä ja erilaisista olemisen tavoista. Nämä olisivat siten pelkkiä ilmennyksiä substanssin ollessa määrätyssä aikaan ja avaruuteen rajoittuvassa mielen tilassa. Tällöin meidänkin olemisemme olisi eräänlainen tihentymä tajunnan virran aika-avaruus-jatkumossa. Siten esimerkiksi liikahduksissamme ja ylipäänsä muutoksissa tajunnan tilan peräkkäiset alueet tihentyisivät ajan yli.

        Puolestaan muutoksia voisi verrata vaikkapa meren aaltoihin tai johonkin kohteeseen tv-ruudulla. Mikäli objektien liike halki tila-avaruuden olisi todella analoginen tv-ruudun poikki kulkevalle liikekuvalle, silloin liikettä voitaisiin pitää pelkkänä illuusiona. Tällöin tavallisetkin arkielämän objektit eivät olisi sen todellisempia kuin aallot ovat objekteja valtamerellä tai, että ne olisivat sen enempää objekteja kuin substanssejakaan, vaan erilaisia tapoja olla, joilla oleminen pyrkisi ilmenemään ja organisoitumaan tajunnan virrassa. Kvanttifysiikkaan liitettynä kyse voisi olla eräänlaisesta "häiriöstä" kvanttikentässä.

        Ei nyt ihan auennut, miten klassinen panpsykismi saadaan yhdistettyä kvanttikenttäteorian kanssa. Eli keskimmäinen kappale meni minulle sanojen sillisalaatiksi, mikä voi hyvinkin olla minun ymmärtämyyttäni.

        Se, että kaikella materialla on loppujen lopuksi aaltoluonne mikromaailmassa ei mitenkään tee siitä vähemmän konkreettista tai illuusiota. Selvästikin aallot ovat olemassa. Aaltoilu on materian (eivät hengen tai mielen) ominaisuus, joten en ymmärrä, miten ne liitetään idealismiin.


      • kiikkustuolista
        inti kirjoitti:

        Olisi siis tietoista happea ja tietoista vetyä ja kun ne yhdistyvät syntyy tietoista vettä?

        Idealismi on kieltämättä outo oppisuuntaus. Se on hyvin kaukana naiivista arkiajattelusta, jonka mukaan vesi on vain vettä ja sillä siisti. Idealismissa mennään syvälle mielen rakenteisiin ja vastaavasti nykyajan fysikalismissa syvälle aineen rakenteisiin. Siellä ne puolestaan lähestyvät toisiaan.


      • kiikkustuolista
        Järkisyitä kirjoitti:

        Ei nyt ihan auennut, miten klassinen panpsykismi saadaan yhdistettyä kvanttikenttäteorian kanssa. Eli keskimmäinen kappale meni minulle sanojen sillisalaatiksi, mikä voi hyvinkin olla minun ymmärtämyyttäni.

        Se, että kaikella materialla on loppujen lopuksi aaltoluonne mikromaailmassa ei mitenkään tee siitä vähemmän konkreettista tai illuusiota. Selvästikin aallot ovat olemassa. Aaltoilu on materian (eivät hengen tai mielen) ominaisuus, joten en ymmärrä, miten ne liitetään idealismiin.

        Ei tietenkään materia todellakaan materialistille mikään illuusio ole. Mutta kun kaikki eivät ole materialisteja.

        Silloin kun objektit koostuvat magneettikentistä, pisteistä ja kokonaisista tila-avaruuden ulottuvuuksista, liike voi olla pelkästään näennäistä, peräkkäisten ulottuvuuksien ottaessa tai menettäessä ominaisuuksia jollain erityisellä tavalla. Tv: n taustasäteily tai kuvaruudulla kiertävä valomainos voi hyvin kuvata tuota ilmiötä, mikä voi hyvinkin olla yleistettävissä.

        Ei liene vaikeaa kuvitella elollisen olennon olevan vain hetkellinen tihentymä ajassa ja tilassa. Voiko siinä näin ollen olla kiinteää ja pysyvää olemusta. Mikäli asia on näin, kyse on pikemmin muodoista, piirteistä ja olemisen tavasta. Puolestaan aika ja avaruus ovat pelkkiä mielteitä.


        On turha toistaa, miten täällä on idealismista kerrottu. Metafysiikasta ja kvanttiteoriasta lisää alan kirjallisuudesta.


      • TerminologianVääristely
        kiikkustuolista kirjoitti:

        Ei tietenkään materia todellakaan materialistille mikään illuusio ole. Mutta kun kaikki eivät ole materialisteja.

        Silloin kun objektit koostuvat magneettikentistä, pisteistä ja kokonaisista tila-avaruuden ulottuvuuksista, liike voi olla pelkästään näennäistä, peräkkäisten ulottuvuuksien ottaessa tai menettäessä ominaisuuksia jollain erityisellä tavalla. Tv: n taustasäteily tai kuvaruudulla kiertävä valomainos voi hyvin kuvata tuota ilmiötä, mikä voi hyvinkin olla yleistettävissä.

        Ei liene vaikeaa kuvitella elollisen olennon olevan vain hetkellinen tihentymä ajassa ja tilassa. Voiko siinä näin ollen olla kiinteää ja pysyvää olemusta. Mikäli asia on näin, kyse on pikemmin muodoista, piirteistä ja olemisen tavasta. Puolestaan aika ja avaruus ovat pelkkiä mielteitä.


        On turha toistaa, miten täällä on idealismista kerrottu. Metafysiikasta ja kvanttiteoriasta lisää alan kirjallisuudesta.

        Fysiikan kvanttiteorialla ei taida olla juurikaan tekemistä kuvailemasi idealismin kanssa. Toki huhaahoitajatkin antavat kvanttikosketushoitoa, mutta fysiikaalisessa mielessä termillä ei ole tuossa kontekstissa mitään järkevää merkitystä.


      • Belisario
        Järkisyitä kirjoitti:

        Ei nyt ihan auennut, miten klassinen panpsykismi saadaan yhdistettyä kvanttikenttäteorian kanssa. Eli keskimmäinen kappale meni minulle sanojen sillisalaatiksi, mikä voi hyvinkin olla minun ymmärtämyyttäni.

        Se, että kaikella materialla on loppujen lopuksi aaltoluonne mikromaailmassa ei mitenkään tee siitä vähemmän konkreettista tai illuusiota. Selvästikin aallot ovat olemassa. Aaltoilu on materian (eivät hengen tai mielen) ominaisuus, joten en ymmärrä, miten ne liitetään idealismiin.

        "Ei nyt ihan auennut, miten klassinen panpsykismi saadaan yhdistettyä kvanttikenttäteorian kanssa. "

        Kvanttikenttäteoria on matemaattinen teoria jossa ei oteta kantaa ontologiaan eli siihen onko se tutkittava asia materiaalinen vai ei-materiaalinen. Ontologian kannalta kvanttimekaniikassa ei vallitse juuri minkäänlaista yksimielisyyttä vaan niitä ontologisia tulkintamalleja on lukuisia.

        https://en.wikipedia.org/wiki/Interpretations_of_quantum_mechanics

        Tuossa wikiartikikkelissa löytyy mielenkiintoinen taulukko jossa näkyy miten kukin tulkinta hahmottaa esim. havaitsijan roolin ja sen onko universaalinen aaltofunktio on todellinen asia vai pelkkä matemaattinen apuväline.

        Kööpenhaminaisessa tulkinnassa havaitsijalla on kausaalinen rooli samoin kuin Wignerin mallissa. Jos havaitsijalla on kausaalinen rooli aaltofunktion romahtamisessa niin sellainen näkemys vastaa täysin idealismia.

        Köpistulkinta lienee edelleen yleisin tulkinta fyysikkojen keskuudessa vaikka sen loogisista ja filosofista seurauksista ei juurikaan julkisuudessa keskustella kun on omaksuttu "turpa-kiinni-ja-laske" metodi jotta pikku fyysikoiden päät eivät menisi aivan pyörälle.... :-)

        "Aaltoilu on materian (eivät hengen tai mielen) ominaisuus, joten en ymmärrä, miten ne liitetään idealismiin."

        Ne mikromaailman aallot ovat todennäköisyyysaaltoja eivätkä siis konkreettisia aineellisia aaltoja kuten meren aallot. Kyseessä on pelkkä vertauskuva.

        Tieteessä yleisin filosofisin kanta lienee jonkinlainen positivismi tai looginen empirismi jossa kaikki filosofiset ja metafysikaaliset tulkinnat pyritään siirtämään luonnontieteiden ulkopuolelle. Tämä tarkoittaa sitten myös toisaalta sitä että pelkästään luonnontieteiden pohjalta ei voi perustella esim. materialismia eikä ateismia.

        Positivismilla ja loogisella empirismillä on myös omat sisäiset filosofiset ongelmansa joiden takia näitä filosofisia suuntauksia on vaikeaa tai mahdotonta seurata koherentisti joutumatta ristiriitoihin.


      • iloinenpiereskelijä
        Belisario kirjoitti:

        "Ei nyt ihan auennut, miten klassinen panpsykismi saadaan yhdistettyä kvanttikenttäteorian kanssa. "

        Kvanttikenttäteoria on matemaattinen teoria jossa ei oteta kantaa ontologiaan eli siihen onko se tutkittava asia materiaalinen vai ei-materiaalinen. Ontologian kannalta kvanttimekaniikassa ei vallitse juuri minkäänlaista yksimielisyyttä vaan niitä ontologisia tulkintamalleja on lukuisia.

        https://en.wikipedia.org/wiki/Interpretations_of_quantum_mechanics

        Tuossa wikiartikikkelissa löytyy mielenkiintoinen taulukko jossa näkyy miten kukin tulkinta hahmottaa esim. havaitsijan roolin ja sen onko universaalinen aaltofunktio on todellinen asia vai pelkkä matemaattinen apuväline.

        Kööpenhaminaisessa tulkinnassa havaitsijalla on kausaalinen rooli samoin kuin Wignerin mallissa. Jos havaitsijalla on kausaalinen rooli aaltofunktion romahtamisessa niin sellainen näkemys vastaa täysin idealismia.

        Köpistulkinta lienee edelleen yleisin tulkinta fyysikkojen keskuudessa vaikka sen loogisista ja filosofista seurauksista ei juurikaan julkisuudessa keskustella kun on omaksuttu "turpa-kiinni-ja-laske" metodi jotta pikku fyysikoiden päät eivät menisi aivan pyörälle.... :-)

        "Aaltoilu on materian (eivät hengen tai mielen) ominaisuus, joten en ymmärrä, miten ne liitetään idealismiin."

        Ne mikromaailman aallot ovat todennäköisyyysaaltoja eivätkä siis konkreettisia aineellisia aaltoja kuten meren aallot. Kyseessä on pelkkä vertauskuva.

        Tieteessä yleisin filosofisin kanta lienee jonkinlainen positivismi tai looginen empirismi jossa kaikki filosofiset ja metafysikaaliset tulkinnat pyritään siirtämään luonnontieteiden ulkopuolelle. Tämä tarkoittaa sitten myös toisaalta sitä että pelkästään luonnontieteiden pohjalta ei voi perustella esim. materialismia eikä ateismia.

        Positivismilla ja loogisella empirismillä on myös omat sisäiset filosofiset ongelmansa joiden takia näitä filosofisia suuntauksia on vaikeaa tai mahdotonta seurata koherentisti joutumatta ristiriitoihin.

        "Ne mikromaailman aallot ovat todennäköisyyysaaltoja eivätkä siis konkreettisia aineellisia aaltoja kuten meren aallot. Kyseessä on pelkkä vertauskuva."

        Kysymyksessä on aaltofunktio, joka kuvaa ihan todellista ilmiötä. Siitä ei saa tehtyä rationaalisesti mkinkäänlaisia idealistisia tulkintoja. Ne ovat pelkkää huuhaata.

        "Tieteessä yleisin filosofisin kanta lienee jonkinlainen positivismi tai looginen empirismi jossa kaikki filosofiset ja metafysikaaliset tulkinnat pyritään siirtämään luonnontieteiden ulkopuolelle."

        Ei ole. Puhut filosofian historiasta. Nykyisin on sikäli pragmaatinen asenne vallalla, että turha metafysiikka leikataan Occamin parturissa sillä veitellä veks. ;)


      • Belisario
        iloinenpiereskelijä kirjoitti:

        "Ne mikromaailman aallot ovat todennäköisyyysaaltoja eivätkä siis konkreettisia aineellisia aaltoja kuten meren aallot. Kyseessä on pelkkä vertauskuva."

        Kysymyksessä on aaltofunktio, joka kuvaa ihan todellista ilmiötä. Siitä ei saa tehtyä rationaalisesti mkinkäänlaisia idealistisia tulkintoja. Ne ovat pelkkää huuhaata.

        "Tieteessä yleisin filosofisin kanta lienee jonkinlainen positivismi tai looginen empirismi jossa kaikki filosofiset ja metafysikaaliset tulkinnat pyritään siirtämään luonnontieteiden ulkopuolelle."

        Ei ole. Puhut filosofian historiasta. Nykyisin on sikäli pragmaatinen asenne vallalla, että turha metafysiikka leikataan Occamin parturissa sillä veitellä veks. ;)

        "Kysymyksessä on aaltofunktio, joka kuvaa ihan todellista ilmiötä. "

        Todellinen se ehkä on mutta ei sitä ilmiöksi voi kutsua koska se tapahtuu aina ennen havaintoa eli se ei varsinaisesti voi olla fysikaalinen ilmiö.

        Aaltofunktio on todennäköisyysajakauma jonka voi tulkita joko siten että se todennäköisyysjakauma (=aaltofunktio) romahtaa yhdeksi niistä todennäköisyyksistä tai sitten multiversumimallissa ne kaikki mahdollisuudet toteutuvat mutta kukin eri maailmankaikkeudessa.

        Tuossa multiversumimallissa olis aika paljon käyttöä sillä Occamin parturille.... :-)

        "Siitä ei saa tehtyä rationaalisesti mkinkäänlaisia idealistisia tulkintoja. Ne ovat pelkkää huuhaata."

        Noinhan materialistit ovat tottuneet ajattelemaan mutta se ole pelkkä uskomus tai metafyysinen tulkinta. Ihminen pitää aina omia uskomuksiaan ja uskontoaan "rationaalisena" joten se on aika tyhjä väite. Idealistin kannalta materialismi on täyttä huuhaata.

        Paras olisi kuin ei edes yritettäisi yhdistää tieteeseen minkäänlaisia ontologioita koska ontologia liittyy aina filosofiaan eikä sitä tieteen metodilla mitenkään vahvistaa.

        "Nykyisin on sikäli pragmaatinen asenne vallalla, että turha metafysiikka leikataan Occamin parturissa sillä veitellä veks. ;)"

        No se on juuri se positivistin tai skientistin asenne jonka kuvitellaan olevan vapaa kaikesta metafysiikasta vaikka se ei pidä alkuunkaan paikkaansa.

        Kannattaa muuten käydä vilkaisemassa wikipediasta miten positivismi, looginen empirismi ja skientismi määritellään ja tulla vasta sitten mutuilemaan.

        .....


        Kek-kek höpötti puskista seuraavan junttimaterialistisen väitteen:

        "Koivuhalkoterapia tekee piintyneistäkin idealisteista äkisti realisteja."

        Tuskin. Idealisti ehtii tarvittaessa käyttämään samaa koivuhalkoa ensin sen ns. "realistin" päähän jos huomaa toisella olevan sellaisia aikeita. Ns. fyysisen kivun kokemus on sekin subjektiivinen kokemus jolla on lisäksi hyödyllinen tarkoitus opettaa mitä kannattaa tehdä ja mitä ei tässä "simulaatiossa".

        Kaikki kokemus on todellista sellaisenaan ja se totuusarvo tulee vasta siinä vaiheessa kun aletaan miettiä mitä se kokemus tarkoittaa ja miten se liittyy kaikkeen muuhun kokemukseen.


      • kiikkustuolista kirjoitti:

        Idealismi on kieltämättä outo oppisuuntaus. Se on hyvin kaukana naiivista arkiajattelusta, jonka mukaan vesi on vain vettä ja sillä siisti. Idealismissa mennään syvälle mielen rakenteisiin ja vastaavasti nykyajan fysikalismissa syvälle aineen rakenteisiin. Siellä ne puolestaan lähestyvät toisiaan.

        "Idealismi on kieltämättä outo oppisuuntaus."

        Eipä tuo ole yhtään sen oudompaa kuin mikään muukaan tietoisuuteen liittyvä. Itse tietoisuus kun on outo juttu. Sellaisenkin ajatuksen olen lukenut (ehkä tältä palstalta), että jos hapella ja vedyllä olisi oma tietoisuutensa, niin yhtyessään vedeksi ne menettäisivät oman tietoisuutensa ja jäljelle jäisi vain veden tietoisuus. Kompleksisemmillakin olioilla, kuten ihminen. olisi vain yksi tietoisuus, eikä niiden osasilla olisi omaa tietoisuutta ainakaan niin kauan kuin ne ovat osa suurempaa kokonaisuutta. Minusta aivan toimivan tuntuinen ajatus (idea).


      • Järkisyitä
        Belisario kirjoitti:

        "Ei nyt ihan auennut, miten klassinen panpsykismi saadaan yhdistettyä kvanttikenttäteorian kanssa. "

        Kvanttikenttäteoria on matemaattinen teoria jossa ei oteta kantaa ontologiaan eli siihen onko se tutkittava asia materiaalinen vai ei-materiaalinen. Ontologian kannalta kvanttimekaniikassa ei vallitse juuri minkäänlaista yksimielisyyttä vaan niitä ontologisia tulkintamalleja on lukuisia.

        https://en.wikipedia.org/wiki/Interpretations_of_quantum_mechanics

        Tuossa wikiartikikkelissa löytyy mielenkiintoinen taulukko jossa näkyy miten kukin tulkinta hahmottaa esim. havaitsijan roolin ja sen onko universaalinen aaltofunktio on todellinen asia vai pelkkä matemaattinen apuväline.

        Kööpenhaminaisessa tulkinnassa havaitsijalla on kausaalinen rooli samoin kuin Wignerin mallissa. Jos havaitsijalla on kausaalinen rooli aaltofunktion romahtamisessa niin sellainen näkemys vastaa täysin idealismia.

        Köpistulkinta lienee edelleen yleisin tulkinta fyysikkojen keskuudessa vaikka sen loogisista ja filosofista seurauksista ei juurikaan julkisuudessa keskustella kun on omaksuttu "turpa-kiinni-ja-laske" metodi jotta pikku fyysikoiden päät eivät menisi aivan pyörälle.... :-)

        "Aaltoilu on materian (eivät hengen tai mielen) ominaisuus, joten en ymmärrä, miten ne liitetään idealismiin."

        Ne mikromaailman aallot ovat todennäköisyyysaaltoja eivätkä siis konkreettisia aineellisia aaltoja kuten meren aallot. Kyseessä on pelkkä vertauskuva.

        Tieteessä yleisin filosofisin kanta lienee jonkinlainen positivismi tai looginen empirismi jossa kaikki filosofiset ja metafysikaaliset tulkinnat pyritään siirtämään luonnontieteiden ulkopuolelle. Tämä tarkoittaa sitten myös toisaalta sitä että pelkästään luonnontieteiden pohjalta ei voi perustella esim. materialismia eikä ateismia.

        Positivismilla ja loogisella empirismillä on myös omat sisäiset filosofiset ongelmansa joiden takia näitä filosofisia suuntauksia on vaikeaa tai mahdotonta seurata koherentisti joutumatta ristiriitoihin.

        "Kvanttikenttäteoria on matemaattinen teoria jossa ei oteta kantaa ontologiaan eli siihen onko se tutkittava asia materiaalinen vai ei-materiaalinen."

        Se on malli, joka selittää miten havaitsemamme materia käyttäytyy. Se, onko materia maailman perusta vaan vain olemassa jossakin mielessä on irrelevanttia. Malli ennustaa sen käyttäytymisen.

        "Köpistulkinta lienee edelleen yleisin tulkinta fyysikkojen keskuudessa .."

        Ehkä oli 100 vuotta sitten muttei enää. Köpistulkintaa kritisoitiin jo aikoinaan, mutta 90-luvulla sitä ei edes mainittu yliopistofysiilassa Otaniemessä. Nykyinen vallalla oleva selitys on sen verran tylsä, ettei se paljon filosofeja kiinnosta, jotka rakastavat mystisempiä selityksiä.

        Ensinnäkin nykyisin kaksoisrakokokeen elektronin todennäköinen reitti saatu jo mitattua ilman aaltofunktion romahtamista. Havaitsijalla ei ole vaikutusta tulokseen vaan oleellista on mittausmenetelmä. Aaltofunktion romahtaminen seuraa siitä, että toinen hiukkanen esimerkiksi vain fotoni vuorovaikuttaa mitattavan elektronin kanssa. Nykyinen tulkinta QM mallista onkin, että hiukkaset ovat olemassa ikäänkuin aaltoina, mutta vuorovaikutus toisten hiukkasten kanssa tapahtuu aina pistemäisesti ja kvantittuneesti. Hiukkastenvälinen vuorovaikutus siis romahduttaa aaltofunktion.


      • Järkisyitä
        Belisario kirjoitti:

        "Ei nyt ihan auennut, miten klassinen panpsykismi saadaan yhdistettyä kvanttikenttäteorian kanssa. "

        Kvanttikenttäteoria on matemaattinen teoria jossa ei oteta kantaa ontologiaan eli siihen onko se tutkittava asia materiaalinen vai ei-materiaalinen. Ontologian kannalta kvanttimekaniikassa ei vallitse juuri minkäänlaista yksimielisyyttä vaan niitä ontologisia tulkintamalleja on lukuisia.

        https://en.wikipedia.org/wiki/Interpretations_of_quantum_mechanics

        Tuossa wikiartikikkelissa löytyy mielenkiintoinen taulukko jossa näkyy miten kukin tulkinta hahmottaa esim. havaitsijan roolin ja sen onko universaalinen aaltofunktio on todellinen asia vai pelkkä matemaattinen apuväline.

        Kööpenhaminaisessa tulkinnassa havaitsijalla on kausaalinen rooli samoin kuin Wignerin mallissa. Jos havaitsijalla on kausaalinen rooli aaltofunktion romahtamisessa niin sellainen näkemys vastaa täysin idealismia.

        Köpistulkinta lienee edelleen yleisin tulkinta fyysikkojen keskuudessa vaikka sen loogisista ja filosofista seurauksista ei juurikaan julkisuudessa keskustella kun on omaksuttu "turpa-kiinni-ja-laske" metodi jotta pikku fyysikoiden päät eivät menisi aivan pyörälle.... :-)

        "Aaltoilu on materian (eivät hengen tai mielen) ominaisuus, joten en ymmärrä, miten ne liitetään idealismiin."

        Ne mikromaailman aallot ovat todennäköisyyysaaltoja eivätkä siis konkreettisia aineellisia aaltoja kuten meren aallot. Kyseessä on pelkkä vertauskuva.

        Tieteessä yleisin filosofisin kanta lienee jonkinlainen positivismi tai looginen empirismi jossa kaikki filosofiset ja metafysikaaliset tulkinnat pyritään siirtämään luonnontieteiden ulkopuolelle. Tämä tarkoittaa sitten myös toisaalta sitä että pelkästään luonnontieteiden pohjalta ei voi perustella esim. materialismia eikä ateismia.

        Positivismilla ja loogisella empirismillä on myös omat sisäiset filosofiset ongelmansa joiden takia näitä filosofisia suuntauksia on vaikeaa tai mahdotonta seurata koherentisti joutumatta ristiriitoihin.

        "Ne mikromaailman aallot ovat todennäköisyyysaaltoja eivätkä siis konkreettisia aineellisia aaltoja kuten meren aallot."

        Minä puhuin QFT mallin kenttäyhtälöistä. Sinä puhut ilmeisesti Schrödingerin todennäköisyysfunktioista, joiden kutsuminen "aalloiksi" on varsin harhaanjohtavaa. Schrödingerin todennäköisyysfunktioden tulkinta onkin oma asiansa....


      • Belisario
        Järkisyitä kirjoitti:

        "Kvanttikenttäteoria on matemaattinen teoria jossa ei oteta kantaa ontologiaan eli siihen onko se tutkittava asia materiaalinen vai ei-materiaalinen."

        Se on malli, joka selittää miten havaitsemamme materia käyttäytyy. Se, onko materia maailman perusta vaan vain olemassa jossakin mielessä on irrelevanttia. Malli ennustaa sen käyttäytymisen.

        "Köpistulkinta lienee edelleen yleisin tulkinta fyysikkojen keskuudessa .."

        Ehkä oli 100 vuotta sitten muttei enää. Köpistulkintaa kritisoitiin jo aikoinaan, mutta 90-luvulla sitä ei edes mainittu yliopistofysiilassa Otaniemessä. Nykyinen vallalla oleva selitys on sen verran tylsä, ettei se paljon filosofeja kiinnosta, jotka rakastavat mystisempiä selityksiä.

        Ensinnäkin nykyisin kaksoisrakokokeen elektronin todennäköinen reitti saatu jo mitattua ilman aaltofunktion romahtamista. Havaitsijalla ei ole vaikutusta tulokseen vaan oleellista on mittausmenetelmä. Aaltofunktion romahtaminen seuraa siitä, että toinen hiukkanen esimerkiksi vain fotoni vuorovaikuttaa mitattavan elektronin kanssa. Nykyinen tulkinta QM mallista onkin, että hiukkaset ovat olemassa ikäänkuin aaltoina, mutta vuorovaikutus toisten hiukkasten kanssa tapahtuu aina pistemäisesti ja kvantittuneesti. Hiukkastenvälinen vuorovaikutus siis romahduttaa aaltofunktion.

        "Ensinnäkin nykyisin kaksoisrakokokeen elektronin todennäköinen reitti saatu jo mitattua ilman aaltofunktion romahtamista."

        Edelleenkin se on laskujen mukaan 'todennäköinen reitti' tai pitäiskö sanoa todennäköisin reitti mutta kyseessä on edelleen samanlainen todennäköisyysjakauma.


        " Havaitsijalla ei ole vaikutusta tulokseen vaan oleellista on mittausmenetelmä. "

        Niin mittausmenetelmän valinnalla on merkitystä ja sen valinnan tekee sen kokeen suunnittelija.

        "Aaltofunktion romahtaminen seuraa siitä, että toinen hiukkanen esimerkiksi vain fotoni vuorovaikuttaa mitattavan elektronin kanssa."

        Tuo on vain yksi niistä monista mahdollisista tulkinnoista jonka sinä olet valinnut ainoaksi "järkeväksi". Ei tuo sinun tulkintasi sulje mitenkään pois niitä muita tulkintoja.

        Aaltofunktioita ja lomittumista on testattu laajemmallakin jopa 3000 atomin muodostamalla kokonaisuudella (esim. Bucky ball C60)

        "Nykyinen tulkinta QM mallista onkin, että hiukkaset ovat olemassa ikäänkuin aaltoina, mutta vuorovaikutus toisten hiukkasten kanssa tapahtuu aina pistemäisesti ja kvantittuneesti. "

        Se on sitten varmaankin se "turpa-kiinni-ja-laske" tulkinta.

        "Hiukkastenvälinen vuorovaikutus siis romahduttaa aaltofunktion."

        Tuskin koska aaltofunktiolla ei ole koolle ylärajaa tiedossa vaikka teknisiä hankaluuksia tulee varmaan koon kasvaessa.

        "Minä puhuin QFT mallin kenttäyhtälöistä. "

        Tuossas QFT:ssä tulee mukaan niitä äärettömyyksiä jotka huijataan pois ns. renormalisaatiolla.

        Matematiittisella fysiikalla sinänsä ei ole oikein mitään annettavaa filosofialle ja tässä ketjussa keskustellaan idealismista joka on taas lähinnä ontologinen asia. Jos kokeen suunnittelijan mittaustavan päätöksellä on merkitystä kokeen lopputuloksen kannalta niin se silti tukee idealistista mallia enemmän kuin mekaanista materialismia.

        ...

        Idealismia on monenlaista ja niissä ainoa yhteinen tekijä on että tietoisuus on kaikessa ensisijainen. Materiaa ei siis ole pakko sulkea kokonaan pois tai palauttaa tietoisuuteen. Voi myös olettaa että ns. materia on aina kaaoottinen, stokastinen ja satunnainen kun taas tietoisuus on ainoa kausaliteetti mikä tuottaa siihen materiaan järjestystä.

        Voin esim. järjestää kaikki n. 40 lottopalloa numerojärjestykseen suuremmasta pienempään mikä taas on äärimmäisen epätodennäköistä tai mahdotonta lottokoneen tai minkään muunkaan satunnaisen prosessin tuloksena. Oikein menneitä arvontoja ei voi lukita ilman että touhu muuttuu älykkääksi suunnitteluksi. Tuontyyppistä lukitsemista Dawkins käytti yrittäessään perustella evoluutioteoriaa mutta joutui sitten myöntämään että se ei toimi.
        (ks. Dawkins weasel)

        ...

        Kun tuossa edellä oli puhetta irrationaaliluvuista. Esim. pi-luku on irrationaaliluku ja sen väitetään sisältävän kaiken mahdollisen ihmisen tuottaman informaation. Tässä linkissä on pi:n miljoonaa ensimmäistä desimaalia.

        https://www.piday.org/million/


        Löytyykö tuosta ainuttakaan numerosarjaa jossa on esim. 10 kpl ykkösiä ("1111111111")?

        0-9 numeroiden esiintyvyys piissä

        http://www.eveandersson.com/pi/precalculated-frequencies

        Kaikki numerot esiintyvät suurin piirtein yhtä usein. Hyvin suurella todennäköisyydellä mistään irrationaaliluvusta ei löydy ainuttakaan edes lyhyttä Shakespearen opusta.


        ....

        Skeptikon sivustolla on useita aika hyviä Bernardo Kastrupin haastatteluja jos haluaa tietää enemmän modernista idealismista:

        http://skeptiko.com/?s=Kastrup

        Kastrup onnistuu mielestäni tähän mennessä parhaiten perustelemaan idealistisen ontologian.


      • Järkisyitä

        ”Niin mittausmenetelmän valinnalla on merkitystä ja sen valinnan tekee sen kokeen suunnittelija.”

        Kokeen suunnittelijan tietoisella valinnalla ei ole merkitystä. Sinä voit tehdä järjestelyn, jossa mittauslaitteen päälläolo määritellään vaikkapa tuulen voimakkuuden mukaan ja mittauksen tulos riippuu tuolloin tuuliolosuhteista.

        ”Tuo on vain yksi niistä monista mahdollisista tulkinnoista jonka sinä olet valinnut ainoaksi "järkeväksi". Ei tuo sinun tulkintasi sulje mitenkään pois niitä muita tulkintoja.”

        Unohdat, että fysiikka on edennyt QM mallin tekemisen jälkeen. Nykyisin fyysikoiden keskuudessa Köpiksentulkinta on pitkälti hylätty. On esitetty useita paradokseja, jotka tekevät siitä kyseenalaisen. Huomaa myös, että Köpiksenmalli puhuu mittauksesta eikä edes edellytä tietoista tarkkailijaa.


      • Belisario
        Järkisyitä kirjoitti:

        ”Niin mittausmenetelmän valinnalla on merkitystä ja sen valinnan tekee sen kokeen suunnittelija.”

        Kokeen suunnittelijan tietoisella valinnalla ei ole merkitystä. Sinä voit tehdä järjestelyn, jossa mittauslaitteen päälläolo määritellään vaikkapa tuulen voimakkuuden mukaan ja mittauksen tulos riippuu tuolloin tuuliolosuhteista.

        ”Tuo on vain yksi niistä monista mahdollisista tulkinnoista jonka sinä olet valinnut ainoaksi "järkeväksi". Ei tuo sinun tulkintasi sulje mitenkään pois niitä muita tulkintoja.”

        Unohdat, että fysiikka on edennyt QM mallin tekemisen jälkeen. Nykyisin fyysikoiden keskuudessa Köpiksentulkinta on pitkälti hylätty. On esitetty useita paradokseja, jotka tekevät siitä kyseenalaisen. Huomaa myös, että Köpiksenmalli puhuu mittauksesta eikä edes edellytä tietoista tarkkailijaa.

        "Unohdat, että fysiikka on edennyt QM mallin tekemisen jälkeen. Nykyisin fyysikoiden keskuudessa Köpiksentulkinta on pitkälti hylätty. On esitetty useita paradokseja, jotka tekevät siitä kyseenalaisen. Huomaa myös, että Köpiksenmalli puhuu mittauksesta eikä edes edellytä tietoista tarkkailijaa."

        Voisitko laittaa linkin niihin paradokseihin.

        ...

        Sitten on sellainen kaksoisrakokokeen viivästetyn valinnan versio (Delayed choice quantum eraser) joka alunperin oli John Wheelerin kehittämä ajatuskoe.

        http://www.bottomlayer.com/bottom/basic_delayed_choice.htm

        Tietoinen tarkkailija tarvitaan aina jossain vaiheessa lukemaan niitä mittausten tuloksia. Entäs jos se aaltofunktio romahtaakin vasta sitten kun joku tietoinen olento lukee ne tulokset?

        Ne mittalaitteidenkin tulokset nimittäin muodostavat myös todennäköisyysjakauman joka romahtaa vasta kun joku lukee ne. Vastaavasti kun rakennetaan jokin uusi mittauslaite (esim. hiukkaskiihdytin) niin ensimmäinen mittaus on satunnainen arvonta todennäköisyysjakaumasta ja kaikki samanlaisten laitteiden myöhemmät mittaustulokset ovat alkuperäisen arvonnan määrittelemiä.

        Ja sitten jos jostain syystä kaikki aikaisempien mittausten tuokset tuhoutuvat tutkijoiden mukana niin todennäköisyysjakauma palautuu alkup. tilaansa ja kun keksitään uudestaam hiukkaskiihdytin niin sen tulokset voivat olla täysin erilaisia kuin sen aikasemman vaiheen tulokset. Tässä olisi siis viivästetty valinta makrokoossa.

        Tietoisuus sinänsä ei riitä aaltofunktion romahtamiseen vaan tarvitaan lisäksi myös se havaintoinformaatio jonka poistaminen palauttaa tilanteen takaisin mikä se oli ennen romahdusta.

        Tom Campbellilla on aika hyvä esitys tästä esim. tällä luentovideolla:

        https://www.youtube.com/watch?v=BhMIz_iJtzQ

        John Wheelerin mukaan elämme osallistuvassa (participatory) kaikkeudessa jossa jokaisen yksilön tietoisuus omalta osaltaan vaikuttaa kokonaisuuteen.

        Fysikaalinen kausaliteetti on illuusio ja kyseessä on lähinnä lomittumisen kaltainen korrelaatio jossa joissain tilanteissa "syy" ja "seuraus" vaihtavat järjestystä. Ns. "seuraus" tai tulevaisuuden templaatti vetää puoleensa ja menneisyyden ("syyn") ehdollistuminen tuuppaa takaapäin.

        Ideoiden platonistinen maailma ei ole staattinen vaan jatkuvassa takaisinkytkennässä aktuaalisen maailman kanssa kuten esim. Sheldraken morfisessa resonanssissa.


      • Järkisyitä

        ”Entäs jos se aaltofunktio romahtaakin vasta sitten kun joku tietoinen olento lukee ne tulokset? ”

        Ok. Pohditaas, mitä tuo tarkoittaisi käytännössä paradoksin avulla.

        Koejärjestely: Tarkkailija on selin kokeeseen eikä kykene havinnoimaan kokeen tulosta, kunnes kääntyy muutamia minuutteja kokeen jäkeen tarkastamaan tuloksen. Mittauslaite ampuu elektronin kaksoisraon läpi ja mittalaite mittaa menikö elektroni vasemmasta vai oikeasta aukosta. Jos elektroni meni oikeasta aukosta mittalaite laukaisee haulikon, joka ampuu tarkkailijan kuoliaaksi. Kuolema on välitön.

        Jos elektronin reitti ratkeaa vasta siinä vaiheessa, kun tietoinen tarkkailija tarkistaa kokeen tuloksen ja tulos on hailikon ampuminen, niin tarkkailija on kuollut jo ennen kuin on kyennyt havainnoimaan kokeen tuloksen. Tämä on paradoksaalista, joten oletus, että tietoinen havainnoija romauttaisi aaltofunktion ratkaissen elektronin reitin on mahdottomuus.

        Köpiksen tulkinnan kritiikkiä ja outouksia on puitu mm:
        https://en.m.wikipedia.org/wiki/Copenhagen_interpretation


      • Järkisyitä
        Belisario kirjoitti:

        "Unohdat, että fysiikka on edennyt QM mallin tekemisen jälkeen. Nykyisin fyysikoiden keskuudessa Köpiksentulkinta on pitkälti hylätty. On esitetty useita paradokseja, jotka tekevät siitä kyseenalaisen. Huomaa myös, että Köpiksenmalli puhuu mittauksesta eikä edes edellytä tietoista tarkkailijaa."

        Voisitko laittaa linkin niihin paradokseihin.

        ...

        Sitten on sellainen kaksoisrakokokeen viivästetyn valinnan versio (Delayed choice quantum eraser) joka alunperin oli John Wheelerin kehittämä ajatuskoe.

        http://www.bottomlayer.com/bottom/basic_delayed_choice.htm

        Tietoinen tarkkailija tarvitaan aina jossain vaiheessa lukemaan niitä mittausten tuloksia. Entäs jos se aaltofunktio romahtaakin vasta sitten kun joku tietoinen olento lukee ne tulokset?

        Ne mittalaitteidenkin tulokset nimittäin muodostavat myös todennäköisyysjakauman joka romahtaa vasta kun joku lukee ne. Vastaavasti kun rakennetaan jokin uusi mittauslaite (esim. hiukkaskiihdytin) niin ensimmäinen mittaus on satunnainen arvonta todennäköisyysjakaumasta ja kaikki samanlaisten laitteiden myöhemmät mittaustulokset ovat alkuperäisen arvonnan määrittelemiä.

        Ja sitten jos jostain syystä kaikki aikaisempien mittausten tuokset tuhoutuvat tutkijoiden mukana niin todennäköisyysjakauma palautuu alkup. tilaansa ja kun keksitään uudestaam hiukkaskiihdytin niin sen tulokset voivat olla täysin erilaisia kuin sen aikasemman vaiheen tulokset. Tässä olisi siis viivästetty valinta makrokoossa.

        Tietoisuus sinänsä ei riitä aaltofunktion romahtamiseen vaan tarvitaan lisäksi myös se havaintoinformaatio jonka poistaminen palauttaa tilanteen takaisin mikä se oli ennen romahdusta.

        Tom Campbellilla on aika hyvä esitys tästä esim. tällä luentovideolla:

        https://www.youtube.com/watch?v=BhMIz_iJtzQ

        John Wheelerin mukaan elämme osallistuvassa (participatory) kaikkeudessa jossa jokaisen yksilön tietoisuus omalta osaltaan vaikuttaa kokonaisuuteen.

        Fysikaalinen kausaliteetti on illuusio ja kyseessä on lähinnä lomittumisen kaltainen korrelaatio jossa joissain tilanteissa "syy" ja "seuraus" vaihtavat järjestystä. Ns. "seuraus" tai tulevaisuuden templaatti vetää puoleensa ja menneisyyden ("syyn") ehdollistuminen tuuppaa takaapäin.

        Ideoiden platonistinen maailma ei ole staattinen vaan jatkuvassa takaisinkytkennässä aktuaalisen maailman kanssa kuten esim. Sheldraken morfisessa resonanssissa.

        ”Ne mittalaitteidenkin tulokset nimittäin muodostavat myös todennäköisyysjakauman joka romahtaa vasta kun joku lukee ne.”

        Ei, ei, ei... Esitin jo yllä paradoksin tämän mahdottomuudeksi, mutta voit ajatella myös tuon väitteen seurauksia. Nauhottakaamme mittaustulos videolle. Mieti onko todellakin mielekästä, että koko videon sisältö ratkeaa vasta siinä vaiheessa, kun me katsomme nauhan.

        ”Vastaavasti kun rakennetaan jokin uusi mittauslaite (esim. hiukkaskiihdytin) niin ensimmäinen mittaus on satunnainen arvonta todennäköisyysjakaumasta ja kaikki samanlaisten laitteiden myöhemmät mittaustulokset ovat alkuperäisen arvonnan määrittelemiä.”

        Se johtuuko todennököisyysjakauma siitä, ettemme tiedä kaikkia piileviä muuttujia vai onko kyseessä aito satunnaisuus on, ainakin omasta mielestäni, epäselvää. Väitteesi, että seuraavat mittaukset olisivat ensimmäisen määrittelemiä on mielestäni yksinkertaisesti väärin. Tämä on kuin sanoisi, että kun heität noppaa kolme kertaa vain ensimmäinen heitto on satunnainen ja seuraavat ratkeavat ensimmäisen heiton perusteella.

        ”Ja sitten jos jostain syystä kaikki aikaisempien mittausten tuokset tuhoutuvat tutkijoiden mukana niin todennäköisyysjakauma palautuu alkup. tilaansa ja kun keksitään uudestaam hiukkaskiihdytin niin sen tulokset voivat olla täysin erilaisia kuin sen aikasemman vaiheen tulokset. Tässä olisi siis viivästetty valinta makrokoossa.”

        Tuo kuulostaa jo monimaailma-ajatukselta. Ottamatta kantaa uskotko abiogenesikseen, niin olettakaamme, että elämä maapallolla syntyi sattumalta. Elämme tuhansien vuosien historian ja jossain vaiheessa kaikki kuolemme. Kun joku tulee katsomaan Maapalloa myöhemmin näkeekö hän sivilisaatiomme rauniot vai palautuuko kaikki alkuperäistilanteeseen ja kenties heidän havaintojemme mukaan elämä ei edes koskaan syntynyt maapallolle, koska todennäikyys määräytyi uudelleen?


      • Belisario
        Järkisyitä kirjoitti:

        ”Entäs jos se aaltofunktio romahtaakin vasta sitten kun joku tietoinen olento lukee ne tulokset? ”

        Ok. Pohditaas, mitä tuo tarkoittaisi käytännössä paradoksin avulla.

        Koejärjestely: Tarkkailija on selin kokeeseen eikä kykene havinnoimaan kokeen tulosta, kunnes kääntyy muutamia minuutteja kokeen jäkeen tarkastamaan tuloksen. Mittauslaite ampuu elektronin kaksoisraon läpi ja mittalaite mittaa menikö elektroni vasemmasta vai oikeasta aukosta. Jos elektroni meni oikeasta aukosta mittalaite laukaisee haulikon, joka ampuu tarkkailijan kuoliaaksi. Kuolema on välitön.

        Jos elektronin reitti ratkeaa vasta siinä vaiheessa, kun tietoinen tarkkailija tarkistaa kokeen tuloksen ja tulos on hailikon ampuminen, niin tarkkailija on kuollut jo ennen kuin on kyennyt havainnoimaan kokeen tuloksen. Tämä on paradoksaalista, joten oletus, että tietoinen havainnoija romauttaisi aaltofunktion ratkaissen elektronin reitin on mahdottomuus.

        Köpiksen tulkinnan kritiikkiä ja outouksia on puitu mm:
        https://en.m.wikipedia.org/wiki/Copenhagen_interpretation

        "Mittauslaite ampuu elektronin kaksoisraon läpi ja mittalaite mittaa menikö elektroni vasemmasta vai oikeasta aukosta. "

        Niin se mittaus tuossa tapauksessa pakottaa elektronin hiukkas-moodiin. Viivästetyn valinnan versio tuosta olisi että se mittauksen data poistetaan ENNEN kuin se ehtii sinne haulikkoon asti.

        "Nauhottakaamme mittaustulos videolle. Mieti onko todellakin mielekästä, että koko videon sisältö ratkeaa vasta siinä vaiheessa, kun me katsomme nauhan."

        Se "mielekkyys" on aika pitkälle riippuvaista perusoletuksistamme mutta jos viivästetyn valinnan koe on yleistettävissä makrotasolle niin se nauhan sisällön aaltofunktio romahtaa vasta kun se nauha katsotaan tai on ylipäätänsä kenen tahansa katsottavissa.

        Jos nauha hävitetään ennen sen katsomista niin aaltofunktio ei ole vielä romahtanut eli tulos ei ole vielä tässä meidän konsensustodellisuudessamme. Sama periaate kuin Schrödingerin kissa ajatuskokeessa.

        Kyse on enemmän epävarmuudesta eikä suoraan liity havaitsijaan. Kvanttifysikiikan Zeno paradoksissa hiukkasen evoluutiota ei pääse tapahtumaan jos sitä tarkkaillaan/mitataan jatkuvasti. Arkikokemuksen makromaailman aaltofunktio on jatkuvassa "hiukkastilassa" tai dehorenssissa koska sitä tarkkaillaan jatkuvasti.

        Metsässä joku puun kaatuminen voi taas olla todennäköisyysmoodissa hyvinkin kauan koska esim. lintujen, hyönteisten ja oravien data ei välttämättä välity ihmisten konsensustodellisuuteen.

        Mitä enemmän epävarmuutta tapahtumissa sitä todellisempi on sen tapahtuman aaltoluonne eli mahdollisuuksien maailma on yhtä todellinen kun aktuaalinenkin maailma sillä edellytyksellä että siihen liittyy epävarmuutta. Fotoni/elektroni kulkee molemmista raoista ja muodostaa todennäköisyys hajonnan koska mittausdataa ei ole saatavilla.

        Paradoksaalisia seurauksia tuosta viivästetyn valinnan kokeesta:

        Historiaa ei ole jos dataa siitä ei ole olemassa. Tähtitieteen tulokset ovat todellisia vain tässä meidän konsensusunessamme. Evoluutiota ei ole siinä mielessä kun nyt ajatellaan. Saatamme siis olla yksin tässä itse luomassamme konsensusunessa. On olemassa vain nykyhetki ja ns. menneisyys ja tulevaisuus riippuu täysin valinnoistamme

        Tämä on jonkinlainen makrotietoisuuden kehittämä simulaatio jonka ainoa tarkoitus on kehittää sitä makrotietoisuutta itseään jakautumalla useammaksi kokijaksi eri tilanteisiin. Kyse on siis jonkinlaisesta pelistä jossa on oma sääntökokoelmansa joka on optimoitu tietoisuuden kehitystä silmällä pitäen.

        Ei tuohon tietenkään tarvitse uskoa enkä itsekään siihen sellaisenaan usko mutta se selittäisi aika hyvin sen miksi tietoisuus tai oikeammin alitajunta kykenee unissa tuottamaan kokonaisia monimuotoisia maailmoja jotka ovat hyvin todentuntuisia ja omasta tietoisesta mielestä yleensä riippumattomia eli kyseessä ei ole mikään tietoisen mielen kontroloima maailma kuten ei tämä valvemaailmakaan ole.

        "Köpiksen tulkinnan kritiikkiä ja outouksia on puitu..."

        Nuo ovat ikivanhoja vastaväitteitä eli jo Einsteinin ja Schrödingerin aikana tiedossa.

        Kvanttifysiikka on erittäin outoa ja monet ovat sanoneetkin ettei kvanttifysiikkaa voi edes ymmärtää ellei tiedosta sitä outoutta vaikka se outous lähinnä liittyykin nykyisen tieteen perusoletuksiin ja tulkintoihin jotka ovat materialismin ideologian mukaisia.


      • iloinenpiereskelijä
        Belisario kirjoitti:

        "Mittauslaite ampuu elektronin kaksoisraon läpi ja mittalaite mittaa menikö elektroni vasemmasta vai oikeasta aukosta. "

        Niin se mittaus tuossa tapauksessa pakottaa elektronin hiukkas-moodiin. Viivästetyn valinnan versio tuosta olisi että se mittauksen data poistetaan ENNEN kuin se ehtii sinne haulikkoon asti.

        "Nauhottakaamme mittaustulos videolle. Mieti onko todellakin mielekästä, että koko videon sisältö ratkeaa vasta siinä vaiheessa, kun me katsomme nauhan."

        Se "mielekkyys" on aika pitkälle riippuvaista perusoletuksistamme mutta jos viivästetyn valinnan koe on yleistettävissä makrotasolle niin se nauhan sisällön aaltofunktio romahtaa vasta kun se nauha katsotaan tai on ylipäätänsä kenen tahansa katsottavissa.

        Jos nauha hävitetään ennen sen katsomista niin aaltofunktio ei ole vielä romahtanut eli tulos ei ole vielä tässä meidän konsensustodellisuudessamme. Sama periaate kuin Schrödingerin kissa ajatuskokeessa.

        Kyse on enemmän epävarmuudesta eikä suoraan liity havaitsijaan. Kvanttifysikiikan Zeno paradoksissa hiukkasen evoluutiota ei pääse tapahtumaan jos sitä tarkkaillaan/mitataan jatkuvasti. Arkikokemuksen makromaailman aaltofunktio on jatkuvassa "hiukkastilassa" tai dehorenssissa koska sitä tarkkaillaan jatkuvasti.

        Metsässä joku puun kaatuminen voi taas olla todennäköisyysmoodissa hyvinkin kauan koska esim. lintujen, hyönteisten ja oravien data ei välttämättä välity ihmisten konsensustodellisuuteen.

        Mitä enemmän epävarmuutta tapahtumissa sitä todellisempi on sen tapahtuman aaltoluonne eli mahdollisuuksien maailma on yhtä todellinen kun aktuaalinenkin maailma sillä edellytyksellä että siihen liittyy epävarmuutta. Fotoni/elektroni kulkee molemmista raoista ja muodostaa todennäköisyys hajonnan koska mittausdataa ei ole saatavilla.

        Paradoksaalisia seurauksia tuosta viivästetyn valinnan kokeesta:

        Historiaa ei ole jos dataa siitä ei ole olemassa. Tähtitieteen tulokset ovat todellisia vain tässä meidän konsensusunessamme. Evoluutiota ei ole siinä mielessä kun nyt ajatellaan. Saatamme siis olla yksin tässä itse luomassamme konsensusunessa. On olemassa vain nykyhetki ja ns. menneisyys ja tulevaisuus riippuu täysin valinnoistamme

        Tämä on jonkinlainen makrotietoisuuden kehittämä simulaatio jonka ainoa tarkoitus on kehittää sitä makrotietoisuutta itseään jakautumalla useammaksi kokijaksi eri tilanteisiin. Kyse on siis jonkinlaisesta pelistä jossa on oma sääntökokoelmansa joka on optimoitu tietoisuuden kehitystä silmällä pitäen.

        Ei tuohon tietenkään tarvitse uskoa enkä itsekään siihen sellaisenaan usko mutta se selittäisi aika hyvin sen miksi tietoisuus tai oikeammin alitajunta kykenee unissa tuottamaan kokonaisia monimuotoisia maailmoja jotka ovat hyvin todentuntuisia ja omasta tietoisesta mielestä yleensä riippumattomia eli kyseessä ei ole mikään tietoisen mielen kontroloima maailma kuten ei tämä valvemaailmakaan ole.

        "Köpiksen tulkinnan kritiikkiä ja outouksia on puitu..."

        Nuo ovat ikivanhoja vastaväitteitä eli jo Einsteinin ja Schrödingerin aikana tiedossa.

        Kvanttifysiikka on erittäin outoa ja monet ovat sanoneetkin ettei kvanttifysiikkaa voi edes ymmärtää ellei tiedosta sitä outoutta vaikka se outous lähinnä liittyykin nykyisen tieteen perusoletuksiin ja tulkintoihin jotka ovat materialismin ideologian mukaisia.

        "Kvanttifysiikka on erittäin outoa ja monet ovat sanoneetkin ettei kvanttifysiikkaa voi edes ymmärtää ellei tiedosta sitä outoutta vaikka se outous lähinnä liittyykin nykyisen tieteen perusoletuksiin ja tulkintoihin jotka ovat materialismin ideologian mukaisia."

        Niinpä näyttääolevan idealisteille. Tuossa yllä tuli sinulta niin huurun täytteistä puppua, ettei tiedä aina onko se parodiaa idealisteista. Yhtään mitään tekemistä jutuillasi ei ole materialismin "ideologian" kanssa, mitä sitten lienet tarkoitat sillä.

        Kvanttielektrodynamiikan aaltofunktio kuvastaa ihan oikeaa ilmiötä, eikä sitä pidä sotkea mihinkään mittaustuloksiin tai saadaan aikaan vain täydellistä sekasotkua.

        Aaltofunktio "realisoituu" mittauksissa approksimaationa, mutta se ei ole todellista muussa kuin matemaattistilastollisessa mielessä.


      • Järkisyitä
        Belisario kirjoitti:

        "Mittauslaite ampuu elektronin kaksoisraon läpi ja mittalaite mittaa menikö elektroni vasemmasta vai oikeasta aukosta. "

        Niin se mittaus tuossa tapauksessa pakottaa elektronin hiukkas-moodiin. Viivästetyn valinnan versio tuosta olisi että se mittauksen data poistetaan ENNEN kuin se ehtii sinne haulikkoon asti.

        "Nauhottakaamme mittaustulos videolle. Mieti onko todellakin mielekästä, että koko videon sisältö ratkeaa vasta siinä vaiheessa, kun me katsomme nauhan."

        Se "mielekkyys" on aika pitkälle riippuvaista perusoletuksistamme mutta jos viivästetyn valinnan koe on yleistettävissä makrotasolle niin se nauhan sisällön aaltofunktio romahtaa vasta kun se nauha katsotaan tai on ylipäätänsä kenen tahansa katsottavissa.

        Jos nauha hävitetään ennen sen katsomista niin aaltofunktio ei ole vielä romahtanut eli tulos ei ole vielä tässä meidän konsensustodellisuudessamme. Sama periaate kuin Schrödingerin kissa ajatuskokeessa.

        Kyse on enemmän epävarmuudesta eikä suoraan liity havaitsijaan. Kvanttifysikiikan Zeno paradoksissa hiukkasen evoluutiota ei pääse tapahtumaan jos sitä tarkkaillaan/mitataan jatkuvasti. Arkikokemuksen makromaailman aaltofunktio on jatkuvassa "hiukkastilassa" tai dehorenssissa koska sitä tarkkaillaan jatkuvasti.

        Metsässä joku puun kaatuminen voi taas olla todennäköisyysmoodissa hyvinkin kauan koska esim. lintujen, hyönteisten ja oravien data ei välttämättä välity ihmisten konsensustodellisuuteen.

        Mitä enemmän epävarmuutta tapahtumissa sitä todellisempi on sen tapahtuman aaltoluonne eli mahdollisuuksien maailma on yhtä todellinen kun aktuaalinenkin maailma sillä edellytyksellä että siihen liittyy epävarmuutta. Fotoni/elektroni kulkee molemmista raoista ja muodostaa todennäköisyys hajonnan koska mittausdataa ei ole saatavilla.

        Paradoksaalisia seurauksia tuosta viivästetyn valinnan kokeesta:

        Historiaa ei ole jos dataa siitä ei ole olemassa. Tähtitieteen tulokset ovat todellisia vain tässä meidän konsensusunessamme. Evoluutiota ei ole siinä mielessä kun nyt ajatellaan. Saatamme siis olla yksin tässä itse luomassamme konsensusunessa. On olemassa vain nykyhetki ja ns. menneisyys ja tulevaisuus riippuu täysin valinnoistamme

        Tämä on jonkinlainen makrotietoisuuden kehittämä simulaatio jonka ainoa tarkoitus on kehittää sitä makrotietoisuutta itseään jakautumalla useammaksi kokijaksi eri tilanteisiin. Kyse on siis jonkinlaisesta pelistä jossa on oma sääntökokoelmansa joka on optimoitu tietoisuuden kehitystä silmällä pitäen.

        Ei tuohon tietenkään tarvitse uskoa enkä itsekään siihen sellaisenaan usko mutta se selittäisi aika hyvin sen miksi tietoisuus tai oikeammin alitajunta kykenee unissa tuottamaan kokonaisia monimuotoisia maailmoja jotka ovat hyvin todentuntuisia ja omasta tietoisesta mielestä yleensä riippumattomia eli kyseessä ei ole mikään tietoisen mielen kontroloima maailma kuten ei tämä valvemaailmakaan ole.

        "Köpiksen tulkinnan kritiikkiä ja outouksia on puitu..."

        Nuo ovat ikivanhoja vastaväitteitä eli jo Einsteinin ja Schrödingerin aikana tiedossa.

        Kvanttifysiikka on erittäin outoa ja monet ovat sanoneetkin ettei kvanttifysiikkaa voi edes ymmärtää ellei tiedosta sitä outoutta vaikka se outous lähinnä liittyykin nykyisen tieteen perusoletuksiin ja tulkintoihin jotka ovat materialismin ideologian mukaisia.

        ”Niin se mittaus tuossa tapauksessa pakottaa elektronin hiukkas-moodiin. Viivästetyn valinnan versio tuosta olisi että se mittauksen data poistetaan ENNEN kuin se ehtii sinne haulikkoon asti.”

        Minulle jäi epäselväksi hyväksyitkö haulikko esimerkin osoittavan, ettei tietoisella havaitsijalla voi olla merkitystä vai keksitkö mitään paradoksin ratkaisuksi? Tuossa yllä sanot, että mittaus - eikä havaitsija romahduttaa aaltofunktion.

        Viivästetty valinta konsepti ei euennut minulle tässä yhteydessä. Jos viivästetty valinta johtaa havaitsijan kuolemiseen ja siten poistaa myös havaitsijan havaintokyvyn ennen kokemusta, niin katsotko, että tapahtuma ei sittenkään vielä ratkennut, koska kuolleella tietoisella havaitsijalla ei ole merkitystä?

        ”Arkikokemuksen makromaailman aaltofunktio on jatkuvassa "hiukkastilassa" tai dehorenssissa koska sitä tarkkaillaan jatkuvasti.”

        Tuo nyt on yksinkertaisesti puppua. Fysiikan mukaan aaltoluonteen merkityksellisyys riippuu kappaleen massasta. Makromaailmankohteiden aaltoluonne on jo merkityksetön. Kun katsot ihmistä, niin tarkkailukykysi jää varsin mitättömäksi, koska et nää pintaa syvemmälle ja tarkkailukykysi on varsin rajallinen.

        ”Metsässä joku puun kaatuminen voi taas olla todennäköisyysmoodissa hyvinkin kauan koska esim. lintujen, hyönteisten ja oravien data ei välttämättä välity ihmisten konsensustodellisuuteen. ”

        Laajennat kavanttimekaniikan satunnaisuuden varsin heppoisesti makromaailmaan, missä teet virheen. Kun toisistaan riippumatonta satunnaisuutta on paljon satunnaisuus katoaa.

        Otetaan esimerkiksi nopanheitto, joka on täysin satunnainen. Heitä noppaa sata kertaa ja jaa tulos nopanheittojen lukumäärällä, niin tulos on aika lähellä 3,5. Eli satunnaisuus pitkälti katosi.


      • Belisario
        iloinenpiereskelijä kirjoitti:

        "Kvanttifysiikka on erittäin outoa ja monet ovat sanoneetkin ettei kvanttifysiikkaa voi edes ymmärtää ellei tiedosta sitä outoutta vaikka se outous lähinnä liittyykin nykyisen tieteen perusoletuksiin ja tulkintoihin jotka ovat materialismin ideologian mukaisia."

        Niinpä näyttääolevan idealisteille. Tuossa yllä tuli sinulta niin huurun täytteistä puppua, ettei tiedä aina onko se parodiaa idealisteista. Yhtään mitään tekemistä jutuillasi ei ole materialismin "ideologian" kanssa, mitä sitten lienet tarkoitat sillä.

        Kvanttielektrodynamiikan aaltofunktio kuvastaa ihan oikeaa ilmiötä, eikä sitä pidä sotkea mihinkään mittaustuloksiin tai saadaan aikaan vain täydellistä sekasotkua.

        Aaltofunktio "realisoituu" mittauksissa approksimaationa, mutta se ei ole todellista muussa kuin matemaattistilastollisessa mielessä.

        Tällä videolla on tuo viivästetyn valinnan koe selitettynä aika yksityiskohtaisesti:

        https://www.youtube.com/watch?v=sfeoE1arF0I

        Se jo aikaisemmin linkkaamani Tom Campbellin video kaksoisrakokokeesta on myös aika hyvä.

        Koe on aika monimutkainen joten kannattaa etsiä hakusanalla "delayed choice quantum eraser". Tuon asian selittäminen aihetta tuntemattomille on aivan liian työlästä ja aikaavievää joten tutkikaa itse ensin ja muodostakaa oma mielipiteenne vasta sitten.

        Tuo haulikkoesimerkki ei mielestäni oikein vastaa sitä kaksoisrakokeen ongelmaa missä välillä saadaan interferenssikuvio ja taas välillä ei riippuen siitä kerätäänkö mittausdataa vai ei. Noppien heitto ei myöskään liity aiheeseen kovin kiinteästi jotta siitä olisi apua.

        .....

        Tietoinen havaitsija tarvitaan mutta olennaisempaa on että tilanteen informaatio on saatavilla ja se kuinka todennäköinen se tilanne on.

        ...


        Piereskelijä:


        "Kvanttielektrodynamiikan aaltofunktio kuvastaa ihan oikeaa ilmiötä, eikä sitä pidä sotkea mihinkään mittaustuloksiin tai saadaan aikaan vain täydellistä sekasotkua."

        "Aaltofunktio "realisoituu" mittauksissa approksimaationa, mutta se ei ole todellista muussa kuin matemaattistilastollisessa mielessä."

        Aaltofunktio on todennäköisyysjakauma ja kaksoisrakokoe muunnoksineen on juuri se koe joka ylipäätänsä aloitti kvanttifysiikan kehityksen. Vielä 1920-luvulla tuota pohdittiin vielä ahkerasti Einsteinin. Heisenbergin ja Schrödingerin johdolla mutta vähitellen tutkijat luovuttivat koska eivät edistyneet kvanttifysiikan ymmärtämisessä. Nykyään ei enää edes yritetä vaan pelkästään lasketaan kaavoilla.

        Tietenkin noiden kvanttifysiikan outouksien filosofiset seuraukset ovat arkijärjen vastaisia ja eräät tulkinnat saattavat viedä aika lähelle jonkinlaista new agea tai magiaa mutta onko sillä oikeasti väliä jos se kvanttifysiikka kuitenkin toimii sillä tavalla riippumatta tutkijoiden ja tieteenharrastajien enemmistön mieltymyksistä ja uskomuksista.

        ......


        En ole koneen ääressä tämän jälkeen moneen tuntiin joten en ehdi vastata pidempään.


      • iloinenpiereskelijä
        Belisario kirjoitti:

        Tällä videolla on tuo viivästetyn valinnan koe selitettynä aika yksityiskohtaisesti:

        https://www.youtube.com/watch?v=sfeoE1arF0I

        Se jo aikaisemmin linkkaamani Tom Campbellin video kaksoisrakokokeesta on myös aika hyvä.

        Koe on aika monimutkainen joten kannattaa etsiä hakusanalla "delayed choice quantum eraser". Tuon asian selittäminen aihetta tuntemattomille on aivan liian työlästä ja aikaavievää joten tutkikaa itse ensin ja muodostakaa oma mielipiteenne vasta sitten.

        Tuo haulikkoesimerkki ei mielestäni oikein vastaa sitä kaksoisrakokeen ongelmaa missä välillä saadaan interferenssikuvio ja taas välillä ei riippuen siitä kerätäänkö mittausdataa vai ei. Noppien heitto ei myöskään liity aiheeseen kovin kiinteästi jotta siitä olisi apua.

        .....

        Tietoinen havaitsija tarvitaan mutta olennaisempaa on että tilanteen informaatio on saatavilla ja se kuinka todennäköinen se tilanne on.

        ...


        Piereskelijä:


        "Kvanttielektrodynamiikan aaltofunktio kuvastaa ihan oikeaa ilmiötä, eikä sitä pidä sotkea mihinkään mittaustuloksiin tai saadaan aikaan vain täydellistä sekasotkua."

        "Aaltofunktio "realisoituu" mittauksissa approksimaationa, mutta se ei ole todellista muussa kuin matemaattistilastollisessa mielessä."

        Aaltofunktio on todennäköisyysjakauma ja kaksoisrakokoe muunnoksineen on juuri se koe joka ylipäätänsä aloitti kvanttifysiikan kehityksen. Vielä 1920-luvulla tuota pohdittiin vielä ahkerasti Einsteinin. Heisenbergin ja Schrödingerin johdolla mutta vähitellen tutkijat luovuttivat koska eivät edistyneet kvanttifysiikan ymmärtämisessä. Nykyään ei enää edes yritetä vaan pelkästään lasketaan kaavoilla.

        Tietenkin noiden kvanttifysiikan outouksien filosofiset seuraukset ovat arkijärjen vastaisia ja eräät tulkinnat saattavat viedä aika lähelle jonkinlaista new agea tai magiaa mutta onko sillä oikeasti väliä jos se kvanttifysiikka kuitenkin toimii sillä tavalla riippumatta tutkijoiden ja tieteenharrastajien enemmistön mieltymyksistä ja uskomuksista.

        ......


        En ole koneen ääressä tämän jälkeen moneen tuntiin joten en ehdi vastata pidempään.

        Lyhyesti kommentoin noihin delayed choice quantum eraser -kokeisiin, että näkemykseni mukaan niistä tehdään virheellisiä filosofisia johtopäätöksiä, koska itse koe ymmärretään väärin. Virhe on siinä, että tietyin ennakko-oletuksin tulosta tulkitessa päädytään johtopäätöksiin, jotka ovat omituisia suhteessa niihin alkuperäisiin ennakko-oletuksiin. Kun ennakko-oletukset poistaa, katoaa asian "outouskin".

        "Aaltofunktio on todennäköisyysjakauma ja kaksoisrakokoe muunnoksineen on juuri se koe joka ylipäätänsä aloitti kvanttifysiikan kehityksen."

        Näin siinä kävi, mutta sittemmin on edistytty ja rakennettu QED (=kvanttielektrodynamiikka), joka käsittelee kvanttikenttiä. Aaltofunktio on kvanttikentän eräs ratkaisu. Se tarkoittaa Feynmanin polkusumman ratkaisua yli kaikkien mahdollisten "ratojen", joilla "hiukkanen" on voinut kulkea. Se kuvastaa oikeaa fysikaalista ilmiötä, joka esitetään matemaattisena tod.näk.jakaumana. Heisenbergin epätarkkuusperiaate estää saamasta tälle aaltofunktiolle mitään "hiukkastason" täsmällistä ratkaisua, jossa sen parametrit olisivat tarkasti samanaikaisesti havaittavissa.

        Kuvitelma, että "hiukkasen" radan (joka on yksi parametri) voisi tarkasti määrittää todellisuudessa, on harhaa. Sille saadaan kyllä approksimaatio jollakin ajanhetkellä, mutta se ei ole "hiukkasen" luonteen todellinen kokonaiskuvaus. Sanoisinko, että silloin on vain tavoitettu vain kangastus.

        "...onko sillä oikeasti väliä jos se kvanttifysiikka kuitenkin toimii sillä tavalla riippumatta tutkijoiden ja tieteenharrastajien enemmistön mieltymyksistä ja uskomuksista."

        Kvanttifysiikka nimenomana toimii ja se toimii niiden rajoitteiden sisällä, minkä luonnonlait sille asettavat. On turhaa kuvitella todellisuuteen mitään muuta kuin minkä aaltofunktio sinne antaa.


      • Järkisyitä
        Belisario kirjoitti:

        Tällä videolla on tuo viivästetyn valinnan koe selitettynä aika yksityiskohtaisesti:

        https://www.youtube.com/watch?v=sfeoE1arF0I

        Se jo aikaisemmin linkkaamani Tom Campbellin video kaksoisrakokokeesta on myös aika hyvä.

        Koe on aika monimutkainen joten kannattaa etsiä hakusanalla "delayed choice quantum eraser". Tuon asian selittäminen aihetta tuntemattomille on aivan liian työlästä ja aikaavievää joten tutkikaa itse ensin ja muodostakaa oma mielipiteenne vasta sitten.

        Tuo haulikkoesimerkki ei mielestäni oikein vastaa sitä kaksoisrakokeen ongelmaa missä välillä saadaan interferenssikuvio ja taas välillä ei riippuen siitä kerätäänkö mittausdataa vai ei. Noppien heitto ei myöskään liity aiheeseen kovin kiinteästi jotta siitä olisi apua.

        .....

        Tietoinen havaitsija tarvitaan mutta olennaisempaa on että tilanteen informaatio on saatavilla ja se kuinka todennäköinen se tilanne on.

        ...


        Piereskelijä:


        "Kvanttielektrodynamiikan aaltofunktio kuvastaa ihan oikeaa ilmiötä, eikä sitä pidä sotkea mihinkään mittaustuloksiin tai saadaan aikaan vain täydellistä sekasotkua."

        "Aaltofunktio "realisoituu" mittauksissa approksimaationa, mutta se ei ole todellista muussa kuin matemaattistilastollisessa mielessä."

        Aaltofunktio on todennäköisyysjakauma ja kaksoisrakokoe muunnoksineen on juuri se koe joka ylipäätänsä aloitti kvanttifysiikan kehityksen. Vielä 1920-luvulla tuota pohdittiin vielä ahkerasti Einsteinin. Heisenbergin ja Schrödingerin johdolla mutta vähitellen tutkijat luovuttivat koska eivät edistyneet kvanttifysiikan ymmärtämisessä. Nykyään ei enää edes yritetä vaan pelkästään lasketaan kaavoilla.

        Tietenkin noiden kvanttifysiikan outouksien filosofiset seuraukset ovat arkijärjen vastaisia ja eräät tulkinnat saattavat viedä aika lähelle jonkinlaista new agea tai magiaa mutta onko sillä oikeasti väliä jos se kvanttifysiikka kuitenkin toimii sillä tavalla riippumatta tutkijoiden ja tieteenharrastajien enemmistön mieltymyksistä ja uskomuksista.

        ......


        En ole koneen ääressä tämän jälkeen moneen tuntiin joten en ehdi vastata pidempään.

        ”Tuo haulikkoesimerkki ei mielestäni oikein vastaa sitä kaksoisrakokeen ongelmaa missä välillä saadaan interferenssikuvio ja taas välillä ei riippuen siitä kerätäänkö mittausdataa vai ei.”

        Haulikkoparadoksi kumoaa vain sinun tulkintasi, missä väitit, että tietoisella havaitsijalla voisi olla vaikutusta kokeen tulokseen, koska tuo tulkinta aiheuttaa mahdottoman paradoksin. Schrödingerin kissa paradoksi oli tarkoitettu samaan, mutta jotkut valopäät lähtivätkin vääntämään siitä mystiikkaa. Haulikkoparadoksi lienee tehokkaampi.

        ”Tällä videolla on tuo viivästetyn valinnan koe selitettynä aika yksityiskohtaisesti: https://www.youtube.com/watch?v=sfeoE1arF0I

        Viivästetty valinta on tuttu, mutten ymmärrä miten se liittyy haulikkoparadoksiini. Tuo viivästetty valinta quantum eraser oli minulle uusi, mutta aika kiva. Edelleenkään en näe miten se liittyy haulikkoparadoksiini.

        https://en.m.wikipedia.org/wiki/Delayed_choice_quantum_eraser

        Kannattaa mieluummin lukea wikipediasta koe suoraan, koska tuon videon tekijä ei ollut edes ymmärtänyt koetta. Oleellisin virhe oli, että D0 detektorissa ei ole interferenssikuviota vaan kun D0 osuneet fotoneista valitaan D1 (tai D2) osuneiden fotoneiden parit, niin nuo jälkikäteen valitut D0 osuneet fotonit ovat inteferenssikuviomaisia. Ajatus, että tulevaisuus vaikuttaa menneisyyteen, on virheellinen. Selitys ilmiölle löytyy mm:
        https://physics.stackexchange.com/questions/255127/is-my-understanding-of-the-delayed-choice-quantum-eraser-correct

        En nyt ihan ymmärrä, miten nämä QM kokeet liittyvät idealismiin millään tavalla ellei tarkoituksena ole osoittaa, että idealistit eivät ymmärrä fysiikkaa ja poimivat heidän ajatuksiaan tukevat tulkinnat välittämättä, että nämä tulkinnat on osoitettavissa virheellisiksi.


    • Koivuhalkoterapia tekee piintyneistäkin idealisteista äkisti realisteja.

      • kiikkustuolista

        Ilmeisesti idealistin realismi merkitsee sitä, että tuo koivuhalkoterapiakin on illuusio. Idealistit eivät kiellä, etteikö kipu- ja kärsimyskokemuksia olisi olemassa.

        Materialististen mielenteorioiden mukaan mieli on yhtä kuin aivot. Tai pitäisikö sanoa yhtä kuin aivojen neuro- tai hermoverkosto. Näin ainakin identiteettiteoria antaa ymmärtää. Aivot puolestaan ovat materiaa. Näin ollen aineen pitäisi olla vähintäänkin potentiaalisesti tietoista. Luulisi siten panpsykismin tapaisten teorioiden olevan ilouutinen materialisteille.

        Joku täällä kummasteli, voiko vesi olla komponentteineen tietoista. Eikös ihmisen kehossa ole suurin osa silkkaa vettä? Ilmeisesti myös hermoverkostot saavat osansa siitä. Näin ollen materialistisesti ajatellen vesi olisi merittävässä asemassa tietoisuuden synnyssä.


      • Järkisyitä

        Materialistisen käsityksen mukaan mieli ei ole aivojen neuroverkko vaan neuroverkossa tapahtuvan prosessin tuotos/ilmentymä. Ajatus, että aine sinänsä olisi tietoista, on materialistisesta näkökulmasta älyvapaa ajatus. Tuollainen menee jonnekkin taikauskon ja mystiikan puolelle.


      • kiikkustuolista

        Mikäli tuo on totta, silloin materialistit ajautuvat ristiriitaan varsinkin, elleivät pysty selittämään mielen syntymekanismia. Teknisessä mielessä mielen rakenne sinänsä on prosessimainen, jota voidaan kuvata esimerkiksi virtauskaavion avulla.

        Toinen mieleen liittyvä ongelma on sen paikka maailmassa. Toisille mieli on ihmisen sisäinen ilmentymä, (internalismi), Toisille taas ulkoinen maailma jotenkin pitää mieltä yllä, (eksternalismi).

        Kolmas ongelma varsinkin materialismin kohdalla on mielen kvalitatiivisuus. Mistä syntyvät kaikki nuo "joltakin tuntuvat" kokemukset? Ne eivät oikein tahdo sopia materialistisiin arkkitehtuureihin.

        Vaikuttaa siltä, että ainakin perinteisessä mielessä sekä idealistit, että materialistit ovat tavallaan heittäneet pyyhkeen kehään, sillä he ovat lukinneet kaikki päätelmänsä dogmeiksi, joita ei sovi muutella. Monet mielen teoriat kylläkin nostavat esille osatotuuksia, mutta yhtenäinen mielen teoria antaa odottaa.


      • NA-jargonia
        kiikkustuolista kirjoitti:

        Mikäli tuo on totta, silloin materialistit ajautuvat ristiriitaan varsinkin, elleivät pysty selittämään mielen syntymekanismia. Teknisessä mielessä mielen rakenne sinänsä on prosessimainen, jota voidaan kuvata esimerkiksi virtauskaavion avulla.

        Toinen mieleen liittyvä ongelma on sen paikka maailmassa. Toisille mieli on ihmisen sisäinen ilmentymä, (internalismi), Toisille taas ulkoinen maailma jotenkin pitää mieltä yllä, (eksternalismi).

        Kolmas ongelma varsinkin materialismin kohdalla on mielen kvalitatiivisuus. Mistä syntyvät kaikki nuo "joltakin tuntuvat" kokemukset? Ne eivät oikein tahdo sopia materialistisiin arkkitehtuureihin.

        Vaikuttaa siltä, että ainakin perinteisessä mielessä sekä idealistit, että materialistit ovat tavallaan heittäneet pyyhkeen kehään, sillä he ovat lukinneet kaikki päätelmänsä dogmeiksi, joita ei sovi muutella. Monet mielen teoriat kylläkin nostavat esille osatotuuksia, mutta yhtenäinen mielen teoria antaa odottaa.

        "Ne eivät oikein tahdo sopia materialistisiin arkkitehtuureihin."

        Miksi eivät sopisi.
        Mielestä ilman biologista viitekehystä ei ole pienintäkään näyttöä. Ulkoisen maailmaan ylläpitämä mieli on silkkaa hörhöilyä.


      • Belisario
        NA-jargonia kirjoitti:

        "Ne eivät oikein tahdo sopia materialistisiin arkkitehtuureihin."

        Miksi eivät sopisi.
        Mielestä ilman biologista viitekehystä ei ole pienintäkään näyttöä. Ulkoisen maailmaan ylläpitämä mieli on silkkaa hörhöilyä.

        "Mielestä ilman biologista viitekehystä ei ole pienintäkään näyttöä. "

        Tuo on vähän samantapainen väite kuin:

        "Ilman television viitekehystä uutistenlukijasta ei ole minkäänlaista näyttöä"

        Tarkkaan ottaen meillä ei ole suoraa näyttöä kuin omasta mielestämme. Kaikkien muiden mieli päätellään aina käyttäytymisestä ja muiden kielellisistä ilmaisuista jotka on helppo feikata esim. tietotekniikalla.

        Viimeaikasissa kokeissa DMT-huumeen kanssa huomattiin että aivojen aktiviteetti väheni merkittävästi samalla kuin koehenkilöiden kokemusmaailma rikastui ja laajeni. Samantapainen ilmiö lienee ns. kuolemanrajakokemuksissa. Aivot ovat pelkkä suodatin ja 3.persoonan kuvaus tietoisuuden lokalisaatioista ja dissosiaatiosta

        Huvittaa tuo NA (new age) vihjaaminen.

        Jos esim. telepatia, kaukonäkeminen yms. toimivat niin niillä ei ole mitään tekemistä new agen kanssa. Tiedeyhteisöllä on kummallisia estoja, ennakkoasenteita ja ennakkoluuloja mitkä ominaisuudet ovatkin tyypillisiä kaikilla ns, skeptikoille (=tieteen papiston pikku apulaisille)

        Jos jokin asia toimii todistettavasti tai vähintään yhtä hyvin todistein kuin mikä tahansa muukin asia niin se sitten toimii riippumatta siitä onko se asia tai kyky yhteensopiva olemassaolevan "tieteellisen" uskomusjärjestelmän kanssa.


      • kiikkustuolista
        NA-jargonia kirjoitti:

        "Ne eivät oikein tahdo sopia materialistisiin arkkitehtuureihin."

        Miksi eivät sopisi.
        Mielestä ilman biologista viitekehystä ei ole pienintäkään näyttöä. Ulkoisen maailmaan ylläpitämä mieli on silkkaa hörhöilyä.

        Biologiset viitekehykset palvelevat lähinnä käytännön elämän tarpeita, eikä niitä voida sitoa yksioikoisesti mielen käsitteeseen. Esimerkkinä vaikkapa kipu. Syvässä narkoottisessa tilassa, jolloin tietoisuus ei ole läsnä, et tunne kipua ja näin suuretkin kirurgiset operaatiot voidaan tehdä huoletta.

        Eksternalismi on lähinnä sosiologien vastaus mielen ja tietoisuuden ongelmaan. Hehän pitävät ihmistä läpeensä sosiaalisena olentona. En tyrmäisi aivan kokonaan sosiaalisuuden vaikutusta mielen syntyyn. Esimerkiksi "kieli", joka toimii ajatusten "rahtilaivana", on mitä ilmeisemmin sosiaalisen kanssakäymisen tulosta.


      • iloinenpiereskelijä
        Belisario kirjoitti:

        "Mielestä ilman biologista viitekehystä ei ole pienintäkään näyttöä. "

        Tuo on vähän samantapainen väite kuin:

        "Ilman television viitekehystä uutistenlukijasta ei ole minkäänlaista näyttöä"

        Tarkkaan ottaen meillä ei ole suoraa näyttöä kuin omasta mielestämme. Kaikkien muiden mieli päätellään aina käyttäytymisestä ja muiden kielellisistä ilmaisuista jotka on helppo feikata esim. tietotekniikalla.

        Viimeaikasissa kokeissa DMT-huumeen kanssa huomattiin että aivojen aktiviteetti väheni merkittävästi samalla kuin koehenkilöiden kokemusmaailma rikastui ja laajeni. Samantapainen ilmiö lienee ns. kuolemanrajakokemuksissa. Aivot ovat pelkkä suodatin ja 3.persoonan kuvaus tietoisuuden lokalisaatioista ja dissosiaatiosta

        Huvittaa tuo NA (new age) vihjaaminen.

        Jos esim. telepatia, kaukonäkeminen yms. toimivat niin niillä ei ole mitään tekemistä new agen kanssa. Tiedeyhteisöllä on kummallisia estoja, ennakkoasenteita ja ennakkoluuloja mitkä ominaisuudet ovatkin tyypillisiä kaikilla ns, skeptikoille (=tieteen papiston pikku apulaisille)

        Jos jokin asia toimii todistettavasti tai vähintään yhtä hyvin todistein kuin mikä tahansa muukin asia niin se sitten toimii riippumatta siitä onko se asia tai kyky yhteensopiva olemassaolevan "tieteellisen" uskomusjärjestelmän kanssa.

        "Tarkkaan ottaen meillä ei ole suoraa näyttöä kuin omasta mielestämme."

        Tämäkään ei pidä paikaansa. Näyttö tarkoittaa sellaista evidenssiä, joka voidaan esittää jollekulle (muulle kuin itselle) ja tätä näyttöä arvioimalla tuo vastaanottaja joko vakuuttuu tai ei vakuutu väitteen totuudesta.

        Niinpä mielestä ei ole mitään näyttöä lainkaan.

        Omasta mielestä jokaisella on välitön kokemus, joka on täysin eri asia kuin näyttö, jota voidaan julkisesti arvioida.


      • kiikkustuolista
        Belisario kirjoitti:

        "Mielestä ilman biologista viitekehystä ei ole pienintäkään näyttöä. "

        Tuo on vähän samantapainen väite kuin:

        "Ilman television viitekehystä uutistenlukijasta ei ole minkäänlaista näyttöä"

        Tarkkaan ottaen meillä ei ole suoraa näyttöä kuin omasta mielestämme. Kaikkien muiden mieli päätellään aina käyttäytymisestä ja muiden kielellisistä ilmaisuista jotka on helppo feikata esim. tietotekniikalla.

        Viimeaikasissa kokeissa DMT-huumeen kanssa huomattiin että aivojen aktiviteetti väheni merkittävästi samalla kuin koehenkilöiden kokemusmaailma rikastui ja laajeni. Samantapainen ilmiö lienee ns. kuolemanrajakokemuksissa. Aivot ovat pelkkä suodatin ja 3.persoonan kuvaus tietoisuuden lokalisaatioista ja dissosiaatiosta

        Huvittaa tuo NA (new age) vihjaaminen.

        Jos esim. telepatia, kaukonäkeminen yms. toimivat niin niillä ei ole mitään tekemistä new agen kanssa. Tiedeyhteisöllä on kummallisia estoja, ennakkoasenteita ja ennakkoluuloja mitkä ominaisuudet ovatkin tyypillisiä kaikilla ns, skeptikoille (=tieteen papiston pikku apulaisille)

        Jos jokin asia toimii todistettavasti tai vähintään yhtä hyvin todistein kuin mikä tahansa muukin asia niin se sitten toimii riippumatta siitä onko se asia tai kyky yhteensopiva olemassaolevan "tieteellisen" uskomusjärjestelmän kanssa.

        Aivojen lisäksi myös mieli on eräänlainen "suodatin". Se nimittäin jäsentää ja järjestää kokemuksen eri osat mielekkääksi ja yhtenäiseksi kokemukseksi. Toisaalta se myös erottelee, luokittelee ja normittaa kokemuksen kaikesta muusta asiaan kuulumattomasta. Lähtökohtana on siten inhimillinen kokemus ja päättelyn lopputuloksena inhimillinen järkeily.

        Kyseinen prosessi edellyttää tietenkin tietoisuuden jatkuvaa läsnä oloa. I Kantia tulkiten tiedostuksen voidaan sanoa syntyvän, kun ymmärrys ja aistimus yhdistyvät toisiinsa. Tässä kyseisessä yhtymisessä puhtaiden ymmärryskäsitteiden tarjoama "muoto" liittyy aistihavainnossa (kokemuksessa) annettuun "sisältöön". Noista tajunnan ja tietoisuuden virroista syntyy mielen sisältä. Tietoisuuden intentionaalisuudesta johtuen, kokemuksen ei tarvitse välttämättä olla materialistisesti "aistillinen".


      • Belisario
        kiikkustuolista kirjoitti:

        Biologiset viitekehykset palvelevat lähinnä käytännön elämän tarpeita, eikä niitä voida sitoa yksioikoisesti mielen käsitteeseen. Esimerkkinä vaikkapa kipu. Syvässä narkoottisessa tilassa, jolloin tietoisuus ei ole läsnä, et tunne kipua ja näin suuretkin kirurgiset operaatiot voidaan tehdä huoletta.

        Eksternalismi on lähinnä sosiologien vastaus mielen ja tietoisuuden ongelmaan. Hehän pitävät ihmistä läpeensä sosiaalisena olentona. En tyrmäisi aivan kokonaan sosiaalisuuden vaikutusta mielen syntyyn. Esimerkiksi "kieli", joka toimii ajatusten "rahtilaivana", on mitä ilmeisemmin sosiaalisen kanssakäymisen tulosta.

        Sekä biologia että sosiaalisuus moduloivat tietoisuutta mutta eivät tuota sitä. Sekä biologia että sosiaalisuus edellyttävät jo sellaisenaan tietoisuuden olemassaolon. Samalla tavalla kehon fyysinen kunto, vireystila ja alkoholi/lääkkeet/huumeet moduloivat tietoisuutta.

        Jopa yksisoluisella parameciun oliolla on melko monimuotoinen tietoisuus mitä ainakin Christoph Kochin mukaan ei pysty mallintamaan materialismin pohjalta. Koch on yksi IIT teorian kehittäjistä ja nykyään kannattaa panpsykismiä.

        Kochin mukaan tietoisuus on sitä miltä aine tuntuu "sisäpuolelta" tarkasteltuna. Stuart Hameroff on taas sitä mieltä että evoluution prosessi ei edes toimi ilman jonkinlaista kokemuksellista tietoisuutta.

        Idealismi sinänsä ei tietenkään sulje pois biologiaa, sosiologiaa eikä mitään muitakaan tieteitä ja niiden tuloksia varsinkin jos niitä tieteen tuloksia tarkastellaan ilman ontologista materialistista painolastia joka ei sinänsä tieteeseen edes kuulu.

        Erilaiset viitekehykset eivät välttämättä sulje pois toisiaan vaan ovat toisiaan täydentäviä ja ne kaikki ovat yhteensovitettavissa idealismin kanssa. Looginen päättely toimii muutenkin vain saman viitekehyksen puitteissa ja jos samaan päättelyyn sotketaan asioita erilaisista viitekehyksistä niin lopputulos ei ole toimiva vaan sekasotkua.

        Idealismin mukaan tietoisuus "tuottaa" sekä aivot että hermoston eikä päinvastoin niin kuin materialismissa oletetaan. Kyseessä on pelkkä korrelaatio eikä kausaliteetti kumpaakaan suuntaan.


      • iloinenpiereskelijä
        Belisario kirjoitti:

        Sekä biologia että sosiaalisuus moduloivat tietoisuutta mutta eivät tuota sitä. Sekä biologia että sosiaalisuus edellyttävät jo sellaisenaan tietoisuuden olemassaolon. Samalla tavalla kehon fyysinen kunto, vireystila ja alkoholi/lääkkeet/huumeet moduloivat tietoisuutta.

        Jopa yksisoluisella parameciun oliolla on melko monimuotoinen tietoisuus mitä ainakin Christoph Kochin mukaan ei pysty mallintamaan materialismin pohjalta. Koch on yksi IIT teorian kehittäjistä ja nykyään kannattaa panpsykismiä.

        Kochin mukaan tietoisuus on sitä miltä aine tuntuu "sisäpuolelta" tarkasteltuna. Stuart Hameroff on taas sitä mieltä että evoluution prosessi ei edes toimi ilman jonkinlaista kokemuksellista tietoisuutta.

        Idealismi sinänsä ei tietenkään sulje pois biologiaa, sosiologiaa eikä mitään muitakaan tieteitä ja niiden tuloksia varsinkin jos niitä tieteen tuloksia tarkastellaan ilman ontologista materialistista painolastia joka ei sinänsä tieteeseen edes kuulu.

        Erilaiset viitekehykset eivät välttämättä sulje pois toisiaan vaan ovat toisiaan täydentäviä ja ne kaikki ovat yhteensovitettavissa idealismin kanssa. Looginen päättely toimii muutenkin vain saman viitekehyksen puitteissa ja jos samaan päättelyyn sotketaan asioita erilaisista viitekehyksistä niin lopputulos ei ole toimiva vaan sekasotkua.

        Idealismin mukaan tietoisuus "tuottaa" sekä aivot että hermoston eikä päinvastoin niin kuin materialismissa oletetaan. Kyseessä on pelkkä korrelaatio eikä kausaliteetti kumpaakaan suuntaan.

        "Jopa yksisoluisella parameciun oliolla on melko monimuotoinen tietoisuus mitä ainakin Christoph Kochin mukaan ei pysty mallintamaan materialismin pohjalta."

        Heheh! Panspykistien mukaan juu kivelläkin on "tietoisuus". Ja yleisö jopa uskoo tällaiseen new age roskaan. Tuollainen kielenkäyttö edellyttäisi edes pienen pientä näyttöä, miten se kivi ajattelee ja miten kiven ajattelu ilmenisi sen "toiminnassa".

        Mitenköhän kivi muuten toimii? Kivi "istuu" maassa. Se kokee ehkä olevansa "kaiken perusta".


      • Järkisyitä
        kiikkustuolista kirjoitti:

        Biologiset viitekehykset palvelevat lähinnä käytännön elämän tarpeita, eikä niitä voida sitoa yksioikoisesti mielen käsitteeseen. Esimerkkinä vaikkapa kipu. Syvässä narkoottisessa tilassa, jolloin tietoisuus ei ole läsnä, et tunne kipua ja näin suuretkin kirurgiset operaatiot voidaan tehdä huoletta.

        Eksternalismi on lähinnä sosiologien vastaus mielen ja tietoisuuden ongelmaan. Hehän pitävät ihmistä läpeensä sosiaalisena olentona. En tyrmäisi aivan kokonaan sosiaalisuuden vaikutusta mielen syntyyn. Esimerkiksi "kieli", joka toimii ajatusten "rahtilaivana", on mitä ilmeisemmin sosiaalisen kanssakäymisen tulosta.

        ”Syvässä narkoottisessa tilassa, jolloin tietoisuus ei ole läsnä, et tunne kipua ja näin suuretkin kirurgiset operaatiot voidaan tehdä huoletta.”

        Puudutuksessa estetään hermostosta tulevat sähköimpulssit, jotka välittävät tiedon kivusta. Anektesiassa taas häiritään aivojen hermosolujen toimintaa, niin että ihmisen tietoisuus sammuu. Evätkö nämä nimen omaan ole todisteita siitä, että tietoisuus syntyy hermosolujen toiminnan synnyttämänä?


      • Belisario
        iloinenpiereskelijä kirjoitti:

        "Jopa yksisoluisella parameciun oliolla on melko monimuotoinen tietoisuus mitä ainakin Christoph Kochin mukaan ei pysty mallintamaan materialismin pohjalta."

        Heheh! Panspykistien mukaan juu kivelläkin on "tietoisuus". Ja yleisö jopa uskoo tällaiseen new age roskaan. Tuollainen kielenkäyttö edellyttäisi edes pienen pientä näyttöä, miten se kivi ajattelee ja miten kiven ajattelu ilmenisi sen "toiminnassa".

        Mitenköhän kivi muuten toimii? Kivi "istuu" maassa. Se kokee ehkä olevansa "kaiken perusta".

        "Mitenköhän kivi muuten toimii? Kivi "istuu" maassa. Se kokee ehkä olevansa "kaiken perusta"."

        Tyypillinen olkiukko.

        Kochin ja Tononin integroidun informaation malli luokittelee tietoiseksi vain sellaiset asiat joissa tietoisuus on riittävän integroitunut. Koch käyttää itse paramecium yksisoluista esimerkkinä tällaisesta vaikka 1-soluisilla ei ole edes erilaistunutta hermostoa puhumattakaan aivoista. Kiven informaatio ei ole riittävän integroitunutta ollakseen tietoista.

        ...

        Neurokorrelaatiot eivät mitenkään sellaisenaan todista että ne tuottavat tietoisuuden koska ei ole olemassa ensimmäistäkään toimivaa teoriaa miten se käytännössä tapahtuu. Kyse on vain korrelaatiosta - ei kausaliteetista.


      • Belisario
        iloinenpiereskelijä kirjoitti:

        "Tarkkaan ottaen meillä ei ole suoraa näyttöä kuin omasta mielestämme."

        Tämäkään ei pidä paikaansa. Näyttö tarkoittaa sellaista evidenssiä, joka voidaan esittää jollekulle (muulle kuin itselle) ja tätä näyttöä arvioimalla tuo vastaanottaja joko vakuuttuu tai ei vakuutu väitteen totuudesta.

        Niinpä mielestä ei ole mitään näyttöä lainkaan.

        Omasta mielestä jokaisella on välitön kokemus, joka on täysin eri asia kuin näyttö, jota voidaan julkisesti arvioida.

        "Omasta mielestä jokaisella on välitön kokemus, joka on täysin eri asia kuin näyttö, jota voidaan julkisesti arvioida."

        Voi sen noinkin muotoilla jos välttämättä haluaa pilkkua viilata viimeisen päälle.... :-)

        Korvaa sana "näyttö" sanalla "välitön varma kokemus" jos se auttaa asiaa....

        Tais tulla nyt meikäläisellä päivän kiintiö täyteen täällä.


      • kiikkustuolista
        Järkisyitä kirjoitti:

        ”Syvässä narkoottisessa tilassa, jolloin tietoisuus ei ole läsnä, et tunne kipua ja näin suuretkin kirurgiset operaatiot voidaan tehdä huoletta.”

        Puudutuksessa estetään hermostosta tulevat sähköimpulssit, jotka välittävät tiedon kivusta. Anektesiassa taas häiritään aivojen hermosolujen toimintaa, niin että ihmisen tietoisuus sammuu. Evätkö nämä nimen omaan ole todisteita siitä, että tietoisuus syntyy hermosolujen toiminnan synnyttämänä?

        Argumenttisi ei edelleenkään vastaa kysymykseen, miten tietoisuus syntyy hermotoimintojen synnyttämänä. Se ei myöskään kerro, miten tietoisuus häviää tai tässä tapauksessa ei ole "läsnä" kyseisessä tapahtumassa. Puudutus kertoo vain sen, että jotain aistimusta ei tapahdu. Puolestaan anestesiassa sen, että tietoisuus tietyssä operaatiossa ei ole läsnä.

        Vaikka tietoisuus ei olekkaan kyseisessä operaatiossa läsnä, niin potilas voi silti olla tajuissaan. Nimittäin tajunta, joka on elämysten kokonaisuus, voi esiintyä myös alitajuisena ja tiedostamattomana. Kansanomainen määritelmä "tajuttomuudesta" ei siis päde kaikissa tapauksissa.

        Emme voi perustaa vastauksiamme pelkästään joihinkin rajatapuksiin, emmekä selitysten monimutkaisuuksiin.


      • Järkisyitä
        kiikkustuolista kirjoitti:

        Argumenttisi ei edelleenkään vastaa kysymykseen, miten tietoisuus syntyy hermotoimintojen synnyttämänä. Se ei myöskään kerro, miten tietoisuus häviää tai tässä tapauksessa ei ole "läsnä" kyseisessä tapahtumassa. Puudutus kertoo vain sen, että jotain aistimusta ei tapahdu. Puolestaan anestesiassa sen, että tietoisuus tietyssä operaatiossa ei ole läsnä.

        Vaikka tietoisuus ei olekkaan kyseisessä operaatiossa läsnä, niin potilas voi silti olla tajuissaan. Nimittäin tajunta, joka on elämysten kokonaisuus, voi esiintyä myös alitajuisena ja tiedostamattomana. Kansanomainen määritelmä "tajuttomuudesta" ei siis päde kaikissa tapauksissa.

        Emme voi perustaa vastauksiamme pelkästään joihinkin rajatapuksiin, emmekä selitysten monimutkaisuuksiin.

        ”Argumenttisi ei edelleenkään vastaa kysymykseen, miten tietoisuus syntyy hermotoimintojen synnyttämänä.”

        Totta. Tuo on ns ”hard problem”, johon on toki selityksiä, mutta johon kukaan ei ole vielä esittänyt ratkaisua. Idealistit, panpsykismin, IIT:n kannattajat eivät myöskään ratkaise tietoisuuden synnyn ongelmaa vaan vain ohittavat sen.

        ”Se ei myöskään kerro, miten tietoisuus häviää tai tässä tapauksessa ei ole "läsnä" kyseisessä tapahtumassa.”

        Tietoisuuden häviäminen johtuu selvästi aivojen neuroverkon toiminnan häirinnästä, Toki tietoisuuden havaitseminen mittaamalla on vielä ongelmallista. Toki joitakin asioita kuten se ajatteleeko ihminen mielessään kuvia saadaan mitattua ja tietoisuus on paikannettu aivojen etuosaan.

        Minulle tajunta ja tietoisuus ovat synonyymejä. Alitajunta on tiedostomaton toiminta, mutta sekin on vielä varsin laaja käsite. On ymmärtääkseni tavaittu tilanteita, jossa näköinformaatiota käsittelevä aivojen osa toimii, mutta tietoisuus on sammunut. Katsoisinkin, että suuri osa aivojen prosesseista muistuttaa keinoälyneuraverkkojen informaation käsittelyä, johon ei tarvita tietoisuutta.

        Esimerkiksi voidaan osoittaa, että kuva, joka välittyy tietoisuuteemme, on aivojen interpoloima tulevaisuuden ennustus, joka on rakennettu kuvien perusteella, mitä aivot näkivät joitakin kymmeniä millisekunteja sitten. Tämä auttaa meitä reagoimaan nykyhetken vaatimalla tavalla sen sijaan, että olisimme aivojen tietojenprosessointiajan verran menneisyydessä.


      • kiikkustuolista

        Tajunta ja tietoisuus eroavat selvästi toisistaan. Tajunta on "elämysten kokonaisuus". Tietoisuus puolestaan "tietoisten elämysten kokonaisuus". Mieli taas on "luokittelija", eräänlainen separaattori, joka erottelee olennaisen epäolennaisesta. Jos tuo separointi ei onnistu, puhutaan mielettömyydestä.

        Mielen ja komputaation ero tulee ilmi Searlen visioimassa "kiinalaisessa huoneessa".

        Tietoisen ja tiedottoman toiminnan ero selviää ns. Zombiargumentissa. Tietoinen toiminta "tuntuu joltakin", tiedoton toiminta ei. Juuri tuohon tietoisuuden kvalitatiivisuuteen teoreetikot hakkaavat päätään.


    • Belisario

      Kaikki mahdollinen tiede, tekniikka, filosofia ja matematiikka on aina tietoisuuden tuottamaa.

      Tietoisuus on ainoa asia joka on jo annettuna olemassa. Tietoisuutta ei voi reduktionistisesti palauttaa mihinkään ei-tietoiseen eikä tietoisuutta voi selittää millään mikä ei ole tietoista.

      Tämä on hyvin selkeä tosiasia jota on mahdotonta kumota vaikka viimeiset pari sataaa vuotta on tieteessä melko kuumeisesti yritetty todistella tietoisuutta olemattomaksi tai jostain ei-tietoisesta johtuvaksi. Tällaiset yritelmät kertovat paljon enemmän niiden esittäjistä kuin tietoisuudesta tai mistään muustakaan todellisesta. Tietoisuus ei pysty mitenkään todistamaan itseään itselleen olemattomaksi.

      Unissa me kaikki kehitämme hyvin monimuotoisen, todentuntuisen kokemusmaailman jossa ei ole täysin samanlaiset säännöt kuin valvetilan kokemusmaailmassa ja tavataan usein ihmisiä ja olijoita joita ei edes ennestään tunneta. Tämä unimaailma on usein niin valvetilassa koetun kaltaista että siinä unessa on usein mahdotonta tietää kummasta on kyse.

      En tiedä mistä ja miten ne unimaailman ilmiöt tulevat. Oikeastaan ihan samanlainen tilanne on valvemaailman ilmiöiden kanssa. Sekä unimaailman että valvemaailman tapahtumien säännönmukaisuuksista voi muodostaa matemaattisia kaavoja ja sitä kautta rakentaa jonkinlaista tekniikkaa vaikka unimaailmassa se jatkuvuus ei ole niin selkeää vaikka itse ainakin näen ajoittain unia jotka jatkavat siitä mihin edellinen uni jäi.

      Valvemaailmasta voi tutkia vain miten se toimii. 'Miksi' ja 'mitä' kysymyksiin ei löydy vastauksia valvemaaiilman eikä unimaailman puitteissa. Jonkinlainen universaali tietoisuus eli idealismi on kuitenkin varsin toimiva filosofinen valinta. Muita filosofisia valintoja ovat sitten materialismin eri muodot mutta nekin ovat kaikki mielen luomuksia.

      Jos on erilaiset filosofiset valinnat näkemysten tai uskomusjärjestelmän aksioomina niin keskustelu ei voi johtaa mihinkään vain jää loputtomaan eipäs-juupas luuppiin kuten aina aikaisemminkin ja varmaan aina tulevaisuudessakin.

      • iloinenpiereskelijä

        "Kaikki mahdollinen tiede, tekniikka, filosofia ja matematiikka on aina tietoisuuden tuottamaa.

        Tietoisuus on ainoa asia joka on jo annettuna olemassa."

        Väärin ja väärin.

        Tiede ei edellytä tietoisuutta. Sitä ei tieto-opissa tarvita lainkaan. Tietoisuus ei myöskään ole mitenkään "annettuna olemassa". Sen ilmentymistapoja tutkitaan. Sekin on mahdollista, ettei sitä sillä tavalla ole olemassa kuin arkikokemus ilmentää.

        "Tämä on hyvin selkeä tosiasia jota on mahdotonta kumota vaikka viimeiset pari sataaa vuotta on tieteessä melko kuumeisesti yritetty todistella tietoisuutta olemattomaksi tai jostain ei-tietoisesta johtuvaksi."

        Jaa, vai niin. Tietoisuudelle on yritetty löytää selitystä, mikä on tieteen tehtävä.

        "Tämä unimaailma on usein niin valvetilassa koetun kaltaista että siinä unessa on usein mahdotonta tietää kummasta on kyse."

        Väärin. Unet ovat usein sellaisia, että herättyään ihminen heti tajuaa, että uni ei voinut olla totta. Unessa ollessaan unennäkijä sen sijaan usein kokee unen toden tuntuisena. Siihen vaikuttaa unen näkemiseen liittyvät prosessit.


      • Belisario
        iloinenpiereskelijä kirjoitti:

        "Kaikki mahdollinen tiede, tekniikka, filosofia ja matematiikka on aina tietoisuuden tuottamaa.

        Tietoisuus on ainoa asia joka on jo annettuna olemassa."

        Väärin ja väärin.

        Tiede ei edellytä tietoisuutta. Sitä ei tieto-opissa tarvita lainkaan. Tietoisuus ei myöskään ole mitenkään "annettuna olemassa". Sen ilmentymistapoja tutkitaan. Sekin on mahdollista, ettei sitä sillä tavalla ole olemassa kuin arkikokemus ilmentää.

        "Tämä on hyvin selkeä tosiasia jota on mahdotonta kumota vaikka viimeiset pari sataaa vuotta on tieteessä melko kuumeisesti yritetty todistella tietoisuutta olemattomaksi tai jostain ei-tietoisesta johtuvaksi."

        Jaa, vai niin. Tietoisuudelle on yritetty löytää selitystä, mikä on tieteen tehtävä.

        "Tämä unimaailma on usein niin valvetilassa koetun kaltaista että siinä unessa on usein mahdotonta tietää kummasta on kyse."

        Väärin. Unet ovat usein sellaisia, että herättyään ihminen heti tajuaa, että uni ei voinut olla totta. Unessa ollessaan unennäkijä sen sijaan usein kokee unen toden tuntuisena. Siihen vaikuttaa unen näkemiseen liittyvät prosessit.

        "Tiede ei edellytä tietoisuutta. Sitä ei tieto-opissa tarvita lainkaan."

        Heh. Pistetään koko tiedeyhteisö nukutukseen ja katsotaan sitten miten se tieteenteko onnistuu. Tieto-opissa sitä ei varmaan tarvita koska tietoisuus on jo niin alkeellinen perusedellytys että useimmille sitä ei tarvitse erikseen mainita.

        "Tietoisuus ei myöskään ole mitenkään "annettuna olemassa"."

        Väitätkö että itselläsi ei ole minkäänlaista kokemuksellista tietoisuutta? Oletko robotti?

        "Sen ilmentymistapoja tutkitaan. Sekin on mahdollista, ettei sitä sillä tavalla ole olemassa kuin arkikokemus ilmentää."

        Tietoisuuden tai oikeammin tietoiseksi oletetun käyttäytymisen tutkiminen on aina eri asia kuin kokemuksellinen tietoisuus joka on aina ensimmäisen persoonan kokemus.

        "Tietoisuudelle on yritetty löytää selitystä, mikä on tieteen tehtävä."

        Tiede on aina 3. persoonan ulkopuolinen näkökulma. Useimmille länsimaisille ihmisille se oma kokemuksellinen tietoisuus on aika vieras asia kun tarkkaavaisuus on koko ajan suuntautunut ulospäin.

        Tietoisuutta voi kyllä tutkia mutta se edellyttää kunkin oman tietoisuuden kehittämistä esim. jonkinlaisen meditaation avulla. Kaikissa muissa tapauksissa oikeasti tutkitaan vain yksilön tai ryhmän käyttäytymistä sekä hermoston & aivojen korrelaatioita suhteessa siihen 1. persoonan kokemukseen ja introspektioon.

        "Väärin. Unet ovat usein sellaisia, että herättyään ihminen heti tajuaa, että uni ei voinut olla totta."

        Niin huomaa vasta herättyään että kokemuksen "datastream" muuttui toiseksi mutta se ei sinänsä tee sitä unimaailma yhtään vähemmän todelliseksi kuin valveilla olon maailmaa. Ne ovat erilaisia maailmoja erilaisilla sääntökokoelmilla. Usemmat ihmiset voivat nähdä samaa unta - sellaisia tapauksia on useita ja ne lienevät jopa suhteellisen yleisiä.


      • iloinenpiereskelijä
        Belisario kirjoitti:

        "Tiede ei edellytä tietoisuutta. Sitä ei tieto-opissa tarvita lainkaan."

        Heh. Pistetään koko tiedeyhteisö nukutukseen ja katsotaan sitten miten se tieteenteko onnistuu. Tieto-opissa sitä ei varmaan tarvita koska tietoisuus on jo niin alkeellinen perusedellytys että useimmille sitä ei tarvitse erikseen mainita.

        "Tietoisuus ei myöskään ole mitenkään "annettuna olemassa"."

        Väitätkö että itselläsi ei ole minkäänlaista kokemuksellista tietoisuutta? Oletko robotti?

        "Sen ilmentymistapoja tutkitaan. Sekin on mahdollista, ettei sitä sillä tavalla ole olemassa kuin arkikokemus ilmentää."

        Tietoisuuden tai oikeammin tietoiseksi oletetun käyttäytymisen tutkiminen on aina eri asia kuin kokemuksellinen tietoisuus joka on aina ensimmäisen persoonan kokemus.

        "Tietoisuudelle on yritetty löytää selitystä, mikä on tieteen tehtävä."

        Tiede on aina 3. persoonan ulkopuolinen näkökulma. Useimmille länsimaisille ihmisille se oma kokemuksellinen tietoisuus on aika vieras asia kun tarkkaavaisuus on koko ajan suuntautunut ulospäin.

        Tietoisuutta voi kyllä tutkia mutta se edellyttää kunkin oman tietoisuuden kehittämistä esim. jonkinlaisen meditaation avulla. Kaikissa muissa tapauksissa oikeasti tutkitaan vain yksilön tai ryhmän käyttäytymistä sekä hermoston & aivojen korrelaatioita suhteessa siihen 1. persoonan kokemukseen ja introspektioon.

        "Väärin. Unet ovat usein sellaisia, että herättyään ihminen heti tajuaa, että uni ei voinut olla totta."

        Niin huomaa vasta herättyään että kokemuksen "datastream" muuttui toiseksi mutta se ei sinänsä tee sitä unimaailma yhtään vähemmän todelliseksi kuin valveilla olon maailmaa. Ne ovat erilaisia maailmoja erilaisilla sääntökokoelmilla. Usemmat ihmiset voivat nähdä samaa unta - sellaisia tapauksia on useita ja ne lienevät jopa suhteellisen yleisiä.

        "Pistetään koko tiedeyhteisö nukutukseen ja katsotaan sitten miten se tieteenteko onnistuu. Tieto-opissa sitä ei varmaan tarvita koska tietoisuus on jo niin alkeellinen perusedellytys että useimmille sitä ei tarvitse erikseen mainita."

        Höpsistä! Tiedettä voisivat tehdä tiedottomat robotitkin. Tiede on ensisijassa metodi tiedon hankintaan. Sen tekeminen ei edellytä edes ihmisiä.

        Tieto-oppi ei tutki tietoisuutta, joten sellaista ei edellytetä siinä. Tietoisuus on tarpeetonta olettaa tieto-opissa millään tavalla.

        "Väitätkö että itselläsi ei ole minkäänlaista kokemuksellista tietoisuutta?"

        Jokaisella on välitön kokemus tietoisuudestaan; niin myös minulla.

        "Tietoisuuden tai oikeammin tietoiseksi oletetun käyttäytymisen tutkiminen on aina eri asia kuin kokemuksellinen tietoisuus joka on aina ensimmäisen persoonan kokemus."

        Näin on. (Itsestään selvää)

        "Tiede on aina 3. persoonan ulkopuolinen näkökulma."

        No, sitähän mä tässä olen sinulle selittänyt. Näyttö jostakin on oltava julkisesti kritiikin kohteena. Ja siksipä juuri tieteen tekeminen ei edellytä tietoisuutta. Tieteen metodia voi käyttää (tiedoton) robottikin.

        "Tietoisuutta voi kyllä tutkia mutta se edellyttää kunkin oman tietoisuuden kehittämistä esim. jonkinlaisen meditaation avulla."

        Mainitsemasi ei ole tietoisuuden tieteellistä tutkimista. Viittaan edellä sanomaani.

        "Niin huomaa vasta herättyään että kokemuksen "datastream" muuttui toiseksi mutta se ei sinänsä tee sitä unimaailma yhtään vähemmän todelliseksi kuin valveilla olon maailmaa."

        Niin muuttui ja silloin huomaa, että uni ei ollut totta! Jos sekoitat unen ja todellisuuden, sinun realiteetin tajussasi on ongelmia. Suosittelen pikaista kääntymistä psykiatrin puheille.

        Vai samaa unta näkisi muutkin??? Nyt taisi päästä aivopieru. Huh huh kun haisee!

        Onneksi olen iloinen piereskelijä, niin ei tunne pahalta. ;)


      • Belisario
        iloinenpiereskelijä kirjoitti:

        "Pistetään koko tiedeyhteisö nukutukseen ja katsotaan sitten miten se tieteenteko onnistuu. Tieto-opissa sitä ei varmaan tarvita koska tietoisuus on jo niin alkeellinen perusedellytys että useimmille sitä ei tarvitse erikseen mainita."

        Höpsistä! Tiedettä voisivat tehdä tiedottomat robotitkin. Tiede on ensisijassa metodi tiedon hankintaan. Sen tekeminen ei edellytä edes ihmisiä.

        Tieto-oppi ei tutki tietoisuutta, joten sellaista ei edellytetä siinä. Tietoisuus on tarpeetonta olettaa tieto-opissa millään tavalla.

        "Väitätkö että itselläsi ei ole minkäänlaista kokemuksellista tietoisuutta?"

        Jokaisella on välitön kokemus tietoisuudestaan; niin myös minulla.

        "Tietoisuuden tai oikeammin tietoiseksi oletetun käyttäytymisen tutkiminen on aina eri asia kuin kokemuksellinen tietoisuus joka on aina ensimmäisen persoonan kokemus."

        Näin on. (Itsestään selvää)

        "Tiede on aina 3. persoonan ulkopuolinen näkökulma."

        No, sitähän mä tässä olen sinulle selittänyt. Näyttö jostakin on oltava julkisesti kritiikin kohteena. Ja siksipä juuri tieteen tekeminen ei edellytä tietoisuutta. Tieteen metodia voi käyttää (tiedoton) robottikin.

        "Tietoisuutta voi kyllä tutkia mutta se edellyttää kunkin oman tietoisuuden kehittämistä esim. jonkinlaisen meditaation avulla."

        Mainitsemasi ei ole tietoisuuden tieteellistä tutkimista. Viittaan edellä sanomaani.

        "Niin huomaa vasta herättyään että kokemuksen "datastream" muuttui toiseksi mutta se ei sinänsä tee sitä unimaailma yhtään vähemmän todelliseksi kuin valveilla olon maailmaa."

        Niin muuttui ja silloin huomaa, että uni ei ollut totta! Jos sekoitat unen ja todellisuuden, sinun realiteetin tajussasi on ongelmia. Suosittelen pikaista kääntymistä psykiatrin puheille.

        Vai samaa unta näkisi muutkin??? Nyt taisi päästä aivopieru. Huh huh kun haisee!

        Onneksi olen iloinen piereskelijä, niin ei tunne pahalta. ;)

        "Tiedettä voisivat tehdä tiedottomat robotitkin. Tiede on ensisijassa metodi tiedon hankintaan. Sen tekeminen ei edellytä edes ihmisiä."

        Ei se ihmistä edellytä mutta sen havaitsemisen, uskomisen ja kokemisen subjektia eli tietoisuutta edellyttää. Robotit ovat tietoisuuden kehittämiä ja kelpaavat pelkästään tieteen apuvälineiksi kuten kaukoputket ja tietokoneet.

        Käypä lukemassa täältä tieto-opista:

        https://fi.wikipedia.org/wiki/Tietoteoria

        Kyllä se tietoisuus oletetaan tietoteoriassa implisiittisesti vaikka sitä "tietoisuus" käsitettä ei erikseen mainita.

        "Näyttö jostakin on oltava julkisesti kritiikin kohteena. Ja siksipä juuri tieteen tekeminen ei edellytä tietoisuutta."

        Et taida ymmärtää että tietoisuus kaiken mahdollisen tiedon ja tieteen subjektina ei voi olla edes tieteen objekti ainakaan sillä tavalla että se olisi julkista tietoa samalla tavalla kuin esim. fysiikan tai kemian tieto.

        "Mainitsemasi ei ole tietoisuuden tieteellistä tutkimista."

        Tieteen metodilla voi tutkia vain käyttäytymistä mikä ei ole sama asia kuin kokeminen.

        Tietoisuuden subjektiiviset tutkimismetodit ovat kuitenkin ainoita mahdollisia kun halutaan tutkia sitä kvaliaa eli kokemuksellisuutta eli 1. persoonan kokemusta. Sen subjektiivisen komponentin voi kyllä sitten yrittää yhdistää niihin tieteen metodilla saavutettuihin 3. persoonan käyttäytymiseen liittyvään tietoon.

        "Jos sekoitat unen ja todellisuuden, sinun realiteetin tajussasi on ongelmia. Suosittelen pikaista kääntymistä psykiatrin puheille."

        Minusta kyse on kahdesta erilaisesta todellisuudesta joita itse asiassa on lukemattomia jos otetaan mukaan kaikki harvinaisemmat tietoisuuden ilmiöt.

        Ei niitä sekoita helposti toisiinsa muuta kuin silloin kun on uppoutunut johonkin niistä. Myös unessa voi huomata olevansa unessa ja silloin jotkut pystyvät jopa kontrolloimaan ja toimimaan tietoisesti niissä unissa.

        Ns. "tavallinen" valvetila on sekin useimmilla aika kaukana todellisuudesta ainakin jos teikäläisten juttuja kuuntelee. Pelkkiä mielikuvia, uskomuksia ja asenteita on ns, normi-ihmisen kokemusmaailma täynnä.

        Jos pystyy kontrolloimaan ihmisten enemmistön uskomuksia niin samalla kontrolloi heidän todellisuuttaan joka niistä uskomuksista vähitellen viiveellä ilmaantuu siihen fysikaaliseen todellisuuteen.

        "Vai samaa unta näkisi muutkin??"

        Jos jokin asia on sinulle vieras ja outo niin siitä ei seuraa että se ei voisi pitää paikkaansa.

        Olipas taas runsaasti aivopieruja sinulta. Nikkisi on kyllä hyvin osuva.... :-)


      • Järkisyitä
        Belisario kirjoitti:

        "Tiede ei edellytä tietoisuutta. Sitä ei tieto-opissa tarvita lainkaan."

        Heh. Pistetään koko tiedeyhteisö nukutukseen ja katsotaan sitten miten se tieteenteko onnistuu. Tieto-opissa sitä ei varmaan tarvita koska tietoisuus on jo niin alkeellinen perusedellytys että useimmille sitä ei tarvitse erikseen mainita.

        "Tietoisuus ei myöskään ole mitenkään "annettuna olemassa"."

        Väitätkö että itselläsi ei ole minkäänlaista kokemuksellista tietoisuutta? Oletko robotti?

        "Sen ilmentymistapoja tutkitaan. Sekin on mahdollista, ettei sitä sillä tavalla ole olemassa kuin arkikokemus ilmentää."

        Tietoisuuden tai oikeammin tietoiseksi oletetun käyttäytymisen tutkiminen on aina eri asia kuin kokemuksellinen tietoisuus joka on aina ensimmäisen persoonan kokemus.

        "Tietoisuudelle on yritetty löytää selitystä, mikä on tieteen tehtävä."

        Tiede on aina 3. persoonan ulkopuolinen näkökulma. Useimmille länsimaisille ihmisille se oma kokemuksellinen tietoisuus on aika vieras asia kun tarkkaavaisuus on koko ajan suuntautunut ulospäin.

        Tietoisuutta voi kyllä tutkia mutta se edellyttää kunkin oman tietoisuuden kehittämistä esim. jonkinlaisen meditaation avulla. Kaikissa muissa tapauksissa oikeasti tutkitaan vain yksilön tai ryhmän käyttäytymistä sekä hermoston & aivojen korrelaatioita suhteessa siihen 1. persoonan kokemukseen ja introspektioon.

        "Väärin. Unet ovat usein sellaisia, että herättyään ihminen heti tajuaa, että uni ei voinut olla totta."

        Niin huomaa vasta herättyään että kokemuksen "datastream" muuttui toiseksi mutta se ei sinänsä tee sitä unimaailma yhtään vähemmän todelliseksi kuin valveilla olon maailmaa. Ne ovat erilaisia maailmoja erilaisilla sääntökokoelmilla. Usemmat ihmiset voivat nähdä samaa unta - sellaisia tapauksia on useita ja ne lienevät jopa suhteellisen yleisiä.

        Unimaailma on itseasiassa hyvä esimerkki. Unimaailma muistuttaa mielensisäistä maailmaa. Siitä puuttuu arkimaailman systemaattinen toiminta sekä siinä ei pysty hankkimaan tietoa, joka ei olisi mahdollista saavuttaa itsenäisesti päättelemällä. Ota unimaailmassa fysiikankirja ja yritä lukea uusista fysiikanteorioista ja huomaat eron realimaailmaan, missä voit lukea itseäsi fiksumpien tutkijoiden tuloksia.


      • iloinenpiereskelijä
        Belisario kirjoitti:

        "Tiedettä voisivat tehdä tiedottomat robotitkin. Tiede on ensisijassa metodi tiedon hankintaan. Sen tekeminen ei edellytä edes ihmisiä."

        Ei se ihmistä edellytä mutta sen havaitsemisen, uskomisen ja kokemisen subjektia eli tietoisuutta edellyttää. Robotit ovat tietoisuuden kehittämiä ja kelpaavat pelkästään tieteen apuvälineiksi kuten kaukoputket ja tietokoneet.

        Käypä lukemassa täältä tieto-opista:

        https://fi.wikipedia.org/wiki/Tietoteoria

        Kyllä se tietoisuus oletetaan tietoteoriassa implisiittisesti vaikka sitä "tietoisuus" käsitettä ei erikseen mainita.

        "Näyttö jostakin on oltava julkisesti kritiikin kohteena. Ja siksipä juuri tieteen tekeminen ei edellytä tietoisuutta."

        Et taida ymmärtää että tietoisuus kaiken mahdollisen tiedon ja tieteen subjektina ei voi olla edes tieteen objekti ainakaan sillä tavalla että se olisi julkista tietoa samalla tavalla kuin esim. fysiikan tai kemian tieto.

        "Mainitsemasi ei ole tietoisuuden tieteellistä tutkimista."

        Tieteen metodilla voi tutkia vain käyttäytymistä mikä ei ole sama asia kuin kokeminen.

        Tietoisuuden subjektiiviset tutkimismetodit ovat kuitenkin ainoita mahdollisia kun halutaan tutkia sitä kvaliaa eli kokemuksellisuutta eli 1. persoonan kokemusta. Sen subjektiivisen komponentin voi kyllä sitten yrittää yhdistää niihin tieteen metodilla saavutettuihin 3. persoonan käyttäytymiseen liittyvään tietoon.

        "Jos sekoitat unen ja todellisuuden, sinun realiteetin tajussasi on ongelmia. Suosittelen pikaista kääntymistä psykiatrin puheille."

        Minusta kyse on kahdesta erilaisesta todellisuudesta joita itse asiassa on lukemattomia jos otetaan mukaan kaikki harvinaisemmat tietoisuuden ilmiöt.

        Ei niitä sekoita helposti toisiinsa muuta kuin silloin kun on uppoutunut johonkin niistä. Myös unessa voi huomata olevansa unessa ja silloin jotkut pystyvät jopa kontrolloimaan ja toimimaan tietoisesti niissä unissa.

        Ns. "tavallinen" valvetila on sekin useimmilla aika kaukana todellisuudesta ainakin jos teikäläisten juttuja kuuntelee. Pelkkiä mielikuvia, uskomuksia ja asenteita on ns, normi-ihmisen kokemusmaailma täynnä.

        Jos pystyy kontrolloimaan ihmisten enemmistön uskomuksia niin samalla kontrolloi heidän todellisuuttaan joka niistä uskomuksista vähitellen viiveellä ilmaantuu siihen fysikaaliseen todellisuuteen.

        "Vai samaa unta näkisi muutkin??"

        Jos jokin asia on sinulle vieras ja outo niin siitä ei seuraa että se ei voisi pitää paikkaansa.

        Olipas taas runsaasti aivopieruja sinulta. Nikkisi on kyllä hyvin osuva.... :-)

        Melkoista idealistista soopaahan sieltä tuli, kuten arvata saattoi.

        En käy kiistelemään tieto-opista, koska tulkitset senkin idealismin laseillasi päin helv*ttiä.

        "Kyllä se tietoisuus oletetaan tietoteoriassa implisiittisesti vaikka sitä "tietoisuus" käsitettä ei erikseen mainita."

        Tieto-oppi koskee tiedon hankintaa ja sitä, mitä näin on saatu kerätyksi. Näin yksinkertaista se on. Tietoisuus on ihan muuta.

        "Et taida ymmärtää että tietoisuus kaiken mahdollisen tiedon ja tieteen subjektina ei voi olla edes tieteen objekti ainakaan sillä tavalla että se olisi julkista tietoa ..."

        Kyllä se voi olla. Tieteen metodilla ei ole tässä mielessä rajoja.

        "Tietoisuuden subjektiiviset tutkimismetodit ovat kuitenkin ainoita mahdollisia kun halutaan tutkia sitä kvaliaa eli kokemuksellisuutta eli 1. persoonan kokemusta."

        Väärin.

        Kokemuksista voidaan tehdä kysely niiltä tietoisilta olioilta ja tutkia näiden kokemusten luonnetta ja syntytapaa. Voidaan löytää korrelaatioita aivojen toiminnan ja kokemusten välille.

        "Jos pystyy kontrolloimaan ihmisten enemmistön uskomuksia niin samalla kontrolloi heidän todellisuuttaan..."

        Ja myös toisinpäin. Jos pystyt kontrolloimaan ihmisen aivotoimintoja niin samalla kontrolloit ihmisen kokemusta. Tästä on mainio esimerkki koehenkilön aivojen erään alueen stimulointi, jolloin koehenkilö koki itse liikuttavansa raajaansa, vaikka todellisuudessa raaja liikkui stimuloinnin vuoksi.

        Minunkin mielestäni nimimerkkini on erinomaisesti valittu. Antaa ajattelemisen aihetta itse kullekin!


      • Belisario
        Järkisyitä kirjoitti:

        Unimaailma on itseasiassa hyvä esimerkki. Unimaailma muistuttaa mielensisäistä maailmaa. Siitä puuttuu arkimaailman systemaattinen toiminta sekä siinä ei pysty hankkimaan tietoa, joka ei olisi mahdollista saavuttaa itsenäisesti päättelemällä. Ota unimaailmassa fysiikankirja ja yritä lukea uusista fysiikanteorioista ja huomaat eron realimaailmaan, missä voit lukea itseäsi fiksumpien tutkijoiden tuloksia.

        "Ota unimaailmassa fysiikankirja ja yritä lukea uusista fysiikanteorioista ja huomaat eron realimaailmaan, missä voit lukea itseäsi fiksumpien tutkijoiden tuloksia."

        Fysiikan kirjan lukeminen unimaailmassa on haaskausta.

        Unessa alitajunta toimii parhaiten ja voi tuottaa parhaat ideat kun tietoinen arkitajunta (ja sen analytical overlay) ei ole sotkemassa ja jumittamassa. LSD:n ja muiden kemikalien käytöllä lienee vastaavia ominaisuuksia.

        Tietenkin pitää se perusymmärrys ja tieto ko. aiheesta olla ensin jotta se intuitio unessa toimisi hyvin. Nukkumaan mennessä voi asettaa itselleen konkreettisen ja koherentin intention sitä uniprosessointia varten. Kuulemma toimii myös ns. selkounissa (lucid dreams) sekä OBE-kokemusten suhteen.

        .....

        piereskelijä kirjoitti:

        "Jos pystyt kontrolloimaan ihmisen aivotoimintoja niin samalla kontrolloit ihmisen kokemusta. Tästä on mainio esimerkki koehenkilön aivojen erään alueen stimulointi, jolloin koehenkilö koki itse liikuttavansa raajaansa, vaikka todellisuudessa raaja liikkui stimuloinnin vuoksi."

        Niin koska aivotoiminta on se miltä lokalisoitunut tietoisuus näyttää 3. persoonan näkökulmasta. Onhan tuota mielen kontrollointia aivoja manipuloimalla tutkittu hyvin kauan.

        Hypnoosi on yksi tehokas keino vaikuttaa mielen kautta ja on olemassa sitten myös kemiallisia ja sähkömagneettisia keinoja sekä erilaisten stressitilojen tuottamista kuten unen estämistä ja tauotonta tietyn musiikin tai puheen toistamista jne, Natsi-Saksassa tuo tutkimus oli aika pitkälle koska koehenkilöitä (keskitysleirivankeja) oli käytettävissä lähes rajattomasti. Neuvostoliitossa tutkittiin samoja asioita sekä USA:ssa mm. MK-Ultra ja vastaavat projektit.

        Jo renesanssiaikana oli musiikissa affekti-teoria jolla laskelmoidusti pyrittiin vaikuttamaan kuulijan tunnetilaan ja vastaavaa tietysti on kaikissa uskonnoissa joissa luodaan tietty hypnoosille otollinen tunnelma (ambience) arkkitehtuurilla, rituaaleilla ja musiikilla.


        Et tajua että samat asiat voi yhtä hyvin hahmottaa sekä materialismin että idealismin kautta ja oikeastaan vain se asenne siihen koettuun todellisuuteen on näissä aika erilainen. Materialismi on 3. persoonan näkökulma ja idealismi taas 1. persoonan näkäkulma ja aistihavainnot kuuluvat nekin siihen 1. persoonan näkökulmaan.

        Idealismi nojautuu enemmän suoraan kokemuksellisuuteen kun taas materialismissa on vahva ideologinen painolasti joka on pohjimmiltaan täysin tarpeeton koska ne samat asiat voi käsitellä myös neutraalin ontologian kannalta.

        Materialisteilla on pakkomielteinen tarve todistella tietoisuus jotenkin epäolennaiseksi ja se ilmeisesti johtuu aika lapsellisesta vastareaktiosta siihen koettuun kristilliseen kasvatukseen ja manipulaatioon mitä Suomessa ja monessa muussakin maassa on kärsitty vuosisatoja.

        Ateismia nykyisessä muodossaan ei olisi edes olemassa ilman teismiä ja monet tiedemiehet ovatkin rasittavan kiinnostuneita hyökkäämään uskontoja vastaan. 6000 v vanha maapallo on tietenkin idioottimainen idea mutta ainakaan omien tutkimuksieni mukaan sillä ei ole mitään tekemistä alkup, kristinuskon kanssa. Itse asiassa Isaac Newton ja Kepler olivat niitä jotka laskeskelivat maapallon syntyneen n. 4000 ekR.

        Itse en ole koskaan kuulunut mihinkään uskontokuntaan ja erosin koulussa uskonnonopetuksesta heti kuin se tuli mahdolliseksi. Ns. Jumalan suhteen olen agnostikko eli jätän asian avoimeksi ja oma jumalakuvani varmaan muutenkin poikkeaisi hyvin paljon länsimaisesta kristillisestä teismistä.

        Minulle on itsestään selvää että tietoisuus on kaikessa ensisijainen mutta jos se yksilön tietoisuus on hyvin kehittymätön ja surkastunut niin materialismi voi houkutella enemmän.

        Itse muuten luen parhaillaan kirjaa mielen manipulaatiosta eri menetelmin ja niiden menetelmien historiaa.


      • iloinenpiereskelijä
        Belisario kirjoitti:

        "Ota unimaailmassa fysiikankirja ja yritä lukea uusista fysiikanteorioista ja huomaat eron realimaailmaan, missä voit lukea itseäsi fiksumpien tutkijoiden tuloksia."

        Fysiikan kirjan lukeminen unimaailmassa on haaskausta.

        Unessa alitajunta toimii parhaiten ja voi tuottaa parhaat ideat kun tietoinen arkitajunta (ja sen analytical overlay) ei ole sotkemassa ja jumittamassa. LSD:n ja muiden kemikalien käytöllä lienee vastaavia ominaisuuksia.

        Tietenkin pitää se perusymmärrys ja tieto ko. aiheesta olla ensin jotta se intuitio unessa toimisi hyvin. Nukkumaan mennessä voi asettaa itselleen konkreettisen ja koherentin intention sitä uniprosessointia varten. Kuulemma toimii myös ns. selkounissa (lucid dreams) sekä OBE-kokemusten suhteen.

        .....

        piereskelijä kirjoitti:

        "Jos pystyt kontrolloimaan ihmisen aivotoimintoja niin samalla kontrolloit ihmisen kokemusta. Tästä on mainio esimerkki koehenkilön aivojen erään alueen stimulointi, jolloin koehenkilö koki itse liikuttavansa raajaansa, vaikka todellisuudessa raaja liikkui stimuloinnin vuoksi."

        Niin koska aivotoiminta on se miltä lokalisoitunut tietoisuus näyttää 3. persoonan näkökulmasta. Onhan tuota mielen kontrollointia aivoja manipuloimalla tutkittu hyvin kauan.

        Hypnoosi on yksi tehokas keino vaikuttaa mielen kautta ja on olemassa sitten myös kemiallisia ja sähkömagneettisia keinoja sekä erilaisten stressitilojen tuottamista kuten unen estämistä ja tauotonta tietyn musiikin tai puheen toistamista jne, Natsi-Saksassa tuo tutkimus oli aika pitkälle koska koehenkilöitä (keskitysleirivankeja) oli käytettävissä lähes rajattomasti. Neuvostoliitossa tutkittiin samoja asioita sekä USA:ssa mm. MK-Ultra ja vastaavat projektit.

        Jo renesanssiaikana oli musiikissa affekti-teoria jolla laskelmoidusti pyrittiin vaikuttamaan kuulijan tunnetilaan ja vastaavaa tietysti on kaikissa uskonnoissa joissa luodaan tietty hypnoosille otollinen tunnelma (ambience) arkkitehtuurilla, rituaaleilla ja musiikilla.


        Et tajua että samat asiat voi yhtä hyvin hahmottaa sekä materialismin että idealismin kautta ja oikeastaan vain se asenne siihen koettuun todellisuuteen on näissä aika erilainen. Materialismi on 3. persoonan näkökulma ja idealismi taas 1. persoonan näkäkulma ja aistihavainnot kuuluvat nekin siihen 1. persoonan näkökulmaan.

        Idealismi nojautuu enemmän suoraan kokemuksellisuuteen kun taas materialismissa on vahva ideologinen painolasti joka on pohjimmiltaan täysin tarpeeton koska ne samat asiat voi käsitellä myös neutraalin ontologian kannalta.

        Materialisteilla on pakkomielteinen tarve todistella tietoisuus jotenkin epäolennaiseksi ja se ilmeisesti johtuu aika lapsellisesta vastareaktiosta siihen koettuun kristilliseen kasvatukseen ja manipulaatioon mitä Suomessa ja monessa muussakin maassa on kärsitty vuosisatoja.

        Ateismia nykyisessä muodossaan ei olisi edes olemassa ilman teismiä ja monet tiedemiehet ovatkin rasittavan kiinnostuneita hyökkäämään uskontoja vastaan. 6000 v vanha maapallo on tietenkin idioottimainen idea mutta ainakaan omien tutkimuksieni mukaan sillä ei ole mitään tekemistä alkup, kristinuskon kanssa. Itse asiassa Isaac Newton ja Kepler olivat niitä jotka laskeskelivat maapallon syntyneen n. 4000 ekR.

        Itse en ole koskaan kuulunut mihinkään uskontokuntaan ja erosin koulussa uskonnonopetuksesta heti kuin se tuli mahdolliseksi. Ns. Jumalan suhteen olen agnostikko eli jätän asian avoimeksi ja oma jumalakuvani varmaan muutenkin poikkeaisi hyvin paljon länsimaisesta kristillisestä teismistä.

        Minulle on itsestään selvää että tietoisuus on kaikessa ensisijainen mutta jos se yksilön tietoisuus on hyvin kehittymätön ja surkastunut niin materialismi voi houkutella enemmän.

        Itse muuten luen parhaillaan kirjaa mielen manipulaatiosta eri menetelmin ja niiden menetelmien historiaa.

        "Niin koska aivotoiminta on se miltä lokalisoitunut tietoisuus näyttää 3. persoonan näkökulmasta."

        Se ei ollut vain 3. persoonan näkemys, koska koehenilö kertoi 1. pers näkemyksen tapahtuneeseen.

        Koehenkilöllä oli "aito" tietoinen kokemus 1. persoonassaan, mutta se oli synnytetty mielen ulkopuolelta stimuloimalla.

        "Et tajua että samat asiat voi yhtä hyvin hahmottaa sekä materialismin että idealismin kautta ja oikeastaan vain se asenne siihen koettuun todellisuuteen on näissä aika erilainen."

        Sinä et tajua, että idealismi on jo kumottu em koejärjestelyin.

        "Idealismi nojautuu enemmän suoraan kokemuksellisuuteen kun taas materialismissa on vahva ideologinen painolasti joka on pohjimmiltaan täysin tarpeeton koska ne samat asiat voi käsitellä myös neutraalin ontologian kannalta."

        Tämä on päinvastoin. Idealismi nojautuu ideologiseen painolastiin, josta se pitää dogmaattisesti kiinni. Idealismi on perimmäisissä kysymyksissä täysin väärässä ja siten täysin tarpeeton. Occamilla leikeltävissä huisin v***uun.

        "Materialisteilla on pakkomielteinen tarve todistella tietoisuus jotenkin epäolennaiseksi ja se ilmeisesti johtuu aika lapsellisesta vastareaktiosta siihen koettuun kristilliseen kasvatukseen ja manipulaatioon mitä Suomessa ja monessa muussakin maassa on kärsitty vuosisatoja."

        Ei påidä paikkaansa. Koejärjestelyin tutkitaan tietoisuuden toimintaa. Jos tulokset eivät idealistia miellytä, en voi mitään. Idealisteilla on usein lapsellinen denialismi koetuloksien suhteen. Kasvakaa ihmisinä ja meditoikaa; kyllä se siitä helpottaa. Ei tietoisuus kokemuksena katoa, vaikka sitä mieltä voikin manipuloida. Se on vain sitä ideologista painolastianne. Luopukaa siitä ja saavuttakaa "korkeampi tietoisuuden taso".

        "Itse asiassa Isaac Newton ja Kepler olivat niitä jotka laskeskelivat maapallon syntyneen n. 4000 ekR."

        Mainitut herrat olivat uskovaisia deistejä. Kepler tutki luonto hartaudella yrittäen löytää jumalan tekojen tarkoituksen niistä. Melkoisia hörhöjä siis molemmat!!

        Mutta päätyivät siltikin tieteen teossaan totuutta kohti. Ehkä ihmiskunnalla on tosiaan toivoa vielä. Pitää vain hyväksyä tutkimustulokset ja luopua painolastista.

        "Itse en ole koskaan kuulunut mihinkään uskontokuntaan ja erosin koulussa uskonnonopetuksesta heti kuin se tuli mahdolliseksi."

        En minäkään ja lintsasin jatkuvasti lukiossa uskonnontunnit.

        Tiede on agnostista.

        "Minulle on itsestään selvää että tietoisuus on kaikessa ensisijainen mutta jos se yksilön tietoisuus on hyvin kehittymätön ja surkastunut niin materialismi voi houkutella enemmän."

        Monilla idealisteilla on kysymys narsimista. Heidän oma ego paisuu liiallisuuksiin ja siitä tehdään maailmankuvan kulmakivi. Pitäisi vain tunnustaa todellisuus eikä elätellä sairaita ja dogmaattisia mielikuvia.

        Itse luen kirjaa, jossa yritetään saada kokonaiskuva maailmasta.


      • Belisario
        iloinenpiereskelijä kirjoitti:

        "Niin koska aivotoiminta on se miltä lokalisoitunut tietoisuus näyttää 3. persoonan näkökulmasta."

        Se ei ollut vain 3. persoonan näkemys, koska koehenilö kertoi 1. pers näkemyksen tapahtuneeseen.

        Koehenkilöllä oli "aito" tietoinen kokemus 1. persoonassaan, mutta se oli synnytetty mielen ulkopuolelta stimuloimalla.

        "Et tajua että samat asiat voi yhtä hyvin hahmottaa sekä materialismin että idealismin kautta ja oikeastaan vain se asenne siihen koettuun todellisuuteen on näissä aika erilainen."

        Sinä et tajua, että idealismi on jo kumottu em koejärjestelyin.

        "Idealismi nojautuu enemmän suoraan kokemuksellisuuteen kun taas materialismissa on vahva ideologinen painolasti joka on pohjimmiltaan täysin tarpeeton koska ne samat asiat voi käsitellä myös neutraalin ontologian kannalta."

        Tämä on päinvastoin. Idealismi nojautuu ideologiseen painolastiin, josta se pitää dogmaattisesti kiinni. Idealismi on perimmäisissä kysymyksissä täysin väärässä ja siten täysin tarpeeton. Occamilla leikeltävissä huisin v***uun.

        "Materialisteilla on pakkomielteinen tarve todistella tietoisuus jotenkin epäolennaiseksi ja se ilmeisesti johtuu aika lapsellisesta vastareaktiosta siihen koettuun kristilliseen kasvatukseen ja manipulaatioon mitä Suomessa ja monessa muussakin maassa on kärsitty vuosisatoja."

        Ei påidä paikkaansa. Koejärjestelyin tutkitaan tietoisuuden toimintaa. Jos tulokset eivät idealistia miellytä, en voi mitään. Idealisteilla on usein lapsellinen denialismi koetuloksien suhteen. Kasvakaa ihmisinä ja meditoikaa; kyllä se siitä helpottaa. Ei tietoisuus kokemuksena katoa, vaikka sitä mieltä voikin manipuloida. Se on vain sitä ideologista painolastianne. Luopukaa siitä ja saavuttakaa "korkeampi tietoisuuden taso".

        "Itse asiassa Isaac Newton ja Kepler olivat niitä jotka laskeskelivat maapallon syntyneen n. 4000 ekR."

        Mainitut herrat olivat uskovaisia deistejä. Kepler tutki luonto hartaudella yrittäen löytää jumalan tekojen tarkoituksen niistä. Melkoisia hörhöjä siis molemmat!!

        Mutta päätyivät siltikin tieteen teossaan totuutta kohti. Ehkä ihmiskunnalla on tosiaan toivoa vielä. Pitää vain hyväksyä tutkimustulokset ja luopua painolastista.

        "Itse en ole koskaan kuulunut mihinkään uskontokuntaan ja erosin koulussa uskonnonopetuksesta heti kuin se tuli mahdolliseksi."

        En minäkään ja lintsasin jatkuvasti lukiossa uskonnontunnit.

        Tiede on agnostista.

        "Minulle on itsestään selvää että tietoisuus on kaikessa ensisijainen mutta jos se yksilön tietoisuus on hyvin kehittymätön ja surkastunut niin materialismi voi houkutella enemmän."

        Monilla idealisteilla on kysymys narsimista. Heidän oma ego paisuu liiallisuuksiin ja siitä tehdään maailmankuvan kulmakivi. Pitäisi vain tunnustaa todellisuus eikä elätellä sairaita ja dogmaattisia mielikuvia.

        Itse luen kirjaa, jossa yritetään saada kokonaiskuva maailmasta.

        "Se ei ollut vain 3. persoonan näkemys, koska koehenilö kertoi 1. pers näkemyksen tapahtuneeseen."

        Kielellinen kokemuksen kuvaus on aivan eri asia kuin se kokemus. Kokemuksista voi puhua vain lähinnä vertauskuvin.


        "Sinä et tajua, että idealismi on jo kumottu em koejärjestelyin."

        Kuten esimerkiksi.....

        Jos nyt ajatellaan esim. matematiikkaa niin se perustuu nimenomaan ideoihin ja niiden loogisiin suhteisiin eikä matematiikalla ole mitään välttämätöntä suhdetta aistikokemukseen. Koska tieteen kieli on matemaattista ja loogista niin siinä ei ole mitään materiaan liittyvä.

        Ylipäätänsä "materia" tai ns. "aine" on tyhjä käsite joka liittyy ainoastaan aistikokemuksessa esiintyviin säännönmukaisuuksiiin.


        "Monilla idealisteilla on kysymys narsimista. Heidän oma ego paisuu liiallisuuksiin ja siitä tehdään maailmankuvan kulmakivi. Pitäisi vain tunnustaa todellisuus eikä elätellä sairaita ja dogmaattisia mielikuvia."

        Tuo pitää kyllä paikkansa ja sopii hyvin erilaisten uskonnollisten lahkojen perustajiin (kuten esim. mormonien Joseph Smith joka kaiken muun sekoilun lisäksi sotkeutui mm. pankkihuijaukseen yli sata vuotta ennen nykyisen finanssikapitalismin kuplia. )

        Meditaatiossa ja tietoisuuden kehittämisessä on omat vaaransa mutta kyllä se materialismikin nostaa helposti keltaisen nesteen päähän varsinkin kun tarkkailee touhukkaita miljardöörejä kuten Rockefellerit, Bill Gates ym. busybodies sotkemassa maailmaa "loistavilla" ideoillaan ja "yes" porukka vain nyökkäilee.

        Ihminen on lajityypillisesti erittäin taitava huijaamaan sekä muita että itseään mutta siihenkin vaivaan löytyy erilaisia toimivia lääkkeitä.

        .....

        Sinun keskustelutyylisi on aika reaktiivista kun et vaivaudu yhtään ajattelemaan.

        https://www.urbandictionary.com/define.php?term=kneejerk


      • iloinenpiereskelijä
        Belisario kirjoitti:

        "Se ei ollut vain 3. persoonan näkemys, koska koehenilö kertoi 1. pers näkemyksen tapahtuneeseen."

        Kielellinen kokemuksen kuvaus on aivan eri asia kuin se kokemus. Kokemuksista voi puhua vain lähinnä vertauskuvin.


        "Sinä et tajua, että idealismi on jo kumottu em koejärjestelyin."

        Kuten esimerkiksi.....

        Jos nyt ajatellaan esim. matematiikkaa niin se perustuu nimenomaan ideoihin ja niiden loogisiin suhteisiin eikä matematiikalla ole mitään välttämätöntä suhdetta aistikokemukseen. Koska tieteen kieli on matemaattista ja loogista niin siinä ei ole mitään materiaan liittyvä.

        Ylipäätänsä "materia" tai ns. "aine" on tyhjä käsite joka liittyy ainoastaan aistikokemuksessa esiintyviin säännönmukaisuuksiiin.


        "Monilla idealisteilla on kysymys narsimista. Heidän oma ego paisuu liiallisuuksiin ja siitä tehdään maailmankuvan kulmakivi. Pitäisi vain tunnustaa todellisuus eikä elätellä sairaita ja dogmaattisia mielikuvia."

        Tuo pitää kyllä paikkansa ja sopii hyvin erilaisten uskonnollisten lahkojen perustajiin (kuten esim. mormonien Joseph Smith joka kaiken muun sekoilun lisäksi sotkeutui mm. pankkihuijaukseen yli sata vuotta ennen nykyisen finanssikapitalismin kuplia. )

        Meditaatiossa ja tietoisuuden kehittämisessä on omat vaaransa mutta kyllä se materialismikin nostaa helposti keltaisen nesteen päähän varsinkin kun tarkkailee touhukkaita miljardöörejä kuten Rockefellerit, Bill Gates ym. busybodies sotkemassa maailmaa "loistavilla" ideoillaan ja "yes" porukka vain nyökkäilee.

        Ihminen on lajityypillisesti erittäin taitava huijaamaan sekä muita että itseään mutta siihenkin vaivaan löytyy erilaisia toimivia lääkkeitä.

        .....

        Sinun keskustelutyylisi on aika reaktiivista kun et vaivaudu yhtään ajattelemaan.

        https://www.urbandictionary.com/define.php?term=kneejerk

        "Kielellinen kokemuksen kuvaus on aivan eri asia kuin se kokemus."

        Kyllä, mutta mahdollistaa kokemuksien tutkimisen.

        "Jos nyt ajatellaan esim. matematiikkaa niin se perustuu nimenomaan ideoihin ja niiden loogisiin suhteisiin eikä matematiikalla ole mitään välttämätöntä suhdetta aistikokemukseen."

        Matematiikka on tautologiaa. Aksiomista johdettua, jossa lopputulos on jo perimmäisessä mielessä olemassa alusta alkaen. Sillä ei ole mitään kosketusta filosofisesti idealismiin.

        "Ylipäätänsä "materia" tai ns. "aine" on tyhjä käsite joka liittyy ainoastaan aistikokemuksessa esiintyviin säännönmukaisuuksiiin."

        Eli kun idealisti hyppää pirvenpiirtäjän katolta alas, häntä vain vähän sattuu, kun katu onkin vain "tyhjä käsite". Oman tietämykseni mukaan siinä tapahtuu idealistillekin paljon muuta. Voipi henki lähteä liihottelemaan! :D

        Jospa olen jo ajatellut ja syvällisemmin kuin sinä.


      • Järkisyitä
        Belisario kirjoitti:

        "Ota unimaailmassa fysiikankirja ja yritä lukea uusista fysiikanteorioista ja huomaat eron realimaailmaan, missä voit lukea itseäsi fiksumpien tutkijoiden tuloksia."

        Fysiikan kirjan lukeminen unimaailmassa on haaskausta.

        Unessa alitajunta toimii parhaiten ja voi tuottaa parhaat ideat kun tietoinen arkitajunta (ja sen analytical overlay) ei ole sotkemassa ja jumittamassa. LSD:n ja muiden kemikalien käytöllä lienee vastaavia ominaisuuksia.

        Tietenkin pitää se perusymmärrys ja tieto ko. aiheesta olla ensin jotta se intuitio unessa toimisi hyvin. Nukkumaan mennessä voi asettaa itselleen konkreettisen ja koherentin intention sitä uniprosessointia varten. Kuulemma toimii myös ns. selkounissa (lucid dreams) sekä OBE-kokemusten suhteen.

        .....

        piereskelijä kirjoitti:

        "Jos pystyt kontrolloimaan ihmisen aivotoimintoja niin samalla kontrolloit ihmisen kokemusta. Tästä on mainio esimerkki koehenkilön aivojen erään alueen stimulointi, jolloin koehenkilö koki itse liikuttavansa raajaansa, vaikka todellisuudessa raaja liikkui stimuloinnin vuoksi."

        Niin koska aivotoiminta on se miltä lokalisoitunut tietoisuus näyttää 3. persoonan näkökulmasta. Onhan tuota mielen kontrollointia aivoja manipuloimalla tutkittu hyvin kauan.

        Hypnoosi on yksi tehokas keino vaikuttaa mielen kautta ja on olemassa sitten myös kemiallisia ja sähkömagneettisia keinoja sekä erilaisten stressitilojen tuottamista kuten unen estämistä ja tauotonta tietyn musiikin tai puheen toistamista jne, Natsi-Saksassa tuo tutkimus oli aika pitkälle koska koehenkilöitä (keskitysleirivankeja) oli käytettävissä lähes rajattomasti. Neuvostoliitossa tutkittiin samoja asioita sekä USA:ssa mm. MK-Ultra ja vastaavat projektit.

        Jo renesanssiaikana oli musiikissa affekti-teoria jolla laskelmoidusti pyrittiin vaikuttamaan kuulijan tunnetilaan ja vastaavaa tietysti on kaikissa uskonnoissa joissa luodaan tietty hypnoosille otollinen tunnelma (ambience) arkkitehtuurilla, rituaaleilla ja musiikilla.


        Et tajua että samat asiat voi yhtä hyvin hahmottaa sekä materialismin että idealismin kautta ja oikeastaan vain se asenne siihen koettuun todellisuuteen on näissä aika erilainen. Materialismi on 3. persoonan näkökulma ja idealismi taas 1. persoonan näkäkulma ja aistihavainnot kuuluvat nekin siihen 1. persoonan näkökulmaan.

        Idealismi nojautuu enemmän suoraan kokemuksellisuuteen kun taas materialismissa on vahva ideologinen painolasti joka on pohjimmiltaan täysin tarpeeton koska ne samat asiat voi käsitellä myös neutraalin ontologian kannalta.

        Materialisteilla on pakkomielteinen tarve todistella tietoisuus jotenkin epäolennaiseksi ja se ilmeisesti johtuu aika lapsellisesta vastareaktiosta siihen koettuun kristilliseen kasvatukseen ja manipulaatioon mitä Suomessa ja monessa muussakin maassa on kärsitty vuosisatoja.

        Ateismia nykyisessä muodossaan ei olisi edes olemassa ilman teismiä ja monet tiedemiehet ovatkin rasittavan kiinnostuneita hyökkäämään uskontoja vastaan. 6000 v vanha maapallo on tietenkin idioottimainen idea mutta ainakaan omien tutkimuksieni mukaan sillä ei ole mitään tekemistä alkup, kristinuskon kanssa. Itse asiassa Isaac Newton ja Kepler olivat niitä jotka laskeskelivat maapallon syntyneen n. 4000 ekR.

        Itse en ole koskaan kuulunut mihinkään uskontokuntaan ja erosin koulussa uskonnonopetuksesta heti kuin se tuli mahdolliseksi. Ns. Jumalan suhteen olen agnostikko eli jätän asian avoimeksi ja oma jumalakuvani varmaan muutenkin poikkeaisi hyvin paljon länsimaisesta kristillisestä teismistä.

        Minulle on itsestään selvää että tietoisuus on kaikessa ensisijainen mutta jos se yksilön tietoisuus on hyvin kehittymätön ja surkastunut niin materialismi voi houkutella enemmän.

        Itse muuten luen parhaillaan kirjaa mielen manipulaatiosta eri menetelmin ja niiden menetelmien historiaa.

        ”samat asiat voi yhtä hyvin hahmottaa sekä materialismin että idealismin kautta ja oikeastaan vain se asenne siihen koettuun todellisuuteen on näissä aika erilainen.”

        Täysin samaa mieltä. Olen eri mieltä noiden katsontakantojen tulkinnasta. Materialismin kannaltakin on täysin selvää, että voimme katsoa realimaailmaa vain oman tietoisuutemme muodostaman kokemuksellisen kuvan kautta, joka on vain heijastus todellisuudesta. Idealistin kokemuksellisuus on täysin totta, mutta idealismissa ongelmaksi tulee selittää realimaailman systemaattisuus. Mielikuvat ovat satunnaisia kuten unissa ja realimaailman loogisuus ja systemaattisuus vaati selityksen. Samoin idealisti ei pysty selittämään tietoisuuden syntyä - se vain on.

        ”Materialisteilla on pakkomielteinen tarve todistella tietoisuus jotenkin epäolennaiseksi ja se ilmeisesti johtuu aika lapsellisesta vastareaktiosta ....”

        Täysin eri mieltä! Materialistinakin pidän tietoisuutta *tärkeimpänä* asiana mitä on, koska koko kokemuksellisuuteni perustuu siihen.


      • Belisario
        Järkisyitä kirjoitti:

        ”samat asiat voi yhtä hyvin hahmottaa sekä materialismin että idealismin kautta ja oikeastaan vain se asenne siihen koettuun todellisuuteen on näissä aika erilainen.”

        Täysin samaa mieltä. Olen eri mieltä noiden katsontakantojen tulkinnasta. Materialismin kannaltakin on täysin selvää, että voimme katsoa realimaailmaa vain oman tietoisuutemme muodostaman kokemuksellisen kuvan kautta, joka on vain heijastus todellisuudesta. Idealistin kokemuksellisuus on täysin totta, mutta idealismissa ongelmaksi tulee selittää realimaailman systemaattisuus. Mielikuvat ovat satunnaisia kuten unissa ja realimaailman loogisuus ja systemaattisuus vaati selityksen. Samoin idealisti ei pysty selittämään tietoisuuden syntyä - se vain on.

        ”Materialisteilla on pakkomielteinen tarve todistella tietoisuus jotenkin epäolennaiseksi ja se ilmeisesti johtuu aika lapsellisesta vastareaktiosta ....”

        Täysin eri mieltä! Materialistinakin pidän tietoisuutta *tärkeimpänä* asiana mitä on, koska koko kokemuksellisuuteni perustuu siihen.

        "Mielikuvat ovat satunnaisia kuten unissa ja realimaailman loogisuus ja systemaattisuus vaati selityksen."

        Ns. reaalimaailman loogisuus ja systemaattisuus viittaavat nimenomaan siihen että ns. aine/energia on rakentunut informaatiosta ja sen informaation tuottaa nimenomaan tietoisuus.

        Ns. aineen pohjimmainen taso näyttää sen sijaan olevan täysin satunnainen ja kaoottinen sekä aivojen mitatut sähkösignaalit taas hyvin "meluisia" eli täysin päinvastaisia kuin (normaaali) kokemuksellisuus.

        Jostain se systemaattisuus ja loogisuus tulee tietenkin mutta ei se näytä tulevan ainakaan aineen mikrotasolta eikä aivojen/hermoston tasolta.

        Viittaan aivojen osalta vielä Lorberin tutkimukseen jonka sinä kuittasit aivojen plastisuudella.
        Jos aivokudosta on hyvin vähän jäljellä ja silti yksilö toimii normaalilla on jopa keskitasoa älykkyympi niin kaikki mahdolliset teoriat tietoisuuden tiettyjen komponenttien paikallistamisesta joihinkin aivojen osiin ovat kyseenalaisia.

        Jos taas tämä reaalitodellisuus on jonkinlainen globalin ja epälokaalin tietoisuuden tuottama tapahtumakeskeinen (event driven) simulaatio niin se selittäisi miksi tiettyjä asioita fysiikkatieteessä voidaan laskea ja kuvata hyvinkin suurella tarkkuudella.

        Simulaatio on rakennettu matematiikan ja logiikan pohjalta ja jokaiselle kokonaisuudesta dissosiaatioidulle kokijalle tuotetaan vain sen verran datastreamiä kun on kussakin tilanteessa tarpeellista ottaen samalla huomioon että kokonaisuuden on oltava ristiriidaton.

        Simulaatiomallissa sekä keho että aivot ovat virtuaalisia eivät oikeasti kausaalisesti vaikuta yhtään mihinkään kuten esim. moninpeleissä tapahtuu eli pelin/simulaation serveri tuottaa jokaiselle pelaajalle sen pelaajan valintoja vastaavan lopputuloksen.

        Pelissä pelihahmon keho voi eri tavoin vaurioitua ja kuolla mutta se ei vaikuta itse pelaajaan muuten kuin sen simulaation tuottaman kokemuksen kautta. Vaikeasti vammaisen pelihahmon pelaaminen on vain paljon haastavampaa ja menestyminen palkitsevampaa kuin sitä normaalia ja tervettä pelihahmoa pelaamalla (esim. Stephen Hawking) .

        Pelatessa pelaaja myös oppii monenlaisia asioita joita ns. puhtaalla "henkisellä" tasolla ei pystyisi oppimaan eli kehitys on nopeampaa kun olosuhteet ovat riittävän haastavia ja pelin vaikeusaste lisääntyy tai vähenee sen mukaan miten pelaaja pärjää "elämän koulussa".

        Entropia eli rakenteiden vähittäinen hajoaminen taas selittyy sillä että kun simulaatio ei enää ylläpidä jotain rakennetta niin se vähitellen hajoaa siihen kaaottiseen ja satunnaiseen alkutilaansa joka näkyy mikromaailmassa.

        "Samoin idealisti ei pysty selittämään tietoisuuden syntyä - se vain on."

        Tietoisuutta voi syntyä vain tietoisuudesta joko fuusion tai fission avulla. Tietoisuus ei voi syntyä jostain ei-tietoisesta. Voi hyvinkin olla että se ensimmäinen prototietoisuus oli niin yksinkertainen että se kykeni erottamaan vain kaksi vastakkaista kokemusta.

        Sitten hieman soluautomaatin tavoin se alkoi jakautua ja tuottaa itsestään erilaistuvaa ympäristöä itselleen jolloin eriytyneiden tietoisuuksien kokemusmaailma monimuotoistui vähitellen ja sitten kunkin osatietoisuuden kokemukset integroitiin takaisinkytkennällä siihen kokonaisuuteen ja seuraavassa iteraatiossa monimutkaistuminen ja monimuotoutuminen jatkui kiihdyttäen vauhtia päätyen lopulta siihen missä nyt olemme.

        Monimutkaisia osatietoisuuksia voidaan sitten kutsua biologiseksi elämäksi ja niiden toisilleen tuottamaa informaatiota aineeksi.

        Tämä on tietysti puhtaasti spekulaatiota ja arvelua mutta jotenkin edellä mainitsemaani tyyliin se voisi tapahtua. Tämä on myös suurinpiirtein se kuvaus mitä Tom Campbell käyttää.


        ....

        Siitä kvanttifysiikasta vielä kun en erikseen siihen jaksa nyt vastata:

        Pelkkä tietoisuus ei riitä aaltofunktion romahdukseen vaan se informaatio saatavuus sekä sen informaation epävarmuus ovat kaikki yhtä tarpeellisia.

        Kokonaisuuden täytyy olla myös koherentti eli jos tähän konsensustodellisuuteen jokin aktualisoituu "hiukkasmoodissa" niin se myös täällä pysyy hiukkasmoodissa niin kauan kuin informaatio ja se kokija ovat vaikuttamassa tässä todellisuudessa. Jos taas se informaatio että kokija katoavat niin kokonaisuuden koherenssin kannalta voidaan suorittaa täysin uusi satunnaisarvonta joka tuottaa erilaisen mutta siltikin aina sen kokonaisuuden koherenssin säilyttävä lopputulos.

        Satunnaisuus on eri asia kuin epävarmuus koska tiedetään varmasti että nopanheitosta tulee arvot 1-6 mutta ei esim. tiedetä mitä jossain viidakon tutkimattomassa kolkassa tai vieraalla planeetalla niin niissä voi satunnaisesti arpoa sellaisiakin asioita joita emme pysty edes kuvittelemaan.

        Kuten huomaat niin idealismi on aika luontevasti yhdistettävissä simulaatiomalliin jos sen simulaation serveri on kaikkien tietoisuuksien kokonaisuus ja yksittäiset pelaajat sen kokonaisuuden dissosiaatioita ja kaikki aine/energia tai oikeammin niiden rakenne taas informaatiota. Ns. fysikaalista ainetta ei tarvita.


      • Järkisyitä

        ”Tietoisuutta voi syntyä vain tietoisuudesta joko fuusion tai fission avulla.”

        Tuo on tyhjä väite, jota et pysty perustelemaan mitenkään. Eikä se edes selitä, mistä ensimmäinen tietoisuus tuli ja miksi alkuperäisestä tietoisuudesta pilkotut osatietoisuudet eivät ole alkuperäisestä tietoisuudesta tietoisia tai miten pitkää tietoisuutta voi pilkkoa, kunnes jäljelle jää vain murunen, jonka tietoisuus on minimalistinen.Mistä minun tietoisuuteni on peräisin?

        ”Tietoisuus ei voi syntyä jostain ei-tietoisesta.”

        Taas tyhjä väite, jota et pysty perustelemaan. Miksei voisi syntyä?


      • HengetKuviteltuja
        Järkisyitä kirjoitti:

        ”Tietoisuutta voi syntyä vain tietoisuudesta joko fuusion tai fission avulla.”

        Tuo on tyhjä väite, jota et pysty perustelemaan mitenkään. Eikä se edes selitä, mistä ensimmäinen tietoisuus tuli ja miksi alkuperäisestä tietoisuudesta pilkotut osatietoisuudet eivät ole alkuperäisestä tietoisuudesta tietoisia tai miten pitkää tietoisuutta voi pilkkoa, kunnes jäljelle jää vain murunen, jonka tietoisuus on minimalistinen.Mistä minun tietoisuuteni on peräisin?

        ”Tietoisuus ei voi syntyä jostain ei-tietoisesta.”

        Taas tyhjä väite, jota et pysty perustelemaan. Miksei voisi syntyä?

        ”Tietoisuus ei voi syntyä jostain ei-tietoisesta.”

        Elämä syntyy, kun ei-eläviä molekyylejä järjestyy riittävän suuri määrä sopivaan muotoon. Vitaalivoimaa ei tarvita.
        Tietoisuus syntyy, kun eliölle kehittyy riittävän monimutkainen keskushermosto. Mitään "sielua", "henkeä" tai muutakaan ei-aterialistista ei olla havaittu, eikä sellaista tarvita. Sillä ei olisi toimivaa liityntäpintaa biologiseen järjestelmään.


      • Belisario
        Järkisyitä kirjoitti:

        ”Tietoisuutta voi syntyä vain tietoisuudesta joko fuusion tai fission avulla.”

        Tuo on tyhjä väite, jota et pysty perustelemaan mitenkään. Eikä se edes selitä, mistä ensimmäinen tietoisuus tuli ja miksi alkuperäisestä tietoisuudesta pilkotut osatietoisuudet eivät ole alkuperäisestä tietoisuudesta tietoisia tai miten pitkää tietoisuutta voi pilkkoa, kunnes jäljelle jää vain murunen, jonka tietoisuus on minimalistinen.Mistä minun tietoisuuteni on peräisin?

        ”Tietoisuus ei voi syntyä jostain ei-tietoisesta.”

        Taas tyhjä väite, jota et pysty perustelemaan. Miksei voisi syntyä?

        "Eikä se edes selitä, mistä ensimmäinen tietoisuus tuli ja miksi alkuperäisestä tietoisuudesta pilkotut osatietoisuudet eivät ole alkuperäisestä tietoisuudesta tietoisia tai miten pitkää tietoisuutta voi pilkkoa, kunnes jäljelle jää vain murunen, jonka tietoisuus on minimalistinen.Mistä minun tietoisuuteni on peräisin?"

        Niin se oli spekulatiivinen hahmotelma siitä miten voisin sen kuvitella.

        Alkuperäisestä tietoisuudesta jakautuneet osatietoisuudet voivat esim. meditaation ja muiden ns. henkisten tekniikoiden avulla olla siitä laajemmasta tietoisuudesta tietoisia. Hyvä esimerkki on esim. tuo kaukonäkeminen (remote viewing) tai oikeammin kaukokokeminen ja monet muut ilmiöt ja taidot joihin materialismiin jumittunut tiede ei edes halua tutkia koska pitää niitä jo oletusarvoisesti mahdottomina.

        Ei se tietoisuus "pilkkoudu" samalla tavalla kuin jokin fysikaalinen kappale vaan se osatietoisuuskin koetaan aina täydellisenä samalla tavalla kuin hologrammin osa vastaa kokonaisuutta mistä se on irrotettu.

        Sen tietoisuuden "fission" voi todeta jakautuneesta persoonallisuudesta kärsivillä yksilöillä ja sen "fuusion" taas ainakin jollain tasolla kaikissa ryhmäkäyttäytymisen muodoissa.

        En itse pidä idealismia enkä varsinkaan materialismia mitenkään itsetarkoituksellisina ideologioina vaan lähinnä näkökulmina tai työkaluina jatkotutkimukselle.

        Minusta se materialismin näkökulma alkaa olla jo aika loppuunkaluttu kun taas idealismi esim. Bernardo Kastrupin ja Tom Campbellin ideoissa tarjoaa uutta ja mielenkiintoista lähestymistapaa koska ne voisivat olla mahdollisia nykyisen tiedon ja ymmärryksen puitteissa.


        ....

        HengetKuviteltuja:

        "Elämä syntyy, kun ei-eläviä molekyylejä järjestyy riittävän suuri määrä sopivaan muotoon. "

        Tarkoitat varmaan sillä "sopivalla muodolla" riittävää organisoitumista ja esim. Tononin IIT-teoriassa se olisi riittävän integroitunut informaatio. Tuo nykyisen evoluutioteorian näkemys on lähinnä vain molekyylietason kuvaus eikä sellaisenaan selitä mistä se "sopiva muoto" syntyy eikä sitten hermoston & aivojen tasolla kerro sitäkään miten niiden oletetaan tuottavan sen tietoisuuden.

        "Tietoisuus syntyy, kun eliölle kehittyy riittävän monimutkainen keskushermosto."

        Sekin on pelkkä spekulaatio kun ei tiedetä mikä se riittävä on. Ainoa mitä oikeasti tiedetään on se että ns. biologiset oliot alkavat käyttäytyä sillä tavalla että voimme tulkita ne jollain tavalla tietoiseksi vertaamalla niiden käytöstä omaamme kun vain itse tiedämme olevamme tietoisia.

        ...

        Tieteellinen tietomme on vielä hyvin fragmentoitunutta ja alkeellista kaikesta hypetyksestä huolimatta. Tekoälytutkimus kieltämättä on jo paljastanut monet yksinkertaisina pidetyt ihmisen psyyken ilmiöt hyvinkin monimutkaisiksi ja vaikeiksi toteuttaa tietotekniikan pohjalta.

        Aidosti tietoinen tekoäly projisoidaan kuvitelmissa aina jonnekin kymmenien vuosien päähän kuten monet muutkin tieteiden haaveet ja päämäärät ovat toistaiseksi vain haaveita.

        Jos pystyisin vastaamaan kaikkiin mahdollisiin kuviteltavissa oleviin vastaväitteisiin vakuuttavasti niin minun pitäisi ymmärtää ja tietää se kaikki mikä on ylipäätänsä tiedettävissä.

        En pidä jankkausta kovin kiinnostavana muutenkaan vaan keskityn mielummin itse tutkimaan ja opiskelemaan lisää asioista.

        Pidän myös aika typeränä taktiikkana uskoa johonkin teoriaan tai malliin vain sen takia että se tuottaa jonkinlaisen valheellisen varmuuden tunteen koska todellisuudessa mitään sen tyyppistä varmuutta ei mikään tiede eikä filosofia voi koskaan tarjota koska mikään tiede eikä filosofia tule koskaan valmiiksi.

        Taidan jäädä taas vähäksi aikaa tauolle kun tämä touhu vie liikaa aikaa ja energiaa.


      • kiikkustuolista
        HengetKuviteltuja kirjoitti:

        ”Tietoisuus ei voi syntyä jostain ei-tietoisesta.”

        Elämä syntyy, kun ei-eläviä molekyylejä järjestyy riittävän suuri määrä sopivaan muotoon. Vitaalivoimaa ei tarvita.
        Tietoisuus syntyy, kun eliölle kehittyy riittävän monimutkainen keskushermosto. Mitään "sielua", "henkeä" tai muutakaan ei-aterialistista ei olla havaittu, eikä sellaista tarvita. Sillä ei olisi toimivaa liityntäpintaa biologiseen järjestelmään.

        Tuo on tyypillinen elämän ja tietoisuuden emergenttinen teoria. Siihen liittyy kuitenkin monia ratkaisemattomia ongelmia.

        Ensiksikin, mikä on se riittävä määrä molekyylejä ja millainen on se sopiva muoto, että elämää syntyy?

        Toiseksi, mikä on riittävän monimutkainen hermoverkosto, että tietoisuus syntyy? Jos se on esimerkiksi oma hermoverkostosi, niin voiko pienemmän hermoverkoston omaavat eläimet olla vailla tietoisuutta? (Tutkimusten mukaan aivojen koolla ei ole merkitystä älykkyyteen.)

        Kolmanneksi, millaisen rakenteellisen arkkitehtuurin tuo kaikki edellyttää ja millaisia yhdistäviä voimia se tarvitsee?

        Liioin kaikki oleva ei ole suoraan havaittavissa. Joidenkin entiteettien olemassa olo voidaan päätellä niiden aiheuttamien kausaalisten seurausten vuoksi. Siten esimerkiksi mieli voidaan tunnistaa, koska sen toiminnan seurauksena syntyy ajattelua.

        Noista ongelmista huolimatta emergenttisyys voi olla tosiasia.

        Sielun ja hengen suhteen en ota kantaa.


      • iloinenpiereskelijä
        Belisario kirjoitti:

        "Eikä se edes selitä, mistä ensimmäinen tietoisuus tuli ja miksi alkuperäisestä tietoisuudesta pilkotut osatietoisuudet eivät ole alkuperäisestä tietoisuudesta tietoisia tai miten pitkää tietoisuutta voi pilkkoa, kunnes jäljelle jää vain murunen, jonka tietoisuus on minimalistinen.Mistä minun tietoisuuteni on peräisin?"

        Niin se oli spekulatiivinen hahmotelma siitä miten voisin sen kuvitella.

        Alkuperäisestä tietoisuudesta jakautuneet osatietoisuudet voivat esim. meditaation ja muiden ns. henkisten tekniikoiden avulla olla siitä laajemmasta tietoisuudesta tietoisia. Hyvä esimerkki on esim. tuo kaukonäkeminen (remote viewing) tai oikeammin kaukokokeminen ja monet muut ilmiöt ja taidot joihin materialismiin jumittunut tiede ei edes halua tutkia koska pitää niitä jo oletusarvoisesti mahdottomina.

        Ei se tietoisuus "pilkkoudu" samalla tavalla kuin jokin fysikaalinen kappale vaan se osatietoisuuskin koetaan aina täydellisenä samalla tavalla kuin hologrammin osa vastaa kokonaisuutta mistä se on irrotettu.

        Sen tietoisuuden "fission" voi todeta jakautuneesta persoonallisuudesta kärsivillä yksilöillä ja sen "fuusion" taas ainakin jollain tasolla kaikissa ryhmäkäyttäytymisen muodoissa.

        En itse pidä idealismia enkä varsinkaan materialismia mitenkään itsetarkoituksellisina ideologioina vaan lähinnä näkökulmina tai työkaluina jatkotutkimukselle.

        Minusta se materialismin näkökulma alkaa olla jo aika loppuunkaluttu kun taas idealismi esim. Bernardo Kastrupin ja Tom Campbellin ideoissa tarjoaa uutta ja mielenkiintoista lähestymistapaa koska ne voisivat olla mahdollisia nykyisen tiedon ja ymmärryksen puitteissa.


        ....

        HengetKuviteltuja:

        "Elämä syntyy, kun ei-eläviä molekyylejä järjestyy riittävän suuri määrä sopivaan muotoon. "

        Tarkoitat varmaan sillä "sopivalla muodolla" riittävää organisoitumista ja esim. Tononin IIT-teoriassa se olisi riittävän integroitunut informaatio. Tuo nykyisen evoluutioteorian näkemys on lähinnä vain molekyylietason kuvaus eikä sellaisenaan selitä mistä se "sopiva muoto" syntyy eikä sitten hermoston & aivojen tasolla kerro sitäkään miten niiden oletetaan tuottavan sen tietoisuuden.

        "Tietoisuus syntyy, kun eliölle kehittyy riittävän monimutkainen keskushermosto."

        Sekin on pelkkä spekulaatio kun ei tiedetä mikä se riittävä on. Ainoa mitä oikeasti tiedetään on se että ns. biologiset oliot alkavat käyttäytyä sillä tavalla että voimme tulkita ne jollain tavalla tietoiseksi vertaamalla niiden käytöstä omaamme kun vain itse tiedämme olevamme tietoisia.

        ...

        Tieteellinen tietomme on vielä hyvin fragmentoitunutta ja alkeellista kaikesta hypetyksestä huolimatta. Tekoälytutkimus kieltämättä on jo paljastanut monet yksinkertaisina pidetyt ihmisen psyyken ilmiöt hyvinkin monimutkaisiksi ja vaikeiksi toteuttaa tietotekniikan pohjalta.

        Aidosti tietoinen tekoäly projisoidaan kuvitelmissa aina jonnekin kymmenien vuosien päähän kuten monet muutkin tieteiden haaveet ja päämäärät ovat toistaiseksi vain haaveita.

        Jos pystyisin vastaamaan kaikkiin mahdollisiin kuviteltavissa oleviin vastaväitteisiin vakuuttavasti niin minun pitäisi ymmärtää ja tietää se kaikki mikä on ylipäätänsä tiedettävissä.

        En pidä jankkausta kovin kiinnostavana muutenkaan vaan keskityn mielummin itse tutkimaan ja opiskelemaan lisää asioista.

        Pidän myös aika typeränä taktiikkana uskoa johonkin teoriaan tai malliin vain sen takia että se tuottaa jonkinlaisen valheellisen varmuuden tunteen koska todellisuudessa mitään sen tyyppistä varmuutta ei mikään tiede eikä filosofia voi koskaan tarjota koska mikään tiede eikä filosofia tule koskaan valmiiksi.

        Taidan jäädä taas vähäksi aikaa tauolle kun tämä touhu vie liikaa aikaa ja energiaa.

        "Pidän myös aika typeränä taktiikkana uskoa johonkin teoriaan tai malliin vain sen takia että se tuottaa jonkinlaisen valheellisen varmuuden tunteen koska todellisuudessa mitään sen tyyppistä varmuutta ei mikään tiede eikä filosofia voi koskaan tarjota koska mikään tiede eikä filosofia tule koskaan valmiiksi."

        No niin. Siinä sitä taas ollaan. Heh!

        Kyllä kaikki tietellinen tieto on epävarmaa. Itselleni mikään tieteellinen toria tai tieto ei synnytä "varmuuden tunnetta", koska sellaista ei voi edes periaatteessa olla olemassa tieteenfilosofian perusteella.

        Mutta tämä ei liity aiheeseen mitenkään. Vaikka tietellinen tieto on epävarmaa, voin hyvin todeta, että idealismi on roskaa. Johtopäätös on toki epävarma niin kuin kaikki tieteellinen tieto on. Ei se silti muuta idealismin luonnetta roskasta miksikään!!

        Pitäisi löytä jotakin todella vakuuttavaa uutta tietoa, joka tukisi idealistisia pilvilinnoja edes murusen verran, että syntyisi tarvetta tarkastella edelliä todettua uudestaan. Itse luulin Belisariota parodiaksi, kunnes ymmärsin, että taitaahan se spekuloinneissaan ihan "tosissaan" olla. Hyvä asia on tietenkin, että hän sen tunnustaa spekuloinniksi.


      • kiikkustuolista kirjoitti:

        Tuo on tyypillinen elämän ja tietoisuuden emergenttinen teoria. Siihen liittyy kuitenkin monia ratkaisemattomia ongelmia.

        Ensiksikin, mikä on se riittävä määrä molekyylejä ja millainen on se sopiva muoto, että elämää syntyy?

        Toiseksi, mikä on riittävän monimutkainen hermoverkosto, että tietoisuus syntyy? Jos se on esimerkiksi oma hermoverkostosi, niin voiko pienemmän hermoverkoston omaavat eläimet olla vailla tietoisuutta? (Tutkimusten mukaan aivojen koolla ei ole merkitystä älykkyyteen.)

        Kolmanneksi, millaisen rakenteellisen arkkitehtuurin tuo kaikki edellyttää ja millaisia yhdistäviä voimia se tarvitsee?

        Liioin kaikki oleva ei ole suoraan havaittavissa. Joidenkin entiteettien olemassa olo voidaan päätellä niiden aiheuttamien kausaalisten seurausten vuoksi. Siten esimerkiksi mieli voidaan tunnistaa, koska sen toiminnan seurauksena syntyy ajattelua.

        Noista ongelmista huolimatta emergenttisyys voi olla tosiasia.

        Sielun ja hengen suhteen en ota kantaa.

        "Tutkimusten mukaan aivojen koolla ei ole merkitystä älykkyyteen."

        Ei kai ihan näinkään. Aivojen koko suhteessa ruumiin kokoon korreloi älykkyyden kanssa, joskaan ei suoraviivaisesti. Koon lisäksi esimerkiksi aivokuoren poimuttuneisuus vaikuttaa kognitiivisiin kykyihin.

        Paljonkin ihmistä pienemmän hermoverkon omaavat eläimet voivat olla tietoisia itsestään. Toisaalta kun puhumme eläinten tietoisuudesta, emme mitenkään voi tietää, miten eläin asian kokee. Voimme tutkia vain eläinten reaktioita erilaisin kokein. Telkänrannan melko tuore "Millaista on olla eläin" valaisee asiaa uusimman tutkimuksen kannalta.


      • Järkisyitä
        Belisario kirjoitti:

        "Eikä se edes selitä, mistä ensimmäinen tietoisuus tuli ja miksi alkuperäisestä tietoisuudesta pilkotut osatietoisuudet eivät ole alkuperäisestä tietoisuudesta tietoisia tai miten pitkää tietoisuutta voi pilkkoa, kunnes jäljelle jää vain murunen, jonka tietoisuus on minimalistinen.Mistä minun tietoisuuteni on peräisin?"

        Niin se oli spekulatiivinen hahmotelma siitä miten voisin sen kuvitella.

        Alkuperäisestä tietoisuudesta jakautuneet osatietoisuudet voivat esim. meditaation ja muiden ns. henkisten tekniikoiden avulla olla siitä laajemmasta tietoisuudesta tietoisia. Hyvä esimerkki on esim. tuo kaukonäkeminen (remote viewing) tai oikeammin kaukokokeminen ja monet muut ilmiöt ja taidot joihin materialismiin jumittunut tiede ei edes halua tutkia koska pitää niitä jo oletusarvoisesti mahdottomina.

        Ei se tietoisuus "pilkkoudu" samalla tavalla kuin jokin fysikaalinen kappale vaan se osatietoisuuskin koetaan aina täydellisenä samalla tavalla kuin hologrammin osa vastaa kokonaisuutta mistä se on irrotettu.

        Sen tietoisuuden "fission" voi todeta jakautuneesta persoonallisuudesta kärsivillä yksilöillä ja sen "fuusion" taas ainakin jollain tasolla kaikissa ryhmäkäyttäytymisen muodoissa.

        En itse pidä idealismia enkä varsinkaan materialismia mitenkään itsetarkoituksellisina ideologioina vaan lähinnä näkökulmina tai työkaluina jatkotutkimukselle.

        Minusta se materialismin näkökulma alkaa olla jo aika loppuunkaluttu kun taas idealismi esim. Bernardo Kastrupin ja Tom Campbellin ideoissa tarjoaa uutta ja mielenkiintoista lähestymistapaa koska ne voisivat olla mahdollisia nykyisen tiedon ja ymmärryksen puitteissa.


        ....

        HengetKuviteltuja:

        "Elämä syntyy, kun ei-eläviä molekyylejä järjestyy riittävän suuri määrä sopivaan muotoon. "

        Tarkoitat varmaan sillä "sopivalla muodolla" riittävää organisoitumista ja esim. Tononin IIT-teoriassa se olisi riittävän integroitunut informaatio. Tuo nykyisen evoluutioteorian näkemys on lähinnä vain molekyylietason kuvaus eikä sellaisenaan selitä mistä se "sopiva muoto" syntyy eikä sitten hermoston & aivojen tasolla kerro sitäkään miten niiden oletetaan tuottavan sen tietoisuuden.

        "Tietoisuus syntyy, kun eliölle kehittyy riittävän monimutkainen keskushermosto."

        Sekin on pelkkä spekulaatio kun ei tiedetä mikä se riittävä on. Ainoa mitä oikeasti tiedetään on se että ns. biologiset oliot alkavat käyttäytyä sillä tavalla että voimme tulkita ne jollain tavalla tietoiseksi vertaamalla niiden käytöstä omaamme kun vain itse tiedämme olevamme tietoisia.

        ...

        Tieteellinen tietomme on vielä hyvin fragmentoitunutta ja alkeellista kaikesta hypetyksestä huolimatta. Tekoälytutkimus kieltämättä on jo paljastanut monet yksinkertaisina pidetyt ihmisen psyyken ilmiöt hyvinkin monimutkaisiksi ja vaikeiksi toteuttaa tietotekniikan pohjalta.

        Aidosti tietoinen tekoäly projisoidaan kuvitelmissa aina jonnekin kymmenien vuosien päähän kuten monet muutkin tieteiden haaveet ja päämäärät ovat toistaiseksi vain haaveita.

        Jos pystyisin vastaamaan kaikkiin mahdollisiin kuviteltavissa oleviin vastaväitteisiin vakuuttavasti niin minun pitäisi ymmärtää ja tietää se kaikki mikä on ylipäätänsä tiedettävissä.

        En pidä jankkausta kovin kiinnostavana muutenkaan vaan keskityn mielummin itse tutkimaan ja opiskelemaan lisää asioista.

        Pidän myös aika typeränä taktiikkana uskoa johonkin teoriaan tai malliin vain sen takia että se tuottaa jonkinlaisen valheellisen varmuuden tunteen koska todellisuudessa mitään sen tyyppistä varmuutta ei mikään tiede eikä filosofia voi koskaan tarjota koska mikään tiede eikä filosofia tule koskaan valmiiksi.

        Taidan jäädä taas vähäksi aikaa tauolle kun tämä touhu vie liikaa aikaa ja energiaa.

        ”Hyvä esimerkki on esim. tuo kaukonäkeminen (remote viewing) tai oikeammin kaukokokeminen ja monet muut ilmiöt ja taidot joihin materialismiin jumittunut tiede ei edes halua tutkia koska pitää niitä jo oletusarvoisesti mahdottomina.”

        CIA tutki 1970-luvulla kaiken maailman psyykkisiä ilmiöitä. Nuo tutkimukset julkaistiin joku aika sitten ja yllätys yllätys: mitään psyykkisiä ilmiöitä kuten kaukonäkeminen ei löydetty ja tutkimus lakkautettiin tuloksettomana. Jos kaukonäkeminen oikeasti toimisi, niin toki moinen olisi jo löydetty ja se aiheuttaisi mm. tietoturva alalle ja vakoilutoimintaan ihan uusia haasteita. Täyttä huuhaata!


      • kiikkustuolista
        agnoskepo kirjoitti:

        "Tutkimusten mukaan aivojen koolla ei ole merkitystä älykkyyteen."

        Ei kai ihan näinkään. Aivojen koko suhteessa ruumiin kokoon korreloi älykkyyden kanssa, joskaan ei suoraviivaisesti. Koon lisäksi esimerkiksi aivokuoren poimuttuneisuus vaikuttaa kognitiivisiin kykyihin.

        Paljonkin ihmistä pienemmän hermoverkon omaavat eläimet voivat olla tietoisia itsestään. Toisaalta kun puhumme eläinten tietoisuudesta, emme mitenkään voi tietää, miten eläin asian kokee. Voimme tutkia vain eläinten reaktioita erilaisin kokein. Telkänrannan melko tuore "Millaista on olla eläin" valaisee asiaa uusimman tutkimuksen kannalta.

        Myös Thomas Nagel on teoksessaan "Millaista on olla lepakko" syventynyt samaan tematiikkaan. Siinä hän nostaa esille mielen ja kehon välisen suhteen ehdot ja ongelmat. Hänen mukaansa lepakon kaikukuvakokemuksen ensimmäisen persoonan näkökulma poikkeaa ulkopuolisesta lepakon aivotoiminnan kuvauksen kolmannen persoonan näkökulmasta. Mikäli tuo pitää paikkansa, niin kokemusta ei voida selittää pelkällä fysiikalla.

        Mitä tulee eläimiin kohdistuviin tutkimuksiin, niin on huomattava, että kokemuksen ominaispiirteet ja kokemus ominaispiirteistä poikkeavat toisistaan. Samoin tarkasteltavan kohteen kvalitatiivisuus on eri asia kuin kokemuksen kvalitatiivisuus. Tällöin on vaarana, että tutkija peilaakin eläimen kohdalla pelkästään omia reaktioitaan.

        Puolestaan instrumentalisti Daniel Dennet on muotoillut kokonaisen mielen rakenteellisen hierarkian. Hänen mukaansa vain ihminen mieleen sisältyvän "lisäkapasiteetin" ansiosta tämä kykenee reflektoimaan itselleen kvalitatiivisen kivun tuntemuksen. Sen sijaan eläin, hänen mukaansa kylläkin reagoi kipuärsytykseen, mutta kykenemättömänä itsereflektioon, ei tuntisi kärsimystä.

        Itse en ole aivan vakuuttunut tuosta Dennetin argumentaatiosta. Toisaalta niin kuin totesit, emme voi varmuudella tietää, miten eläin kipuärsytyksen kokee.


      • Järkisyitä
        kiikkustuolista kirjoitti:

        Tuo on tyypillinen elämän ja tietoisuuden emergenttinen teoria. Siihen liittyy kuitenkin monia ratkaisemattomia ongelmia.

        Ensiksikin, mikä on se riittävä määrä molekyylejä ja millainen on se sopiva muoto, että elämää syntyy?

        Toiseksi, mikä on riittävän monimutkainen hermoverkosto, että tietoisuus syntyy? Jos se on esimerkiksi oma hermoverkostosi, niin voiko pienemmän hermoverkoston omaavat eläimet olla vailla tietoisuutta? (Tutkimusten mukaan aivojen koolla ei ole merkitystä älykkyyteen.)

        Kolmanneksi, millaisen rakenteellisen arkkitehtuurin tuo kaikki edellyttää ja millaisia yhdistäviä voimia se tarvitsee?

        Liioin kaikki oleva ei ole suoraan havaittavissa. Joidenkin entiteettien olemassa olo voidaan päätellä niiden aiheuttamien kausaalisten seurausten vuoksi. Siten esimerkiksi mieli voidaan tunnistaa, koska sen toiminnan seurauksena syntyy ajattelua.

        Noista ongelmista huolimatta emergenttisyys voi olla tosiasia.

        Sielun ja hengen suhteen en ota kantaa.

        ”Ensiksikin, mikä on se riittävä määrä molekyylejä ja millainen on se sopiva muoto, että elämää syntyy?”

        Yksisoluisten alkeellisten eliöiden toiminta tunnetaan jo aika hyvin. Ei siinä ole mitään mystiikkaa vaan elämä on todistetusti kemiallinen kone.

        Yksinkertaisin tunnettu mikrobi on tietääkseni ihmisen itse valmistama:
        https://www.ft.com/content/8e67c552-f1b4-11e5-aff5-19b4e253664a

        ”Toiseksi, mikä on riittävän monimutkainen hermoverkosto, että tietoisuus syntyy? ”

        Tällä hetkellä tietoisuuden syntymekanismia tunneta. Varmaa on, että hermoston pitää olla erittäin monimutkainen, mutta pelkkä monimutkaisuus ei riitä - myös arkkitehtuurin on oltava sopiva.

        Tässä yksi tämän hetken monimutkaisimpia neuroverkkoihin perustuvia keinoälyjä, joita yritetään opettaa kuin ihmislapsia:
        https://www.youtube.com/watch?v=ZcTwO2dpX8A


      • kiikkustuolista kirjoitti:

        Myös Thomas Nagel on teoksessaan "Millaista on olla lepakko" syventynyt samaan tematiikkaan. Siinä hän nostaa esille mielen ja kehon välisen suhteen ehdot ja ongelmat. Hänen mukaansa lepakon kaikukuvakokemuksen ensimmäisen persoonan näkökulma poikkeaa ulkopuolisesta lepakon aivotoiminnan kuvauksen kolmannen persoonan näkökulmasta. Mikäli tuo pitää paikkansa, niin kokemusta ei voida selittää pelkällä fysiikalla.

        Mitä tulee eläimiin kohdistuviin tutkimuksiin, niin on huomattava, että kokemuksen ominaispiirteet ja kokemus ominaispiirteistä poikkeavat toisistaan. Samoin tarkasteltavan kohteen kvalitatiivisuus on eri asia kuin kokemuksen kvalitatiivisuus. Tällöin on vaarana, että tutkija peilaakin eläimen kohdalla pelkästään omia reaktioitaan.

        Puolestaan instrumentalisti Daniel Dennet on muotoillut kokonaisen mielen rakenteellisen hierarkian. Hänen mukaansa vain ihminen mieleen sisältyvän "lisäkapasiteetin" ansiosta tämä kykenee reflektoimaan itselleen kvalitatiivisen kivun tuntemuksen. Sen sijaan eläin, hänen mukaansa kylläkin reagoi kipuärsytykseen, mutta kykenemättömänä itsereflektioon, ei tuntisi kärsimystä.

        Itse en ole aivan vakuuttunut tuosta Dennetin argumentaatiosta. Toisaalta niin kuin totesit, emme voi varmuudella tietää, miten eläin kipuärsytyksen kokee.

        Olen tutustunut Nagelin ajatuksiin pintapuolisesti. Nagel lähestyy filosofina aihetta eri tavoin kuin eläinten käyttäytymistä tutkiva biologi. Lähestymistapa on kuitenkin tutkimuksen kannalta arvokas, ettei kävisi kuten totesit:
        "Tällöin on vaarana, että tutkija peilaakin eläimen kohdalla pelkästään omia reaktioitaan."

        Olen lukenut enemmän kuin yhden populaarin kirjan, missä kirjoittaja inhimmillistää liikaa kohdettaan.

        Tuosta Dennetin ajatuksesta asiaa tutkineet biologit taitavat olla eri mieltä. Itse kallistuisin enemmän biologien puoleen jo pelkästään varovaisuusperiaatteen takia. Humen giljotiinin tietenkin huomioiden.


      • kiikkustuolista
        Järkisyitä kirjoitti:

        ”Ensiksikin, mikä on se riittävä määrä molekyylejä ja millainen on se sopiva muoto, että elämää syntyy?”

        Yksisoluisten alkeellisten eliöiden toiminta tunnetaan jo aika hyvin. Ei siinä ole mitään mystiikkaa vaan elämä on todistetusti kemiallinen kone.

        Yksinkertaisin tunnettu mikrobi on tietääkseni ihmisen itse valmistama:
        https://www.ft.com/content/8e67c552-f1b4-11e5-aff5-19b4e253664a

        ”Toiseksi, mikä on riittävän monimutkainen hermoverkosto, että tietoisuus syntyy? ”

        Tällä hetkellä tietoisuuden syntymekanismia tunneta. Varmaa on, että hermoston pitää olla erittäin monimutkainen, mutta pelkkä monimutkaisuus ei riitä - myös arkkitehtuurin on oltava sopiva.

        Tässä yksi tämän hetken monimutkaisimpia neuroverkkoihin perustuvia keinoälyjä, joita yritetään opettaa kuin ihmislapsia:
        https://www.youtube.com/watch?v=ZcTwO2dpX8A

        Tuo termi "kemiallinen kone" viittaa vääjäämättä koneen rakentajaan, olkoon se sitten Jumala tai insinööri.

        Emergenssiin liittyvä tasoajattelu on vielä sikäli ongelmallista, että fyysisen perustason yhteydet ylemmällä, eli mentaalisella tasolla olevaan emergenssiin ei selitä fysikaalisen ja mentaalisen maailman välistä suhdetta. Tuo eri todellisuustasojen ja olemistapojen välinen emergenssi voi siten olla täysin mahdoton. Tämä johtuu siitä, että ylemmän tason olemassa olo riippuu perustasosta, joten emergenssi ei voi ylittää fysikaalista ja mentaalista vastakkaisuutta. Nämä kaksihan määritellään jo lähtökohdissaan ominaisuuksiltaan vastakkaisiksi.

        Mikäli siis mentaalisuus olisi aineen emergentti ominaisuus ja mentaalinen ylätaso riippuisi aineellisesta perustasosta, joka puolestaan selitetään aineen perustason ominaisuuksilla, silloin mentaalisuus jäisi pelkäksi aivoprosessin heijastukseksi, jolla ei siten voisi olla kausaalista yhteyttä takaisin aivoprosesseihin. Mentaalisuudella kuitenkin tiedetään olevan vaikutuksia kehollisiin toimintoihin. Näin ollen emergenttisyys voi toimia pelkästään olemisen fyysisellä perustasolla.

        Kysymys olemisesta ja olemisen luonteesta on synnyttänyt kokonaisen filosofisen suuntauksen, nimittäin eksistentialismin. Siinä oleminen jaetaan kahteen perustyyppiin, nimittäin "oleva itselleen" ja "oleva itsessään". Ensin mainittu tarkoittaa sitä, että subjektin olemus, kuten vaikkapa ihmisellä, seuraa olemista. Vastaavasti "oleva itsessään" olemus, kuten esimerkiksi robotin tai tietokoneen kohdalla, edeltää sen olemista.

        Nuo mainitut olemisen perustyypit merkitsevät ratkaisevaa eroa myötäsyntyisen olennon ja keinotekoisen esineen välillä. Näin ollen hienoimmatkin robotit ja tietokoneet voivat vain etäisesti muistuttaa myötäsyntyisiä olentoja, kuten ihmisiä ja eläimiä.


      • kiikkustuolista
        agnoskepo kirjoitti:

        Olen tutustunut Nagelin ajatuksiin pintapuolisesti. Nagel lähestyy filosofina aihetta eri tavoin kuin eläinten käyttäytymistä tutkiva biologi. Lähestymistapa on kuitenkin tutkimuksen kannalta arvokas, ettei kävisi kuten totesit:
        "Tällöin on vaarana, että tutkija peilaakin eläimen kohdalla pelkästään omia reaktioitaan."

        Olen lukenut enemmän kuin yhden populaarin kirjan, missä kirjoittaja inhimmillistää liikaa kohdettaan.

        Tuosta Dennetin ajatuksesta asiaa tutkineet biologit taitavat olla eri mieltä. Itse kallistuisin enemmän biologien puoleen jo pelkästään varovaisuusperiaatteen takia. Humen giljotiinin tietenkin huomioiden.

        Daniel Dennet ei suinkaan kiellä biologiaa, pikemminkin päinvastoin. Hän nimittäin jakaa mielen rakenteellisesti intentionaaliseen, design ja fyysiseen asennekuvaan. Tällöin mieleen kohdistuvat kuvaukset ovat paljon biologisia kuvauksia laajemmat. Hänen mukaansa mieli saa voimansa representaatioista, eli mentaalisten tilojen kyvystä esittää jotain asiaintilaa tai erilaisista ilmennyksistä muodostuvista vaikutelmista.

        Mitä tulee hänen esittämäänsä mielen "lisäkapasiteettiin", niin se mahdollistuisi ajatellun kielen kautta. Kieli viestinnän välineenä ja ajattelun rahtilaivana tuo aivan eri ulottuvuuden verrattuna signaaleista muodostuvaan viestintään nähden. Juuri kieli on laajentanut tietoisuuttamme todellisuudesta, sen kvalitaatioista ja valinnan mahdollisuuksista. Se on myös mahdollistanut tämän meidän käymämme pohdiskelun mielen ja tietoisuuden peruskysymyksistä.


      • koittakaakehitellä
        kiikkustuolista kirjoitti:

        Tuo termi "kemiallinen kone" viittaa vääjäämättä koneen rakentajaan, olkoon se sitten Jumala tai insinööri.

        Emergenssiin liittyvä tasoajattelu on vielä sikäli ongelmallista, että fyysisen perustason yhteydet ylemmällä, eli mentaalisella tasolla olevaan emergenssiin ei selitä fysikaalisen ja mentaalisen maailman välistä suhdetta. Tuo eri todellisuustasojen ja olemistapojen välinen emergenssi voi siten olla täysin mahdoton. Tämä johtuu siitä, että ylemmän tason olemassa olo riippuu perustasosta, joten emergenssi ei voi ylittää fysikaalista ja mentaalista vastakkaisuutta. Nämä kaksihan määritellään jo lähtökohdissaan ominaisuuksiltaan vastakkaisiksi.

        Mikäli siis mentaalisuus olisi aineen emergentti ominaisuus ja mentaalinen ylätaso riippuisi aineellisesta perustasosta, joka puolestaan selitetään aineen perustason ominaisuuksilla, silloin mentaalisuus jäisi pelkäksi aivoprosessin heijastukseksi, jolla ei siten voisi olla kausaalista yhteyttä takaisin aivoprosesseihin. Mentaalisuudella kuitenkin tiedetään olevan vaikutuksia kehollisiin toimintoihin. Näin ollen emergenttisyys voi toimia pelkästään olemisen fyysisellä perustasolla.

        Kysymys olemisesta ja olemisen luonteesta on synnyttänyt kokonaisen filosofisen suuntauksen, nimittäin eksistentialismin. Siinä oleminen jaetaan kahteen perustyyppiin, nimittäin "oleva itselleen" ja "oleva itsessään". Ensin mainittu tarkoittaa sitä, että subjektin olemus, kuten vaikkapa ihmisellä, seuraa olemista. Vastaavasti "oleva itsessään" olemus, kuten esimerkiksi robotin tai tietokoneen kohdalla, edeltää sen olemista.

        Nuo mainitut olemisen perustyypit merkitsevät ratkaisevaa eroa myötäsyntyisen olennon ja keinotekoisen esineen välillä. Näin ollen hienoimmatkin robotit ja tietokoneet voivat vain etäisesti muistuttaa myötäsyntyisiä olentoja, kuten ihmisiä ja eläimiä.

        "Tuo eri todellisuustasojen ja olemistapojen välinen emergenssi voi siten olla täysin mahdoton. Tämä johtuu siitä, että ylemmän tason olemassa olo riippuu perustasosta, joten emergenssi ei voi ylittää fysikaalista ja mentaalista vastakkaisuutta. Nämä kaksihan määritellään jo lähtökohdissaan ominaisuuksiltaan vastakkaisiksi."

        Tähän voi sanoa, että lopeta määrittely vastakkaisiksi. Silloin pääset pois virheellisestä kysymyksen asettelusta.

        "Mikäli siis mentaalisuus olisi aineen emergentti ominaisuus ja mentaalinen ylätaso riippuisi aineellisesta perustasosta, joka puolestaan selitetään aineen perustason ominaisuuksilla, silloin mentaalisuus jäisi pelkäksi aivoprosessin heijastukseksi, jolla ei siten voisi olla kausaalista yhteyttä takaisin aivoprosesseihin."

        Emergessiä ei määritellä noin. Mainitsemasi olisi reduktiota, jossa fysikaalisen tason ominaisuuksilla selitettään tyhjentävästi mentaalisen tason ominaisuuksia. Emergenssi tarkoittaa sitä, että mentaalisella tasolla ilmenevät ominaisuudet eivät ole palautettavissa kokonaan fysikaalisen tason ilmiöihin. Kielikuvana emergentti on enemmän kuin sen osiensa summa.

        Mikä tuo "enemmän" on, siihen ei ole yleensä kyetty vastaamaan. Seitysvaihtoehtoina on prosessien kompleksisuus synnyttäisi jollakin tavalla uusia ominaisuuksia ylemmälle tasolle.

        " Mentaalisuudella kuitenkin tiedetään olevan vaikutuksia kehollisiin toimintoihin."

        Tämä on kylläkin oletus, joka perustuu tietoisuuden saamaan vaikutelmaan omista kyvyistään vaikuttaa kehoon. Todisteita sille ei ole, koska asian luonnetta ei ymmärretä.

        "Nuo mainitut olemisen perustyypit merkitsevät ratkaisevaa eroa myötäsyntyisen olennon ja keinotekoisen esineen välillä. Näin ollen hienoimmatkin robotit ja tietokoneet voivat vain etäisesti muistuttaa myötäsyntyisiä olentoja, kuten ihmisiä ja eläimiä."

        Kirjoittamasi on tältä osin spekulaatiota, joka ei perustu mihinkään. (Olen perehtynyt hyvin Heideggerin eksistentialismiin ja pidän hänen pohdintojaan tärkeinä ihmisen itseymmärryksen lisääjänä. Todellisuuden kanssa niillä ei kuitenkaan välttämättä ole mitään tekemistä.)


      • Belisario
        Järkisyitä kirjoitti:

        ”Hyvä esimerkki on esim. tuo kaukonäkeminen (remote viewing) tai oikeammin kaukokokeminen ja monet muut ilmiöt ja taidot joihin materialismiin jumittunut tiede ei edes halua tutkia koska pitää niitä jo oletusarvoisesti mahdottomina.”

        CIA tutki 1970-luvulla kaiken maailman psyykkisiä ilmiöitä. Nuo tutkimukset julkaistiin joku aika sitten ja yllätys yllätys: mitään psyykkisiä ilmiöitä kuten kaukonäkeminen ei löydetty ja tutkimus lakkautettiin tuloksettomana. Jos kaukonäkeminen oikeasti toimisi, niin toki moinen olisi jo löydetty ja se aiheuttaisi mm. tietoturva alalle ja vakoilutoimintaan ihan uusia haasteita. Täyttä huuhaata!

        "CIA tutki 1970-luvulla kaiken maailman psyykkisiä ilmiöitä. Nuo tutkimukset julkaistiin joku aika sitten ja yllätys yllätys: mitään psyykkisiä ilmiöitä kuten kaukonäkeminen ei löydetty ja tutkimus lakkautettiin tuloksettomana. Jos kaukonäkeminen oikeasti toimisi, niin toki moinen olisi jo löydetty ja se aiheuttaisi mm. tietoturva alalle ja vakoilutoimintaan ihan uusia haasteita. Täyttä huuhaata!"

        Pakko puuttua tähän.

        Kaukonäkemistä rahoitettiin n. 20 vuotta n. 1975-1995 USA:n eri "aakkosvirastojen" toimesta ja rahoitus arvioitiin uudestaan joka vuosi. Kylmän sodan aikana USA:n vakoilukoneistoa ja tutkimuksen rahoittajia kiinnosti ainoastaan se toimiko se vai ei ja jos se kaukonäkeminen ei olisi toiminut niin tuskin sitä oli rahoitettu yli 20 vuotta varsinkin kun kyse oli ilmeisen kiistanalaisesta asiasta varsinkin jos sitä valtavirtatieteen kannalta arvioi.

        Kaukonäkemisprojekti oli koko toimintansa ajan salainen ja julkistettiin vasta 90-luvun lopussa. Kyllä siitä aineistosta näkee että se toimi ja sitä käytettiin menestyksellisesti vakoilun apuvälineenä. Neuvostoliitolla oli oma vastaava projektinsa. Ei yhden skeptikon lausunto projektin julkistamisvaiheessa kovin paljon paina kun löytyy mm. tilastotieteen professori Jessica Utts joka tämän skeptikon kanssa täysin eri mieltä kaukonäkemisen toimivuudesta.

        Jo ennen 2. maailmansotaa Saksa perusti Ahnenerbe-laitoksen jonka tarkoituksena oli tutkia kaikki mahdollinen arkeologinen sekä parapsykologiaan liityvä aineisto tarkoituksena löytyy sieltä mitä tahansa mitä voisi käyttää sotilaallisiin tarkoituksiin. Suuri osa tutkimuksesta tietenkin tuotti paljon huuhaata mutta suurin osa Ahnenerben aineistosta on edelleenkin salaista.

        "Jos kaukonäkeminen oikeasti toimisi, niin toki moinen olisi jo löydetty ja se aiheuttaisi mm. tietoturva alalle ja vakoilutoimintaan ihan uusia haasteita. Täyttä huuhaata!"

        Älä viitsi naurattaa! Ne haasteet ilmaantuivat jo 1970-luvulla tai mahdollisesti jo aikaisemmin kun saatiin vihiä N-liiton vastaavista projekteista. Kaukonäkemisen idea ei muutenkaan ole mitenkään vasta viime vuosisadan keksintöä vaikka vasta Ingo Schwann muokkasi siitä selkeän teknisen protokollan jota alettiin kutsua kontrolloiduksi kaukonäkemiseksi.

        Vastaavia "paranormaaleja" kokemuksia on ihmisillä ollut niin kauan kuin kirjallista aineistoa on olemassa eli tuhansia vuosia ja vaikuttaa siltä että tullaiset kyvyt ovat hyvin yleisiä kaikilla biologisilla olioilla vaikka harvemmin nykyään esim, ihmisellä ovat tietoisia ja tahdonalaisia. Kännykät ja internet ovat varmaan tehneet telepatian yms. melko tarpeettomaksi.

        Tottakai julkisuudessa halutaan kaukonäkemisen tyyppiset asiat häivyttää marginaaliin kuten hypnoosillekin on aika pitkälle tapahtunut. Myös ns. ufoista löytyy valtavasti julkistettua virallista aineistoa ja sitten myös vastaavasti valtavasti disinformaatiota kuten parapsykologian ilmiöistäkin.

        Tyypillisen löysää argumentoitia taas sinulta. Ei yhden wiki-sivun lukemisesta paljoa ymmärrä eikä varsinkaan skeposivustojen tietoisesta disinformaatiosta. Kiistanalaisten asioiden suhteen wikipedia melko epäluotettava lähde muutenkin.

        ....


        Nyt voitte jatkaa harjoituksia materialismin ja idealismin syvimmästä olemuksesta. Itse siirryn nyt takaisin katsomon puolelle. Pitäkää hauskaa!!


      • iloinenpiereskelijä
        Belisario kirjoitti:

        "CIA tutki 1970-luvulla kaiken maailman psyykkisiä ilmiöitä. Nuo tutkimukset julkaistiin joku aika sitten ja yllätys yllätys: mitään psyykkisiä ilmiöitä kuten kaukonäkeminen ei löydetty ja tutkimus lakkautettiin tuloksettomana. Jos kaukonäkeminen oikeasti toimisi, niin toki moinen olisi jo löydetty ja se aiheuttaisi mm. tietoturva alalle ja vakoilutoimintaan ihan uusia haasteita. Täyttä huuhaata!"

        Pakko puuttua tähän.

        Kaukonäkemistä rahoitettiin n. 20 vuotta n. 1975-1995 USA:n eri "aakkosvirastojen" toimesta ja rahoitus arvioitiin uudestaan joka vuosi. Kylmän sodan aikana USA:n vakoilukoneistoa ja tutkimuksen rahoittajia kiinnosti ainoastaan se toimiko se vai ei ja jos se kaukonäkeminen ei olisi toiminut niin tuskin sitä oli rahoitettu yli 20 vuotta varsinkin kun kyse oli ilmeisen kiistanalaisesta asiasta varsinkin jos sitä valtavirtatieteen kannalta arvioi.

        Kaukonäkemisprojekti oli koko toimintansa ajan salainen ja julkistettiin vasta 90-luvun lopussa. Kyllä siitä aineistosta näkee että se toimi ja sitä käytettiin menestyksellisesti vakoilun apuvälineenä. Neuvostoliitolla oli oma vastaava projektinsa. Ei yhden skeptikon lausunto projektin julkistamisvaiheessa kovin paljon paina kun löytyy mm. tilastotieteen professori Jessica Utts joka tämän skeptikon kanssa täysin eri mieltä kaukonäkemisen toimivuudesta.

        Jo ennen 2. maailmansotaa Saksa perusti Ahnenerbe-laitoksen jonka tarkoituksena oli tutkia kaikki mahdollinen arkeologinen sekä parapsykologiaan liityvä aineisto tarkoituksena löytyy sieltä mitä tahansa mitä voisi käyttää sotilaallisiin tarkoituksiin. Suuri osa tutkimuksesta tietenkin tuotti paljon huuhaata mutta suurin osa Ahnenerben aineistosta on edelleenkin salaista.

        "Jos kaukonäkeminen oikeasti toimisi, niin toki moinen olisi jo löydetty ja se aiheuttaisi mm. tietoturva alalle ja vakoilutoimintaan ihan uusia haasteita. Täyttä huuhaata!"

        Älä viitsi naurattaa! Ne haasteet ilmaantuivat jo 1970-luvulla tai mahdollisesti jo aikaisemmin kun saatiin vihiä N-liiton vastaavista projekteista. Kaukonäkemisen idea ei muutenkaan ole mitenkään vasta viime vuosisadan keksintöä vaikka vasta Ingo Schwann muokkasi siitä selkeän teknisen protokollan jota alettiin kutsua kontrolloiduksi kaukonäkemiseksi.

        Vastaavia "paranormaaleja" kokemuksia on ihmisillä ollut niin kauan kuin kirjallista aineistoa on olemassa eli tuhansia vuosia ja vaikuttaa siltä että tullaiset kyvyt ovat hyvin yleisiä kaikilla biologisilla olioilla vaikka harvemmin nykyään esim, ihmisellä ovat tietoisia ja tahdonalaisia. Kännykät ja internet ovat varmaan tehneet telepatian yms. melko tarpeettomaksi.

        Tottakai julkisuudessa halutaan kaukonäkemisen tyyppiset asiat häivyttää marginaaliin kuten hypnoosillekin on aika pitkälle tapahtunut. Myös ns. ufoista löytyy valtavasti julkistettua virallista aineistoa ja sitten myös vastaavasti valtavasti disinformaatiota kuten parapsykologian ilmiöistäkin.

        Tyypillisen löysää argumentoitia taas sinulta. Ei yhden wiki-sivun lukemisesta paljoa ymmärrä eikä varsinkaan skeposivustojen tietoisesta disinformaatiosta. Kiistanalaisten asioiden suhteen wikipedia melko epäluotettava lähde muutenkin.

        ....


        Nyt voitte jatkaa harjoituksia materialismin ja idealismin syvimmästä olemuksesta. Itse siirryn nyt takaisin katsomon puolelle. Pitäkää hauskaa!!

        "Kylmän sodan aikana USA:n vakoilukoneistoa ja tutkimuksen rahoittajia kiinnosti ainoastaan se toimiko se vai ei ja jos se kaukonäkeminen ei olisi toiminut niin tuskin sitä oli rahoitettu yli 20 vuotta varsinkin kun kyse oli ilmeisen kiistanalaisesta asiasta varsinkin jos sitä valtavirtatieteen kannalta arvioi."

        Miksei olisi rahoitettu, koska kylmä sota oli erittäin paranoidi ajanjakso, jossa kaikkea toisen osapuolen tekemää vastaan oli löydettävä vastavaikutus. Niinpä rahoittajaa petettiin surutta, jos siltä rahaa oli saatavissa ihan mihin tahansa huuhaahan. 20 v ei ole pitkä aika, kun kylmän sodan paine oli koko ajan päällä.

        Etkö yhtään ihmettele, että "toimivan" ilmiön tutkiminen loppui sen jälkeen, kun 1991 Neuvostoliitto romahti?


      • kiikkustuolista
        koittakaakehitellä kirjoitti:

        "Tuo eri todellisuustasojen ja olemistapojen välinen emergenssi voi siten olla täysin mahdoton. Tämä johtuu siitä, että ylemmän tason olemassa olo riippuu perustasosta, joten emergenssi ei voi ylittää fysikaalista ja mentaalista vastakkaisuutta. Nämä kaksihan määritellään jo lähtökohdissaan ominaisuuksiltaan vastakkaisiksi."

        Tähän voi sanoa, että lopeta määrittely vastakkaisiksi. Silloin pääset pois virheellisestä kysymyksen asettelusta.

        "Mikäli siis mentaalisuus olisi aineen emergentti ominaisuus ja mentaalinen ylätaso riippuisi aineellisesta perustasosta, joka puolestaan selitetään aineen perustason ominaisuuksilla, silloin mentaalisuus jäisi pelkäksi aivoprosessin heijastukseksi, jolla ei siten voisi olla kausaalista yhteyttä takaisin aivoprosesseihin."

        Emergessiä ei määritellä noin. Mainitsemasi olisi reduktiota, jossa fysikaalisen tason ominaisuuksilla selitettään tyhjentävästi mentaalisen tason ominaisuuksia. Emergenssi tarkoittaa sitä, että mentaalisella tasolla ilmenevät ominaisuudet eivät ole palautettavissa kokonaan fysikaalisen tason ilmiöihin. Kielikuvana emergentti on enemmän kuin sen osiensa summa.

        Mikä tuo "enemmän" on, siihen ei ole yleensä kyetty vastaamaan. Seitysvaihtoehtoina on prosessien kompleksisuus synnyttäisi jollakin tavalla uusia ominaisuuksia ylemmälle tasolle.

        " Mentaalisuudella kuitenkin tiedetään olevan vaikutuksia kehollisiin toimintoihin."

        Tämä on kylläkin oletus, joka perustuu tietoisuuden saamaan vaikutelmaan omista kyvyistään vaikuttaa kehoon. Todisteita sille ei ole, koska asian luonnetta ei ymmärretä.

        "Nuo mainitut olemisen perustyypit merkitsevät ratkaisevaa eroa myötäsyntyisen olennon ja keinotekoisen esineen välillä. Näin ollen hienoimmatkin robotit ja tietokoneet voivat vain etäisesti muistuttaa myötäsyntyisiä olentoja, kuten ihmisiä ja eläimiä."

        Kirjoittamasi on tältä osin spekulaatiota, joka ei perustu mihinkään. (Olen perehtynyt hyvin Heideggerin eksistentialismiin ja pidän hänen pohdintojaan tärkeinä ihmisen itseymmärryksen lisääjänä. Todellisuuden kanssa niillä ei kuitenkaan välttämättä ole mitään tekemistä.)

        Missä kohtaa olen todennut, että emergenssi ei olisi enempää kuin osiensa summa?

        "Emergenssi tarkoittaa sitä, että mentaalisella tasolla ilmenevät tapahtumat eivät ole palautettavissa kokonaan fysikaalisen tason ilmiöihin."

        Olet siis sitä mieltä, että osa palautuu. Mikä osa?

        Ethän vaan itse sekoita emergenssiä ja reduktiota toisiinsa? Ongelmia ei ratkaista siten, että muutellaan käsitteitä ja väännetään "todellisuutta"vastauksiin sopivaksi. Niin meneteltäessä kyse on pelkästään subjektiivisistä mielipiteistä.


      • koittakaakehitellä
        kiikkustuolista kirjoitti:

        Missä kohtaa olen todennut, että emergenssi ei olisi enempää kuin osiensa summa?

        "Emergenssi tarkoittaa sitä, että mentaalisella tasolla ilmenevät tapahtumat eivät ole palautettavissa kokonaan fysikaalisen tason ilmiöihin."

        Olet siis sitä mieltä, että osa palautuu. Mikä osa?

        Ethän vaan itse sekoita emergenssiä ja reduktiota toisiinsa? Ongelmia ei ratkaista siten, että muutellaan käsitteitä ja väännetään "todellisuutta"vastauksiin sopivaksi. Niin meneteltäessä kyse on pelkästään subjektiivisistä mielipiteistä.

        "mentaalinen ylätaso riippuisi aineellisesta perustasosta, joka puolestaan selitetään aineen perustason ominaisuuksilla, silloin mentaalisuus jäisi pelkäksi aivoprosessin heijastuksi"

        mMitä sitten tarkoitit yllä olevalla? Sen mukaan Mentaalisuus jäisi pelkäksi heijastukseksi aivoprosesseja. Itse tulkitsen kirjoittamasi reduktioksi, jossa mentaalisuus olisi siis palautettavissa fysikaalisen tason prosesseihin tyhjentävästi. Jos tarkoitit jotakin muuta, en löydä sitä mainitusta kohdasta.

        Emergenssistä on monta tulkintaa, mutta se edellyttää nähdäkseni ainakin jonkin ominaisuuden olemista palautumaton alemman tason ominaisuuksista.

        Tähän kysymykseesi "Olet siis sitä mieltä, että osa palautuu. Mikä osa?" vastaan: ei hajuakaan.


      • Järkisyitä
        kiikkustuolista kirjoitti:

        Tuo termi "kemiallinen kone" viittaa vääjäämättä koneen rakentajaan, olkoon se sitten Jumala tai insinööri.

        Emergenssiin liittyvä tasoajattelu on vielä sikäli ongelmallista, että fyysisen perustason yhteydet ylemmällä, eli mentaalisella tasolla olevaan emergenssiin ei selitä fysikaalisen ja mentaalisen maailman välistä suhdetta. Tuo eri todellisuustasojen ja olemistapojen välinen emergenssi voi siten olla täysin mahdoton. Tämä johtuu siitä, että ylemmän tason olemassa olo riippuu perustasosta, joten emergenssi ei voi ylittää fysikaalista ja mentaalista vastakkaisuutta. Nämä kaksihan määritellään jo lähtökohdissaan ominaisuuksiltaan vastakkaisiksi.

        Mikäli siis mentaalisuus olisi aineen emergentti ominaisuus ja mentaalinen ylätaso riippuisi aineellisesta perustasosta, joka puolestaan selitetään aineen perustason ominaisuuksilla, silloin mentaalisuus jäisi pelkäksi aivoprosessin heijastukseksi, jolla ei siten voisi olla kausaalista yhteyttä takaisin aivoprosesseihin. Mentaalisuudella kuitenkin tiedetään olevan vaikutuksia kehollisiin toimintoihin. Näin ollen emergenttisyys voi toimia pelkästään olemisen fyysisellä perustasolla.

        Kysymys olemisesta ja olemisen luonteesta on synnyttänyt kokonaisen filosofisen suuntauksen, nimittäin eksistentialismin. Siinä oleminen jaetaan kahteen perustyyppiin, nimittäin "oleva itselleen" ja "oleva itsessään". Ensin mainittu tarkoittaa sitä, että subjektin olemus, kuten vaikkapa ihmisellä, seuraa olemista. Vastaavasti "oleva itsessään" olemus, kuten esimerkiksi robotin tai tietokoneen kohdalla, edeltää sen olemista.

        Nuo mainitut olemisen perustyypit merkitsevät ratkaisevaa eroa myötäsyntyisen olennon ja keinotekoisen esineen välillä. Näin ollen hienoimmatkin robotit ja tietokoneet voivat vain etäisesti muistuttaa myötäsyntyisiä olentoja, kuten ihmisiä ja eläimiä.

        Tarkoitukseni ei ollut viitata ”kemiallinen kone” määritelmällä, että elämä olisi suunniteltu, vaan ainoastaan kuvata sitä, että elämä on kemiallinen prosessi, jonka mekanismi on jo hyvin tiedossa etenkin yksinkertaisimpien mikrobien osalta.

        Ymmärrän mieli-ruumis ongelman ja myönnän, ettei mekanismia mielen syntymiselle materian pohjalta tunneta. Meillä on kuitenkin jo paljonkin viitteitä, että mieli liittyy aivoihin ja ajatukset korreloivat aivojen sähkötoiminnan kanssa.

        ”Mikäli siis mentaalisuus olisi aineen emergentti ominaisuus ja mentaalinen ylätaso riippuisi aineellisesta perustasosta, joka puolestaan selitetään aineen perustason ominaisuuksilla, silloin mentaalisuus jäisi pelkäksi aivoprosessin heijastukseksi, jolla ei siten voisi olla kausaalista yhteyttä takaisin aivoprosesseihin.”

        Tuota päättelyketjua en hyväksy. Katson, että ajattelu on aivojen prosessi. Tuolla prosessilla on sekä tuloja (aistit) että lähtöjä (lihasten ohjaus yms). Kaikki perustuvat sähköimpulsseihin ja kemikaaleihin, jotka voivat säädellä koko koneiston toimintaa. Tämä on siis kuin kuvaamasi robotti, mutta katson (useimpien materialistien lailla), että tuo prosessi synnyttää tietoisuuden, joka kokee tuon hermotoiminnan ajatteluna. Tuo syntynyt tietoisuus siis ”on” se prosessi.

        Aivojen hermoverkko on kuin kirjan sivulla oleva muste, joka on vain fyysistä mustetta. Mutta kun muste muodostaa tiettyjä kuvioita siitä tulee jotain enemmän - siitä tulee romaanin hieno tarina. Ja lopulta tuo romaanin hieno tarina on kuitenkin vain mustetta paperilla.


      • kiikkustuolista
        koittakaakehitellä kirjoitti:

        "mentaalinen ylätaso riippuisi aineellisesta perustasosta, joka puolestaan selitetään aineen perustason ominaisuuksilla, silloin mentaalisuus jäisi pelkäksi aivoprosessin heijastuksi"

        mMitä sitten tarkoitit yllä olevalla? Sen mukaan Mentaalisuus jäisi pelkäksi heijastukseksi aivoprosesseja. Itse tulkitsen kirjoittamasi reduktioksi, jossa mentaalisuus olisi siis palautettavissa fysikaalisen tason prosesseihin tyhjentävästi. Jos tarkoitit jotakin muuta, en löydä sitä mainitusta kohdasta.

        Emergenssistä on monta tulkintaa, mutta se edellyttää nähdäkseni ainakin jonkin ominaisuuden olemista palautumaton alemman tason ominaisuuksista.

        Tähän kysymykseesi "Olet siis sitä mieltä, että osa palautuu. Mikä osa?" vastaan: ei hajuakaan.

        Vastaus tuohon kysymykseesi sisältyy sen kappaleen alkuosaan, josta olet lainauksen erottanut. Lainauksesi on siis osa kokonaisuudesta. Ehkä ymmärryksesi vahvistuu, kun tarkastelet asiaa deduktiivisesti, eli siirryt kokonaisuudesta osiin ja takaisin osista kokonaisuuteen.


      • kiikkustuolista
        Järkisyitä kirjoitti:

        Tarkoitukseni ei ollut viitata ”kemiallinen kone” määritelmällä, että elämä olisi suunniteltu, vaan ainoastaan kuvata sitä, että elämä on kemiallinen prosessi, jonka mekanismi on jo hyvin tiedossa etenkin yksinkertaisimpien mikrobien osalta.

        Ymmärrän mieli-ruumis ongelman ja myönnän, ettei mekanismia mielen syntymiselle materian pohjalta tunneta. Meillä on kuitenkin jo paljonkin viitteitä, että mieli liittyy aivoihin ja ajatukset korreloivat aivojen sähkötoiminnan kanssa.

        ”Mikäli siis mentaalisuus olisi aineen emergentti ominaisuus ja mentaalinen ylätaso riippuisi aineellisesta perustasosta, joka puolestaan selitetään aineen perustason ominaisuuksilla, silloin mentaalisuus jäisi pelkäksi aivoprosessin heijastukseksi, jolla ei siten voisi olla kausaalista yhteyttä takaisin aivoprosesseihin.”

        Tuota päättelyketjua en hyväksy. Katson, että ajattelu on aivojen prosessi. Tuolla prosessilla on sekä tuloja (aistit) että lähtöjä (lihasten ohjaus yms). Kaikki perustuvat sähköimpulsseihin ja kemikaaleihin, jotka voivat säädellä koko koneiston toimintaa. Tämä on siis kuin kuvaamasi robotti, mutta katson (useimpien materialistien lailla), että tuo prosessi synnyttää tietoisuuden, joka kokee tuon hermotoiminnan ajatteluna. Tuo syntynyt tietoisuus siis ”on” se prosessi.

        Aivojen hermoverkko on kuin kirjan sivulla oleva muste, joka on vain fyysistä mustetta. Mutta kun muste muodostaa tiettyjä kuvioita siitä tulee jotain enemmän - siitä tulee romaanin hieno tarina. Ja lopulta tuo romaanin hieno tarina on kuitenkin vain mustetta paperilla.

        Vaikka mielen ja materian välistä yhteyttä ei tunneta, niin pidät sitä vakaasti ehdottomana totuutena? Etkä näe siinä mitään ongelmaa? Sillä tavalla.

        Lebnizin kuuluisa myllyvertaus kuvaa hyvin mielen ja materian välistä ongelmallisuutta. Modernissa versiossa voisimme kuvitella niin suuret aivot, että mahdumme sen sisälle tarkastelemaan sen rakennetta. Aivojen rakenne on hyvinkin kuvailemasi kaltainen. Mutta emme havaitsisi mitään sellaista, joka kuvaisi mieltä. Vain ja ainoastaan sähkökemiallisia prosesseja.

        Näin ollen argumenttisi sisältää pelkkiä olettamuksia ja yleistyksiä. Se on pelkkää predikaateilla kikkailua.

        Lisäksi funkionalinen tapasi tarkastella tuota prosessia sivuuttaa mielen kvalitatiivisen puolen kokonaan.


      • koittakaakehitellä
        kiikkustuolista kirjoitti:

        Vastaus tuohon kysymykseesi sisältyy sen kappaleen alkuosaan, josta olet lainauksen erottanut. Lainauksesi on siis osa kokonaisuudesta. Ehkä ymmärryksesi vahvistuu, kun tarkastelet asiaa deduktiivisesti, eli siirryt kokonaisuudesta osiin ja takaisin osista kokonaisuuteen.

        Lainaisn alunperin koko kappaleen:
        "Mikäli siis mentaalisuus olisi aineen emergentti ominaisuus ja mentaalinen ylätaso riippuisi aineellisesta perustasosta, joka puolestaan selitetään aineen perustason ominaisuuksilla, silloin mentaalisuus jäisi pelkäksi aivoprosessin heijastukseksi, jolla ei siten voisi olla kausaalista yhteyttä takaisin aivoprosesseihin."

        Joten tulkitsen, että et ymmärrä emergenssiä oikein.

        Jos mentaalisuus olisi emergenssi ominaisuus ja "ylätaso" (mentaalinen taso) riippuisi aineellisesta perustastaan, silloin mentaalisuus EI jää pelkäksi heijastukseksi aivoprosesseista.

        Emergenssinä prosessina mentaaliselle tasolle syntyy uusia ominaisuuksia, joilla voi olla esim. toivomasi kausaalinen yhteys takaksin aivoprosesseihin. Tämä on luonteenomaista emergenssille muutokselle silloin, kun sen selittävän aineellisen tason prosessit ovat riittävän komplekseja.


      • Belisario
        iloinenpiereskelijä kirjoitti:

        "Kylmän sodan aikana USA:n vakoilukoneistoa ja tutkimuksen rahoittajia kiinnosti ainoastaan se toimiko se vai ei ja jos se kaukonäkeminen ei olisi toiminut niin tuskin sitä oli rahoitettu yli 20 vuotta varsinkin kun kyse oli ilmeisen kiistanalaisesta asiasta varsinkin jos sitä valtavirtatieteen kannalta arvioi."

        Miksei olisi rahoitettu, koska kylmä sota oli erittäin paranoidi ajanjakso, jossa kaikkea toisen osapuolen tekemää vastaan oli löydettävä vastavaikutus. Niinpä rahoittajaa petettiin surutta, jos siltä rahaa oli saatavissa ihan mihin tahansa huuhaahan. 20 v ei ole pitkä aika, kun kylmän sodan paine oli koko ajan päällä.

        Etkö yhtään ihmettele, että "toimivan" ilmiön tutkiminen loppui sen jälkeen, kun 1991 Neuvostoliitto romahti?

        "Etkö yhtään ihmettele, että "toimivan" ilmiön tutkiminen loppui sen jälkeen, kun 1991 Neuvostoliitto romahti?"

        Julkinen rahoitus loppui 1995 ja projekti oli pakko julkistaa koska projektin entiset jäsenet alkoivat kirjoittaa kirjoja aiheesta eli se siihen asti salainen projekti olisi tullut ilmi joka tapauksessa. Tietysti voi tästä aiheesta mutuilla mitä tahansa kukin mielensä mukaan kun ei ole yhtään tai on vain hyvin pintapuolisesti perehtynyt aiheeseen.

        Tietysti nuo ns. paranormaalit ilmiöt ovat kiusallisia joillekin varsinkin siksi ettei niitä voi mitenkään helposti yhdistää vallitsevaan reduktionistisen materialismin maailmankuvaan eikä niille ole olemassa oikeastaan mitään teoreettista toimivaa mallia eikä mielestäni sellaista teoriaa tai mallia voi eikä kannata edes kehittää koska ylipäätänsä teorioilla ja malleilla on arvoa pelkästään mahdollisen tulevaisuuden tekniikan kannalta ja jos tekniikka on jos valmiiksi olemassa niin miksi ylipäätänsä kehitellä mitään mallia kun voi kehitellä sitä tekniikkaa paremmaksi .

        Totuuden ymmärtämisen kannalta kaikki mahdolliset tieteelliset ja filosofiset mallit ja suuntaukset ovat virheellisiä, yksipuolisia ja turhia ja korkeintaan apuvälineenä hyödyllisiä niin kauan kuin niistä saa vielä jotain käytännön kannalta hyödyllistä irti. Noilla on pelkkä välinearvo eikä minkäänlaista itseisarvoa.

        Mihinkään ei kannata jäädä jumittamaan. Julistaudun nyt tässä tavallaan antitieteen ja antifilosofian kannattajaksi.... :-)

        .....

        Tuli tuosta idealismi vs materialismi aiheesta vielä mieleen että materialisti ei voi oikeastaan muuta kuin kannattaa materialismia koska hänen kokemusmaailmansa on materiakeskeinen ja materia tuntuu ainoalta konkreettiselta asialta kun taas tietoisuus koetaan vastaavasti epätodellisena ja häilyvänä.

        Sama pätee idealistin suhteen jolle taas materia ja biologinen taso tuntee jotenkin vieraalta ja se oma sisäinen todellisuus paljon konkreettisemmalta kun sen ns. ulkomaailman todellisuus.

        Luonnollinen kieli on hyvin pitkälle rakentunut aineellisten "käsin kosketeltavissa" olevien asioiden käsittelyyn ja sitä kokemusmaailma pystytään sillä kielellä kuvaamaan aika heikosti ja hyvin pitkälle samalla tavalla kuin esineitten maailmaa.

        Ihmisiä on hyvin erilaisia ja eri tasolla olevia ja aika pitkälle tuntuu olevan kovakoodattua perimään kokeeko yksilö itsensä ensi sijassa äly-(tiedenörttityyppi), tunne (taitelijatyyppi) - vai fyysiseksi(urheilijatyyppi) ihmiseksi noin karkeasti ilmaistuna.

        Kaikissa noissa kolmessa on vielä huomioitava laadullinen kehitystaso. Alin taso on automaattinen joka voi olla hyvinkin monimutkaista ja vaikuttaa kehittyneeltä mutta on todellisuudessa pelkkää ympäristön ja muiden matkimista. Tämä on se taso mihin ns. tekoäly voi parhaimmillaan yltää. Tämä automaattinen taso on myös se taso mihin ehdollistavalla koulutuksella korkeintaan päästään ja millaisia ihmiset yleensä "luonnon tuottamina" ovat.

        Tekoäly ei kykene koskaan tuottamaan mitään uutta vaan ainoastaaan simuloimaan rakenteellisella tasolla Searlen kiinalaisen huoneen algoritmeja vaikkakin hyvin nopeasti.

        Vauvakasvoiset "söpöt" muka 1-vuotiaan tasolla olevat robotit ja maailmaa kiertävät "naispuoliset" tekoälyn "lähettiläät" (joille eräs ihmisoikeuksien suhteen primitiivinen valtio on myöntänyt jopa kansallisuuden) vaikuttavat olevan aika rankanpuoleista ihmisten manipulaatiota ja bluffia jolla yritetään huijata lähinnä lisää rahoitusta tekoälyn käärmeöljykauppiaiden taskuun.

        Se automaattinen tai mekaaninen taso tuntuu myös olevan se taso
        missä ei oikein pystytä hahmottamaan laajoja kokonaisuuksia vaan mielummin turvaudutaan jonkinlaiseen algoritmiseen tai mekaanisen loogiseen tai matemaattiseen tapaan ajatella kun intuitio ja minkäänlainen luovuus ei luonnistu.

        Ihmisen voi kyllä aika helposti muokata samalle tai huonommalle tasolle kuin robotin mutta robotteja ei voi muokata sille tasolle mihin kehittyneimmät ihmisyksilöt parhaimmillaan pystyvät.

        Vastailen jatkossa jos sattuu huvittamaan tai ehdin muilta askareilta vaikka kieltämättä tämä ketju on ainakin vähän kiinnostavampi verrattuna palstan loputtomiin jaaritteluihin lukemattomissa kosmisen pierun (BB)-ketjuissa.


      • LuetHömppää
        Belisario kirjoitti:

        "Etkö yhtään ihmettele, että "toimivan" ilmiön tutkiminen loppui sen jälkeen, kun 1991 Neuvostoliitto romahti?"

        Julkinen rahoitus loppui 1995 ja projekti oli pakko julkistaa koska projektin entiset jäsenet alkoivat kirjoittaa kirjoja aiheesta eli se siihen asti salainen projekti olisi tullut ilmi joka tapauksessa. Tietysti voi tästä aiheesta mutuilla mitä tahansa kukin mielensä mukaan kun ei ole yhtään tai on vain hyvin pintapuolisesti perehtynyt aiheeseen.

        Tietysti nuo ns. paranormaalit ilmiöt ovat kiusallisia joillekin varsinkin siksi ettei niitä voi mitenkään helposti yhdistää vallitsevaan reduktionistisen materialismin maailmankuvaan eikä niille ole olemassa oikeastaan mitään teoreettista toimivaa mallia eikä mielestäni sellaista teoriaa tai mallia voi eikä kannata edes kehittää koska ylipäätänsä teorioilla ja malleilla on arvoa pelkästään mahdollisen tulevaisuuden tekniikan kannalta ja jos tekniikka on jos valmiiksi olemassa niin miksi ylipäätänsä kehitellä mitään mallia kun voi kehitellä sitä tekniikkaa paremmaksi .

        Totuuden ymmärtämisen kannalta kaikki mahdolliset tieteelliset ja filosofiset mallit ja suuntaukset ovat virheellisiä, yksipuolisia ja turhia ja korkeintaan apuvälineenä hyödyllisiä niin kauan kuin niistä saa vielä jotain käytännön kannalta hyödyllistä irti. Noilla on pelkkä välinearvo eikä minkäänlaista itseisarvoa.

        Mihinkään ei kannata jäädä jumittamaan. Julistaudun nyt tässä tavallaan antitieteen ja antifilosofian kannattajaksi.... :-)

        .....

        Tuli tuosta idealismi vs materialismi aiheesta vielä mieleen että materialisti ei voi oikeastaan muuta kuin kannattaa materialismia koska hänen kokemusmaailmansa on materiakeskeinen ja materia tuntuu ainoalta konkreettiselta asialta kun taas tietoisuus koetaan vastaavasti epätodellisena ja häilyvänä.

        Sama pätee idealistin suhteen jolle taas materia ja biologinen taso tuntee jotenkin vieraalta ja se oma sisäinen todellisuus paljon konkreettisemmalta kun sen ns. ulkomaailman todellisuus.

        Luonnollinen kieli on hyvin pitkälle rakentunut aineellisten "käsin kosketeltavissa" olevien asioiden käsittelyyn ja sitä kokemusmaailma pystytään sillä kielellä kuvaamaan aika heikosti ja hyvin pitkälle samalla tavalla kuin esineitten maailmaa.

        Ihmisiä on hyvin erilaisia ja eri tasolla olevia ja aika pitkälle tuntuu olevan kovakoodattua perimään kokeeko yksilö itsensä ensi sijassa äly-(tiedenörttityyppi), tunne (taitelijatyyppi) - vai fyysiseksi(urheilijatyyppi) ihmiseksi noin karkeasti ilmaistuna.

        Kaikissa noissa kolmessa on vielä huomioitava laadullinen kehitystaso. Alin taso on automaattinen joka voi olla hyvinkin monimutkaista ja vaikuttaa kehittyneeltä mutta on todellisuudessa pelkkää ympäristön ja muiden matkimista. Tämä on se taso mihin ns. tekoäly voi parhaimmillaan yltää. Tämä automaattinen taso on myös se taso mihin ehdollistavalla koulutuksella korkeintaan päästään ja millaisia ihmiset yleensä "luonnon tuottamina" ovat.

        Tekoäly ei kykene koskaan tuottamaan mitään uutta vaan ainoastaaan simuloimaan rakenteellisella tasolla Searlen kiinalaisen huoneen algoritmeja vaikkakin hyvin nopeasti.

        Vauvakasvoiset "söpöt" muka 1-vuotiaan tasolla olevat robotit ja maailmaa kiertävät "naispuoliset" tekoälyn "lähettiläät" (joille eräs ihmisoikeuksien suhteen primitiivinen valtio on myöntänyt jopa kansallisuuden) vaikuttavat olevan aika rankanpuoleista ihmisten manipulaatiota ja bluffia jolla yritetään huijata lähinnä lisää rahoitusta tekoälyn käärmeöljykauppiaiden taskuun.

        Se automaattinen tai mekaaninen taso tuntuu myös olevan se taso
        missä ei oikein pystytä hahmottamaan laajoja kokonaisuuksia vaan mielummin turvaudutaan jonkinlaiseen algoritmiseen tai mekaanisen loogiseen tai matemaattiseen tapaan ajatella kun intuitio ja minkäänlainen luovuus ei luonnistu.

        Ihmisen voi kyllä aika helposti muokata samalle tai huonommalle tasolle kuin robotin mutta robotteja ei voi muokata sille tasolle mihin kehittyneimmät ihmisyksilöt parhaimmillaan pystyvät.

        Vastailen jatkossa jos sattuu huvittamaan tai ehdin muilta askareilta vaikka kieltämättä tämä ketju on ainakin vähän kiinnostavampi verrattuna palstan loputtomiin jaaritteluihin lukemattomissa kosmisen pierun (BB)-ketjuissa.

        "Julkinen rahoitus loppui 1995 ja projekti oli pakko julkistaa koska projektin entiset jäsenet alkoivat kirjoittaa kirjoja aiheesta eli se siihen asti salainen projekti olisi tullut ilmi joka tapauksessa."

        Kuinka uskottavina pidät tuollaisia kirjoja?
        Ensinnäkin salassapitovelvollisuus on elinikäinen, ellei salassapidolle ole erikseen asetettu takarajaa. Jos oikeasti projektissa mukana olleet olisivat paljastaneet salaiseksi luokiteltua tietoa, olisi häkki heilahtanut välittömästi.

        Mikä onkaan helpompaa kuin väittää olleensa sisäpiiriläinen. Salaisten projektien henkilöstöä kun harvoin julkaistaan ennen kuin projekti on menettänyt salattavuutensa. Kirjoja on voinut kirjoittaa kuka tahansa ja niihin on voinut satuilla mitä tahansa. Virallisen näköisiä asiakirjoja on helppo väärentää.

        Missä lähdekritiikki?


      • Belisario
        LuetHömppää kirjoitti:

        "Julkinen rahoitus loppui 1995 ja projekti oli pakko julkistaa koska projektin entiset jäsenet alkoivat kirjoittaa kirjoja aiheesta eli se siihen asti salainen projekti olisi tullut ilmi joka tapauksessa."

        Kuinka uskottavina pidät tuollaisia kirjoja?
        Ensinnäkin salassapitovelvollisuus on elinikäinen, ellei salassapidolle ole erikseen asetettu takarajaa. Jos oikeasti projektissa mukana olleet olisivat paljastaneet salaiseksi luokiteltua tietoa, olisi häkki heilahtanut välittömästi.

        Mikä onkaan helpompaa kuin väittää olleensa sisäpiiriläinen. Salaisten projektien henkilöstöä kun harvoin julkaistaan ennen kuin projekti on menettänyt salattavuutensa. Kirjoja on voinut kirjoittaa kuka tahansa ja niihin on voinut satuilla mitä tahansa. Virallisen näköisiä asiakirjoja on helppo väärentää.

        Missä lähdekritiikki?

        "Ensinnäkin salassapitovelvollisuus on elinikäinen, ellei salassapidolle ole erikseen asetettu takarajaa. Jos oikeasti projektissa mukana olleet olisivat paljastaneet salaiseksi luokiteltua tietoa, olisi häkki heilahtanut välittömästi."

        Ei se itse kaukonäkemisen tekniikka ollut salaista koska esim. telepatiasta ja vastaavista on kirjoitettu tieteellisiä tutkimuksia jo n. 150 vuotta. Salaista olivat tietenkin ne yksityiskohdat mitä vakoiltiin kunnes osa niistäkin julkistettiin myöhemmin koska kylmä sota oli jo loppunut.

        Ilmeisesti niitä kaukonäkemistä koskevia kirjoja alkoi ilmestyä vasta kun se salainen projekti oli muutettu ei-salaiseksi mutta joka tapauksessa sen projektin paljastuminen lienee ollut muutenkin vain ajan kysymys.

        "Mikä onkaan helpompaa kuin väittää olleensa sisäpiiriläinen. Salaisten projektien henkilöstöä kun harvoin julkaistaan ennen kuin projekti on menettänyt salattavuutensa. Kirjoja on voinut kirjoittaa kuka tahansa ja niihin on voinut satuilla mitä tahansa. Virallisen näköisiä asiakirjoja on helppo väärentää. "

        Jenkeissä oli ns. Freedom of Information act laki joka perusteella mm. ufoja koskevaa aineistoa julkistettin.

        https://en.wikipedia.org/wiki/Freedom_of_Information_Act_(United_States)

        Kyseessä on siis USA:n hallituksen viralliset julkistetut asiakirjat joissa osa informaatiosta on usein peitetty. Eihän tuota salaisten kaukonäkemisprojektien olemassaoloa kukaan vähänkään asiaa tunteva kiellä nykyään (ei edes sellaiset skeptikot jotka ovat vaivautuneet edes vähän tutkimaan asiaa).

        Sitten on tietysti erikseen ns. vuodetut asiakirjat (esim. ufopiireissä ns. MJ 12 dokumentit) joiden todenperäisyys voidaan kyseenalaistaa. Ns. sisäpiirijuttuihin ja "pillinpuhaltajien" juttuihin saa ja pitääkin suhtautua hyvin kriittisesti koska kuka tahansa voi periaatteessa väittää mitä tahansa (esim. joku Corey Goode yms.) mutta aineistoa löytyy kyllä ihan virallisistakin lähteistä ja joidenkin tiedeyhteisössä vaikuttavien tutkijoiden julkaisuista joka sekin voi olla tietysti harhaanjohtavaa mutta jos useampi toisistaan riippumaton lähde viittaa samaan suuntaan niin asia on ainakin tutkimisen arvoinen. Ei sen parempia lähteitä löydy muistakaan akateemisen tutkimuksen aiheista varsinkin kun on kyse historiasta.

        "Missä lähdekritiikki?"

        Jos harrastaa näiden asioiden suhteen täydellistä denialismia niin mitä vaan voi väittää.Skeptikoiden nykyinen taktiikka taitaa olla kaiken olemassaolevan 150 vuoden parapsykologian tutkimuksen tuloksien täydellinen kieltäminen ja ehdoton haluttomuus edes perehtyä niihin tutkimuksiin. Ainoa argumentti on ettei sellaista voi olla olemassa kun se muka on vastoin tunnettuja luonnonlakeja (vaikka ei edes välttämättä ole kaikissa tapauksissa).

        Ylipäätänsä ns skeptikoiden kanssa väittely tästä aiheesta on ajan tuhlausta. Vielä parikymmentä vuotta sitten oli skepoillakin jonkinlaista yritystä mutta ovat ilmeisesti tulleet joko laiskaksi tai todenneet että asiapohjalta joutuvat alakynteen väistämättä.

        Skeptikkosivustojen ja skepojen höpötykset uppoavat pääsääntöisti vain niihin jotka eivät aiheesta ennestään mitään tiedä eivätkä jaksa edes perehtyä tarkemmin. Sokea luottamus tieteen valepuvussa esiintyvään materialismiin tai fysikalismin ideologiaan taitaa olla hyvin vahvaa nykyään.

        Kyllä edelleenkin on olemassa useita paranormaalista kiinnostuneita tutkijoita jotka ovat valmiita vaarantamaan maineensa saadakseen totuuden julki. 1800-luvun lopussa sellaisia tutkijoita olivat mm. fyysikko ja Royal Societyn jäsen William Crookes ja evoteorian toinen kehittäjä Alfred Russell Wallace. Tällä vuosituhannella heitä on vähemmän ilmapiirin kiristymisen vuoksi (esim. Nobel palkittua Brian Josepsonia ei päästetty luennoimaan koska kuultiin hänen olevan kiinnostunut parapsykologiasta). Nykyään tiedeyhteisön ja skepojen parapsykologian vastainen asenne lähentelee jo jonkinlaista joukkopsykoottista vainoharhaa.

        Melko naurettavia ja kökköjä vastaväitteitä taas teikäläisiltä.

        ......

        Sitten varsinaiseen ketjun aiheeseen:

        Idealismin vastakohta on fysikalismi eli sellainen materialismi joka hyväksyy olemassaolevaksi ainoastaan sellaiset asiat jotka voi mitata olemassaolevilla laitteistoilla ja joihin on olemassa matemaattiset teoriat ja mallinnukset.

        Fysikalistit tietenkin kieltävät kaiken ns. paranormaalin olemassaolon koska kuvittelevat tietoisuuden syntyvän sähkökemiallisesti aivojen ja hermoston sivuotuotteena vaikka eivät edelleenkään pysty alkuunkaan selittämään miten kokemuksellinen tietoisuus voi syntyä siitä sähkökemiasta. Korrelaatiota hermoston ja tietoisuuden välillä kukaan ei tietenkään kiistä.


      • kiikkustuolista
        koittakaakehitellä kirjoitti:

        Lainaisn alunperin koko kappaleen:
        "Mikäli siis mentaalisuus olisi aineen emergentti ominaisuus ja mentaalinen ylätaso riippuisi aineellisesta perustasosta, joka puolestaan selitetään aineen perustason ominaisuuksilla, silloin mentaalisuus jäisi pelkäksi aivoprosessin heijastukseksi, jolla ei siten voisi olla kausaalista yhteyttä takaisin aivoprosesseihin."

        Joten tulkitsen, että et ymmärrä emergenssiä oikein.

        Jos mentaalisuus olisi emergenssi ominaisuus ja "ylätaso" (mentaalinen taso) riippuisi aineellisesta perustastaan, silloin mentaalisuus EI jää pelkäksi heijastukseksi aivoprosesseista.

        Emergenssinä prosessina mentaaliselle tasolle syntyy uusia ominaisuuksia, joilla voi olla esim. toivomasi kausaalinen yhteys takaksin aivoprosesseihin. Tämä on luonteenomaista emergenssille muutokselle silloin, kun sen selittävän aineellisen tason prosessit ovat riittävän komplekseja.

        En kyllä ole toivonut yhtään mitään. Olen vain todennut, että aineen emergoituminen tietoisuudeksi on ongelmallista. Jos maailma koostuu pelkästään aineellisista objekteista, niin on epätodennäköistä, että niistä emergoituisi mitään aineetonta. Jos taas materialistit väittävät tietoisuutta aineelliseksi, niin mitkä ovat tietoisuuden materiaaliset ominaisuudet ja ulottuvuudet? Materialistit eivät näköjään ymmärrä, että materialismi on pelkkä teoreettisen järjen muodollinen idea, apriori, jolla ei ole sisältöä.


      • Järkisyitä
        kiikkustuolista kirjoitti:

        Vaikka mielen ja materian välistä yhteyttä ei tunneta, niin pidät sitä vakaasti ehdottomana totuutena? Etkä näe siinä mitään ongelmaa? Sillä tavalla.

        Lebnizin kuuluisa myllyvertaus kuvaa hyvin mielen ja materian välistä ongelmallisuutta. Modernissa versiossa voisimme kuvitella niin suuret aivot, että mahdumme sen sisälle tarkastelemaan sen rakennetta. Aivojen rakenne on hyvinkin kuvailemasi kaltainen. Mutta emme havaitsisi mitään sellaista, joka kuvaisi mieltä. Vain ja ainoastaan sähkökemiallisia prosesseja.

        Näin ollen argumenttisi sisältää pelkkiä olettamuksia ja yleistyksiä. Se on pelkkää predikaateilla kikkailua.

        Lisäksi funkionalinen tapasi tarkastella tuota prosessia sivuuttaa mielen kvalitatiivisen puolen kokonaan.

        "Vaikka mielen ja materian välistä yhteyttä ei tunneta, niin pidät sitä vakaasti ehdottomana totuutena?"

        En pidä mitään ehdottoman totena. Uskon siihen, mihin havainnot mielestäni viittaavat. Kun nykyisin ihmisen mielessään näkema kuva saadaan luettua mittaamalla aivoja, niin eikö tämä ole aika vakuuttava todiste hermotoiminnan ja tietoisuuden yhteydestä?

        "Lebnizin kuuluisa myllyvertaus kuvaa hyvin mielen ja materian ongelmallisuutta .... Mutta emme havaitsisi mitään sellaista, joka kuvaisi mieltä. Vain ja ainoastaan sähkökemiallisia prosesseja."

        Nuo prosessit ovat ajatuksia. Tuo on kuin menisi kaiuttimen sisään katselemaan kaiuttimen kalvon värinää ja valittaisi, ettei näe ääntä missään.


      • koittakaakehitellä
        kiikkustuolista kirjoitti:

        En kyllä ole toivonut yhtään mitään. Olen vain todennut, että aineen emergoituminen tietoisuudeksi on ongelmallista. Jos maailma koostuu pelkästään aineellisista objekteista, niin on epätodennäköistä, että niistä emergoituisi mitään aineetonta. Jos taas materialistit väittävät tietoisuutta aineelliseksi, niin mitkä ovat tietoisuuden materiaaliset ominaisuudet ja ulottuvuudet? Materialistit eivät näköjään ymmärrä, että materialismi on pelkkä teoreettisen järjen muodollinen idea, apriori, jolla ei ole sisältöä.

        " Jos maailma koostuu pelkästään aineellisista objekteista, niin on epätodennäköistä, että niistä emergoituisi mitään aineetonta."

        Onko jossain tutkimushankkeessa näin todettu, vai mihin perustuu tod.näköisyytesi?

        Mielestäni asiaa pitää tutkia lisää ja monissa aivotutkimuksissa on viitteitä juuri siitä, että emergenssi selittäisi miten aineettomasta todellakin nousee esiin tietoisuuden subjektiivinen kokemus. Ei näe mitään estettä, miksei aineettomasti voisi syntyä mielen (aineettomia) kokemuksia.

        "Jos taas materialistit väittävät tietoisuutta aineelliseksi, niin mitkä ovat tietoisuuden materiaaliset ominaisuudet ja ulottuvuudet?"

        Fysikalistit, kuten Kari Enqvist näin ajattelee. (Hän tosin mainitsee myös emergenssin, mutta pitää sitä vain tietämättömyydestä johtuvana "karkeistuksena", jolloin tietoisuus siis olisi täysin materiaalinen, joka syntyy kompleksin aivorakenteen ja ympäristön kokonaisvaikutuksena.) Käsitykseni mukaan Enqvist voisi nähdä tietoisuuden toiminnan, jos olisi olemassa vain riittävän suuren laskutehon omaava supertietokone, joka kalkyloisi kaikki em vuorovaikutukset.


      • koittakaakehitellä
        koittakaakehitellä kirjoitti:

        " Jos maailma koostuu pelkästään aineellisista objekteista, niin on epätodennäköistä, että niistä emergoituisi mitään aineetonta."

        Onko jossain tutkimushankkeessa näin todettu, vai mihin perustuu tod.näköisyytesi?

        Mielestäni asiaa pitää tutkia lisää ja monissa aivotutkimuksissa on viitteitä juuri siitä, että emergenssi selittäisi miten aineettomasta todellakin nousee esiin tietoisuuden subjektiivinen kokemus. Ei näe mitään estettä, miksei aineettomasti voisi syntyä mielen (aineettomia) kokemuksia.

        "Jos taas materialistit väittävät tietoisuutta aineelliseksi, niin mitkä ovat tietoisuuden materiaaliset ominaisuudet ja ulottuvuudet?"

        Fysikalistit, kuten Kari Enqvist näin ajattelee. (Hän tosin mainitsee myös emergenssin, mutta pitää sitä vain tietämättömyydestä johtuvana "karkeistuksena", jolloin tietoisuus siis olisi täysin materiaalinen, joka syntyy kompleksin aivorakenteen ja ympäristön kokonaisvaikutuksena.) Käsitykseni mukaan Enqvist voisi nähdä tietoisuuden toiminnan, jos olisi olemassa vain riittävän suuren laskutehon omaava supertietokone, joka kalkyloisi kaikki em vuorovaikutukset.

        Korjaan: Mielestäni asiaa pitää tutkia lisää ja monissa aivotutkimuksissa on viitteitä juuri siitä, että emergenssi selittäisi miten aineellisesta todellakin nousee esiin tietoisuuden subjektiivinen kokemus. Ei näe mitään estettä, miksei aineellisesta voisi syntyä mielen (aineettomia) kokemuksia.


      • kiikkustuolista

        Alkaa menemään jankutukseksi.


      • Belisario
        Järkisyitä kirjoitti:

        "Vaikka mielen ja materian välistä yhteyttä ei tunneta, niin pidät sitä vakaasti ehdottomana totuutena?"

        En pidä mitään ehdottoman totena. Uskon siihen, mihin havainnot mielestäni viittaavat. Kun nykyisin ihmisen mielessään näkema kuva saadaan luettua mittaamalla aivoja, niin eikö tämä ole aika vakuuttava todiste hermotoiminnan ja tietoisuuden yhteydestä?

        "Lebnizin kuuluisa myllyvertaus kuvaa hyvin mielen ja materian ongelmallisuutta .... Mutta emme havaitsisi mitään sellaista, joka kuvaisi mieltä. Vain ja ainoastaan sähkökemiallisia prosesseja."

        Nuo prosessit ovat ajatuksia. Tuo on kuin menisi kaiuttimen sisään katselemaan kaiuttimen kalvon värinää ja valittaisi, ettei näe ääntä missään.

        "Kun nykyisin ihmisen mielessään näkema kuva saadaan luettua mittaamalla aivoja, niin eikö tämä ole aika vakuuttava todiste hermotoiminnan ja tietoisuuden yhteydestä?"

        Onhan hermotoiminnan ja tietoisuuden välillä tietenkin yhteys - nimittäin se neurokorrelaatio. Nykyään kun aivojen kuvantaminen on edistynyt jo aika pitkälle niin periaatteessa voitaisiin rakentaa jonkinlainen tietokanta erilaisista ajatuksista, tunteista ja aistimuksista kun tarpeeksi saadaan kerättyä aineistoa eri yksilöiden kokemuksesta yhdistettynä heidän kertomuksiinsa siitä mitä he kokevat.

        Todennäköisesti tätä samaa tietokantaa voisi myös käyttää näiden samojen psyykkisten prosessien aiheuttamiseen tietokantatemplaatilla aivoissa. Jos ja kun aivojen kuvantamislaitteet saadaan sillä tavalla pieneen kokoon että koehenkilöt voivat pitää niitä päällä jatkuvasti kaikissa mahdollisissa normaaleissa toiminnoissaan niin tällaisesta tietokannasta voitaisiin saada varsin kattava ja toimiva.

        Patentteja tähän suuntaan on olemassa jo 1970 ja 1980-luvuilta;

        Device for the induction of specific brain patterns 22.6.1982

        http://pdfpiw.uspto.gov/.piw?Docid=4335710&idkey=NONE&homeurl=http://patft.uspto.gov/netahtml/PTO/patimg.htm

        Tuo ei kyllä sinänsä mielestäni tue sitä ideaa että pelkästään tunnetun aineen tietty rakenne tuottaisi tietoisuuden vaan todennäköisempi mahdollisuus minusta on juuri se J.G. Bennettin energiataulukko jossa korkeammanlaatuiset energiat organisoivat niitä alemman laatuisia energioita jolloin se alempilaatuinen energiaa korreloi sen ylempilaatuisen kanssa ts. psyyken energia organisoi aivojen sähkökemiallista tasoa.

        Esoteerisissa opeissa erotetaan toisistaan fysikaalinen keho, eetterikeho joka antaa muodon fysikaaliselle keholle, astraalikeho joka vastaa tunteita sekä älykeho joka on älyn käyttöväline.

        Nämä eivät ole sitä fyysistä ja eetterikehoa lukuunottamatta vielä toimivia käyttövälineitä vaan pelkästään aihioita jotka elämän aikana joko organisoituvat suhteellisen itsenäiseksi käyttövälineeksi tai sitten vähitellen surkastuvat pois riippuen siitä onnistuuko yksilö omilla ponnistuksillaan saavuttamaan automaattisen energian tasoa kehittyneemmän tunne-elämän & älyelämän joka voisi sitten säilyä myös fyysisen kehon kuoleman jälkeen kun fyysisiä energioita ei ole enää mukana konvertoimassa alemman tason energiaa ylemmän tason energiaksi. Ylemmän tason energiakehot ovat ikäänkuin perhonen kuoriutumassa fyysisen tason toukka-asteesta.

        Eetterikeho on havaittavissa esim. Kirlian kuvauksella kun taas tunne & älykehot ovat ainoastaan subjektiivisesti koettavissa. Tämäkin malli on oikeastaan idealistinen vaikka ei suljekaan pois materian tasoa vaan tulkitsee materian eri laadut informaation integroitumistason perusteella. Kyse ei ole myöskään dualismista koska aineen ja psyyken vuorovaikutus selitetään eri energian/aineen tasoilla.


      • Belisario
        koittakaakehitellä kirjoitti:

        " Jos maailma koostuu pelkästään aineellisista objekteista, niin on epätodennäköistä, että niistä emergoituisi mitään aineetonta."

        Onko jossain tutkimushankkeessa näin todettu, vai mihin perustuu tod.näköisyytesi?

        Mielestäni asiaa pitää tutkia lisää ja monissa aivotutkimuksissa on viitteitä juuri siitä, että emergenssi selittäisi miten aineettomasta todellakin nousee esiin tietoisuuden subjektiivinen kokemus. Ei näe mitään estettä, miksei aineettomasti voisi syntyä mielen (aineettomia) kokemuksia.

        "Jos taas materialistit väittävät tietoisuutta aineelliseksi, niin mitkä ovat tietoisuuden materiaaliset ominaisuudet ja ulottuvuudet?"

        Fysikalistit, kuten Kari Enqvist näin ajattelee. (Hän tosin mainitsee myös emergenssin, mutta pitää sitä vain tietämättömyydestä johtuvana "karkeistuksena", jolloin tietoisuus siis olisi täysin materiaalinen, joka syntyy kompleksin aivorakenteen ja ympäristön kokonaisvaikutuksena.) Käsitykseni mukaan Enqvist voisi nähdä tietoisuuden toiminnan, jos olisi olemassa vain riittävän suuren laskutehon omaava supertietokone, joka kalkyloisi kaikki em vuorovaikutukset.

        "Käsitykseni mukaan Enqvist voisi nähdä tietoisuuden toiminnan, jos olisi olemassa vain riittävän suuren laskutehon omaava supertietokone, joka kalkyloisi kaikki em vuorovaikutukset."


        Tuo on samaa haaveilua kuin La Placella aikoinaan. Vaikka pystyttäisiin kartoittamaan jonkun ihmisen aivot atomien tarkkuudella jonkinlaiseen simulaatioon niin se tuskin toimisi samalla tavalla kuin se alkup. ihminen koska se simuloitu rakenne olisi vain pelkkä pysäytyskuva ilman niitä prosesseja mitä aivoissa tapahtuu. Ehkä vasta sillä tavalla kuitenkin sitten vähitellen varmistuu että ko. malli ei toimi.

        Tekoäytutkimus todennäköisesti törmää myös umpikujaan ellei jo ole törmännyt vaikka se ei tarkoitakaan että tekoäly olisi täysin hyödytöntä vaan että se kelpaa vain apuvälineeksi ei päättäjäksi eikä tärkeiden valintojen tekijäksi.

        Tietoisuuden toimintaa ei voi "nähdä" kun se ei ole sama asia kuin niiden aivojen atomien kulloinenkin konfiguraatio josta voidaan saada tietoa vain jos se kyseinen koehenkilö itse kertoo mitä tuntee kokevansa ja simulaatiossa ei ole sitä kokijaa joka voisi kertoa mitä kokee.

        Aika näyttää aikanaan mikä on totuus ja siinä matkan varrella ennen sitä totuuden löytämistä aivan varmasti kokeillaan kaikkea kuviteltavissa olevaa ja sitten vihdoin huomataan mitä jää jäljelle ainoaksi mahdollisuudeksi eli juuri se idealismi jossa tietoisuus on aina ensisijainen ja ns. aine aina joko toissijainen tai pelkkä tietoisuuden tuottama illuusio.


      • HaaveitaVain
        Belisario kirjoitti:

        "Käsitykseni mukaan Enqvist voisi nähdä tietoisuuden toiminnan, jos olisi olemassa vain riittävän suuren laskutehon omaava supertietokone, joka kalkyloisi kaikki em vuorovaikutukset."


        Tuo on samaa haaveilua kuin La Placella aikoinaan. Vaikka pystyttäisiin kartoittamaan jonkun ihmisen aivot atomien tarkkuudella jonkinlaiseen simulaatioon niin se tuskin toimisi samalla tavalla kuin se alkup. ihminen koska se simuloitu rakenne olisi vain pelkkä pysäytyskuva ilman niitä prosesseja mitä aivoissa tapahtuu. Ehkä vasta sillä tavalla kuitenkin sitten vähitellen varmistuu että ko. malli ei toimi.

        Tekoäytutkimus todennäköisesti törmää myös umpikujaan ellei jo ole törmännyt vaikka se ei tarkoitakaan että tekoäly olisi täysin hyödytöntä vaan että se kelpaa vain apuvälineeksi ei päättäjäksi eikä tärkeiden valintojen tekijäksi.

        Tietoisuuden toimintaa ei voi "nähdä" kun se ei ole sama asia kuin niiden aivojen atomien kulloinenkin konfiguraatio josta voidaan saada tietoa vain jos se kyseinen koehenkilö itse kertoo mitä tuntee kokevansa ja simulaatiossa ei ole sitä kokijaa joka voisi kertoa mitä kokee.

        Aika näyttää aikanaan mikä on totuus ja siinä matkan varrella ennen sitä totuuden löytämistä aivan varmasti kokeillaan kaikkea kuviteltavissa olevaa ja sitten vihdoin huomataan mitä jää jäljelle ainoaksi mahdollisuudeksi eli juuri se idealismi jossa tietoisuus on aina ensisijainen ja ns. aine aina joko toissijainen tai pelkkä tietoisuuden tuottama illuusio.

        "...huomataan mitä jää jäljelle ainoaksi mahdollisuudeksi eli juuri se idealismi jossa tietoisuus on aina ensisijainen ja ns. aine aina joko toissijainen tai pelkkä tietoisuuden tuottama illuusio. "

        Näin tuskin ikinä tulee käymään. Panpsykismi ei ole millään järjellisellä määrittelyllä luonnontiedettä. Kysessä on lähinnä uskonnollinen uskomus, eikä tiede ole ikinä historiansa aikana kulkenut takaisin kohti taikauskoa.

        Mitä tekoälyyn tulee, on sen kehittyminen väistämätöntä. Sillä rintamalla kehitys kiihtyy lähes eksponentiaalisesti, eikä suinkaan ole pysähtynyt, kuten väitit. En tiedä, mitä tärkeillä valinnoilla tarkoitat, mutta tosiasia on, että kone peittoaa ihmisen jo nyt lukuisilla eri aloilla, eikä kehitykselle näy olevan rajaa.


      • koittakaakehitellä
        Belisario kirjoitti:

        "Kun nykyisin ihmisen mielessään näkema kuva saadaan luettua mittaamalla aivoja, niin eikö tämä ole aika vakuuttava todiste hermotoiminnan ja tietoisuuden yhteydestä?"

        Onhan hermotoiminnan ja tietoisuuden välillä tietenkin yhteys - nimittäin se neurokorrelaatio. Nykyään kun aivojen kuvantaminen on edistynyt jo aika pitkälle niin periaatteessa voitaisiin rakentaa jonkinlainen tietokanta erilaisista ajatuksista, tunteista ja aistimuksista kun tarpeeksi saadaan kerättyä aineistoa eri yksilöiden kokemuksesta yhdistettynä heidän kertomuksiinsa siitä mitä he kokevat.

        Todennäköisesti tätä samaa tietokantaa voisi myös käyttää näiden samojen psyykkisten prosessien aiheuttamiseen tietokantatemplaatilla aivoissa. Jos ja kun aivojen kuvantamislaitteet saadaan sillä tavalla pieneen kokoon että koehenkilöt voivat pitää niitä päällä jatkuvasti kaikissa mahdollisissa normaaleissa toiminnoissaan niin tällaisesta tietokannasta voitaisiin saada varsin kattava ja toimiva.

        Patentteja tähän suuntaan on olemassa jo 1970 ja 1980-luvuilta;

        Device for the induction of specific brain patterns 22.6.1982

        http://pdfpiw.uspto.gov/.piw?Docid=4335710&idkey=NONE&homeurl=http://patft.uspto.gov/netahtml/PTO/patimg.htm

        Tuo ei kyllä sinänsä mielestäni tue sitä ideaa että pelkästään tunnetun aineen tietty rakenne tuottaisi tietoisuuden vaan todennäköisempi mahdollisuus minusta on juuri se J.G. Bennettin energiataulukko jossa korkeammanlaatuiset energiat organisoivat niitä alemman laatuisia energioita jolloin se alempilaatuinen energiaa korreloi sen ylempilaatuisen kanssa ts. psyyken energia organisoi aivojen sähkökemiallista tasoa.

        Esoteerisissa opeissa erotetaan toisistaan fysikaalinen keho, eetterikeho joka antaa muodon fysikaaliselle keholle, astraalikeho joka vastaa tunteita sekä älykeho joka on älyn käyttöväline.

        Nämä eivät ole sitä fyysistä ja eetterikehoa lukuunottamatta vielä toimivia käyttövälineitä vaan pelkästään aihioita jotka elämän aikana joko organisoituvat suhteellisen itsenäiseksi käyttövälineeksi tai sitten vähitellen surkastuvat pois riippuen siitä onnistuuko yksilö omilla ponnistuksillaan saavuttamaan automaattisen energian tasoa kehittyneemmän tunne-elämän & älyelämän joka voisi sitten säilyä myös fyysisen kehon kuoleman jälkeen kun fyysisiä energioita ei ole enää mukana konvertoimassa alemman tason energiaa ylemmän tason energiaksi. Ylemmän tason energiakehot ovat ikäänkuin perhonen kuoriutumassa fyysisen tason toukka-asteesta.

        Eetterikeho on havaittavissa esim. Kirlian kuvauksella kun taas tunne & älykehot ovat ainoastaan subjektiivisesti koettavissa. Tämäkin malli on oikeastaan idealistinen vaikka ei suljekaan pois materian tasoa vaan tulkitsee materian eri laadut informaation integroitumistason perusteella. Kyse ei ole myöskään dualismista koska aineen ja psyyken vuorovaikutus selitetään eri energian/aineen tasoilla.

        "Onhan hermotoiminnan ja tietoisuuden välillä tietenkin yhteys - nimittäin se neurokorrelaatio."

        Eihän se ole vain korrelaatio, kun stimuloimalla aivoja saadaan aikaiseksi, että koehenkilö itse kokee liikuttavansa raajaansa. Se on syy-yhteys neuronista koehenkilön 1. persoona kokemukseen "tahdon liikuttaa varvastani". Emergentti muutos aineellisesta aineettomaan siis tapahtui.


      • Belisario
        HaaveitaVain kirjoitti:

        "...huomataan mitä jää jäljelle ainoaksi mahdollisuudeksi eli juuri se idealismi jossa tietoisuus on aina ensisijainen ja ns. aine aina joko toissijainen tai pelkkä tietoisuuden tuottama illuusio. "

        Näin tuskin ikinä tulee käymään. Panpsykismi ei ole millään järjellisellä määrittelyllä luonnontiedettä. Kysessä on lähinnä uskonnollinen uskomus, eikä tiede ole ikinä historiansa aikana kulkenut takaisin kohti taikauskoa.

        Mitä tekoälyyn tulee, on sen kehittyminen väistämätöntä. Sillä rintamalla kehitys kiihtyy lähes eksponentiaalisesti, eikä suinkaan ole pysähtynyt, kuten väitit. En tiedä, mitä tärkeillä valinnoilla tarkoitat, mutta tosiasia on, että kone peittoaa ihmisen jo nyt lukuisilla eri aloilla, eikä kehitykselle näy olevan rajaa.

        "Panpsykismi ei ole millään järjellisellä määrittelyllä luonnontiedettä. "

        Luonnontiede ei voikaan sellaisenaan tuottaa mitään lopullisia vastauksia ontologisiin kysymyksiin vaan ainoastaan mallintaa havaittuja säännönmukaisuuksia kaavoiksi jotka muuttuvat yhä monimutkaisemmiksi kun siirrytään fysiikan tasolta tarkastelemaan kemian ja biologian ilmiöitä.

        Panpsykismi ei muuten ole edes sama asia kuin idealismi vaikka molemmissa otetaan huomioon myös se 1. persoonan näkökulma ja siihen liittyvät kokemukset. Määrittelyjen tasolla "ajatteleminen" tai oikeammin prosessoiminen ei itse asiassa ole edes ajattelua vaan mekaanista käsitteiden manipulointia.

        Ymmärtäminen taas ei ole pohjimmiltaan verbaalista ja oikeastaan edellyttää sitä että pidättäydytään reaktiivisesta käsitteiden prosessoinnista ja yritetään sen sijaan hahmottaa mitä tarkoitetaan tai yritetään vajavaisen kielen keinoin ilmaista. Ihmisen automaattisella 2-arvoisen logiikan reaktiivisella tasolla ymmärtäminen ei ole edes mahdollista.

        "Kysessä on lähinnä uskonnollinen uskomus, eikä tiede ole ikinä historiansa aikana kulkenut takaisin kohti taikauskoa. "

        Uskonnolinen kokemus eikä muukaan kokemus ei ole taikauskoa vaan voi olla myös tieteellisen tutkimuksen kohteena. (esim. William James Varieties of religious experience)


        "Mitä tekoälyyn tulee, on sen kehittyminen väistämätöntä. "

        Jonkinlainen kehittyminen tietenkin mutta aika pitkälle siinäkin on mahdollisia samat asiat kuin tietotekniikassa eikä oikeastaan yhtään sen enempää. Väistämätöntä ei ole mikään tässä maailmassa.

        "Sillä rintamalla kehitys kiihtyy lähes eksponentiaalisesti, eikä suinkaan ole pysähtynyt, kuten väitit. "

        En kirjoittanut että se on pysähtynyt vaan että se törmää jossain vaiheessa umpikujaan koska se nimenomaan nojautuu niin vahvasti pohjimmiltaan mekaanisiin algoritmeihin joista ei millään konstilla saada aikaiseksi mitään tietoisuutta muistuttavaakaan eli siinä Searlen kiinalaisen huoneen ongelmassa jumitetaan varmaan loppumattomiin,

        "En tiedä, mitä tärkeillä valinnoilla tarkoitat, mutta tosiasia on, että kone peittoaa ihmisen jo nyt lukuisilla eri aloilla, eikä kehitykselle näy olevan rajaa."

        Kyllä se mekaanisen prosessoinnin nopeudessa "peittoaa" mikä tietenkin sitten vapauttaisi ihmisen keskittymään tärkeämpiin asioihin. Tekoälyn ja tietotekniikan perusongelma on sitten se että kone ei oikeasti ymmärrä mistään mitään eikä osaa hahmottaa sellaisia asioita joita sille ei ole ohjelmoitu joten ne koneiden mokat ovat sitten usein aika laajakantoisia ja sellaisia joihin kukaan normaali ihminen ei sortuisi.

        Aika pitkälle tietotekniikassa ja tekoälyssä on kyse siitä kuinka paljon sen tekniikan suunnittelija osaa ennakoida niitä mahdollisia ja odottomattomia tilanteita mihin se kone voi joutua. Kyllä tekoälyn laajamittaisesa käyttönotossa on olemassa kaikki ennennäkemättömän laajan katastrofin ainekset - todella suurten mokien tekemiseen tarvitaan suuria neroja.... :-)


      • Järkisyitä
        Belisario kirjoitti:

        "Kun nykyisin ihmisen mielessään näkema kuva saadaan luettua mittaamalla aivoja, niin eikö tämä ole aika vakuuttava todiste hermotoiminnan ja tietoisuuden yhteydestä?"

        Onhan hermotoiminnan ja tietoisuuden välillä tietenkin yhteys - nimittäin se neurokorrelaatio. Nykyään kun aivojen kuvantaminen on edistynyt jo aika pitkälle niin periaatteessa voitaisiin rakentaa jonkinlainen tietokanta erilaisista ajatuksista, tunteista ja aistimuksista kun tarpeeksi saadaan kerättyä aineistoa eri yksilöiden kokemuksesta yhdistettynä heidän kertomuksiinsa siitä mitä he kokevat.

        Todennäköisesti tätä samaa tietokantaa voisi myös käyttää näiden samojen psyykkisten prosessien aiheuttamiseen tietokantatemplaatilla aivoissa. Jos ja kun aivojen kuvantamislaitteet saadaan sillä tavalla pieneen kokoon että koehenkilöt voivat pitää niitä päällä jatkuvasti kaikissa mahdollisissa normaaleissa toiminnoissaan niin tällaisesta tietokannasta voitaisiin saada varsin kattava ja toimiva.

        Patentteja tähän suuntaan on olemassa jo 1970 ja 1980-luvuilta;

        Device for the induction of specific brain patterns 22.6.1982

        http://pdfpiw.uspto.gov/.piw?Docid=4335710&idkey=NONE&homeurl=http://patft.uspto.gov/netahtml/PTO/patimg.htm

        Tuo ei kyllä sinänsä mielestäni tue sitä ideaa että pelkästään tunnetun aineen tietty rakenne tuottaisi tietoisuuden vaan todennäköisempi mahdollisuus minusta on juuri se J.G. Bennettin energiataulukko jossa korkeammanlaatuiset energiat organisoivat niitä alemman laatuisia energioita jolloin se alempilaatuinen energiaa korreloi sen ylempilaatuisen kanssa ts. psyyken energia organisoi aivojen sähkökemiallista tasoa.

        Esoteerisissa opeissa erotetaan toisistaan fysikaalinen keho, eetterikeho joka antaa muodon fysikaaliselle keholle, astraalikeho joka vastaa tunteita sekä älykeho joka on älyn käyttöväline.

        Nämä eivät ole sitä fyysistä ja eetterikehoa lukuunottamatta vielä toimivia käyttövälineitä vaan pelkästään aihioita jotka elämän aikana joko organisoituvat suhteellisen itsenäiseksi käyttövälineeksi tai sitten vähitellen surkastuvat pois riippuen siitä onnistuuko yksilö omilla ponnistuksillaan saavuttamaan automaattisen energian tasoa kehittyneemmän tunne-elämän & älyelämän joka voisi sitten säilyä myös fyysisen kehon kuoleman jälkeen kun fyysisiä energioita ei ole enää mukana konvertoimassa alemman tason energiaa ylemmän tason energiaksi. Ylemmän tason energiakehot ovat ikäänkuin perhonen kuoriutumassa fyysisen tason toukka-asteesta.

        Eetterikeho on havaittavissa esim. Kirlian kuvauksella kun taas tunne & älykehot ovat ainoastaan subjektiivisesti koettavissa. Tämäkin malli on oikeastaan idealistinen vaikka ei suljekaan pois materian tasoa vaan tulkitsee materian eri laadut informaation integroitumistason perusteella. Kyse ei ole myöskään dualismista koska aineen ja psyyken vuorovaikutus selitetään eri energian/aineen tasoilla.

        "Bennettin energiataulukko jossa korkeammanlaatuiset energiat organisoivat niitä alemman laatuisia energioita jolloin se alempilaatuinen energiaa korreloi sen ylempilaatuisen kanssa ts. psyyken energia organisoi aivojen sähkökemiallista tasoa."

        Minua tökkii noissa se, että sepitellään mitä ihmeellisempää mentaalienergia soopaa ilman, että takana on mitään tutkimusta. Yhtä hyvin voit lukea buddhalaisuuden maailmamallista eli kyse on uakonnollisesta filosodiasta eikä aidosta halusta tutkia maailmaa.

        Voisin sanoa, että hammasratraiden liike on peräisin korkeammasta energiatasosta ja liikkeen välittyminen hammasrattaasta toiseen ei ole fyysisen maailman ilmiö vaan heijastusta tuosta korkeammasta energiatasosta. Mutta mitä järkeä tälläisessä höpötyksessä on?


      • Belisario
        Järkisyitä kirjoitti:

        "Bennettin energiataulukko jossa korkeammanlaatuiset energiat organisoivat niitä alemman laatuisia energioita jolloin se alempilaatuinen energiaa korreloi sen ylempilaatuisen kanssa ts. psyyken energia organisoi aivojen sähkökemiallista tasoa."

        Minua tökkii noissa se, että sepitellään mitä ihmeellisempää mentaalienergia soopaa ilman, että takana on mitään tutkimusta. Yhtä hyvin voit lukea buddhalaisuuden maailmamallista eli kyse on uakonnollisesta filosodiasta eikä aidosta halusta tutkia maailmaa.

        Voisin sanoa, että hammasratraiden liike on peräisin korkeammasta energiatasosta ja liikkeen välittyminen hammasrattaasta toiseen ei ole fyysisen maailman ilmiö vaan heijastusta tuosta korkeammasta energiatasosta. Mutta mitä järkeä tälläisessä höpötyksessä on?

        "Minua tökkii noissa se, että sepitellään mitä ihmeellisempää mentaalienergia soopaa ilman, että takana on mitään tutkimusta. "

        Eikö introspektio ole tutkimusta? Kyllä ne automaattitason, sensitiivitason, tietoisuuden tason sekä luovuuden tasot ovat kokemuksen tasolla selkeästi erilaisia toisiinsa nähden. Siihen toimivaan introspektioonkin tarvitaan systemaattista harjoittelua että osataan erottaa ne erilaiset psyyken ja käyttäytymisen taustalla olevat "voimat" toisistaan.

        Ns. "normaalissa" psyyken tilassa operoidaan älytasolla kyllä-ei vastakohtien kautta ja tunne ja fyysisillä tasoilla vastaavalla tavalla vastakohtien kautta (mielihyvä vs. kärsimys, hyvä vs paha jne) kun taas sillä korkeammalla tasolla koetaan samat asiat sillä tavalla että ymmärretään joka asiassa olevan ne yhtäaikaa ne molemmat vastakohtaiset asiat mutta eri suhteissa keskenään. Sitten on aina vielä aina kolmas neutralisoiva vaikutus niiden "plus" ja "miinus" vaikutteiden lisäksi jota ei taas normaalisti edes huomata.

        Kyse on tietenkin lähinnä käsitteellisistä työkaluista joilla sitä omaa kokemusta voi jäsentää. Se on sitten taas eri asia että nykyisen tekniikan mahdollistamilla mittauslaitteilla ei kyetä noita psyyken energioita ainakaan toistaiseksi mitenkään mittaamaan mutta se onkin taas fysikalismin ideologian aiheuttama puute.

        "Voisin sanoa, että hammasratraiden liike on peräisin korkeammasta energiatasosta ja liikkeen välittyminen hammasrattaasta toiseen ei ole fyysisen maailman ilmiö vaan heijastusta tuosta korkeammasta energiatasosta. Mutta mitä järkeä tälläisessä höpötyksessä on?"

        Ei tuossa sinun olkinukessasi olekaan mitään järkeä koska hammasrattaiden liike on selitettävissä tunnetuilla fysiikan energioilla kun taas psyyken toiminnot eivät ole.


      • VainMielenHarhaa
        Belisario kirjoitti:

        "Minua tökkii noissa se, että sepitellään mitä ihmeellisempää mentaalienergia soopaa ilman, että takana on mitään tutkimusta. "

        Eikö introspektio ole tutkimusta? Kyllä ne automaattitason, sensitiivitason, tietoisuuden tason sekä luovuuden tasot ovat kokemuksen tasolla selkeästi erilaisia toisiinsa nähden. Siihen toimivaan introspektioonkin tarvitaan systemaattista harjoittelua että osataan erottaa ne erilaiset psyyken ja käyttäytymisen taustalla olevat "voimat" toisistaan.

        Ns. "normaalissa" psyyken tilassa operoidaan älytasolla kyllä-ei vastakohtien kautta ja tunne ja fyysisillä tasoilla vastaavalla tavalla vastakohtien kautta (mielihyvä vs. kärsimys, hyvä vs paha jne) kun taas sillä korkeammalla tasolla koetaan samat asiat sillä tavalla että ymmärretään joka asiassa olevan ne yhtäaikaa ne molemmat vastakohtaiset asiat mutta eri suhteissa keskenään. Sitten on aina vielä aina kolmas neutralisoiva vaikutus niiden "plus" ja "miinus" vaikutteiden lisäksi jota ei taas normaalisti edes huomata.

        Kyse on tietenkin lähinnä käsitteellisistä työkaluista joilla sitä omaa kokemusta voi jäsentää. Se on sitten taas eri asia että nykyisen tekniikan mahdollistamilla mittauslaitteilla ei kyetä noita psyyken energioita ainakaan toistaiseksi mitenkään mittaamaan mutta se onkin taas fysikalismin ideologian aiheuttama puute.

        "Voisin sanoa, että hammasratraiden liike on peräisin korkeammasta energiatasosta ja liikkeen välittyminen hammasrattaasta toiseen ei ole fyysisen maailman ilmiö vaan heijastusta tuosta korkeammasta energiatasosta. Mutta mitä järkeä tälläisessä höpötyksessä on?"

        Ei tuossa sinun olkinukessasi olekaan mitään järkeä koska hammasrattaiden liike on selitettävissä tunnetuilla fysiikan energioilla kun taas psyyken toiminnot eivät ole.

        "Eikö introspektio ole tutkimusta? "

        Se on subjektiivisuutta pahimmillaan, josta tutkimuksen pitäisi päästä eroon.


      • Belisario
        VainMielenHarhaa kirjoitti:

        "Eikö introspektio ole tutkimusta? "

        Se on subjektiivisuutta pahimmillaan, josta tutkimuksen pitäisi päästä eroon.

        "Se on subjektiivisuutta pahimmillaan, josta tutkimuksen pitäisi päästä eroon."

        Heh. Mikäs olis sitten subjektiivisuutta parhaimmillaan vai oisko paras ettei olis kokemuksia ollenkaan?

        Varmaan siitä subjektiivisuudesta päästään eroon kun korvataan kaikki tieteiljät tekoälyllä ja aistit mittareilla ja saadaan siten toteutettua materistin unelma. Kun vielä siirretään sen materialistin pelkkä äly sinne koneeseen niin niistä ikävistä tunteista sekä kivusta ja särystä pääsee samalla eroon ja voi sitten elää ikuisesti. Kannattaa kyllä varoa mitä toivoo - se voi vielä toteutuakin ja tuottaa erinäisiä ylläreitä siinä sivussa.... :-)


      • Järkisyitä
        Belisario kirjoitti:

        "Minua tökkii noissa se, että sepitellään mitä ihmeellisempää mentaalienergia soopaa ilman, että takana on mitään tutkimusta. "

        Eikö introspektio ole tutkimusta? Kyllä ne automaattitason, sensitiivitason, tietoisuuden tason sekä luovuuden tasot ovat kokemuksen tasolla selkeästi erilaisia toisiinsa nähden. Siihen toimivaan introspektioonkin tarvitaan systemaattista harjoittelua että osataan erottaa ne erilaiset psyyken ja käyttäytymisen taustalla olevat "voimat" toisistaan.

        Ns. "normaalissa" psyyken tilassa operoidaan älytasolla kyllä-ei vastakohtien kautta ja tunne ja fyysisillä tasoilla vastaavalla tavalla vastakohtien kautta (mielihyvä vs. kärsimys, hyvä vs paha jne) kun taas sillä korkeammalla tasolla koetaan samat asiat sillä tavalla että ymmärretään joka asiassa olevan ne yhtäaikaa ne molemmat vastakohtaiset asiat mutta eri suhteissa keskenään. Sitten on aina vielä aina kolmas neutralisoiva vaikutus niiden "plus" ja "miinus" vaikutteiden lisäksi jota ei taas normaalisti edes huomata.

        Kyse on tietenkin lähinnä käsitteellisistä työkaluista joilla sitä omaa kokemusta voi jäsentää. Se on sitten taas eri asia että nykyisen tekniikan mahdollistamilla mittauslaitteilla ei kyetä noita psyyken energioita ainakaan toistaiseksi mitenkään mittaamaan mutta se onkin taas fysikalismin ideologian aiheuttama puute.

        "Voisin sanoa, että hammasratraiden liike on peräisin korkeammasta energiatasosta ja liikkeen välittyminen hammasrattaasta toiseen ei ole fyysisen maailman ilmiö vaan heijastusta tuosta korkeammasta energiatasosta. Mutta mitä järkeä tälläisessä höpötyksessä on?"

        Ei tuossa sinun olkinukessasi olekaan mitään järkeä koska hammasrattaiden liike on selitettävissä tunnetuilla fysiikan energioilla kun taas psyyken toiminnot eivät ole.

        ”Ei tuossa sinun olkinukessasi olekaan mitään järkeä koska hammasrattaiden liike on selitettävissä tunnetuilla fysiikan energioilla kun taas psyyken toiminnot eivät ole.”

        Jos emme ymmärtäisi hammasrattaiden toimintaa, niin olisiko mielestäsi silloin tuollainen selittely järkevää? Mielestäni se on juuri sitä, mitä mentaalienergioista puhuvat tekevät. Emme ymmärrä jotain, siispä jotkut höpöttävät selityksen.


      • koittakaakehitellä
        Belisario kirjoitti:

        "Minua tökkii noissa se, että sepitellään mitä ihmeellisempää mentaalienergia soopaa ilman, että takana on mitään tutkimusta. "

        Eikö introspektio ole tutkimusta? Kyllä ne automaattitason, sensitiivitason, tietoisuuden tason sekä luovuuden tasot ovat kokemuksen tasolla selkeästi erilaisia toisiinsa nähden. Siihen toimivaan introspektioonkin tarvitaan systemaattista harjoittelua että osataan erottaa ne erilaiset psyyken ja käyttäytymisen taustalla olevat "voimat" toisistaan.

        Ns. "normaalissa" psyyken tilassa operoidaan älytasolla kyllä-ei vastakohtien kautta ja tunne ja fyysisillä tasoilla vastaavalla tavalla vastakohtien kautta (mielihyvä vs. kärsimys, hyvä vs paha jne) kun taas sillä korkeammalla tasolla koetaan samat asiat sillä tavalla että ymmärretään joka asiassa olevan ne yhtäaikaa ne molemmat vastakohtaiset asiat mutta eri suhteissa keskenään. Sitten on aina vielä aina kolmas neutralisoiva vaikutus niiden "plus" ja "miinus" vaikutteiden lisäksi jota ei taas normaalisti edes huomata.

        Kyse on tietenkin lähinnä käsitteellisistä työkaluista joilla sitä omaa kokemusta voi jäsentää. Se on sitten taas eri asia että nykyisen tekniikan mahdollistamilla mittauslaitteilla ei kyetä noita psyyken energioita ainakaan toistaiseksi mitenkään mittaamaan mutta se onkin taas fysikalismin ideologian aiheuttama puute.

        "Voisin sanoa, että hammasratraiden liike on peräisin korkeammasta energiatasosta ja liikkeen välittyminen hammasrattaasta toiseen ei ole fyysisen maailman ilmiö vaan heijastusta tuosta korkeammasta energiatasosta. Mutta mitä järkeä tälläisessä höpötyksessä on?"

        Ei tuossa sinun olkinukessasi olekaan mitään järkeä koska hammasrattaiden liike on selitettävissä tunnetuilla fysiikan energioilla kun taas psyyken toiminnot eivät ole.

        "Eikö introspektio ole tutkimusta? Kyllä ne automaattitason, sensitiivitason, tietoisuuden tason sekä luovuuden tasot ovat kokemuksen tasolla selkeästi erilaisia toisiinsa nähden. Siihen toimivaan introspektioonkin tarvitaan systemaattista harjoittelua että osataan erottaa ne erilaiset psyyken ja käyttäytymisen taustalla olevat "voimat" toisistaan."

        Introspektio on itsensä kehittämistä mentaalisella tasolla, mutta se ei ole tieteellistä missään mielessä. Tiede ei toimi subjektiivisessa maailmassa.

        Näkemyksesi on liian musta-valkoinen. Materialisti kykenee täysin itsensä kehittämiseen, jos on sellaista kyvykkyyttä hankkinut siinä kuin muutkin. Filosofiset kysymyksen asettelut eivät rajaa ihmisen kokemusmaailmaa eikä itsensä kehittämisen mahdollisuuksia, ellei henkilö ole dogmaattinen kannanotoissan.

        "Varmaan siitä subjektiivisuudesta päästään eroon kun korvataan kaikki tieteiljät tekoälyllä ja aistit mittareilla ja saadaan siten toteutettua materistin unelma."

        En ymmärrä, miksi subjektiivisuudesta pitäisi eroon päästä, mutta tieteen tekemisessä se ei käy päinsä, koska tieteen kuuluu olla julkista ja avoimelle kritiikille altistettavaa. Subjektiivisia kokemuksia ei voi tällaiseen alistaa käytönnön syistä.

        Mentaalienergioista puhuminen on huvittavaa, koska kielikuva energiasta viittaa fysiikan maailmaan, vaikka subjektiivisessa kokemuksessa ei ole sellaisesta kysymys. Parempi olisi puhua vaikkapa mielen sisäisestä voimasta tms. subjektiiviseen maailman kuvailuun paremmin sopivista käsitteistä.


      • Järkisyitä
        Belisario kirjoitti:

        "Minua tökkii noissa se, että sepitellään mitä ihmeellisempää mentaalienergia soopaa ilman, että takana on mitään tutkimusta. "

        Eikö introspektio ole tutkimusta? Kyllä ne automaattitason, sensitiivitason, tietoisuuden tason sekä luovuuden tasot ovat kokemuksen tasolla selkeästi erilaisia toisiinsa nähden. Siihen toimivaan introspektioonkin tarvitaan systemaattista harjoittelua että osataan erottaa ne erilaiset psyyken ja käyttäytymisen taustalla olevat "voimat" toisistaan.

        Ns. "normaalissa" psyyken tilassa operoidaan älytasolla kyllä-ei vastakohtien kautta ja tunne ja fyysisillä tasoilla vastaavalla tavalla vastakohtien kautta (mielihyvä vs. kärsimys, hyvä vs paha jne) kun taas sillä korkeammalla tasolla koetaan samat asiat sillä tavalla että ymmärretään joka asiassa olevan ne yhtäaikaa ne molemmat vastakohtaiset asiat mutta eri suhteissa keskenään. Sitten on aina vielä aina kolmas neutralisoiva vaikutus niiden "plus" ja "miinus" vaikutteiden lisäksi jota ei taas normaalisti edes huomata.

        Kyse on tietenkin lähinnä käsitteellisistä työkaluista joilla sitä omaa kokemusta voi jäsentää. Se on sitten taas eri asia että nykyisen tekniikan mahdollistamilla mittauslaitteilla ei kyetä noita psyyken energioita ainakaan toistaiseksi mitenkään mittaamaan mutta se onkin taas fysikalismin ideologian aiheuttama puute.

        "Voisin sanoa, että hammasratraiden liike on peräisin korkeammasta energiatasosta ja liikkeen välittyminen hammasrattaasta toiseen ei ole fyysisen maailman ilmiö vaan heijastusta tuosta korkeammasta energiatasosta. Mutta mitä järkeä tälläisessä höpötyksessä on?"

        Ei tuossa sinun olkinukessasi olekaan mitään järkeä koska hammasrattaiden liike on selitettävissä tunnetuilla fysiikan energioilla kun taas psyyken toiminnot eivät ole.

        ”Eikö introspektio ole tutkimusta?”

        Kyllä sitä voi käyttää tutkimisen menetelmänä, mutta se rajoittuu vain itse kokemuksen tutkimiseen.

        Tässä yksi loistava video introspektiosta parhaimmillaan:
        https://www.youtube.com/watch?v=lyu7v7nWzfo


      • Belisario
        Järkisyitä kirjoitti:

        ”Ei tuossa sinun olkinukessasi olekaan mitään järkeä koska hammasrattaiden liike on selitettävissä tunnetuilla fysiikan energioilla kun taas psyyken toiminnot eivät ole.”

        Jos emme ymmärtäisi hammasrattaiden toimintaa, niin olisiko mielestäsi silloin tuollainen selittely järkevää? Mielestäni se on juuri sitä, mitä mentaalienergioista puhuvat tekevät. Emme ymmärrä jotain, siispä jotkut höpöttävät selityksen.

        "Jos emme ymmärtäisi hammasrattaiden toimintaa, niin olisiko mielestäsi silloin tuollainen selittely järkevää? "

        Ei tietenkään.

        "Mielestäni se on juuri sitä, mitä mentaalienergioista puhuvat tekevät. Emme ymmärrä jotain, siispä jotkut höpöttävät selityksen."

        Materialistin näkökulmasta tietoisuuden ymmärtäminen ilmeisesti tarkoittaa sitä että se tietoisuus pystytään palauttamaan johonkin ei-tietoiseen kuten esim. jonkinlaiseen rakenteeseen tai mekanismiin.

        Ontologiassa on kyse aina jostain ontologisesta primitivistä jonka pohjalta pyritään selittämään kaikki muu olevainen. Fysiikassa se ontologinen primitiivi tai perusoletus taitaa olla jonkinlainen kvanttikenttä jonka eksitaatioita ovat alkeishiukkaset joista taas oletetaan muodostuvan kaikki muu.

        Idealismissa se ontologinen primitiivi on itse se tietoisuus - ei kuitenkaan yksilön tietoisuus vaan kaiken kattava universaalitietoisuus jonka osia ne kaikki yksilölliset tietoisuudet ovat.

        Materialismissa se materia on kokemuksen säännönmukaisuuksista rakentunut abstraktio johon on postuloitu mm. aika, avaruus. hiukkaset, massa, varaus jne. Materialismi on hyvin pitkälle siitä alkuperäisestä kokemuksesta kehitetty abstraktio kun taas idealismissa ne kokemukset eivät ole abstrakteja muuten kuin jos niitä aletaan käsitteellisesti kommunikoida.

        Idealismi on siis perusoletusten suhteen paljon säästäväisempi kuin materialismi. Molemmat ovat kuitenkin ontologisia valintoja eli filosofisia valintoja ja asenteita suhteessa siihen välittömästi koettuun kokemusmaailmaan.

        Tieteen metodi ei sellaisenaan edellytä minkäänlaista ontologiaa vaikka varsinkin tieteen popularisoinneissa on lähes aina mukana materialistiseen ontologiaan viittaavia oletuksia.

        .....

        Mentalienergioista:

        Olen tuota jo aikaisemmin yrittänyt selittää sinulle aika laajasti mutta ilmeisesti torjuntasi tai "tökkimisesi" on niin voimakasta ettet pääse sen ylitse kun jo valmiiksi oletat että se huuhaata niin et viitsi yhtään vaivautua.

        Energioiden laatu (josta tässä on nyt kyse) on selkeä jatkumo tunnetuista energian laaduista eli alimmasta hajaenergiasta (lämpö), suuntaenergiaan (esim. sähkö), sidosenergiaan (kemian sidokset ja atomienergia) aina sinne mentaalienergioihin asti jotka voidaan kokea vain subjektiivisesti mutta joiden vaikutukset näkyvät selkeästi käyttäytymisessä ja siinä mitä saadaan aikaiseksi. Energia on kyky tehdä työtä ja jokainen erilainen työ vaatii toimiakseen oikean laatuista energiaa oikeassa määrässä ja oikeassa intensiteetissä.

        Ihmisellä on 3 erilaista aivoa tai keskusta (äly, tunne,fyysinen) joista jokainen voi toimia eri aikoina eri laatuisella energialla.

        Alin mentaalienergia on automaattienergia eli mekaaninen taso ja siinä kuluu suurin osa ajasta koska se suurin osa on nimenomaan opittua ja ehdollistettua toimintaa. Automaattinen taso voi olla hyvin nopea ja tehokas mutta siinä vaiheessa kun vasta opetellaan jotain niin tarvitaan korkeamman laatuista energiaa eli tarkkaavaisuutta jonka määrä ja intensiteetti on aina rajoitettua kuten kaikkien muidenkin energioiden.

        Nämä kaksi mentaalienergian tasoa pitäisi jokaisen pystyä erottamaan toisistaan. Muut korkeammat tasot ovat tavallisesti sen verran satunnaisesti ja harvoin ilmeneviä ettei niistä kannata tässä yhteydessä edes puhua.

        Itse asiassa noidenkin kahden alimman tason mentaalitoiminnan erottamiseen toisistaan tarvitaan jonkinasteista harjoittelua ainakin jo kyseessä on ns. ekstrovertti jonka tarkkaavaisuus on lähes aina suuntautunut ulospäin eikä juuri koskaan sisäänpäin paitsi ehkä joissain tunne-elämän kriiseissä ja silloinkin siitä sisäisen maailman kokemisesta halutaan päästä nopeasti eroon.

        ts. jos sinulle on materialistina tärkeintä nimenomaan se materia eri muodoissa (jopa uskonnot voivat olla hyvinkin materialistisia kuten esim. tuo mormonismi ja lestadiolaisuus) niin tietenkin se sisäinen kokemusmaailma jää aika vieraaksi etkä pidä sitä edes kovin tärkeänä.

        Oman tietoisuutesi kehittäminen ei ole sinusta mielenkiintoista vaan sinulle mielenkiintoista on kehittää koneelle se kuvitelmasi tietoisuudesta joka tuntuu helpolta koska se oma tietoisuuskin koetaan melko merkityksettömänä asiana joka kulkee siinä materiaalisen toiminnan sivussa.

        Sitten on tietenkin ne länsimaiseen kulttuuriin syvälle juurtuneet asenteet kaiken uskonnolliseen vähänkin viittaavien asioiden väheksyminen kun kaiken toimivan pitää tietenkin olla jotenkin aineellista ja mekaanista.

        Etävaikutukset kuten telepatia on tietenkin mahdotonta koska vuorovaikutukset voivat välittyä vain suoran kosketuksen kautta kuten biljardipallofysiikassa tapahtuu. Kvanttifysiikan tiettyjä asioita ei haluta ottaa vakavasti vaan nekin mieluitenn tulkitaan siten ettei vaan mikään tietoisuuteen viittaava voi olla mukana.

        Tekoälyä pidetään jonkinlaisena todisteena siitä että tietoisuus muodostuu algoritmeista ja mekaanisen fyysiikan toiminnoista ja sitten tietenkin unohdetaan kokonaan se että tekoäly ei synny itsestään.


      • Belisario
        koittakaakehitellä kirjoitti:

        "Eikö introspektio ole tutkimusta? Kyllä ne automaattitason, sensitiivitason, tietoisuuden tason sekä luovuuden tasot ovat kokemuksen tasolla selkeästi erilaisia toisiinsa nähden. Siihen toimivaan introspektioonkin tarvitaan systemaattista harjoittelua että osataan erottaa ne erilaiset psyyken ja käyttäytymisen taustalla olevat "voimat" toisistaan."

        Introspektio on itsensä kehittämistä mentaalisella tasolla, mutta se ei ole tieteellistä missään mielessä. Tiede ei toimi subjektiivisessa maailmassa.

        Näkemyksesi on liian musta-valkoinen. Materialisti kykenee täysin itsensä kehittämiseen, jos on sellaista kyvykkyyttä hankkinut siinä kuin muutkin. Filosofiset kysymyksen asettelut eivät rajaa ihmisen kokemusmaailmaa eikä itsensä kehittämisen mahdollisuuksia, ellei henkilö ole dogmaattinen kannanotoissan.

        "Varmaan siitä subjektiivisuudesta päästään eroon kun korvataan kaikki tieteiljät tekoälyllä ja aistit mittareilla ja saadaan siten toteutettua materistin unelma."

        En ymmärrä, miksi subjektiivisuudesta pitäisi eroon päästä, mutta tieteen tekemisessä se ei käy päinsä, koska tieteen kuuluu olla julkista ja avoimelle kritiikille altistettavaa. Subjektiivisia kokemuksia ei voi tällaiseen alistaa käytönnön syistä.

        Mentaalienergioista puhuminen on huvittavaa, koska kielikuva energiasta viittaa fysiikan maailmaan, vaikka subjektiivisessa kokemuksessa ei ole sellaisesta kysymys. Parempi olisi puhua vaikkapa mielen sisäisestä voimasta tms. subjektiiviseen maailman kuvailuun paremmin sopivista käsitteistä.

        "Introspektio on itsensä kehittämistä mentaalisella tasolla, mutta se ei ole tieteellistä missään mielessä. Tiede ei toimi subjektiivisessa maailmassa."

        Neurotiede ei toimi ilman introspektiota eli sitä miten saadaan selville ne neurokorrelaatit. Siinä sen tutkijan itsensä pitää olla koekaniinina jossa hän suoraan voi kokea miten jonkun aivojen osan stimulointi vaikuttaa siihen omaan kokemukseen tai sitten joku muu koehenkilö joka sanallisesti kertoo mitä kokee. Minusta se tutkija itse on samalla koehenkilö on toimivin lähestystapa.

        'Subjektiivinen' ja 'objektiivinen' käsitteet menevät aika iloisesti sekaisin koko ajan. Tiede perustuu aina yksilöiden aistihavaintoon joka on aina osa sen yksilön subjektiivisesta kokemusmaailmasta. Varsinaisesti mitään välitöntä tieteellistä ja objektiivista havaintoa ei ole edes olemassa eikä voi edes olla olemassa. Kaikki on aina joistain perustelemattomista perusoletuksista pääteltyä ja tulkittua.

        "Materialisti kykenee täysin itsensä kehittämiseen, jos on sellaista kyvykkyyttä hankkinut siinä kuin muutkin. Filosofiset kysymyksen asettelut eivät rajaa ihmisen kokemusmaailmaa eikä itsensä kehittämisen mahdollisuuksia, ellei henkilö ole dogmaattinen kannanotoissan."

        Tietenkin kykenee ja materialismia on hyvin erilaista ja esim. tuo J.G. Bennettin energioiden laadut voidaan tulkita myös puhtaasti materialistiselta kannalta. Idealismiakin on monenlaista eli Bernardo Kastrupin malli jossa hän pyrkii palauttamaan kaiken olemassaolevan tietoisuudessa olevaksi ja tekee sen varsin johdonmukaisesti. Sitten on tietysti kaikenlaista new age ja hippihöttöä joissa logiikka ja koherenssi on aika avutonta.

        "En ymmärrä, miksi subjektiivisuudesta pitäisi eroon päästä, mutta tieteen tekemisessä se ei käy päinsä, koska tieteen kuuluu olla julkista ja avoimelle kritiikille altistettavaa. Subjektiivisia kokemuksia ei voi tällaiseen alistaa käytönnön syistä."

        Niin se oli tarkoitettu lähinnä sarkasmiksi tai vitsiksi siihen mitä siinä kommentoin.

        Toistan vielä sen että ns. objektiivisia kokemuksia ei voi olla tieteessä eikä missään muussakaan koska kokemus on aina pohjimmiltaan subjektiivinen. Samasta syystä itse tietoisuutta ei voi tutkia objektiivisesti eikä se ole objektiivisesti kuvattavissa. Vain käyttäytymistä voidaan tutkia suhteellisen koherentisti. Tietoisuus on aina kaiken tutkimisen subjekti - ei koskaan objekti. Se näennäinen objektiivisuus on lähinnä loogista koherenssia muiden samanlaisten kokijoiden kokemusten kuvausten suhteen.


      • Belisario
        Järkisyitä kirjoitti:

        ”Eikö introspektio ole tutkimusta?”

        Kyllä sitä voi käyttää tutkimisen menetelmänä, mutta se rajoittuu vain itse kokemuksen tutkimiseen.

        Tässä yksi loistava video introspektiosta parhaimmillaan:
        https://www.youtube.com/watch?v=lyu7v7nWzfo

        "Kyllä sitä voi käyttää tutkimisen menetelmänä, mutta se rajoittuu vain itse kokemuksen tutkimiseen."


        Onko jotain muuta olemassa suoraan annettuna kuin se itse kokemus?

        "Tässä yksi loistava video introspektiosta parhaimmillaan:
        https://www.youtube.com/watch?v=lyu7v7nWzfo"

        Voi sen noinkin hahmottaa mutta tuossa käsitteen "aivot" voi aivan hyvin muuttaa käsitteeksi "tietoisuus" eikä lopputulos muutu muuten kuin ontologian kannalta. Tuon Anil Sethin ideat ovat muuten kyllä aika lähellä Kastrupin ja Tom Campbellin vastaavia vaikka ontologinen tulkinta onkin erilainen.

        Tiede sinänsä ei tarvitse ontologisia valintoja eikä tulkintoja vaan kaiken voi yhtä hyvin kuvata neutraalisti esim. systeemiteorian keinoin ja erilaisilla matemaattisilla malleilla jotka eivät nekään ota kantaa ontologiaan.

        Yleensä nykyään ihmiset muutenkin käyttävät käsitettä "aivot" kun puhuvat omasta kokemuksestaan. Materialismin ideologia on mennyt siis aika hyvin perille ns. common sense maailmankuvaan joka on usein melkein 100 vuotta ajastaan jäljessä.

        Tuollaisessa ajattelussa lisäksi sotketaan keskenään 2 ontologiaa eli se 1. persoonan kokemuksellisuus siihen 3. persoonan aistihavainnon kuvaukseen jossa aivot ovat vain lokalisoituneen tietoisuuden kuva. Kun näkökulmia sotketaan keskenään samaan päättelyyn niin lopputulos on lähinnä sekoilua.


      • koittakaakehitellä
        Belisario kirjoitti:

        "Introspektio on itsensä kehittämistä mentaalisella tasolla, mutta se ei ole tieteellistä missään mielessä. Tiede ei toimi subjektiivisessa maailmassa."

        Neurotiede ei toimi ilman introspektiota eli sitä miten saadaan selville ne neurokorrelaatit. Siinä sen tutkijan itsensä pitää olla koekaniinina jossa hän suoraan voi kokea miten jonkun aivojen osan stimulointi vaikuttaa siihen omaan kokemukseen tai sitten joku muu koehenkilö joka sanallisesti kertoo mitä kokee. Minusta se tutkija itse on samalla koehenkilö on toimivin lähestystapa.

        'Subjektiivinen' ja 'objektiivinen' käsitteet menevät aika iloisesti sekaisin koko ajan. Tiede perustuu aina yksilöiden aistihavaintoon joka on aina osa sen yksilön subjektiivisesta kokemusmaailmasta. Varsinaisesti mitään välitöntä tieteellistä ja objektiivista havaintoa ei ole edes olemassa eikä voi edes olla olemassa. Kaikki on aina joistain perustelemattomista perusoletuksista pääteltyä ja tulkittua.

        "Materialisti kykenee täysin itsensä kehittämiseen, jos on sellaista kyvykkyyttä hankkinut siinä kuin muutkin. Filosofiset kysymyksen asettelut eivät rajaa ihmisen kokemusmaailmaa eikä itsensä kehittämisen mahdollisuuksia, ellei henkilö ole dogmaattinen kannanotoissan."

        Tietenkin kykenee ja materialismia on hyvin erilaista ja esim. tuo J.G. Bennettin energioiden laadut voidaan tulkita myös puhtaasti materialistiselta kannalta. Idealismiakin on monenlaista eli Bernardo Kastrupin malli jossa hän pyrkii palauttamaan kaiken olemassaolevan tietoisuudessa olevaksi ja tekee sen varsin johdonmukaisesti. Sitten on tietysti kaikenlaista new age ja hippihöttöä joissa logiikka ja koherenssi on aika avutonta.

        "En ymmärrä, miksi subjektiivisuudesta pitäisi eroon päästä, mutta tieteen tekemisessä se ei käy päinsä, koska tieteen kuuluu olla julkista ja avoimelle kritiikille altistettavaa. Subjektiivisia kokemuksia ei voi tällaiseen alistaa käytönnön syistä."

        Niin se oli tarkoitettu lähinnä sarkasmiksi tai vitsiksi siihen mitä siinä kommentoin.

        Toistan vielä sen että ns. objektiivisia kokemuksia ei voi olla tieteessä eikä missään muussakaan koska kokemus on aina pohjimmiltaan subjektiivinen. Samasta syystä itse tietoisuutta ei voi tutkia objektiivisesti eikä se ole objektiivisesti kuvattavissa. Vain käyttäytymistä voidaan tutkia suhteellisen koherentisti. Tietoisuus on aina kaiken tutkimisen subjekti - ei koskaan objekti. Se näennäinen objektiivisuus on lähinnä loogista koherenssia muiden samanlaisten kokijoiden kokemusten kuvausten suhteen.

        "Neurotiede ei toimi ilman introspektiota eli sitä miten saadaan selville ne neurokorrelaatit."

        Tuossa suhteessa subjektiivinen kokemus on vain tutkimuksen objekti. Ja se on mahdollista, vaikka sinä teetkin ontologisia oletuksia, ettei sellainen voisi olla mahdollista.

        Koehenkilön subjektiivinen kertomus kokemastaan on objektiivinen kohde.

        "'Subjektiivinen' ja 'objektiivinen' käsitteet menevät aika iloisesti sekaisin koko ajan. Tiede perustuu aina yksilöiden aistihavaintoon joka on aina osa sen yksilön subjektiivisesta kokemusmaailmasta."

        Mielestäni käsitesekaannusta ei ole mitenkään. Tiede ei perustu yksilön aistihavaintoon, vaikka jokainen tiedemies toki on yksilö. Tieteeseen kuuluu julkinen prosessi, jossa tehtyjä havaintoja voidaan kriittisesti tarkastella, jolloin niiden subjektiivisuutta voidaan hälventää. Näkökantasi vaikuttaa edustavan episteemisesti varvin voimakasta relativismia. Sen perimmäiseksi ongelmaksi tulee jossakin kohtaa solipsismi, jossa ei ole varmuudella muita tiedonhankintatapoja kuin yksilön kokemus.

        " Varsinaisesti mitään välitöntä tieteellistä ja objektiivista havaintoa ei ole edes olemassa eikä voi edes olla olemassa."

        Ei tietenkään välitöntä havaintoa ole objektiivisessa mielessä olemassa. Havainnot liittyvät (parhaaassa tapauksessa) johonkin johonkin koherenttiin teoriaan, jonka pohjalta niitä voidaan tulkita tiedeyhteisössä. (Tässä mielessä siis tieto tiedeyhteisön tulkitsemana on relatiivista, josta voitaisiin käyttää myös käsitettä intersubjektiivinen.)

        "Toistan vielä sen että ns. objektiivisia kokemuksia ei voi olla tieteessä eikä missään muussakaan koska kokemus on aina pohjimmiltaan subjektiivinen."

        Tästä olemme täysin eri mieltä. Mitään syytä ei ole luopua tieteessä objektiivisen tiedon mahdollisuudesta, vaikka eri tieteen aloilla on tässä suhteessa jonkin asteisia eroja, miten paljon voidaan tutkijasta riippumattomasti havaintoja tehdä (viittaan yleisesti joihinkin yhteiskuntatieteisiin).

        Välitön subjektiivinen kokemus kelpaa hyvin aivotutkimuksessa objektiiviseksi tutkimuskohteeksi ja hyviä tuloksia onkin saatu aikaan neurotieteissä.


      • Belisario
        koittakaakehitellä kirjoitti:

        "Neurotiede ei toimi ilman introspektiota eli sitä miten saadaan selville ne neurokorrelaatit."

        Tuossa suhteessa subjektiivinen kokemus on vain tutkimuksen objekti. Ja se on mahdollista, vaikka sinä teetkin ontologisia oletuksia, ettei sellainen voisi olla mahdollista.

        Koehenkilön subjektiivinen kertomus kokemastaan on objektiivinen kohde.

        "'Subjektiivinen' ja 'objektiivinen' käsitteet menevät aika iloisesti sekaisin koko ajan. Tiede perustuu aina yksilöiden aistihavaintoon joka on aina osa sen yksilön subjektiivisesta kokemusmaailmasta."

        Mielestäni käsitesekaannusta ei ole mitenkään. Tiede ei perustu yksilön aistihavaintoon, vaikka jokainen tiedemies toki on yksilö. Tieteeseen kuuluu julkinen prosessi, jossa tehtyjä havaintoja voidaan kriittisesti tarkastella, jolloin niiden subjektiivisuutta voidaan hälventää. Näkökantasi vaikuttaa edustavan episteemisesti varvin voimakasta relativismia. Sen perimmäiseksi ongelmaksi tulee jossakin kohtaa solipsismi, jossa ei ole varmuudella muita tiedonhankintatapoja kuin yksilön kokemus.

        " Varsinaisesti mitään välitöntä tieteellistä ja objektiivista havaintoa ei ole edes olemassa eikä voi edes olla olemassa."

        Ei tietenkään välitöntä havaintoa ole objektiivisessa mielessä olemassa. Havainnot liittyvät (parhaaassa tapauksessa) johonkin johonkin koherenttiin teoriaan, jonka pohjalta niitä voidaan tulkita tiedeyhteisössä. (Tässä mielessä siis tieto tiedeyhteisön tulkitsemana on relatiivista, josta voitaisiin käyttää myös käsitettä intersubjektiivinen.)

        "Toistan vielä sen että ns. objektiivisia kokemuksia ei voi olla tieteessä eikä missään muussakaan koska kokemus on aina pohjimmiltaan subjektiivinen."

        Tästä olemme täysin eri mieltä. Mitään syytä ei ole luopua tieteessä objektiivisen tiedon mahdollisuudesta, vaikka eri tieteen aloilla on tässä suhteessa jonkin asteisia eroja, miten paljon voidaan tutkijasta riippumattomasti havaintoja tehdä (viittaan yleisesti joihinkin yhteiskuntatieteisiin).

        Välitön subjektiivinen kokemus kelpaa hyvin aivotutkimuksessa objektiiviseksi tutkimuskohteeksi ja hyviä tuloksia onkin saatu aikaan neurotieteissä.

        "Tuossa suhteessa subjektiivinen kokemus on vain tutkimuksen objekti. Ja se on mahdollista, vaikka sinä teetkin ontologisia oletuksia, ettei sellainen voisi olla mahdollista."

        Aikaisempi subjektiivinen kokemus ei ole sama asia kuin tietoisuus. Tuossa tietoisuus subjektina tarkastelee aikaisempaa kokemustaan objektina eikä se aikaisempi kokemus ole tietenkään enää tietoisuus. Se aikaisempi kokemus on käsitteiden avulla jäsennetty ja kuvattu jolloin mukaan tulee ne kaikki oletukset ja tulkinnat mitä siitä ontologiasta mikä on omaksuttu.

        "Koehenkilön subjektiivinen kertomus kokemastaan on objektiivinen kohde."

        Nyt annat taas käsitteiden automaattisen tulkinnan ajatella puolestasi. Kaikki ns. "objektiiviset" kohteet edellyttävät sen tutkijan subjektiivista tulkintaa siitä tutkittavasta asiasta.

        " Tiede ei perustu yksilön aistihavaintoon, vaikka jokainen tiedemies toki on yksilö. Tieteeseen kuuluu julkinen prosessi, jossa tehtyjä havaintoja voidaan kriittisesti tarkastella, jolloin niiden subjektiivisuutta voidaan hälventää. "

        Ei se subjektiivisuus mihinkään hälvene vaan se subjektiivisuus vain kollektivisoituu ja on jaettua subjektiivisuuta. Ns. kaikki tieteen ja filosofian ns. koulukunnat edustavat aina subjektiivisuutta.

        "Näkökantasi vaikuttaa edustavan episteemisesti varvin voimakasta relativismia. Sen perimmäiseksi ongelmaksi tulee jossakin kohtaa solipsismi, jossa ei ole varmuudella muita tiedonhankintatapoja kuin yksilön kokemus."

        Niin todellisuudessa kukin meistä (normaalisti) kokee vain omat kokemuksensa ja omat ajatuksensa ja tunteensa ja kaikki muu on sitten käyttäytymistä. Se näennäinen objektiivisuus syntyy siitä että eri yksilöiden kokemukset ovat kunkin ryhmän konsensustodellisuudessa rakenneyhtäläisiä keskenään koherentteja ja siihen keskinäiseen koherenssiin päästään nimenomaan koulutuksella ja samanlaisella kielellä tai jargonilla. Kyse on kuitenkin aina sellaisesta "todellisuudesta" joka on aina tiettyjen oletusten varassa muodostettua.

        "Ei tietenkään välitöntä havaintoa ole objektiivisessa mielessä olemassa. Havainnot liittyvät (parhaaassa tapauksessa) johonkin johonkin koherenttiin teoriaan, jonka pohjalta niitä voidaan tulkita tiedeyhteisössä. (Tässä mielessä siis tieto tiedeyhteisön tulkitsemana on relatiivista, josta voitaisiin käyttää myös käsitettä intersubjektiivinen.)"

        Niin tuossa nyt pääsit oikeastaan samaan johtopäätökseen kuin minä vaikka eri vähän eri kautta.

        "Mitään syytä ei ole luopua tieteessä objektiivisen tiedon mahdollisuudesta, vaikka eri tieteen aloilla on tässä suhteessa jonkin asteisia eroja, miten paljon voidaan tutkijasta riippumattomasti havaintoja tehdä (viittaan yleisesti joihinkin yhteiskuntatieteisiin)."

        Parempi ettei siitä objektiivisuudesta edes puhuta koska tiede parhaimmillaankin voi olla korkeintaan melkein objektiivista esim. jossain kiinteiden kappaleiden fysiikassa ja tavanomaisten mekaanisten koneiden kuvaamisessa.

        Sitten on olemassa monimutkaisempia systeemejä ja rakenteita tai ilmiöitä jotka eivät ole kovin yksiselitteisiä ja joissa se tarkka matemaattinen tai reduktionistinen alemman tason kautta kuvaaminen tuottaa hyvin monimutkaisen ja vaikeasti tulkittavan lopputuloksen (kuten juuri niissä yhteiskuntatieteissä ja periaatteessa kyllä kaikissa vähänkin muutaman kappaleen fysiikkaa monimutkaisemmissa ilmiöissä kuten kemiassa ja biologiassa joissa sitten turvaudutaan tilastotieteeseen ja arvioituihin todennnäköisyyksiin.

        "Välitön subjektiivinen kokemus kelpaa hyvin aivotutkimuksessa objektiiviseksi tutkimuskohteeksi ja hyviä tuloksia onkin saatu aikaan neurotieteissä."

        Se käsittein ilmaistu kokemus ei ole enää välitön kokemus. Niistä neurotieteen hyvistä tuloksistakin voi olla eri mieltä kun ne tuntuvat olevan hyvinkin tarkoitushakuisia eli pyrkimyksenä on jotenkin vahvistaa materialismin ideologiaa vaikka todellisuudessa pystytään vain kuvailemaan sähkökemian tasolla olevia korrelaatioita tietoisuuden suhteen.


        Idealismissa ne perusoletukset ovat

        1. Tietoisuus on olemassa
        2. Vapaa tahto on olemassa
        3. Aika on olemassa
        4. Aine on illuusio tai ainakin toissijainen suhteessa tietoisuuteen.

        Tietoisuus, vapaa tahto ja aika liittyvät aina toisiinsa ja edellyttävät toisiaan.


        Materialismissa taas vastaavat perusoletukset ovat:

        1. Materia on olemassa

        2. Determinismi on ainoa tapa selittää erilaisten prosessien kuten tietoisuuden toiminta eli elävät ja ns. tietoiset oliot ovat jonkinlaisia koneita. (satunnaisuutta ja stokastisuutta voi esiintyä mutta niillä sellaisenaan ei ole kovin merkittävää selitysvoimaa biologisen elämän eikä tietoisuuden kuvaamisessa. )

        3. Tietoisuus on jotenkin näennäistä eli syntyy ei-tietoisista prosesseista.

        4. Vapaata tahtoa ei ole olemassa.

        ...

        Paljon melua nousee siitä että tieteeseen sotketaan uskonnollisia tulkintoja mutta aivan yhtä harhaanjohtavaa on ja voi olla materialismin tai minkä muun tahansa ideologian sotkeminen tarkoitushakuisesti mukaan varsinkin tiedepopularisoinneissa.


      • koittakaakehitellä
        Belisario kirjoitti:

        "Tuossa suhteessa subjektiivinen kokemus on vain tutkimuksen objekti. Ja se on mahdollista, vaikka sinä teetkin ontologisia oletuksia, ettei sellainen voisi olla mahdollista."

        Aikaisempi subjektiivinen kokemus ei ole sama asia kuin tietoisuus. Tuossa tietoisuus subjektina tarkastelee aikaisempaa kokemustaan objektina eikä se aikaisempi kokemus ole tietenkään enää tietoisuus. Se aikaisempi kokemus on käsitteiden avulla jäsennetty ja kuvattu jolloin mukaan tulee ne kaikki oletukset ja tulkinnat mitä siitä ontologiasta mikä on omaksuttu.

        "Koehenkilön subjektiivinen kertomus kokemastaan on objektiivinen kohde."

        Nyt annat taas käsitteiden automaattisen tulkinnan ajatella puolestasi. Kaikki ns. "objektiiviset" kohteet edellyttävät sen tutkijan subjektiivista tulkintaa siitä tutkittavasta asiasta.

        " Tiede ei perustu yksilön aistihavaintoon, vaikka jokainen tiedemies toki on yksilö. Tieteeseen kuuluu julkinen prosessi, jossa tehtyjä havaintoja voidaan kriittisesti tarkastella, jolloin niiden subjektiivisuutta voidaan hälventää. "

        Ei se subjektiivisuus mihinkään hälvene vaan se subjektiivisuus vain kollektivisoituu ja on jaettua subjektiivisuuta. Ns. kaikki tieteen ja filosofian ns. koulukunnat edustavat aina subjektiivisuutta.

        "Näkökantasi vaikuttaa edustavan episteemisesti varvin voimakasta relativismia. Sen perimmäiseksi ongelmaksi tulee jossakin kohtaa solipsismi, jossa ei ole varmuudella muita tiedonhankintatapoja kuin yksilön kokemus."

        Niin todellisuudessa kukin meistä (normaalisti) kokee vain omat kokemuksensa ja omat ajatuksensa ja tunteensa ja kaikki muu on sitten käyttäytymistä. Se näennäinen objektiivisuus syntyy siitä että eri yksilöiden kokemukset ovat kunkin ryhmän konsensustodellisuudessa rakenneyhtäläisiä keskenään koherentteja ja siihen keskinäiseen koherenssiin päästään nimenomaan koulutuksella ja samanlaisella kielellä tai jargonilla. Kyse on kuitenkin aina sellaisesta "todellisuudesta" joka on aina tiettyjen oletusten varassa muodostettua.

        "Ei tietenkään välitöntä havaintoa ole objektiivisessa mielessä olemassa. Havainnot liittyvät (parhaaassa tapauksessa) johonkin johonkin koherenttiin teoriaan, jonka pohjalta niitä voidaan tulkita tiedeyhteisössä. (Tässä mielessä siis tieto tiedeyhteisön tulkitsemana on relatiivista, josta voitaisiin käyttää myös käsitettä intersubjektiivinen.)"

        Niin tuossa nyt pääsit oikeastaan samaan johtopäätökseen kuin minä vaikka eri vähän eri kautta.

        "Mitään syytä ei ole luopua tieteessä objektiivisen tiedon mahdollisuudesta, vaikka eri tieteen aloilla on tässä suhteessa jonkin asteisia eroja, miten paljon voidaan tutkijasta riippumattomasti havaintoja tehdä (viittaan yleisesti joihinkin yhteiskuntatieteisiin)."

        Parempi ettei siitä objektiivisuudesta edes puhuta koska tiede parhaimmillaankin voi olla korkeintaan melkein objektiivista esim. jossain kiinteiden kappaleiden fysiikassa ja tavanomaisten mekaanisten koneiden kuvaamisessa.

        Sitten on olemassa monimutkaisempia systeemejä ja rakenteita tai ilmiöitä jotka eivät ole kovin yksiselitteisiä ja joissa se tarkka matemaattinen tai reduktionistinen alemman tason kautta kuvaaminen tuottaa hyvin monimutkaisen ja vaikeasti tulkittavan lopputuloksen (kuten juuri niissä yhteiskuntatieteissä ja periaatteessa kyllä kaikissa vähänkin muutaman kappaleen fysiikkaa monimutkaisemmissa ilmiöissä kuten kemiassa ja biologiassa joissa sitten turvaudutaan tilastotieteeseen ja arvioituihin todennnäköisyyksiin.

        "Välitön subjektiivinen kokemus kelpaa hyvin aivotutkimuksessa objektiiviseksi tutkimuskohteeksi ja hyviä tuloksia onkin saatu aikaan neurotieteissä."

        Se käsittein ilmaistu kokemus ei ole enää välitön kokemus. Niistä neurotieteen hyvistä tuloksistakin voi olla eri mieltä kun ne tuntuvat olevan hyvinkin tarkoitushakuisia eli pyrkimyksenä on jotenkin vahvistaa materialismin ideologiaa vaikka todellisuudessa pystytään vain kuvailemaan sähkökemian tasolla olevia korrelaatioita tietoisuuden suhteen.


        Idealismissa ne perusoletukset ovat

        1. Tietoisuus on olemassa
        2. Vapaa tahto on olemassa
        3. Aika on olemassa
        4. Aine on illuusio tai ainakin toissijainen suhteessa tietoisuuteen.

        Tietoisuus, vapaa tahto ja aika liittyvät aina toisiinsa ja edellyttävät toisiaan.


        Materialismissa taas vastaavat perusoletukset ovat:

        1. Materia on olemassa

        2. Determinismi on ainoa tapa selittää erilaisten prosessien kuten tietoisuuden toiminta eli elävät ja ns. tietoiset oliot ovat jonkinlaisia koneita. (satunnaisuutta ja stokastisuutta voi esiintyä mutta niillä sellaisenaan ei ole kovin merkittävää selitysvoimaa biologisen elämän eikä tietoisuuden kuvaamisessa. )

        3. Tietoisuus on jotenkin näennäistä eli syntyy ei-tietoisista prosesseista.

        4. Vapaata tahtoa ei ole olemassa.

        ...

        Paljon melua nousee siitä että tieteeseen sotketaan uskonnollisia tulkintoja mutta aivan yhtä harhaanjohtavaa on ja voi olla materialismin tai minkä muun tahansa ideologian sotkeminen tarkoitushakuisesti mukaan varsinkin tiedepopularisoinneissa.

        "Aikaisempi subjektiivinen kokemus ei ole sama asia kuin tietoisuus. Tuossa tietoisuus subjektina tarkastelee aikaisempaa kokemustaan objektina eikä se aikaisempi kokemus ole tietenkään enää tietoisuus."

        Sinun tekemien ontologisin oletuksin ei tietenkään enää ole niin, että subjekti ei voisi koskaan tutkia (välitöntä) kokemustaan. Näin voimakasta ontologista rajausta ei ole välttämätöntä tehdä.

        "Kaikki ns. "objektiiviset" kohteet edellyttävät sen tutkijan subjektiivista tulkintaa siitä tutkittavasta asiasta."

        Objektiivista kohdetta tutkittaessa joudutaan tekemään tulkintoja, kuten kaikissa havainnoissa aina joudutaan. Mutta tulkintojen ei ole pakottavasti oltava "subjektiivisia", kuten annat ymmärtää, vaan ainoastaan relatiivisia tiedeyhteisön tapaan hankkia ja jäsentää tietoa. Viimeksi mainittua voidaan pitää niin objektiivisuutta lähestyvänä kuin se on mahdollista avoimessa ja kriittisessä metodissa ylipäänsä (tästä voitaisiin puhua myös "intersubjektiivisuutena", mutta siinä käsitteessä on ongelmansa, kun se viittaa "subjektiivisuuteen" helposti harhaanjohtavalla tavalla.)

        "Se näennäinen objektiivisuus syntyy siitä että eri yksilöiden kokemukset ovat kunkin ryhmän konsensustodellisuudessa rakenneyhtäläisiä keskenään koherentteja ja siihen keskinäiseen koherenssiin päästään nimenomaan koulutuksella ja samanlaisella kielellä tai jargonilla."

        Täaaä mielessä olemme lähellä toistemme näkemystä. En tosin kutsuisi objektiivisuutta näennäiseksi (=epätodellista), vaan relatiiviseksi suhteessa tiedeyhteisöön ja koherenttiin teoriaan (suhde teoriaan on tosin monimutkaisempi eikä ollenkaan välttämätön).

        "Sitten on olemassa monimutkaisempia systeemejä ja rakenteita tai ilmiöitä jotka eivät ole kovin yksiselitteisiä ja joissa se tarkka matemaattinen tai reduktionistinen alemman tason kautta kuvaaminen tuottaa hyvin monimutkaisen ja vaikeasti tulkittavan lopputuloksen"

        Tämä esiin nostamasi kompleksisuus ei mielestäni ole suorassa suhteessa nyt pohdittuun asiaan subjektiivisuus/objektiivisuus. Tulkinnan tarve voi tietenkin nousta, jos systeemi on vaikeasti ymmärrettävä, mutta aina näin ei ole, vaan käsitys ilmiöiden tilasta voi olla mallinnettavissa hyvin yksinkertaisten parametrien avulla, jolloin tulkintaa ei itse asiassa tarvita (näkisin esim. evoluutioteoriassa tämän kaltaista objektiivisuutta).

        "Materialismissa taas vastaavat perusoletukset ovat:

        1. Materia on olemassa

        2. Determinismi on ainoa tapa selittää erilaisten prosessien kuten tietoisuuden toiminta eli elävät ja ns. tietoiset oliot ovat jonkinlaisia koneita. (satunnaisuutta ja stokastisuutta voi esiintyä mutta niillä sellaisenaan ei ole kovin merkittävää selitysvoimaa biologisen elämän eikä tietoisuuden kuvaamisessa. )

        3. Tietoisuus on jotenkin näennäistä eli syntyy ei-tietoisista prosesseista.

        4. Vapaata tahtoa ei ole olemassa."

        Mainitsemista perusoletuksista mielstäni ainoastaan kohta 1 sopii materialismiin (toki käsityksiä materialismista on monia kuten totesitkin aiemmin).

        Kohta 2: Emergentti materialismi pitää mahdollisena esim. tietoisuuden esiin tulemista aineellisista prosesseista ja vuorovaikutuksista ympäristönsä kanssa. Kysymyksessä on silloin aito ontologinen muutos, jossa kompleksinen systeemi synnyttää aineetonta olemusta. Se ei ole vain reduktiivnen prosessi, jossa ylempi taso tyhjentävästi selitettäisiin alemman tason prosessein (kuten fysikalistit tekevät).

        Kohta 3: Tietoisuus on omassa ontologisessa luokassaan, mutta sen selittämiseen ei käytetä ontologisesti idealistisia entiteettejä (esim. panspykismiä).

        Kohta 4: Vapaaseen tahtoon materialismi ei varsinaisesti ota kantaa. Kysymys siitä on enemminkin empiirisen tutkimuksen kohde ja johtopäätöksiä voidaan tehdä sitten, kun on riittävästi evidenssiä tutkimusten pohjalta tehtävissä.

        Popularisoidessa tiedettä ei ole aina selvää, mitkä ovat kirjoittajan sitoumukset ontologisessa mielessä. Kuten Kari Enqvist käyttää esim. emergenssiä tavalla, joka onkin tulkittavissa näennäiseksi. Hän on todellisuudessa fysikalisti, vaikka muunlaisen kuvan voi helposti saada popularisoivista kirjotuksista.


      • Belisario
        koittakaakehitellä kirjoitti:

        "Aikaisempi subjektiivinen kokemus ei ole sama asia kuin tietoisuus. Tuossa tietoisuus subjektina tarkastelee aikaisempaa kokemustaan objektina eikä se aikaisempi kokemus ole tietenkään enää tietoisuus."

        Sinun tekemien ontologisin oletuksin ei tietenkään enää ole niin, että subjekti ei voisi koskaan tutkia (välitöntä) kokemustaan. Näin voimakasta ontologista rajausta ei ole välttämätöntä tehdä.

        "Kaikki ns. "objektiiviset" kohteet edellyttävät sen tutkijan subjektiivista tulkintaa siitä tutkittavasta asiasta."

        Objektiivista kohdetta tutkittaessa joudutaan tekemään tulkintoja, kuten kaikissa havainnoissa aina joudutaan. Mutta tulkintojen ei ole pakottavasti oltava "subjektiivisia", kuten annat ymmärtää, vaan ainoastaan relatiivisia tiedeyhteisön tapaan hankkia ja jäsentää tietoa. Viimeksi mainittua voidaan pitää niin objektiivisuutta lähestyvänä kuin se on mahdollista avoimessa ja kriittisessä metodissa ylipäänsä (tästä voitaisiin puhua myös "intersubjektiivisuutena", mutta siinä käsitteessä on ongelmansa, kun se viittaa "subjektiivisuuteen" helposti harhaanjohtavalla tavalla.)

        "Se näennäinen objektiivisuus syntyy siitä että eri yksilöiden kokemukset ovat kunkin ryhmän konsensustodellisuudessa rakenneyhtäläisiä keskenään koherentteja ja siihen keskinäiseen koherenssiin päästään nimenomaan koulutuksella ja samanlaisella kielellä tai jargonilla."

        Täaaä mielessä olemme lähellä toistemme näkemystä. En tosin kutsuisi objektiivisuutta näennäiseksi (=epätodellista), vaan relatiiviseksi suhteessa tiedeyhteisöön ja koherenttiin teoriaan (suhde teoriaan on tosin monimutkaisempi eikä ollenkaan välttämätön).

        "Sitten on olemassa monimutkaisempia systeemejä ja rakenteita tai ilmiöitä jotka eivät ole kovin yksiselitteisiä ja joissa se tarkka matemaattinen tai reduktionistinen alemman tason kautta kuvaaminen tuottaa hyvin monimutkaisen ja vaikeasti tulkittavan lopputuloksen"

        Tämä esiin nostamasi kompleksisuus ei mielestäni ole suorassa suhteessa nyt pohdittuun asiaan subjektiivisuus/objektiivisuus. Tulkinnan tarve voi tietenkin nousta, jos systeemi on vaikeasti ymmärrettävä, mutta aina näin ei ole, vaan käsitys ilmiöiden tilasta voi olla mallinnettavissa hyvin yksinkertaisten parametrien avulla, jolloin tulkintaa ei itse asiassa tarvita (näkisin esim. evoluutioteoriassa tämän kaltaista objektiivisuutta).

        "Materialismissa taas vastaavat perusoletukset ovat:

        1. Materia on olemassa

        2. Determinismi on ainoa tapa selittää erilaisten prosessien kuten tietoisuuden toiminta eli elävät ja ns. tietoiset oliot ovat jonkinlaisia koneita. (satunnaisuutta ja stokastisuutta voi esiintyä mutta niillä sellaisenaan ei ole kovin merkittävää selitysvoimaa biologisen elämän eikä tietoisuuden kuvaamisessa. )

        3. Tietoisuus on jotenkin näennäistä eli syntyy ei-tietoisista prosesseista.

        4. Vapaata tahtoa ei ole olemassa."

        Mainitsemista perusoletuksista mielstäni ainoastaan kohta 1 sopii materialismiin (toki käsityksiä materialismista on monia kuten totesitkin aiemmin).

        Kohta 2: Emergentti materialismi pitää mahdollisena esim. tietoisuuden esiin tulemista aineellisista prosesseista ja vuorovaikutuksista ympäristönsä kanssa. Kysymyksessä on silloin aito ontologinen muutos, jossa kompleksinen systeemi synnyttää aineetonta olemusta. Se ei ole vain reduktiivnen prosessi, jossa ylempi taso tyhjentävästi selitettäisiin alemman tason prosessein (kuten fysikalistit tekevät).

        Kohta 3: Tietoisuus on omassa ontologisessa luokassaan, mutta sen selittämiseen ei käytetä ontologisesti idealistisia entiteettejä (esim. panspykismiä).

        Kohta 4: Vapaaseen tahtoon materialismi ei varsinaisesti ota kantaa. Kysymys siitä on enemminkin empiirisen tutkimuksen kohde ja johtopäätöksiä voidaan tehdä sitten, kun on riittävästi evidenssiä tutkimusten pohjalta tehtävissä.

        Popularisoidessa tiedettä ei ole aina selvää, mitkä ovat kirjoittajan sitoumukset ontologisessa mielessä. Kuten Kari Enqvist käyttää esim. emergenssiä tavalla, joka onkin tulkittavissa näennäiseksi. Hän on todellisuudessa fysikalisti, vaikka muunlaisen kuvan voi helposti saada popularisoivista kirjotuksista.

        "Tulkinnan tarve voi tietenkin nousta, jos systeemi on vaikeasti ymmärrettävä, mutta aina näin ei ole, vaan käsitys ilmiöiden tilasta voi olla mallinnettavissa hyvin yksinkertaisten parametrien avulla, jolloin tulkintaa ei itse asiassa tarvita (näkisin esim. evoluutioteoriassa tämän kaltaista objektiivisuutta)."

        Erilaisia mahdollisia tulkintoja ja niihin liittyvä ontologioita varmaan riittää aina niin kauan kun on olemassa erilaisia loogisia mahdollisuuksia hahmottaa jokin tutkittava. Kaiken lisäksi näyttää vielä siltä että ne loogiset mahdollisuudet eivät vähene tutkimuksen edistyessä vaan päinvastoin niiden määrä lisääntyy vaikka joitain mahdollisuuksia pystytäänkin melkoisella varmuudella myös sulkemaan pois. Tieteessä kun ei voi todistaa mitään teoriaa oikeaksi vaan ainoastaan niitä muita vaihtoehtoja ainakin osittain vääräksi.

        Evoluutioteoria ei oikeastaan nykyisessä muodossaan selitä oikein mitään vaan pelkästään kuvailee biologista evoluutiota molekyylien eli biokemian tasolla ja samalla oletetaan että satunnaiset mutaatiot ja luonnonvalinta sellaisenaan selittävät koko asian. Tuossakin on nähtävissä voimakas pyrkimys palauttaa elämän ilmiö reduktionistisesti johonkin mikä ei ole elämää samalla tavalla kuin neurotieteessä pyritään palauttamaan tietoisuus johonkin ei- tietoiseen eli tietyllä tavalla organisoituneeseen aineeseen. Materialismi on tiedeyhteisön valtauskonto tai ideologia joka riittää kyllä toimivan tekniikan kehittämiseen mutta ei oikeastaan minkään toimivan ja koherentin maailmankuvan muodostamiseen.
        ......

        Itse luulen että sen materialistisen ontologian kannatuksen taustalla piilee lähinnä alitajuisia irrationaalisia vaikutteita eli jonkinlaista "taikauskon" pelkoa. Materalismin tai oikeammin fysikalismin viitekehyksessä ei ole mitään päämääriä eikä merkityksiä eikä siis minkäänlaista eettistä vastuutakaan mistään. Kaikki on vain "kvarkkien tanssia" jossain väliaikaisessa konfiguraatiossa joka syntyy ei-mistään ja katoaa ei-mihinkään kun tietoisuus sammuu kuollessa.

        Materialismi tuottaa suotuisan asenneympäristön konsumerismille eli kulutusyhteiskunnalle koska on kiire kokea kaikki mahdollinen materiaalisella tasolla eli tavaroiden kautta.

        Mielenkiintoista on lisäksi että yleensä sekä valtauskonnot että tiedeinstituutio suhtautuvat molemmat samalla tavalla hyvin torjuvasti kaikkiin ns. normitietoisuudesta poikkeaviin ilmiöihin (kuten parapsykologian tutkimuskohteet) joiden paikkansapitävyys käytännössä tarkoittaa sitä että pelkällä vahvalla ja koherentilla intentiolla ja intuitiolla voi vaikuttaa hyvinkin dramaattisesti vallitseviin olosuhteisiin. Joskus vielä muutama sata vuotta sitten nuo ilmiöt koettiin hyvinkin yleisinä ja nähtiin ufojen sijasta demoneja, haltioita, enkeleitä yms. ja silloin poltettiin ihmisiä roviolla satoja tuhansia koska epäiltiin taikuudesta eli tietoisuuden tekniikoiden käytöstä epäeettisiin tarkoituksiin.

        Tuollaiset ilmiöt eivät ole idealismin kanssa ristiriidassa vaikka materialistinen uskomusjärjestelmä torjuu ne täysin. Epäilen vahvasti että se ns. todellisuus muokkaantuu pienellä viiveellä juuri sellaiseksi millainen vallitseva uskomusjärjestelmä on ja tuo mukanaan samalla kaikki ne ongelmat mitä siihen uskomusjärjestelmään liityy (ilmastonmuutos, luonnon tuhoutuminen, ihmiselämän pinnallistuminen, mielenterveyshäiriöt yms.).

        Pidän idealismin yhtenä vahvimpana todistusaineistona nimenomaan sitä kuinka samankaltainen unissa nähty todellisuus on tämän konsensustodellisuuden kanssa.

        Kokemuksessa on kyseessä aina jonkinlainen datastream eli informaation vastaanotto joka siinä tilanteessa tulkitaan omien (unissa) tai kollektiivisten (ns. reaalimaailma) uskomusten mukaan. Itse näen parhaimpana vaihtoehtona olla uskomatta (sokeasti) mihinkään eli suhtaudun suurinpiirtein yhtä kriittisesti sekä tieteeseen että uskontoihin ja kaikkiin muihinkin ideologioihin. Vaikka niissä kaikissa voi olla jotain toimivaa niin ne ovat samalla myös jokainen liian yksipuolinen ja ongelmallinen.

        ....

        Koska idealismissa ei pyritä pelkästään aineelliseen hyvinvointiin (joka on aika pitkälle kuviteltua ja mikään ei tunnu riittävän) niin näen idealismiin suuntautumisessa mahdollisuuden selvitä monista nykyisen materialismin ja sitä tukevan tieteen ja tekniikan ongelmista selviämiseen eli ts. pyritään yksinkertaiseen ja tasapainoiseen elämään jossa se elämä koetaan mielekkäänä ja jossa omat valinnat ovat olennaisia ja tärkeitä ja jossa vallitsee objektiivinen etiikka siinä muodossa miten tasapainoisen ihmisen omatunto toimii.

        Toimivan etiikan voi myös rakentaa entropian käsitteen pohjalle eli se mikä vähentää omaa ja ympäristön entropiaa eli epäjärjestystä on hyvää ja se mikä lisää epäjärjestystä on pahaa objektiivisesti.

        Tämä riittänee taas tältä päivältä.....


      • Järkisyitä
        Belisario kirjoitti:

        "Introspektio on itsensä kehittämistä mentaalisella tasolla, mutta se ei ole tieteellistä missään mielessä. Tiede ei toimi subjektiivisessa maailmassa."

        Neurotiede ei toimi ilman introspektiota eli sitä miten saadaan selville ne neurokorrelaatit. Siinä sen tutkijan itsensä pitää olla koekaniinina jossa hän suoraan voi kokea miten jonkun aivojen osan stimulointi vaikuttaa siihen omaan kokemukseen tai sitten joku muu koehenkilö joka sanallisesti kertoo mitä kokee. Minusta se tutkija itse on samalla koehenkilö on toimivin lähestystapa.

        'Subjektiivinen' ja 'objektiivinen' käsitteet menevät aika iloisesti sekaisin koko ajan. Tiede perustuu aina yksilöiden aistihavaintoon joka on aina osa sen yksilön subjektiivisesta kokemusmaailmasta. Varsinaisesti mitään välitöntä tieteellistä ja objektiivista havaintoa ei ole edes olemassa eikä voi edes olla olemassa. Kaikki on aina joistain perustelemattomista perusoletuksista pääteltyä ja tulkittua.

        "Materialisti kykenee täysin itsensä kehittämiseen, jos on sellaista kyvykkyyttä hankkinut siinä kuin muutkin. Filosofiset kysymyksen asettelut eivät rajaa ihmisen kokemusmaailmaa eikä itsensä kehittämisen mahdollisuuksia, ellei henkilö ole dogmaattinen kannanotoissan."

        Tietenkin kykenee ja materialismia on hyvin erilaista ja esim. tuo J.G. Bennettin energioiden laadut voidaan tulkita myös puhtaasti materialistiselta kannalta. Idealismiakin on monenlaista eli Bernardo Kastrupin malli jossa hän pyrkii palauttamaan kaiken olemassaolevan tietoisuudessa olevaksi ja tekee sen varsin johdonmukaisesti. Sitten on tietysti kaikenlaista new age ja hippihöttöä joissa logiikka ja koherenssi on aika avutonta.

        "En ymmärrä, miksi subjektiivisuudesta pitäisi eroon päästä, mutta tieteen tekemisessä se ei käy päinsä, koska tieteen kuuluu olla julkista ja avoimelle kritiikille altistettavaa. Subjektiivisia kokemuksia ei voi tällaiseen alistaa käytönnön syistä."

        Niin se oli tarkoitettu lähinnä sarkasmiksi tai vitsiksi siihen mitä siinä kommentoin.

        Toistan vielä sen että ns. objektiivisia kokemuksia ei voi olla tieteessä eikä missään muussakaan koska kokemus on aina pohjimmiltaan subjektiivinen. Samasta syystä itse tietoisuutta ei voi tutkia objektiivisesti eikä se ole objektiivisesti kuvattavissa. Vain käyttäytymistä voidaan tutkia suhteellisen koherentisti. Tietoisuus on aina kaiken tutkimisen subjekti - ei koskaan objekti. Se näennäinen objektiivisuus on lähinnä loogista koherenssia muiden samanlaisten kokijoiden kokemusten kuvausten suhteen.

        "Toistan vielä sen että ns. objektiivisia kokemuksia ei voi olla tieteessä eikä missään muussakaan koska kokemus on aina pohjimmiltaan subjektiivinen."

        Kokemus on subjektiivinen, mutta se ei mitenkään sulje pois objektiivisia faktoja. Voimme esimekiksi molemmat todeta reaalimaailman faktoja kuten esimerkiksi sen, että jos et koe syömisen kokemusta muutamaan päivään, siitä seuraa nälän tunteminen. Tämä on kaikkien meidän todennettavissa eli se on selvästi objektiivinen fakta.

        "Varsinaisesti mitään välitöntä tieteellistä ja objektiivista havaintoa ei ole edes olemassa eikä voi edes olla olemassa."

        Saivartelua... Jos useat eri havaitsijat tekevät saman havainnon, kutsumme tuota havaintoa objektiiviseksi havainnoksi.

        "Kaikki on aina joistain perustelemattomista perusoletuksista pääteltyä ja tulkittua."

        Ei ole. Kunnollinen tiede lähtee mahdollisimman minimalistisista perusoletuksista. Toki tietyt matemaattiset ja loogiset aksioomat pitää ottaa selviöinä. Samoin realimaailman olemassa olo vähintäänkin yhteisesti jaettuna kokemuksena täytynee hyväksyä itsestäänselvyytenä.


      • Järkisyitä
        Belisario kirjoitti:

        "Tulkinnan tarve voi tietenkin nousta, jos systeemi on vaikeasti ymmärrettävä, mutta aina näin ei ole, vaan käsitys ilmiöiden tilasta voi olla mallinnettavissa hyvin yksinkertaisten parametrien avulla, jolloin tulkintaa ei itse asiassa tarvita (näkisin esim. evoluutioteoriassa tämän kaltaista objektiivisuutta)."

        Erilaisia mahdollisia tulkintoja ja niihin liittyvä ontologioita varmaan riittää aina niin kauan kun on olemassa erilaisia loogisia mahdollisuuksia hahmottaa jokin tutkittava. Kaiken lisäksi näyttää vielä siltä että ne loogiset mahdollisuudet eivät vähene tutkimuksen edistyessä vaan päinvastoin niiden määrä lisääntyy vaikka joitain mahdollisuuksia pystytäänkin melkoisella varmuudella myös sulkemaan pois. Tieteessä kun ei voi todistaa mitään teoriaa oikeaksi vaan ainoastaan niitä muita vaihtoehtoja ainakin osittain vääräksi.

        Evoluutioteoria ei oikeastaan nykyisessä muodossaan selitä oikein mitään vaan pelkästään kuvailee biologista evoluutiota molekyylien eli biokemian tasolla ja samalla oletetaan että satunnaiset mutaatiot ja luonnonvalinta sellaisenaan selittävät koko asian. Tuossakin on nähtävissä voimakas pyrkimys palauttaa elämän ilmiö reduktionistisesti johonkin mikä ei ole elämää samalla tavalla kuin neurotieteessä pyritään palauttamaan tietoisuus johonkin ei- tietoiseen eli tietyllä tavalla organisoituneeseen aineeseen. Materialismi on tiedeyhteisön valtauskonto tai ideologia joka riittää kyllä toimivan tekniikan kehittämiseen mutta ei oikeastaan minkään toimivan ja koherentin maailmankuvan muodostamiseen.
        ......

        Itse luulen että sen materialistisen ontologian kannatuksen taustalla piilee lähinnä alitajuisia irrationaalisia vaikutteita eli jonkinlaista "taikauskon" pelkoa. Materalismin tai oikeammin fysikalismin viitekehyksessä ei ole mitään päämääriä eikä merkityksiä eikä siis minkäänlaista eettistä vastuutakaan mistään. Kaikki on vain "kvarkkien tanssia" jossain väliaikaisessa konfiguraatiossa joka syntyy ei-mistään ja katoaa ei-mihinkään kun tietoisuus sammuu kuollessa.

        Materialismi tuottaa suotuisan asenneympäristön konsumerismille eli kulutusyhteiskunnalle koska on kiire kokea kaikki mahdollinen materiaalisella tasolla eli tavaroiden kautta.

        Mielenkiintoista on lisäksi että yleensä sekä valtauskonnot että tiedeinstituutio suhtautuvat molemmat samalla tavalla hyvin torjuvasti kaikkiin ns. normitietoisuudesta poikkeaviin ilmiöihin (kuten parapsykologian tutkimuskohteet) joiden paikkansapitävyys käytännössä tarkoittaa sitä että pelkällä vahvalla ja koherentilla intentiolla ja intuitiolla voi vaikuttaa hyvinkin dramaattisesti vallitseviin olosuhteisiin. Joskus vielä muutama sata vuotta sitten nuo ilmiöt koettiin hyvinkin yleisinä ja nähtiin ufojen sijasta demoneja, haltioita, enkeleitä yms. ja silloin poltettiin ihmisiä roviolla satoja tuhansia koska epäiltiin taikuudesta eli tietoisuuden tekniikoiden käytöstä epäeettisiin tarkoituksiin.

        Tuollaiset ilmiöt eivät ole idealismin kanssa ristiriidassa vaikka materialistinen uskomusjärjestelmä torjuu ne täysin. Epäilen vahvasti että se ns. todellisuus muokkaantuu pienellä viiveellä juuri sellaiseksi millainen vallitseva uskomusjärjestelmä on ja tuo mukanaan samalla kaikki ne ongelmat mitä siihen uskomusjärjestelmään liityy (ilmastonmuutos, luonnon tuhoutuminen, ihmiselämän pinnallistuminen, mielenterveyshäiriöt yms.).

        Pidän idealismin yhtenä vahvimpana todistusaineistona nimenomaan sitä kuinka samankaltainen unissa nähty todellisuus on tämän konsensustodellisuuden kanssa.

        Kokemuksessa on kyseessä aina jonkinlainen datastream eli informaation vastaanotto joka siinä tilanteessa tulkitaan omien (unissa) tai kollektiivisten (ns. reaalimaailma) uskomusten mukaan. Itse näen parhaimpana vaihtoehtona olla uskomatta (sokeasti) mihinkään eli suhtaudun suurinpiirtein yhtä kriittisesti sekä tieteeseen että uskontoihin ja kaikkiin muihinkin ideologioihin. Vaikka niissä kaikissa voi olla jotain toimivaa niin ne ovat samalla myös jokainen liian yksipuolinen ja ongelmallinen.

        ....

        Koska idealismissa ei pyritä pelkästään aineelliseen hyvinvointiin (joka on aika pitkälle kuviteltua ja mikään ei tunnu riittävän) niin näen idealismiin suuntautumisessa mahdollisuuden selvitä monista nykyisen materialismin ja sitä tukevan tieteen ja tekniikan ongelmista selviämiseen eli ts. pyritään yksinkertaiseen ja tasapainoiseen elämään jossa se elämä koetaan mielekkäänä ja jossa omat valinnat ovat olennaisia ja tärkeitä ja jossa vallitsee objektiivinen etiikka siinä muodossa miten tasapainoisen ihmisen omatunto toimii.

        Toimivan etiikan voi myös rakentaa entropian käsitteen pohjalle eli se mikä vähentää omaa ja ympäristön entropiaa eli epäjärjestystä on hyvää ja se mikä lisää epäjärjestystä on pahaa objektiivisesti.

        Tämä riittänee taas tältä päivältä.....

        "Materialismi tuottaa suotuisan asenneympäristön konsumerismille eli kulutusyhteiskunnalle koska on kiire kokea kaikki mahdollinen materiaalisella tasolla eli tavaroiden kautta."

        Täysin eri mieltä. Kerskakulutus ja arvomaailmat ovat maailman ontologiasta riippumattomia.

        Uskomukset, missä realimaailmaa pidetään illuusiona, päin vastoin voivat johtaa kyvyttömyyteen arvostaa sen ainutlatuisuutta.


      • koittakaakehitellä
        Belisario kirjoitti:

        "Tulkinnan tarve voi tietenkin nousta, jos systeemi on vaikeasti ymmärrettävä, mutta aina näin ei ole, vaan käsitys ilmiöiden tilasta voi olla mallinnettavissa hyvin yksinkertaisten parametrien avulla, jolloin tulkintaa ei itse asiassa tarvita (näkisin esim. evoluutioteoriassa tämän kaltaista objektiivisuutta)."

        Erilaisia mahdollisia tulkintoja ja niihin liittyvä ontologioita varmaan riittää aina niin kauan kun on olemassa erilaisia loogisia mahdollisuuksia hahmottaa jokin tutkittava. Kaiken lisäksi näyttää vielä siltä että ne loogiset mahdollisuudet eivät vähene tutkimuksen edistyessä vaan päinvastoin niiden määrä lisääntyy vaikka joitain mahdollisuuksia pystytäänkin melkoisella varmuudella myös sulkemaan pois. Tieteessä kun ei voi todistaa mitään teoriaa oikeaksi vaan ainoastaan niitä muita vaihtoehtoja ainakin osittain vääräksi.

        Evoluutioteoria ei oikeastaan nykyisessä muodossaan selitä oikein mitään vaan pelkästään kuvailee biologista evoluutiota molekyylien eli biokemian tasolla ja samalla oletetaan että satunnaiset mutaatiot ja luonnonvalinta sellaisenaan selittävät koko asian. Tuossakin on nähtävissä voimakas pyrkimys palauttaa elämän ilmiö reduktionistisesti johonkin mikä ei ole elämää samalla tavalla kuin neurotieteessä pyritään palauttamaan tietoisuus johonkin ei- tietoiseen eli tietyllä tavalla organisoituneeseen aineeseen. Materialismi on tiedeyhteisön valtauskonto tai ideologia joka riittää kyllä toimivan tekniikan kehittämiseen mutta ei oikeastaan minkään toimivan ja koherentin maailmankuvan muodostamiseen.
        ......

        Itse luulen että sen materialistisen ontologian kannatuksen taustalla piilee lähinnä alitajuisia irrationaalisia vaikutteita eli jonkinlaista "taikauskon" pelkoa. Materalismin tai oikeammin fysikalismin viitekehyksessä ei ole mitään päämääriä eikä merkityksiä eikä siis minkäänlaista eettistä vastuutakaan mistään. Kaikki on vain "kvarkkien tanssia" jossain väliaikaisessa konfiguraatiossa joka syntyy ei-mistään ja katoaa ei-mihinkään kun tietoisuus sammuu kuollessa.

        Materialismi tuottaa suotuisan asenneympäristön konsumerismille eli kulutusyhteiskunnalle koska on kiire kokea kaikki mahdollinen materiaalisella tasolla eli tavaroiden kautta.

        Mielenkiintoista on lisäksi että yleensä sekä valtauskonnot että tiedeinstituutio suhtautuvat molemmat samalla tavalla hyvin torjuvasti kaikkiin ns. normitietoisuudesta poikkeaviin ilmiöihin (kuten parapsykologian tutkimuskohteet) joiden paikkansapitävyys käytännössä tarkoittaa sitä että pelkällä vahvalla ja koherentilla intentiolla ja intuitiolla voi vaikuttaa hyvinkin dramaattisesti vallitseviin olosuhteisiin. Joskus vielä muutama sata vuotta sitten nuo ilmiöt koettiin hyvinkin yleisinä ja nähtiin ufojen sijasta demoneja, haltioita, enkeleitä yms. ja silloin poltettiin ihmisiä roviolla satoja tuhansia koska epäiltiin taikuudesta eli tietoisuuden tekniikoiden käytöstä epäeettisiin tarkoituksiin.

        Tuollaiset ilmiöt eivät ole idealismin kanssa ristiriidassa vaikka materialistinen uskomusjärjestelmä torjuu ne täysin. Epäilen vahvasti että se ns. todellisuus muokkaantuu pienellä viiveellä juuri sellaiseksi millainen vallitseva uskomusjärjestelmä on ja tuo mukanaan samalla kaikki ne ongelmat mitä siihen uskomusjärjestelmään liityy (ilmastonmuutos, luonnon tuhoutuminen, ihmiselämän pinnallistuminen, mielenterveyshäiriöt yms.).

        Pidän idealismin yhtenä vahvimpana todistusaineistona nimenomaan sitä kuinka samankaltainen unissa nähty todellisuus on tämän konsensustodellisuuden kanssa.

        Kokemuksessa on kyseessä aina jonkinlainen datastream eli informaation vastaanotto joka siinä tilanteessa tulkitaan omien (unissa) tai kollektiivisten (ns. reaalimaailma) uskomusten mukaan. Itse näen parhaimpana vaihtoehtona olla uskomatta (sokeasti) mihinkään eli suhtaudun suurinpiirtein yhtä kriittisesti sekä tieteeseen että uskontoihin ja kaikkiin muihinkin ideologioihin. Vaikka niissä kaikissa voi olla jotain toimivaa niin ne ovat samalla myös jokainen liian yksipuolinen ja ongelmallinen.

        ....

        Koska idealismissa ei pyritä pelkästään aineelliseen hyvinvointiin (joka on aika pitkälle kuviteltua ja mikään ei tunnu riittävän) niin näen idealismiin suuntautumisessa mahdollisuuden selvitä monista nykyisen materialismin ja sitä tukevan tieteen ja tekniikan ongelmista selviämiseen eli ts. pyritään yksinkertaiseen ja tasapainoiseen elämään jossa se elämä koetaan mielekkäänä ja jossa omat valinnat ovat olennaisia ja tärkeitä ja jossa vallitsee objektiivinen etiikka siinä muodossa miten tasapainoisen ihmisen omatunto toimii.

        Toimivan etiikan voi myös rakentaa entropian käsitteen pohjalle eli se mikä vähentää omaa ja ympäristön entropiaa eli epäjärjestystä on hyvää ja se mikä lisää epäjärjestystä on pahaa objektiivisesti.

        Tämä riittänee taas tältä päivältä.....

        "Materialismi on tiedeyhteisön valtauskonto tai ideologia joka riittää kyllä toimivan tekniikan kehittämiseen mutta ei oikeastaan minkään toimivan ja koherentin maailmankuvan muodostamiseen."

        Itse näen materialismin olevan seurausta luonnontieteellisen tutkimuksen tuloksista. materialismi ei ole tieteen ennakko-oletuksena, vaan monet tieteen tulokset ovat johtaneet filosofiseen johtopäätökseen materialismin puolesta. Maailmankuvan muodostamiseen tarvitaan jokin kokonaiskäsitys, joka on aina enemmän kuin mikään tieteellisten tulosten summa.

        Yhteiskuntatieteissä muukin taustafilosofia on mahdollinen kuin materialistinen, mutta empiiriselle tutkimuksella on paikkansa yhteiskuntatieteissäkin.

        "Itse luulen että sen materialistisen ontologian kannatuksen taustalla piilee lähinnä alitajuisia irrationaalisia vaikutteita eli jonkinlaista "taikauskon" pelkoa. Materalismin tai oikeammin fysikalismin viitekehyksessä ei ole mitään päämääriä eikä merkityksiä eikä siis minkäänlaista eettistä vastuutakaan mistään."

        Mielestäni käsityksesi materialismista on vääristynyt. Taikausko on väistynyt tieteellisen tutkimuksen tulosten myötä, kun tieteen valo lankeaa tietämättömyyden lakeuksiin (oho!, menipä runolliseksi).

        Vastuuta voivat kantaa niin materialistit kuin idealistit. Joillakin idealistiseen ajatteluun taipuvaisilla henmiklöillä vaikuttaa kuitenkin olevan heikko suhde totuuteen. Heillä ilmenee jonkinlaista tieteen vastaisuutta, vaikka tieteellä ei ole ennakko-oletuksena materialismia. (Sinäkin mainitsit antitieteellisyytesi aiemmin.)

        "Mielenkiintoista on lisäksi että yleensä sekä valtauskonnot että tiedeinstituutio suhtautuvat molemmat samalla tavalla hyvin torjuvasti kaikkiin ns. normitietoisuudesta poikkeaviin ilmiöihin (kuten parapsykologian tutkimuskohteet) ..."

        Kyllähän noita on tutkittu tieteellisesti, mutta em ilmiöille ei ole löytynyt vahvistavia tuloksia.

        " Itse näen parhaimpana vaihtoehtona olla uskomatta (sokeasti) mihinkään eli suhtaudun suurinpiirtein yhtä kriittisesti sekä tieteeseen että uskontoihin ja kaikkiin muihinkin ideologioihin."

        Noinkohan asia on kohdallasi. Vaikuttaa siltä, että sinulla on (luonnon?)tiedettä vastaan vahvoja ennakkoasenteita. En oikein muutenkaan ymmärrä, mitä tarkoittaisi suhtautuminen uskontojen maailmankuvaan "yhtä kriittisesti" kuin tieteelliseen maailmankuvaan. Ne ovat tiedonhankinnan metodeiltaan täysin erilaisia. Uskonnollisten uskomusten kautta ei voi tavoittaa mitään luotettavaa tietoa todellisuudesta.

        "Koska idealismissa ei pyritä pelkästään aineelliseen hyvinvointiin (joka on aika pitkälle kuviteltua ja mikään ei tunnu riittävän) niin näen idealismiin suuntautumisessa mahdollisuuden selvitä monista nykyisen materialismin ja sitä tukevan tieteen ja tekniikan ongelmista selviämiseen eli ts. pyritään yksinkertaiseen ja tasapainoiseen elämään jossa se elämä koetaan mielekkäänä ja jossa omat valinnat ovat olennaisia ja tärkeitä ja jossa vallitsee objektiivinen etiikka siinä muodossa miten tasapainoisen ihmisen omatunto toimii."

        Filosofinen kannanotto materialismiin ei tarkoita mitään materian palvontaa. Puhekielessä asiat usein sotketaan keskenään ja puhutaan "materialisteista" tarkoittaen jotakin rahan ja kuuluisuuden tavoittelua. Sillä ei ole mitään tekemistä filosofisen materialismin kanssa (yhtä vähän kuin idealisminkaan).

        Yksinkertainen ja tasapainoinen elämä voi olla monien päämäärä, mutta teknologista kehitystä ei voi kääntää taaksepäin. Teknologian ja sen seurauksena elintason nousun vaikutus on yhteiskuntaa monimuotoista; sillä on sekä myönteisiä että kielteisiä vaikutuksia. Yhteiskunnan murroksissa ilmenee usein valitettavia lieveilmiöitä, joita on ollut aina teknologiasta riippumatta.


      • Belisario
        koittakaakehitellä kirjoitti:

        "Materialismi on tiedeyhteisön valtauskonto tai ideologia joka riittää kyllä toimivan tekniikan kehittämiseen mutta ei oikeastaan minkään toimivan ja koherentin maailmankuvan muodostamiseen."

        Itse näen materialismin olevan seurausta luonnontieteellisen tutkimuksen tuloksista. materialismi ei ole tieteen ennakko-oletuksena, vaan monet tieteen tulokset ovat johtaneet filosofiseen johtopäätökseen materialismin puolesta. Maailmankuvan muodostamiseen tarvitaan jokin kokonaiskäsitys, joka on aina enemmän kuin mikään tieteellisten tulosten summa.

        Yhteiskuntatieteissä muukin taustafilosofia on mahdollinen kuin materialistinen, mutta empiiriselle tutkimuksella on paikkansa yhteiskuntatieteissäkin.

        "Itse luulen että sen materialistisen ontologian kannatuksen taustalla piilee lähinnä alitajuisia irrationaalisia vaikutteita eli jonkinlaista "taikauskon" pelkoa. Materalismin tai oikeammin fysikalismin viitekehyksessä ei ole mitään päämääriä eikä merkityksiä eikä siis minkäänlaista eettistä vastuutakaan mistään."

        Mielestäni käsityksesi materialismista on vääristynyt. Taikausko on väistynyt tieteellisen tutkimuksen tulosten myötä, kun tieteen valo lankeaa tietämättömyyden lakeuksiin (oho!, menipä runolliseksi).

        Vastuuta voivat kantaa niin materialistit kuin idealistit. Joillakin idealistiseen ajatteluun taipuvaisilla henmiklöillä vaikuttaa kuitenkin olevan heikko suhde totuuteen. Heillä ilmenee jonkinlaista tieteen vastaisuutta, vaikka tieteellä ei ole ennakko-oletuksena materialismia. (Sinäkin mainitsit antitieteellisyytesi aiemmin.)

        "Mielenkiintoista on lisäksi että yleensä sekä valtauskonnot että tiedeinstituutio suhtautuvat molemmat samalla tavalla hyvin torjuvasti kaikkiin ns. normitietoisuudesta poikkeaviin ilmiöihin (kuten parapsykologian tutkimuskohteet) ..."

        Kyllähän noita on tutkittu tieteellisesti, mutta em ilmiöille ei ole löytynyt vahvistavia tuloksia.

        " Itse näen parhaimpana vaihtoehtona olla uskomatta (sokeasti) mihinkään eli suhtaudun suurinpiirtein yhtä kriittisesti sekä tieteeseen että uskontoihin ja kaikkiin muihinkin ideologioihin."

        Noinkohan asia on kohdallasi. Vaikuttaa siltä, että sinulla on (luonnon?)tiedettä vastaan vahvoja ennakkoasenteita. En oikein muutenkaan ymmärrä, mitä tarkoittaisi suhtautuminen uskontojen maailmankuvaan "yhtä kriittisesti" kuin tieteelliseen maailmankuvaan. Ne ovat tiedonhankinnan metodeiltaan täysin erilaisia. Uskonnollisten uskomusten kautta ei voi tavoittaa mitään luotettavaa tietoa todellisuudesta.

        "Koska idealismissa ei pyritä pelkästään aineelliseen hyvinvointiin (joka on aika pitkälle kuviteltua ja mikään ei tunnu riittävän) niin näen idealismiin suuntautumisessa mahdollisuuden selvitä monista nykyisen materialismin ja sitä tukevan tieteen ja tekniikan ongelmista selviämiseen eli ts. pyritään yksinkertaiseen ja tasapainoiseen elämään jossa se elämä koetaan mielekkäänä ja jossa omat valinnat ovat olennaisia ja tärkeitä ja jossa vallitsee objektiivinen etiikka siinä muodossa miten tasapainoisen ihmisen omatunto toimii."

        Filosofinen kannanotto materialismiin ei tarkoita mitään materian palvontaa. Puhekielessä asiat usein sotketaan keskenään ja puhutaan "materialisteista" tarkoittaen jotakin rahan ja kuuluisuuden tavoittelua. Sillä ei ole mitään tekemistä filosofisen materialismin kanssa (yhtä vähän kuin idealisminkaan).

        Yksinkertainen ja tasapainoinen elämä voi olla monien päämäärä, mutta teknologista kehitystä ei voi kääntää taaksepäin. Teknologian ja sen seurauksena elintason nousun vaikutus on yhteiskuntaa monimuotoista; sillä on sekä myönteisiä että kielteisiä vaikutuksia. Yhteiskunnan murroksissa ilmenee usein valitettavia lieveilmiöitä, joita on ollut aina teknologiasta riippumatta.

        "materialismi ei ole tieteen ennakko-oletuksena, vaan monet tieteen tulokset ovat johtaneet filosofiseen johtopäätökseen materialismin puolesta. "

        On se koska hyväksytään vain se kolmannen persoonan näkökulma eli aistihavainto jossa kaikki tulkitaan aineelliseksi. Materialismin johtopäätös on ollut oletuksena mukana jo kokeellisen tieteen alusta lähtien eli Galileon ja Newtonin ajoista jolloin tieteessä otettiin käyttöön konevertauskuva.

        "Yhteiskuntatieteissä muukin taustafilosofia on mahdollinen kuin materialistinen, mutta empiiriselle tutkimuksella on paikkansa yhteiskuntatieteissäkin."

        Tuossakin näköjään oletat että empiirinen = materialistinen.

        "Taikausko on väistynyt tieteellisen tutkimuksen tulosten myötä, kun tieteen valo lankeaa tietämättömyyden lakeuksiin (oho!, menipä runolliseksi)."

        Niinhän sitä kuvitellaan mutta jos otetaan huomioon sillä ihan samalla empiirisellä tieteen metodilla saadut tulokset 150 vuoden parapsykologisessa tutkimuksessa sekä sen lisäksi lukemattomat tavallisten ihmisten keskenään koherentit kokemukset tuhansien vuosien ajalta niin maailma ei enää näytäkään niin mekaaniselta "faktahommalta" kuin ns. akateemisissa piireissä laajasti nykyään luullaan.

        Ns. "taikauskosta" luopumisen kääntöpuolena on sitten se että kaikki merkitykset, tarkoitukset ja periaatteessa kaikki yksilön kannalta tärkeimmät asiat jäävät sen tieteen tuottaman maailmankuvan ulkopuolelle. Oikeastaan tuo maailmankuvan köyhtyminen seuraa jo empiirisen tieteen metodista eli siitä että vain se kolmannen persoonan näkökulma eli aineen näkäkulma sallitaan.


        "Joillakin idealistiseen ajatteluun taipuvaisilla henmiklöillä vaikuttaa kuitenkin olevan heikko suhde totuuteen. Heillä ilmenee jonkinlaista tieteen vastaisuutta, vaikka tieteellä ei ole ennakko-oletuksena materialismia."

        Heh. Oletko varma että tiedät mikä on "totuus".

        Minä en vaan suhtaudu kovin tosikkomaisesti mihinkään. Tämän tyyppistä tiedehuumoria kaivattaisiin enemmän:

        https://www.youtube.com/watch?v=-M-vnmejwXo

        https://www.youtube.com/watch?v=FQjgsQ5G8ug


        Lähes aina tieteen historiassa enemmistö helposti kuvittelee että tieteen maailmankuva on jo olennaisilta osiltaan jotenkin lopullinen ja valmis ja vain joitakin yksityiskohtia pitää tarkentaa ja niidenkin kuvitellaan tarkentuvan kunhan uusi ja uljas tekniikka saadaan käyttöön tyypillisesti 10-30 v sisällä.

        Tieteen metodiin ja empirismiin sisältyy materialismi ja vielä yleensä varsin deterministisessä muodossa. Tieteen esikuvina nähdäänkin usein tähtitiede jossa taivaankappaleet näyttävät liikkuvan ikuisten, matemaattisesti tarkkojen kaavojen mukaan jonkinlaisena kellokoneistona.

        Kellon vertauskuva on vain nykyään muuttunut tietokoneen vertauskuvaksi eli tietoisuuskin (joka on aina kaiken kokemuksen subjekti eikä koskaan objekti, objektina voivat olla vain tietoisuuden sisällöt eli kokemukset) on tai kuvitellaan olevan jonkinlainen tietotekninen hermoverkko ja kieltämättä sellaisten vertauskuvien kantama tiede kykenee tuottamaan mielenkiintoista tekniikkaa mutta sen oppivankin tekoälyn taso on pohjimmiltaan aika alkeellinen juuri mekaanisuutensa ja joustamattomuutensa takia.


        "(Sinäkin mainitsit antitieteellisyytesi aiemmin.)"

        Siinä oli hymiö perässä joten se oli tarkoitettu lähinnä vitsiksi. Minä olenkin todellinen skeptikko eli suhtaudun kaikkeen suurinpiirtein yhtä skeptisesti kun taas näennäis-skeptikot ovat kaiken valtavirtatieteen suhteen aina tosiuskovaisia ja hartaita hymistelijöitä kaiken sen suhteen mitä tieteen kulloinenkin ylipapisto tuottaa tiedotusvälineisiin ja populaariin tiedekirjallisuuteen.


        Toisinajattelusta seuraa usein ihan hauskaa ja joskus mielenkiintoistakin keskustelua.

        En tiedä mistä johtuu mutta tuntuu usein vastapuolella olevan hyvin kirjaimellista ja mekaanista se asioiden hahmottaminen eli vähän sama tilanne kuin siinä Searlen kiinalaisessa huoneessa ainakin toisen osapuolen suhteen vaikka puhutaankin samaa kieltä. Mietin välillä itsekseni että pääsisköhän toi kaveri edes läpi Turingin testistä jos minä olisin testaajana.


      • koittakaakehitellä
        Belisario kirjoitti:

        "materialismi ei ole tieteen ennakko-oletuksena, vaan monet tieteen tulokset ovat johtaneet filosofiseen johtopäätökseen materialismin puolesta. "

        On se koska hyväksytään vain se kolmannen persoonan näkökulma eli aistihavainto jossa kaikki tulkitaan aineelliseksi. Materialismin johtopäätös on ollut oletuksena mukana jo kokeellisen tieteen alusta lähtien eli Galileon ja Newtonin ajoista jolloin tieteessä otettiin käyttöön konevertauskuva.

        "Yhteiskuntatieteissä muukin taustafilosofia on mahdollinen kuin materialistinen, mutta empiiriselle tutkimuksella on paikkansa yhteiskuntatieteissäkin."

        Tuossakin näköjään oletat että empiirinen = materialistinen.

        "Taikausko on väistynyt tieteellisen tutkimuksen tulosten myötä, kun tieteen valo lankeaa tietämättömyyden lakeuksiin (oho!, menipä runolliseksi)."

        Niinhän sitä kuvitellaan mutta jos otetaan huomioon sillä ihan samalla empiirisellä tieteen metodilla saadut tulokset 150 vuoden parapsykologisessa tutkimuksessa sekä sen lisäksi lukemattomat tavallisten ihmisten keskenään koherentit kokemukset tuhansien vuosien ajalta niin maailma ei enää näytäkään niin mekaaniselta "faktahommalta" kuin ns. akateemisissa piireissä laajasti nykyään luullaan.

        Ns. "taikauskosta" luopumisen kääntöpuolena on sitten se että kaikki merkitykset, tarkoitukset ja periaatteessa kaikki yksilön kannalta tärkeimmät asiat jäävät sen tieteen tuottaman maailmankuvan ulkopuolelle. Oikeastaan tuo maailmankuvan köyhtyminen seuraa jo empiirisen tieteen metodista eli siitä että vain se kolmannen persoonan näkökulma eli aineen näkäkulma sallitaan.


        "Joillakin idealistiseen ajatteluun taipuvaisilla henmiklöillä vaikuttaa kuitenkin olevan heikko suhde totuuteen. Heillä ilmenee jonkinlaista tieteen vastaisuutta, vaikka tieteellä ei ole ennakko-oletuksena materialismia."

        Heh. Oletko varma että tiedät mikä on "totuus".

        Minä en vaan suhtaudu kovin tosikkomaisesti mihinkään. Tämän tyyppistä tiedehuumoria kaivattaisiin enemmän:

        https://www.youtube.com/watch?v=-M-vnmejwXo

        https://www.youtube.com/watch?v=FQjgsQ5G8ug


        Lähes aina tieteen historiassa enemmistö helposti kuvittelee että tieteen maailmankuva on jo olennaisilta osiltaan jotenkin lopullinen ja valmis ja vain joitakin yksityiskohtia pitää tarkentaa ja niidenkin kuvitellaan tarkentuvan kunhan uusi ja uljas tekniikka saadaan käyttöön tyypillisesti 10-30 v sisällä.

        Tieteen metodiin ja empirismiin sisältyy materialismi ja vielä yleensä varsin deterministisessä muodossa. Tieteen esikuvina nähdäänkin usein tähtitiede jossa taivaankappaleet näyttävät liikkuvan ikuisten, matemaattisesti tarkkojen kaavojen mukaan jonkinlaisena kellokoneistona.

        Kellon vertauskuva on vain nykyään muuttunut tietokoneen vertauskuvaksi eli tietoisuuskin (joka on aina kaiken kokemuksen subjekti eikä koskaan objekti, objektina voivat olla vain tietoisuuden sisällöt eli kokemukset) on tai kuvitellaan olevan jonkinlainen tietotekninen hermoverkko ja kieltämättä sellaisten vertauskuvien kantama tiede kykenee tuottamaan mielenkiintoista tekniikkaa mutta sen oppivankin tekoälyn taso on pohjimmiltaan aika alkeellinen juuri mekaanisuutensa ja joustamattomuutensa takia.


        "(Sinäkin mainitsit antitieteellisyytesi aiemmin.)"

        Siinä oli hymiö perässä joten se oli tarkoitettu lähinnä vitsiksi. Minä olenkin todellinen skeptikko eli suhtaudun kaikkeen suurinpiirtein yhtä skeptisesti kun taas näennäis-skeptikot ovat kaiken valtavirtatieteen suhteen aina tosiuskovaisia ja hartaita hymistelijöitä kaiken sen suhteen mitä tieteen kulloinenkin ylipapisto tuottaa tiedotusvälineisiin ja populaariin tiedekirjallisuuteen.


        Toisinajattelusta seuraa usein ihan hauskaa ja joskus mielenkiintoistakin keskustelua.

        En tiedä mistä johtuu mutta tuntuu usein vastapuolella olevan hyvin kirjaimellista ja mekaanista se asioiden hahmottaminen eli vähän sama tilanne kuin siinä Searlen kiinalaisessa huoneessa ainakin toisen osapuolen suhteen vaikka puhutaankin samaa kieltä. Mietin välillä itsekseni että pääsisköhän toi kaveri edes läpi Turingin testistä jos minä olisin testaajana.

        "On se koska hyväksytään vain se kolmannen persoonan näkökulma eli aistihavainto jossa kaikki tulkitaan aineelliseksi."

        Näin ei ole asianlaita. Ihan hyvin tutkitaan myös 1. persoonan näkökulman kokemuksia, kun kysellään ihmisten kokemuksista monilla eri tieteen aloilla. Ontologista ennakko-olettamaa ei ole tieteen metodissa. (Tästä emme löydä yhteisymmärrystä keskenämme.)

        "Tuossakin näköjään oletat että empiirinen = materialistinen."

        Empiiirinen tutkimus voi koskea aineetonta tietoisuutta (kuten olen aiemmin todennutkin). Koska välitöntä tietoa ei voi saada tietoisuudesta, pitää sitä kyselytutkimuksin selvittää.

        "Oikeastaan tuo maailmankuvan köyhtyminen seuraa jo empiirisen tieteen metodista eli siitä että vain se kolmannen persoonan näkökulma eli aineen näkäkulma sallitaan."

        Ihmisten maailmankuva on viimeisten satojen vuosien aikana laajentunut ja rikastunut huomattavasti aiemmasta. Moniarvoisuus on länsimaissa vallannut alaa yksioikoisten taikauskoisten käsitysten väistyessä. Sallitaan jopa kaikenlaista hömppää! Et joudu inkvisitioon niistäkään.

        Totuus on siitä hieno käsite, että siitä ei ole kenelläkään varmuutta. Silti kaikkea ei tarvitse eikä käytännössä voikaan koko ajan epäillä, vaan voimme pitää joitakin tietojamme luotettavina ja sitten niillä luotettavan tiedon "kiveyksillä" seisten voimme epäillä monia maailmankuvamme osia. Myöhemmin voimme vaihtaa epäilymme kohdetta niinkin, että epäilemme mainittuja luotettavia tietojamme (so. kiveyksiä), jolloin tukeudumme joihinkin toisiin sillä hetkellä luotettavana pitämiimme tietoihin (kuvaannollisesti olisi kuin vaihtaisimme kiveykseltä toiselle). Tämä on aitoa skeptistä metodia (ilman että epäiltäisiin kaikkea mahdollista jatkuvasti).

        "Lähes aina tieteen historiassa enemmistö helposti kuvittelee että tieteen maailmankuva on jo olennaisilta osiltaan jotenkin lopullinen ja valmis ja vain joitakin yksityiskohtia pitää tarkentaa ja niidenkin kuvitellaan tarkentuvan kunhan uusi ja uljas tekniikka saadaan käyttöön tyypillisesti 10-30 v sisällä."

        Niin mikä enemmistö? kenestä puhut? On varmastikin suurta yleisöä, joka ei ole perehtynyt syvällisemmin tieto-oppiin eikä sen vuoksi osaa skeptistä asennetta.

        " Tieteen esikuvina nähdäänkin usein tähtitiede jossa taivaankappaleet näyttävät liikkuvan ikuisten, matemaattisesti tarkkojen kaavojen mukaan jonkinlaisena kellokoneistona."

        Ehkä Newtoniin saakka ajateltiin näin, mutta jo Newton tiesi, että laskelmansa vaativat aina aika ajoin "asetusten päivittämistä". Alkuarvoista ei voitu silloisin mekaanisin laskelmin ennustaa loputtomasti taivaan kappaleiden liikettä. Syytä ei tiedetty, mutta asia selvisi Einsteinin suhteellisuusteorioiden myötä.

        "mutta sen oppivankin tekoälyn taso on pohjimmiltaan aika alkeellinen juuri mekaanisuutensa ja joustamattomuutensa takia."

        Näin on sen ja monen muunkin teknologian kannalta. Mielestäni kännykätkin ovat alkeellisia, kun täytyy jokin luuri laittaa korvalle puhuakseen. Onneksi puhelinnumeroita ei tarvitse enää muistaa, kun ne on vempaimen muistissa. Mutta puhelin voisi olla vaikkapa intergoituna otsaan!


      • Järkisyitä
        Belisario kirjoitti:

        "materialismi ei ole tieteen ennakko-oletuksena, vaan monet tieteen tulokset ovat johtaneet filosofiseen johtopäätökseen materialismin puolesta. "

        On se koska hyväksytään vain se kolmannen persoonan näkökulma eli aistihavainto jossa kaikki tulkitaan aineelliseksi. Materialismin johtopäätös on ollut oletuksena mukana jo kokeellisen tieteen alusta lähtien eli Galileon ja Newtonin ajoista jolloin tieteessä otettiin käyttöön konevertauskuva.

        "Yhteiskuntatieteissä muukin taustafilosofia on mahdollinen kuin materialistinen, mutta empiiriselle tutkimuksella on paikkansa yhteiskuntatieteissäkin."

        Tuossakin näköjään oletat että empiirinen = materialistinen.

        "Taikausko on väistynyt tieteellisen tutkimuksen tulosten myötä, kun tieteen valo lankeaa tietämättömyyden lakeuksiin (oho!, menipä runolliseksi)."

        Niinhän sitä kuvitellaan mutta jos otetaan huomioon sillä ihan samalla empiirisellä tieteen metodilla saadut tulokset 150 vuoden parapsykologisessa tutkimuksessa sekä sen lisäksi lukemattomat tavallisten ihmisten keskenään koherentit kokemukset tuhansien vuosien ajalta niin maailma ei enää näytäkään niin mekaaniselta "faktahommalta" kuin ns. akateemisissa piireissä laajasti nykyään luullaan.

        Ns. "taikauskosta" luopumisen kääntöpuolena on sitten se että kaikki merkitykset, tarkoitukset ja periaatteessa kaikki yksilön kannalta tärkeimmät asiat jäävät sen tieteen tuottaman maailmankuvan ulkopuolelle. Oikeastaan tuo maailmankuvan köyhtyminen seuraa jo empiirisen tieteen metodista eli siitä että vain se kolmannen persoonan näkökulma eli aineen näkäkulma sallitaan.


        "Joillakin idealistiseen ajatteluun taipuvaisilla henmiklöillä vaikuttaa kuitenkin olevan heikko suhde totuuteen. Heillä ilmenee jonkinlaista tieteen vastaisuutta, vaikka tieteellä ei ole ennakko-oletuksena materialismia."

        Heh. Oletko varma että tiedät mikä on "totuus".

        Minä en vaan suhtaudu kovin tosikkomaisesti mihinkään. Tämän tyyppistä tiedehuumoria kaivattaisiin enemmän:

        https://www.youtube.com/watch?v=-M-vnmejwXo

        https://www.youtube.com/watch?v=FQjgsQ5G8ug


        Lähes aina tieteen historiassa enemmistö helposti kuvittelee että tieteen maailmankuva on jo olennaisilta osiltaan jotenkin lopullinen ja valmis ja vain joitakin yksityiskohtia pitää tarkentaa ja niidenkin kuvitellaan tarkentuvan kunhan uusi ja uljas tekniikka saadaan käyttöön tyypillisesti 10-30 v sisällä.

        Tieteen metodiin ja empirismiin sisältyy materialismi ja vielä yleensä varsin deterministisessä muodossa. Tieteen esikuvina nähdäänkin usein tähtitiede jossa taivaankappaleet näyttävät liikkuvan ikuisten, matemaattisesti tarkkojen kaavojen mukaan jonkinlaisena kellokoneistona.

        Kellon vertauskuva on vain nykyään muuttunut tietokoneen vertauskuvaksi eli tietoisuuskin (joka on aina kaiken kokemuksen subjekti eikä koskaan objekti, objektina voivat olla vain tietoisuuden sisällöt eli kokemukset) on tai kuvitellaan olevan jonkinlainen tietotekninen hermoverkko ja kieltämättä sellaisten vertauskuvien kantama tiede kykenee tuottamaan mielenkiintoista tekniikkaa mutta sen oppivankin tekoälyn taso on pohjimmiltaan aika alkeellinen juuri mekaanisuutensa ja joustamattomuutensa takia.


        "(Sinäkin mainitsit antitieteellisyytesi aiemmin.)"

        Siinä oli hymiö perässä joten se oli tarkoitettu lähinnä vitsiksi. Minä olenkin todellinen skeptikko eli suhtaudun kaikkeen suurinpiirtein yhtä skeptisesti kun taas näennäis-skeptikot ovat kaiken valtavirtatieteen suhteen aina tosiuskovaisia ja hartaita hymistelijöitä kaiken sen suhteen mitä tieteen kulloinenkin ylipapisto tuottaa tiedotusvälineisiin ja populaariin tiedekirjallisuuteen.


        Toisinajattelusta seuraa usein ihan hauskaa ja joskus mielenkiintoistakin keskustelua.

        En tiedä mistä johtuu mutta tuntuu usein vastapuolella olevan hyvin kirjaimellista ja mekaanista se asioiden hahmottaminen eli vähän sama tilanne kuin siinä Searlen kiinalaisessa huoneessa ainakin toisen osapuolen suhteen vaikka puhutaankin samaa kieltä. Mietin välillä itsekseni että pääsisköhän toi kaveri edes läpi Turingin testistä jos minä olisin testaajana.

        Näissä kommenteissa paistaa jotenkin yksinkertaistettu olkiukko-versio materialismista. Ensinnäkin kaikki ei ole ainetta (eli aisten havaittavaa massallista materiaa) eikä kaikea voi selittää mekaniikan kautta. Pohjimmiltaan kvanttifysiikkaan mentäessä energia ja hiukkaset ovat hyvin mystisiä eikä niiden perimmäistä olemusta tunneta. Ajatus, että tietoisuuden kokemus voisi syntyä energian ja materian kautta, on ihan yhtä mystinen kuin uskonnollisetkin maailmat. Ainut ero on, että tiede tutkii ja aidosti yrittää ymmärtää maailmaa.


    • jhkhkhkjhj

      Tietoisuus on kvarkkien tanssia.

      • Anonyymi

        Hihi. Kvarkki on historiallisen tietoisuuden substanssi-hyttynen. Populaarifyysikot universaaleja totaali-idiootteja.


    • Anonyymi

      Realistit ja fysikalistit tiedetirehtööreilevät omassa lammaskuplassaan, jossa uskallusta ajatteluun ei esiinny.

      Sitä pahempi tuhoonsa etenevälle kulttuurille, joka ei osaa eikä tahdo ottaa vastuuta ideoistaan.

      Lopunajan virkamiestampioismi. Filosofian päivänlasku.

      • Anonyymi

        Realistit kuplassaan, no ainakin se kupla on maailmankaikkeuden kokoinen.

        Idealistit tuntuvat kärsivän narsismista ja ties muista persoonallisuusrajoitteista.
        Tajunta on mielellään joko minun tajuntani tai minun tajuamani tajunta on kaikki.
        Vessatkin ovat tajuntani represetraatioita, ties mitä muut tajunnat tekevät siellä!

        Onko Kastrupin todellisuus vain jättikokoinen yleinen käymälä?
        Vai Kastrupin kosminen tajunta ylen kiinnostunut käymäläasioista.


      • Anonyymi

        Kuplan kokoa voit arvioida siitä mitä idealistit ovat historiassaan saaneet
        aikaan. Väittelyä kuplansa koosta.

        Ensin oli vain pikkuisia metsän ja luonnon henkiä.
        Sitten hoksattiin jumalat.
        Jumalia oli liikaa, yksi riittäisi.
        Ehkäpä ihmisen tajunta riittäisi.
        Välillä pohdittiin pahantahtoista voimakasta oliota.
        Tekniikan kehittyessä esiin putkahti Matrix.
        Vielä parempaa, ehkäpä korkean sivilisaation simulaatio.
        Kastrupin maksimi, kosminen tajunta, joka kärsii persoonallisuushäiriöstä.

        Siinä esitetty idealismin monituhatvuotisen historian päälinjat.
        Kaikissa esiintyy "tajunnan" sijaintiongelma, idealismin näköjään ratkaisematon
        ongelma.

        Siinä "tajunta" sitaateissa kun eivät ole päässeet edes yksimielisyyteen "tajunnan"
        määritelmästä.


    • Anonyymi

      Eipäskun, eikös idealismi ole Realismin vastakohta, ja silloin se on väärin!

      Luin iltiksen keskusteluista tietäjä nimim. "Realistin" kommentin ja uskoin häntä.

      Muutenkin realismi on tieteellistä, ja idealismi epätieteellistä,. Näin on, mutten nyt jaksa niitä pidempiä rimpsuja, aamen.

      Tiede on jo pitkään keskittynyt rimpuilemaan, miksi lakeijafilosofian pitää olla realismia. Ergo, pitäähän sen olla. Aamen, taas (sori toisto, en ole Uskovainen).

      Tiede on Kriittistä Realismia. Kriittistä ja siis realismia. Tuliks rtää nyt selväksi jatodistettua. Jatketaan tästä ensi kerralla. T: Hegel

    • Anonyymi
      • Anonyymi

        Kyllähän filosofian idealismeilla on yhtymäkohtansa noihinkin merkityssuuntiin. Selviävät tutkimalla vaikka filosofian suuria klassikoita tarkemmin (Platonista voi lähteä liikkeelle, jatkaa esim. saksalaiseen idealismiin, jne.).


      • Anonyymi
        Anonyymi kirjoitti:

        Kyllähän filosofian idealismeilla on yhtymäkohtansa noihinkin merkityssuuntiin. Selviävät tutkimalla vaikka filosofian suuria klassikoita tarkemmin (Platonista voi lähteä liikkeelle, jatkaa esim. saksalaiseen idealismiin, jne.).

        Joo melkeen on aikaa kahlata jotain platoneita läpi. Hienoja ne ns. dialogit muutenki, platon kirjottaa helvetinmoisia monologeja yhden hahmon suuhun, ylensä sokrateen, ja muut keskustelijat kehuu kilpaa kuinka hyviä ajatuksia ne on :D
        Et oot itekkää tainnu jaksaa niitä lukee ku ainoa mitä osaat niistä sanoa on että "no lue ne" hahaha


      • Anonyymi
        Anonyymi kirjoitti:

        Joo melkeen on aikaa kahlata jotain platoneita läpi. Hienoja ne ns. dialogit muutenki, platon kirjottaa helvetinmoisia monologeja yhden hahmon suuhun, ylensä sokrateen, ja muut keskustelijat kehuu kilpaa kuinka hyviä ajatuksia ne on :D
        Et oot itekkää tainnu jaksaa niitä lukee ku ainoa mitä osaat niistä sanoa on että "no lue ne" hahaha

        Joskus tyhmyyskin kai onoma valinta.

        Noista lähteistä ne klassisen idealismin ja etiikan ydinsidokset kuitenkin siis löytyvät, kiinnostuneille.


    • Anonyymi

      Mielen sisäinen? Ei ole erikseen sisäpuolta ja ulkopuolta. Jos siis katsotaan asiaa laajemmasta perspektiivistä niin kaikki on yhtä ja samaa. Jopa maailmankaikkeus ja pieni ihminen. Erillisyyden tunne on harhaa et pääse tästä kokonaisuudesta eroon vaikka haluaisit.

    • Anonyymi

      Todellisuus on juuri sitä, mitä ihmiskunta on koko olemassaoloaikansa rakentanut. Se vaihtelee henkisestä fyysiseen, ja poikkeaa myös muillekin ideaurille, kuten illuusio, matrix, avaruusmuukalaisten koe yms.

      Koska mieli toimii suljettuna luomisympäristönä, sen hahmottaminen tarjoaa mahdollisuudet lähes minkälaisen tahansa "todellisuuskuvan" kehittelyn. Mitä sallivampi ja avoimempi mieli, sitä mielikuvituksellisempia on ideat. Mutta lopulta yksi jää uupumaan: se ultimaattinen totuus.

      Me olemme luoneet kaikki mahdolliset käsitteet, termit, matematiikan, opit yms., joilla yritämme rakentaa tyydyttävää kokonaisuutta todellisuudesta. Siksi kuva todellisuudesta on kovin "ihmismäinen", eli käytännössä voimme myöntää, ettemme tiedä "todellisuutta" kuin sen tiedon perusteella, minkä olemme itse itseämme varten luoneet ja sen puuttuvan oleellisen tiedon vuoksi luomme korviketodellisuuksia. Emme siis koskaan tule saamaan sitä tietoa, mistä tämä kaikki on peräisin, miten se on tullut ja mitä se ylipäätään on, ellei joku ns. ulkopuolinen, joka sen tietää, näytä sitä meille, jolloin voimme oivaltaa, miten väärässä olemmekaan olleet.

      • Anonyymi

        "Koska mieli toimii suljettuna luomisympäristönä, sen hahmottaminen tarjoaa mahdollisuudet lähes minkälaisen tahansa "todellisuuskuvan" kehittelyn. "

        Ja sitten se mitä sanot "kaikki mahdolliset käsitteet, termit, matematiikan, opit.."

        Koneisto mikä muodostaa käsitteet, matematiikan yms ei ole suljettu sillä
        tavalla kun haluat esittää, se ei ole suljettu käsitteellinen ympäristö.

        Pikemmin puoliavoin ellei avoin, joten on mahdollista luoda käsitteitä jotka
        murtavat tuon suljetun ympäristön, niin kuin on tehty useasti filosofian
        historiassa. Tuo suljettu mieli on vain idealistista sanaverhoa ja sumutusta.


      • Anonyymi
        Anonyymi kirjoitti:

        "Koska mieli toimii suljettuna luomisympäristönä, sen hahmottaminen tarjoaa mahdollisuudet lähes minkälaisen tahansa "todellisuuskuvan" kehittelyn. "

        Ja sitten se mitä sanot "kaikki mahdolliset käsitteet, termit, matematiikan, opit.."

        Koneisto mikä muodostaa käsitteet, matematiikan yms ei ole suljettu sillä
        tavalla kun haluat esittää, se ei ole suljettu käsitteellinen ympäristö.

        Pikemmin puoliavoin ellei avoin, joten on mahdollista luoda käsitteitä jotka
        murtavat tuon suljetun ympäristön, niin kuin on tehty useasti filosofian
        historiassa. Tuo suljettu mieli on vain idealistista sanaverhoa ja sumutusta.

        Käsitteiden luominen on ihmisen oppima kyky ja se kyky sijaitsee aivoissa. Se ei murra aivojen, aistien ja mielen perusominaisuuksia yksittäisenä yksikkönä.

        Puoliavoimuudesta voidaan puhua siinä merkityksessä, että yksilöllä on sensorit, joilla voidaan vastaanottaa ulkoisia signaaleita: molekyylien liike/ilmanpaine, hajumolekyylit, fotonit, fyysinen kontakti ja lämpösäteily.

        Se, mitä tarkoitan termillä 'suljettu' perustuu siihen, että me koemme maailman sellaisena kuin aivomme ja mielemme sen muotoilee, emme sen alkuperäisessä muodossa, joka taas selittyy sillä, että me muunnamme alkuperäiset signaalit toiseksi, jotta niistä voidaan luoda mielemme ymmärtämiä "käännöksiä" tai "kopioita". Tämä ominaisuus luo mielen ja ulkomaailman välille selkeästi rajan -> miltä maailma näyttäisi ja kuulostaisi ilman tarvetta muuntaa signaaleja? Mitä näkisimme tai kokisimme? Mistä löytäisimme itsemme?

        Nämä ominaisuudet luovat myös ongelman: aivomme prosessoivat moniulotteisen ilmiön, joka mm. signaalien muuntamisesta johtuen on periaatteessa illuusio; koemme olevamme juuri sellaisessa maailmassa, jona se ympärillämme avautuu... vaikka kuitenkin kaikki se havaittu ja koettu luodaan ja tapahtuu 100% aivoissa, aistien "kääntämänä", aivojen prosessoimana ja mielen tulkitsemana.

        Jos tämä hahmottuu, silloin avautuu mielessä uusi "ovi", joka johtaa monenlaisiin mahdollisuuksiin pohtia, mitä sen rajan takana oikeasti on. Mikä on se todellisuus, josta meillä on nyt vain itsemme luomia versioita? Toki voi aina tyytyä siihen perinteiseen todellisuuskäsitykseen, jos tuntuu liian scifiltä tai huuhaalta, mutta jos on avoin ja salliva mieli, niin se vanha todellisuuskäsitys voi muuttua melkoisesti.


      • Anonyymi
        Anonyymi kirjoitti:

        Käsitteiden luominen on ihmisen oppima kyky ja se kyky sijaitsee aivoissa. Se ei murra aivojen, aistien ja mielen perusominaisuuksia yksittäisenä yksikkönä.

        Puoliavoimuudesta voidaan puhua siinä merkityksessä, että yksilöllä on sensorit, joilla voidaan vastaanottaa ulkoisia signaaleita: molekyylien liike/ilmanpaine, hajumolekyylit, fotonit, fyysinen kontakti ja lämpösäteily.

        Se, mitä tarkoitan termillä 'suljettu' perustuu siihen, että me koemme maailman sellaisena kuin aivomme ja mielemme sen muotoilee, emme sen alkuperäisessä muodossa, joka taas selittyy sillä, että me muunnamme alkuperäiset signaalit toiseksi, jotta niistä voidaan luoda mielemme ymmärtämiä "käännöksiä" tai "kopioita". Tämä ominaisuus luo mielen ja ulkomaailman välille selkeästi rajan -> miltä maailma näyttäisi ja kuulostaisi ilman tarvetta muuntaa signaaleja? Mitä näkisimme tai kokisimme? Mistä löytäisimme itsemme?

        Nämä ominaisuudet luovat myös ongelman: aivomme prosessoivat moniulotteisen ilmiön, joka mm. signaalien muuntamisesta johtuen on periaatteessa illuusio; koemme olevamme juuri sellaisessa maailmassa, jona se ympärillämme avautuu... vaikka kuitenkin kaikki se havaittu ja koettu luodaan ja tapahtuu 100% aivoissa, aistien "kääntämänä", aivojen prosessoimana ja mielen tulkitsemana.

        Jos tämä hahmottuu, silloin avautuu mielessä uusi "ovi", joka johtaa monenlaisiin mahdollisuuksiin pohtia, mitä sen rajan takana oikeasti on. Mikä on se todellisuus, josta meillä on nyt vain itsemme luomia versioita? Toki voi aina tyytyä siihen perinteiseen todellisuuskäsitykseen, jos tuntuu liian scifiltä tai huuhaalta, mutta jos on avoin ja salliva mieli, niin se vanha todellisuuskäsitys voi muuttua melkoisesti.

        "Käsitteiden luominen on ihmisen oppima kyky ja se kyky sijaitsee aivoissa. Se ei
        murra aivojen, aistien ja mielen perusominaisuuksia yksittäisenä yksikkönä."

        Käsitteiden luominen murtaa idealistisen tulkinnan tarpeen tuossa yllämainitussa
        tilanteessa. "Mieli" on hankala termi, sen monitulkintaisuus aiheuttaa vain
        epävarmuutta mitä sillä tarkkaan ottaen tarkoitetaan.

        Ulkomaailma sellaisena kuin se realistisesti ymmärretään ja aistien kautta
        tuleva tieto, jonka aivot käsittelevät ja jonka tajuamme, on osittain
        hämmentävä tilanne. Mutta "mielen" ja ulkomaailman välinen raja, miten
        se pitäisi ymmärtää.

        Kuitenkin luonto evoluution kautta on ratkaissut sen ilmeisesti monta kertaa,
        petoeläinten on löydettävä saaliinsa ja pyydystettävä se, petoeläimillä
        on useita eri tapoja lajinsa ja ominaisuuksiensa mukaan, joten yksi ratkaisu
        ei ilmeisesti riittäisi.

        Ihmiset eivät kuitenkaan eroa merkittävästi muista petoeläimistä, joten
        kykymme "nähdä" todellisuus oikein on vain perintöä esipedoilta, ihmisen
        suurin ero on käsitekoneiston parempi suorituskyky, joka tosin aiheuttaa
        ongelmia jos suurta hyötyä. Viidakossa toikkaroivan idealistin pyydystäminen
        tiikerille on erinomaisen helppo juttu.

        Tiede on selvittänyt esim kuuloaistimuksen merkityksen, ääniä ei ole
        todellisuudessa, ne ovat väliaineen paineenvaihteluiden tulkinta aivoissa,
        värit ovat valonaallonpituuksien ja silmien yhteisvaikutuksen tulkinta aivoissa.

        Voit kokeilla itse näiden aistimusten roolia, mene ulos ja laita korviin
        erittäin tehokkaat melu/äänisuojat, tunnet kyllä ilmavirtauksia mutta et
        ääniä. Värien poistaminen voi olla hankalaa. ehkä on lasit jotka muokkaavat
        valon aallonpituuksia niin että kuva näkyy vain mustavalkoisena.

        Erikoisfilmillä värien poistaminen onnistuu paremmin, sitten näet kuvan
        todellisuudesta sellaisena kuin se on ilman ihmistä.
        Kuvassa näkyisi ehkä kiviä, tietysti koska ne ovat siellä, vastoin idealistien
        puheita "mieli/tajunta" ei tee kiviä todellisuuteen.

        Aivot prosessoivat käsityksen todellisuudesta, illuusio? Taikurit tuottavat
        illuusioita, enpä sanoisi aivot normaali elämässä tuottavat niitä, taikurit kyllä
        pystyvät huijaamaan aivoja, käyttämällä havaintojärjestelmän heikkouksia.

        Taikurit pystyvät huijaamaan koiria, samoilla tekniikoilla.

        Välttäisin idealistisia tarinoita ja selityksiä, kuten proffa kerran kirjoitti,
        "niissä ei ole mitään lisäarvoa".


      • Anonyymi
        Anonyymi kirjoitti:

        "Käsitteiden luominen on ihmisen oppima kyky ja se kyky sijaitsee aivoissa. Se ei
        murra aivojen, aistien ja mielen perusominaisuuksia yksittäisenä yksikkönä."

        Käsitteiden luominen murtaa idealistisen tulkinnan tarpeen tuossa yllämainitussa
        tilanteessa. "Mieli" on hankala termi, sen monitulkintaisuus aiheuttaa vain
        epävarmuutta mitä sillä tarkkaan ottaen tarkoitetaan.

        Ulkomaailma sellaisena kuin se realistisesti ymmärretään ja aistien kautta
        tuleva tieto, jonka aivot käsittelevät ja jonka tajuamme, on osittain
        hämmentävä tilanne. Mutta "mielen" ja ulkomaailman välinen raja, miten
        se pitäisi ymmärtää.

        Kuitenkin luonto evoluution kautta on ratkaissut sen ilmeisesti monta kertaa,
        petoeläinten on löydettävä saaliinsa ja pyydystettävä se, petoeläimillä
        on useita eri tapoja lajinsa ja ominaisuuksiensa mukaan, joten yksi ratkaisu
        ei ilmeisesti riittäisi.

        Ihmiset eivät kuitenkaan eroa merkittävästi muista petoeläimistä, joten
        kykymme "nähdä" todellisuus oikein on vain perintöä esipedoilta, ihmisen
        suurin ero on käsitekoneiston parempi suorituskyky, joka tosin aiheuttaa
        ongelmia jos suurta hyötyä. Viidakossa toikkaroivan idealistin pyydystäminen
        tiikerille on erinomaisen helppo juttu.

        Tiede on selvittänyt esim kuuloaistimuksen merkityksen, ääniä ei ole
        todellisuudessa, ne ovat väliaineen paineenvaihteluiden tulkinta aivoissa,
        värit ovat valonaallonpituuksien ja silmien yhteisvaikutuksen tulkinta aivoissa.

        Voit kokeilla itse näiden aistimusten roolia, mene ulos ja laita korviin
        erittäin tehokkaat melu/äänisuojat, tunnet kyllä ilmavirtauksia mutta et
        ääniä. Värien poistaminen voi olla hankalaa. ehkä on lasit jotka muokkaavat
        valon aallonpituuksia niin että kuva näkyy vain mustavalkoisena.

        Erikoisfilmillä värien poistaminen onnistuu paremmin, sitten näet kuvan
        todellisuudesta sellaisena kuin se on ilman ihmistä.
        Kuvassa näkyisi ehkä kiviä, tietysti koska ne ovat siellä, vastoin idealistien
        puheita "mieli/tajunta" ei tee kiviä todellisuuteen.

        Aivot prosessoivat käsityksen todellisuudesta, illuusio? Taikurit tuottavat
        illuusioita, enpä sanoisi aivot normaali elämässä tuottavat niitä, taikurit kyllä
        pystyvät huijaamaan aivoja, käyttämällä havaintojärjestelmän heikkouksia.

        Taikurit pystyvät huijaamaan koiria, samoilla tekniikoilla.

        Välttäisin idealistisia tarinoita ja selityksiä, kuten proffa kerran kirjoitti,
        "niissä ei ole mitään lisäarvoa".

        Naiivin epäfilosofista realistista rimpuilua ja huutamista; sano suoraan vaan.


      • Anonyymi
        Anonyymi kirjoitti:

        Naiivin epäfilosofista realistista rimpuilua ja huutamista; sano suoraan vaan.

        Tervetuloa vaan realistiseen todellisuuteen, voit lopettaa sen unissakävelyn,
        voit kävellä vaikka auton alle.
        Siinä olisikin varsinainen realismitesti, kun sairaalan vuoteella "etsisit itseäsi",
        kuten jotkut totuuden etsijät tapaavat sanoa.


    • Anonyymi

      Todellisuus on subjektiivinen kokemus. Kyse on myös suhtaumisesta omaan todellisuuteen. Aloittajalla mennee sekaisin Totuus ja todellisuus. Totuus vallitsee aina, mutta me emme sitä saavuta kuin todellisuutta kohtaamalla. Sanotaan elämäksikin. Luonnontieteissä me yritämme mallintaa prosesseja ja nimeämme esimerkiksi suureita.

      • Anonyymi

        Itselläsi menee kylläkin kaikki sekaisin, alkaen "todellisuudesta". No.


    • Anonyymi

      Aivot, keho, niiden anatomia ja fysiologia, vaikuttaa suurelta määrin todellisuuteemme, ja muulta osin vallitsee kulttuurit, jotka sisältää kuvitelmat, joiden järjellisyyden ja todellisuuden määrät vaihtelee, jotka muodostaa isoimman osan ihmiskunnan ongelmista näinä päivinä. Muun ollen edistystä, ja siltä osin voidaan olettaa edistyksen jatkuvan.
      Tällä hetkellä putinistien määrä on 10-90%, että työsarkaa ja häiriötä riittää vielä pitkäksi aikaa. Aina kun putinistien määrä nousee yli puoleen jossain, tai muuten saa vallan, vallitsee pahuus.
      Siinä koko todellisuuteemme pähkinänkuoressa.

      • Anonyymi

        Otitko varmasti riittävän pähkinäarsenaalin?


    Ketjusta on poistettu 4 sääntöjenvastaista viestiä.

    Luetuimmat keskustelut

    1. En ole rakastunut

      Tai ihastunut sinuun. Kiinnostuin kyllä heti koska erotut massasta.
      Ikävä
      412
      4377
    2. Miksi suomalaisia vainajia säilytetään kylmäkonteissa ulkona? Näin kuolleita kohdellaan Suomessa

      Suomesta ei löydy enää tilaa kuolleille. Tänä päivänä vainajia säilytetään ympäri maata ulkona kylmäkonteissa. Kontit
      Maailman menoa
      241
      2151
    3. Olen ärtynyt koska

      minulla on tunteita sinua kohtaan. Tunteita joita en voi ilmaista. Kaipaan kaikkea sinussa. Siksi olen välillä hankala.
      Ikävä
      68
      1590
    4. Suomalaiset marjat loppuvat

      Suomalaiset marjat mätänevät metsään, koska ulkomaalaiset, lähinnä thaimaalaiset poimijat ovat huolehtineet suomalaisten
      Maailman menoa
      158
      1401
    5. Hallitus korottaa yleisen arvonlisäveron 25,5 prosenttiin

      Yleisen arvonlisäveron kiristäminen 1,5 prosenttiyksiköllä on hallituksen järein toimi kehysriihessä. Se voi tuoda valti
      Perussuomalaiset
      260
      1159
    6. Yhteiskuntaa hyväksi käyttäjät

      Kyllä täällä Suomussalmellakin osaavat käyttää näitä Suomen etuja hyväksi. Vuokrataan ns. asunto lapselle että saa asu
      Suomussalmi
      67
      1087
    7. Mitä teen väärin?

      Alkaa pikku hiljaa tympäsemään ainainen pakkien saanti. Eka ennen kun nähdään, miehet ovat kiinnostuneita viestittelemää
      Sinkut
      137
      1054
    8. Joku tukeva täti syyttää suomalaisia rasisteiksi Hesarissa

      ”Kaikki valkoiset ihmiset Suomessa ovat kasvaneet rasistiseen ajatteluun”, sanoo Maija Laura Kauhanen: https://www.hs.
      Maailman menoa
      167
      1045
    9. Haluaisin tietää

      mikä saa sinut tuntemaan olosi rakastetuksi. Ja sitten haluaisin mahdollisuuden tehdä juuri niin. 💔
      Ikävä
      54
      949
    10. Puhutko toisista ihmisistä

      pahaa, jotta näyttäytyisit itse jotenkin paremmassa valossa?
      Ikävä
      117
      933
    Aihe